Bar Prep
Which type of common law defamation cases require pleadings to proof of special damages?
Slander not falling into one of the four per se categories.
The burdens of an equitable servitude will run to successors in interest if:
The burden of an equitable servitude will run to successors in interest if: (i) the covenanting parties intended that successors in interest be bound by the covenant; (ii) the successor in interest has notice of the covenant; and (iii) the covenant touches and concerns the land (i.e., it benefits the covenantor and her successor in their use and enjoyment of the burdened land). In the absence of a writing, however, the servitude will not be enforced unless there is a common scheme for development as explained above.
Product Liability Manufacturer's Negligence, Defective Product
The plaintiff must show that those designing the product knew or should have known of enough facts to put a reasonable manufacturer on notice about the dangers of marketing the product as designed. Negligent design is not shown, however, if the danger of the product becomes apparent only after the product reaches the public.
Duties of a Lessor of Realty
Tort liability an incident of occupation and control. Lessor obligated to give warning to the lessee of existing defects which he knows the lessee is not likely to discover on reasonable inspect. Convenant to repair; liability for unreasonably dangerous conditions. If lessor repairs, subject to liability if he does so negliently; failure to cure defect. Admission to public: liabilty for unreasonably dangerous conditions at time he transfers interest; mere warning not sufficient.
Alternative Causes Negligence
Where two or more persons HAVE BEEN NEGLIGENT but uncertainty as to which person caused plaintiff's injuries, the plaintiff must prove harm caused by at least one.
When must a motion to amend a judgment be filed?
Within 28 days after entry of judgment; time period to file may not be extended.
7th Amendment guarantees trial by jury where claim is:
at law and amount in controversy exceeds 20 dollars.
A prima facie case in products liability based on strict tort liability consists of the following elements:
(i) the defendant is a commercial supplier; (ii) the defendant produced or sold a defective product; (iii) the product was the actual and proximate cause of the plaintiff's injury; and (iv) the plaintiff suffered damages to person or property.
Bailment Duties
1) Sole benefit of Bailor Bailment: bailee only liable for gross negligence, bailor need only inform of known dangerous defects 2) sole benefit of bailee: bailee liable for slight negligence 3) mutual benefit: ordinary due care 4) Bailments for hire: (bailee loans lawnmower for fee), only duty to advise of known defects OR of which he would have known with exercise of reawsonable diligence.
Libel
defamation in written or other permanent form. Damages presumed.
Prima Facie Case: Intentional Torts
i) Act by Defendant; ii) intent; iii) causation Actor need not intend to cause injury. No age requirement.
The negative implications of the Commerce Clause would not prohibit a regulation: A Prohibiting out-of-state wastes from being accepted in private landfills B Favoring local governments when performing government functions, such as exempting local bonds from state tax while subjecting bonds of other states to a tax C Requiring operations to occur in the state, such as requiring all milk sold within the state to be pasteurized in the state D Protecting local businesses, such as by placing a surcharge on out-of-state products
B.
Joinder: Can absentee be joined?
If should be joined, next question is whether he can be; personal jurisdiction and still have subject matter jurisdiction. If court has PJ and joining will not destroy diversity or venue, he MUST be joined.
Negligence: Duty of Care
Legal duty to act as an ordinary prudent reasonable person. Owed to all foreseeable plaintiffs. Specific Situations: Rescuers are foreseeable, danger invites rescue Prenatal injuries: fetus must have been viable Physical characteristics same as defendants but mental ability is general.
Duty to Licensees
Licensee is one who enters land with landowners permission (express or implied) for her own purpose or business. Landowner owes duty to warn of or make safe dangerous condition known to owner that creates unreasonable risk of harm AND that licensee unlikely to discover. No duty to inspect. Social guests are licensees.
Temporary Restraining Order (TRO)
Only one party heard, injury will result before preliminary injunction hearing can be held; Requirements: Specific facts showing iimmediate and irreparable injury, efforts to give notice, security to pay for any costs or damages incurred by adverse party if wrongfully enjoined. Will expire within 14 days unless good cause for extension: if extended beyond 28 days, treated as preliminary injunction for purposes of appeal.
Requirements for defendant to raise the defense of defense of others?
defender must REASONABLY believe there is right to receive assistance. You can still avoid liability for protecting the initial aggressor, and it doesn't have anything to do with the right of the person being helped
When a defendant attempts to remove a case from state court to federal court, the state court _______ have had subject matter jurisdiction over the case. A Need not B Must C Must not
A. Need not.
Which privacy breach requires plaintiff to show publicity?
Publications of facts placing plaintiff in false light and public disclosure of private facts about plaintiff.
What are grounds for court order for an involuntary dismissal against the plaintiff?
Failure to prosecute case, to comply with federal rules or to comply with court order
Notice to class members of dismissal or settlement of a class action is required unless:
the judgment will not bind the class.
Subject Matter Jursidiction: Diversity Among Parties
1) complete diversity, no plaintiff may be citizen of same state as any defendant. When the action is commenced.
If a statute providing for a criminal penalty is applicable to a common law negligence case, the statute's specific duty will replace the more general common law duty of care. What does a plaintiff need to show to replace the CL Duty of Care?
1) standards in statute are clearly defined; 2) statute designed to prevent type of harm suffered; 3) plaintiff in class intended to be protected 4) ACTUAL damages at some point; does not need to be physical injury, can be property.
In shareholder derivatives suit, minority shareholder needs to include in her complaint:
1) action is not a collusive attempt to confer jurisdiction 2) shareholder at time of transaction 3) made demands of directors or why she did not. Does not need to include the number of shareholders in class.
Use of Force in Terms of Property (real or chattels)
A landowner may not use force to regain real property after being tortiously dispossessed. However, owner may use force to repossess chattels so long as other person's possession did not begin lawfully and there was a "demand for return."
What best states who may bring a strict liability action against a defendant engaged in abnormally dangerous activities?
Any Foreseeable plaintiff injured as a result of the dangerous propensity of the activity.
What cases may the supreme court hear?
Appeals from state courts and some directly from federal district courts
Duty to Trespassers
Undiscovered trespassers: no duty Discovered trespassers: known, manmade death traps Anticipated trespassers: known man made death traps
A landowner contracted in writing to sell a lot to a brother and a sister, as joint tenants, for $60,000. The brother and the sister put up $6,000 as earnest money. Before the closing date, the landowner died. Shortly thereafter, and also before the closing date, a garage on the lot burned down. The garage had a fair market value of $6,500 and was a complete loss. After the fire, the brother demanded that the executor of the landowner's estate return the $6,000, because the brother and sister were no longer interested in the property. The executor refused and told the brother that he expected the brother and sister to tender the $54,000 due on the property when the closing date arrived. The brother and sister did not do so. The brother filed suit demanding a refund of the $6,000. The executor countersued, demanding specific performance by the brother and sister or, in the alternative, monetary damages. Absent any applicable statutes, how should the court rule? (A) In favor of the executor, by requiring specific performance of the brother and sister. (B) In favor of the executor, by assessing damages against the brother and sister. (C) In favor of the brother, by ordering the executor to refund the earnest money. (D) The court should rule that the executor is not entitled to either damages or specific performance and that the brother is not entitled to a refund of the earnest money.
(A) The court should require specific performance of the brother and sister. When a transfer of land is preceded by a contract for sale, the risk of loss to the property during that time interval is imposed on the buyer in most jurisdictions. Thus, despite a loss due to fire or other casualty (assuming it was not due to the fault of either party), the buyer must still pay the contract price at the closing date unless the contract provides otherwise. Consequently, the brother and sister were still under a duty to tender the amount due on the property. Their failure to do so puts them in breach of contract, allowing the executor to obtain specific performance (payment of the balance due on the contract). (B) is incorrect because the preferred remedy for the executor is specific performance. While the executor could obtain a damage remedy based on the difference between the contract price and the market value of the land on the date of the breach, that would leave him with the task as executor of trying to resell the property. Because land is considered unique, a buyer is entitled to specific performance of the contract. On mutuality of remedy grounds, the seller (the executor) can also get that remedy. (C) is incorrect because, as discussed above, the destruction of the garage does not allow the brother and sister to avoid performing the contract. They bore the risk of loss and could not obtain a refund of the earnest money even if the executor had not sought to enforce the contract. (D) is incorrect because the contract can be specifically enforced. The reason that the buyer bears the risk of loss is that the doctrine of equitable conversion treats the buyer as the equitable owner of the land once the contract is signed. Hence, the landowner's death would not prevent the brother and sister from specifically enforcing the contract by requiring the executor to transfer legal title. Mutuality of remedy requires that this same remedy be available to the landowner's executor, who is regarded as being owed a debt of the balance due on the contract. The executor can therefore require payment of the balance as specific performance of the contract.
Class Actions
1) Prerequisities: a) class so numerous that joinder of all members impractical; b) questions of law or fact common to class; c) named parties interests are typical of class; d) named representatives will ensure fair and adequate representation of absent members of class; e) action meets the definition of ANY of the following three types of class actions: 1) separate actions by class members would create risk of inconsistent results or as practical matter, would impair interests of other absent members of class or 2) defendant has acted or refused to act on grounds applicable to class and injunctive or declaratory relief appropriate (no money here) 3) questions of fact or law common to members that predominate over individual issues
The victim owned a cottage in an ocean resort area. He stayed there only during the summer months, and left the cottage unoccupied during the balance of the year. The defendant, a resident of a neighboring cottage, was aware of this practice. For a change in his routine, however, the victim decided to spend a week at the cabin in the off-season. Unaware that the victim was occupying the cottage, the defendant decided to borrow a portable television set that he knew the victim kept in the cottage. To avoid being seen, he entered the cottage late at night, using a key under the front doormat. He found the television set, disconnected it, and headed for the rear of the house to leave. He opened the kitchen door and found the victim seated there in the dark, having a late night snack. Both men were startled and neither man recognized the other in the dark. The defendant assumed that the victim was a burglar, and was afraid that he might be armed. Trying to flee the kitchen as quickly as possible, the defendant dropped the television set in the middle of the kitchen floor. As the set hit the floor, the picture tube exploded with a loud noise. The noise so frightened the victim that he had an immediate heart attack and died. If the defendant is charged with felony murder as the result of the victim's death, what is his best defense? A He did not intend to kill the victim. B His only intent was to borrow the television set for a few days. C Larceny is not an inherently dangerous crime, and it was not being committed in an inherently dangerous manner. D The victim's heart attack was an unforeseeable consequence of the defendant's acts.
B. The defendant's best defense to felony murder is that he only intended to borrow the television set for a few days. By establishing this intent, the defendant will show that he did not have the intent to commit a felony and therefore cannot be guilty of felony murder. A killing (even if accidental) committed during the course of a felony is murder. Malice is implied from the intent to commit the underlying felony. To convict a defendant of felony murder, the prosecution must prove that he committed or attempted to commit the underlying felony. Here, the possible felonies being committed by the defendant, during which the victim's death occurred, would be larceny and burglary. Burglary requires the intent to commit a felony within the dwelling, and larceny requires the intent to permanently deprive a person of his interest in property. If the defendant's only intent was to borrow the victim's television set for a few days, then the defendant lacked the intent to permanently deprive the victim of his interest in the set; i.e., the requisite intent for larceny is missing. Likewise, the absence of intent to steal the set would mean that, at the time of breaking and entering the cottage, the defendant did not intend to commit a felony therein. Consequently, the defendant is not guilty of burglary. Because under these circumstances no felony would have been attempted or committed, it cannot be shown that the death of the victim occurred during the commission of a felony. Therefore, the defendant would not be guilty of felony murder.
A landowner conveyed his land to his wife, son, and daughter "as joint tenants with right of survivorship." The daughter then conveyed her interest to a friend. The wife subsequently executed a will devising her interest to the daughter. Then the son mortgaged his interest to a lender, who promptly and properly recorded the mortgage. The wife died, then the daughter's friend died, leaving a will that bequeathed her entire estate to the daughter. The daughter and the son survived. If the jurisdiction follows the title theory, who owns what interest in the land? A The lender owns the fee simple. B The lender and the daughter own unequal shares as tenants in common. C The son and the daughter own unequal shares as tenants in common. D The son and the daughter own equal shares as joint tenants.
B. The lender and the daughter own unequal shares as tenants in common. Creation of a joint tenancy requires four unities: (i) time (interests must vest at the same time); (ii) title (interests must be acquired by the same instrument); (iii) interest (interests must be of the same type and duration); and (iv) possession (interests must give identical rights to enjoyment). When property is held in joint tenancy by three or more joint tenants, a conveyance by one of them destroys the joint tenancy only as to the conveyor's interest. The other joint tenants continue to hold in joint tenancy as between themselves, while the grantee holds her interest as a tenant in common with them, because she does not share the unities of time or title with the joint tenants (i.e., her interest vested at a different time and was acquired by a different instrument). Here, the wife, the son, and the daughter owned the property as joint tenants. When the daughter conveyed her interest to the friend, the joint tenancy was severed as to the daughter's interest. At that point, the wife and the son each held one-third interests as joint tenants because, as between themselves, the four unities were preserved. The friend did not share the unities of time or title with the wife and the son. Thus, the friend took a one-third interest as a tenant in common rather than as a joint tenant. Because this jurisdiction follows the title theory, the son's mortgage also severed the joint tenancy. The minority of states following the title theory regard a mortgage as an actual transfer of title to the property. Thus, a mortgage by one joint tenant transfers the legal title of the joint tenant to the mortgagee (the lender). This action destroys the unity of title and severs the joint tenancy. On the other hand, in a lien theory state (majority), a mortgage is considered a lien on title--one joint tenant's execution of a mortgage on his interest does not, by itself, sever a joint tenancy until default and foreclosure proceedings have been completed. Here, when the son executed the mortgage, title was transferred to the lender, severing the joint tenancy again. At that point, the wife, the lender, and the friend each held one-third interests as tenants in common because each interest was acquired at a different time and by a different title. An interest in a tenancy in common is freely alienable by inter vivos and testamentary transfer, is inheritable, and is subject to claims of the tenant's creditors. The only "unity" involved is possession: Each tenant is entitled to possession of the whole estate. Thus, when the wife died, her interest passed under her will to the daughter. Likewise, when the friend died, her interest passed to the daughter. Thus, the lender holds a one-third interest as a tenant in common with the daughter, who holds the remaining two-thirds interest. (A) is incorrect because the daughter also has an interest in the property (see above). When one joint tenant dies, the right of survivorship operates to free the property of her interest; the surviving joint tenants retain an undivided right in the property. Although a will is inoperative as to joint tenancy property because the decedent's rights in the property evaporate at the instant of death, a will may effectively dispose of an interest held in a tenancy in common, as discussed above. Here, although the wife executed her will while the property was still held in joint tenancy, a will is ambulatory (effective only at death), and at her death the wife held her interest as a tenant in common. Thus, it passed to the daughter under the will. (C) and (D) are incorrect because, as explained above, the son has no interest.
While working on a construction project, a plaintiff was injured when a heavy object struck his knee. Although the plaintiff was fully compensated for his injuries at the time of the incident, he now seeks disability payments from the construction company because he has developed arthritis in the same knee. The construction company claims that the arthritis has nothing to do with the plaintiff's on-the-job injury and refuses to pay him disability money. The plaintiff sues. A doctor takes the stand to testify for the plaintiff. He is qualified as an expert witness and during direct examination states that in his opinion the blow to the plaintiff's knee caused his arthritis. On cross-examination, the construction company's attorney produces a treatise on arthritis and asks the doctor if the treatise is considered to be authoritative. The doctor responds that the treatise is a standard authority in the field, but that he did not rely on it in forming his professional opinion regarding the plaintiff's condition. The attorney then seeks to introduce into evidence a statement in the treatise that "the idea that arthritis can be caused by a single traumatic event is purely folklore, although it is widely believed by the ignorant who have no scientific basis for their beliefs." The plaintiff's attorney objects. How should the court rule on the admissibility of the statement from the treatise? A Admissible, but only for the purpose of impeaching the doctor's testimony. B Admissible, but only as substantive evidence. C Admissible, both as substantive evidence and for purposes of impeaching the doctor. D Inadmissible.
C. The statement from the treatise is admissible to impeach and as substantive evidence. Under the Federal Rules, learned treatises can be used either for impeachment or as substantive evidence. One way the credibility of an expert witness may be attacked is by cross-examining him as to his general knowledge of the field in which he is claiming to be an expert. This can be done by cross-examining the expert on statements contained in any scientific publication that is established as reliable authority. Reliability of a publication may be established by: (i) the direct testimony or cross-examination admission of the expert, (ii) the testimony of another expert, or (iii) judicial notice. The Federal Rules recognize an exception to the hearsay rule for learned treatises and admit them as substantive evidence if: (i) the expert is on the stand and it is called to his attention, and (ii) it is established as reliable authority (see above). The doctor has admitted on cross-examination that the treatise is authoritative in the field. Thus, the attorney may use the statement in the treatise to attack the doctor's general knowledge of the field of arthritis by showing that the doctor's opinion that the blow to the plaintiff's knee caused his arthritis is considered to be ignorant and unfounded in the text of the treatise. As noted above, such an attack on the doctor's general knowledge of the field is a proper means of impeaching his credibility. In addition, pursuant to the Federal Rules, the statement may be read into the record as substantive evidence (i.e., as a means of proving that the plaintiff's arthritis could not have been caused by a single traumatic event, such as the blow to his knee). The statement may be used as substantive evidence because it has been brought to the attention of the doctor during cross-examination and he established it as a reliable authority, and it will be read into evidence while he is on the stand. (A), which reflects the traditional view, is incorrect because the Federal Rules permit the use of the statement in the treatise as substantive evidence. (B) is incorrect because it precludes use of the statement for impeachment purposes. (D) is incorrect because it would not allow introduction of the statement for either impeachment or substantive evidentiary purposes, and thus it is an incorrect statement of the law.
While driving in a city in State A, a citizen of State B struck a pedestrian who was a citizen of State C. The pedestrian sued both the driver and the State A city in federal district court, seeking $100,000. The pedestrian alleged that the driver was liable for negligently operating his car and that the State A city was liable for negligently maintaining a traffic signal. The driver, who owns an office supply wholesaler, also has a contract claim against the State A city for $80,000 worth of furniture that the city allegedly purchased and received but never paid for. Assume that State A has waived any applicable governmental immunity. If the driver files a cross-claim for negligence against the State A city to recover damages for his injuries in the accident with the pedestrian, can the driver join with that cross-claim his contract claim for the purchase price of the furniture and maintain the claim in the same federal action? A No, because defendants cannot add claims against co-defendants that are not related to the original claims asserted by the plaintiff. B No, because the court does not have supplemental jurisdiction over the driver's contract claim against the State A city. C Yes, the driver must assert the contract claim or he will be foreclosed from asserting it later. D Yes, the driver may join his contract claim with his negligence cross-claim against the State A city, but is not required to do so.
D The driver can join his contract claim with his negligence cross-claim, but he is not required to do so. As a general rule, a party may assert a cross-claim against a co-party only if the cross-claim arises from the same transaction or occurrence as that of the original action or of a counterclaim. However, once the party has filed such a cross-claim, he also may join with it any other claim that he has against the same party. Here, the driver's contract claim is unrelated to the pedestrian's negligence claim; however, the driver's negligence claim is related to the pedestrian's negligence claim. Thus, the contract claim also can be asserted in this case. (A) is therefore incorrect. (B) is incorrect because the court would have diversity jurisdiction over the contract case. (C) is incorrect because the contract claim is not a compulsory counterclaim.
The defendant was arrested, given Miranda warnings, and charged with burglary. At the police station, he telephoned his mother and asked her to come to the station to post bail. Instead, his mother immediately called the family attorney. In the meantime, the police had begun questioning the defendant. Although he never told the police to stop the questioning, his answers were at first vague or clearly unresponsive. During the course of the questioning, the family attorney phoned the station and told the police that she had been hired to represent the defendant and would be there in half an hour. The police did not inform the defendant of the attorney's call. Ten minutes later, the defendant admitted to committing the burglary, and signed a statement to that effect prepared by the police. The attorney arrived a few minutes later and advised the defendant to remain silent, but he told her that he had already signed a confession. How should the court rule on the attorney's pretrial motion to exclude the confession as evidence at trial? A Grant the motion, because the police had a duty to inform the defendant that an attorney was coming to represent him. B Grant the motion, because the defendant has been deprived of his Sixth Amendment right to counsel. C Deny the motion, because the defendant's statement admitting the crime was voluntary. D Deny the motion, because the defendant waived his Miranda rights.
D. The defendant's confession should be admitted because he waived his Fifth Amendment privilege against compelled self-incrimination after receiving Miranda warnings. Miranda v. Arizona requires that a person in custody be informed of his right to remain silent and his right to the presence of an attorney during questioning. A suspect may subsequently waive his rights by making a confession, as long as the waiver was knowing and voluntary. In this case, the defendant received proper Miranda warnings, and there is no indication that he did not understand what his rights were. Although his answers during questioning were initially unresponsive, he never asked for an attorney or indicated that he wished to remain silent, and he voluntarily confessed after a relatively short period of interrogation. Hence, he validly waived his Miranda rights. (A) is incorrect because the police have no duty to inform the defendant that an attorney is attempting to see him. The defendant's ignorance of his attorney's efforts has no bearing on whether he made a knowing waiver of his Miranda rights. (B) is incorrect because the defendant's right to counsel was not violated. Although the defendant does have a separate Sixth Amendment right to counsel under Escobedo v. Illinois because he has already been arrested and charged with the crime, this right would only be violated if the defendant, after being informed of his right to counsel, had requested an attorney or had been prevented from seeing his attorney. Here, he made no request to see an attorney - even when he called his mother - and his attorney was allowed to see him immediately upon her arrival. Thus, he has waived his Sixth Amendment right to counsel. (C) is incorrect even though it is true that the defendant made a voluntary statement. Due process requires that for confessions to be admissible, they must be "voluntary," based on the totality of the circumstances, and here all of the circumstances indicate that the defendant's confession was voluntary. However, even a voluntary confession will be inadmissible if it was obtained in violation of Miranda rights. (D) is therefore a better choice than (C).
A state statute makes it a felony for anyone in the corridors or on the grounds of any building in which a court may be in session to make a speech or carry a sign intended to improperly influence judicial proceedings. When the head of a street gang was on trial for murder, a gang member was arrested for carrying a sign on the steps of the courthouse warning that if the gang leader was not freed, "the judge will die." May the gang member be convicted of violating the state statute? A No, because the statute could apply to others whose speech is constitutionally protected. B No, unless he personally intended to harm the judge. C Yes, if there was a clear and present danger that the judge would be influenced by the sign. D Yes, because the statute does not violate the freedom of expression guaranteed by the First Amendment.
D. The gang member can be convicted because the statute does not violate the First Amendment. Certain public property (e.g., public streets or parks) is so historically associated with the exercise of First Amendment rights that speech thereon can be regulated only by content-neutral proscriptions. Other places controlled by the government, however, are not so historically linked to speech activities, and in such locations free speech might interfere with the intended use of such locations. Thus, the government can regulate access to these limited public forums and nonpublic forums based on the subject matter of the speech, as long as the regulation is reasonably related to the purpose served by the property and is not designed merely to suppress a particular point of view. A courthouse and its grounds are not a public forum. (The surrounding sidewalks are, but that is not in issue here.) The statute, although based on the subject matter of speech, is viewpoint neutral and reasonably related to the courthouse purpose of promoting a stable, orderly atmosphere in which judicial proceedings can take place, free of improper outside influence or coercion. Thus, the statute is valid and the gang member can be convicted for his actions. (A) is wrong because it is based on an overbreadth argument and the statute here is not overbroad. A regulation of speech that restricts substantially more speech than necessary is unenforceable, even if the speech in question could have been properly restricted by a narrower statute. This doctrine is inapplicable here because the statute is not overbroad: It reaches only speech in the courthouse or on its grounds and only that speech that might improperly influence the judicial proceedings; it does not limit all speech at that location. (B) is wrong because the gang member's personal intent to harm the judge is irrelevant. The statute makes it a crime to make a speech or carry a sign intended to influence the judicial proceeding. The statute does not require that the violator intend to harm anyone. Since the state is entitled to regulate speech or conduct in the courthouse or on its grounds that might interfere with the judicial proceedings, it is entitled to convict the gang member for his actions here regardless of his intent to harm the judge. (C) is wrong because it improperly applies the "clear and present danger" test to these facts. Under the current version of the "clear and present danger" test, a state cannot forbid advocating the use of force or violation of law unless such advocacy is (i) directed to producing or inciting imminent lawless action, and (ii) likely to produce such action. The state statute here does not purport to punish advocacy of force or lawlessness, but rather seeks to further the purpose of maintaining the stability and integrity of the judicial proceedings by regulating access to certain nonpublic areas. Therefore, the restrictions are constitutionally valid and the "clear and present danger" test is inapplicable.
Defeasible fees
Fee Simple Determinable Fee Simple Subject to Condition Subsequent Fee Simple Subject to Executory Limitation
Statutory Standard of Care
Plaintiff within protected class and the particular harm to be avoided. Excuses for violation: where compliance would cause more danger OR beyond defendant's control. Estabishes conclusive presumption of duty and breach.
False Imprisonment
Prima Facie Case 1) act or omission on part of defendant that confines or restrains plaintiff to bounded area; 2) intent on part of defendant to confine or restrain plaintiff to bounded area; 3) causation No amount of time needed. Physical force directed at him or immediate family, ALSO PROPERTY. Citizens arrest: crime committed for sure and plaintiff probably committed.
Must a junior mortgagee be named as a party to a senior mortgagee's foreclosure action? response - incorrect A No, because foreclosure does not affect interests junior to the mortgage being foreclosed B No, because foreclosure extinguishes interests junior to the mortgage being foreclosed C Yes, because it has the right to pay off the senior mortgage to avoid being wiped out by foreclosure D Yes, because all those with liens on the property are necessary parties to a foreclosure action
Yes, a junior mortgagee must be named as a party to a senior mortgagee's foreclosure action because it has the right to pay off the senior mortgage to avoid being wiped out by foreclosure. Foreclosure destroys interests ( e.g., liens, mortgages, leases, easements) junior to the mortgage being foreclosed. Thus, if a senior mortgage is in default, a junior mortgagee has the right to pay it off (i.e., redeem it) to avoid being wiped out by its foreclosure. Failure to join the junior mortgagee results in the preservation of its interest despite foreclosure and sale. In contrast, those with interests senior to that of the foreclosing party are not necessary parties because their interests are not affected by foreclosure. The buyer at the sale takes subject to senior interests, which remain on the land. As explained above, NOT all those with liens on the property are necessary parties to a foreclosure action. Only those with interests subordinate to that of the foreclosing party must be named in the foreclosure action. Failure to name a senior interest holder does not affect that party's interest. Because foreclosure extinguishes interests junior to the mortgage being foreclosed, as explained above, junior mortgagees are necessary parties to a senior mortgagee's foreclosure action. Foreclosure DOES affect interests junior to the mortgage being foreclosed. As explained above, junior mortgages are extinguished by foreclosure so long as they are joined in the action.
When a force comes into motion after the defendant's negligent act and combines with that act to cause injury to the plaintiff, this is known as:
an indirect cause. If an intervening force cuts off defendant's liability, it will be a superseding cause.
With regard to trespass to chattels, intermeddling is defined specifically as conduct that:
directly DAMAGES plaintiff's chattels. Dispossession is stealing.
If the jury finds for one side, determines damages and relief to be given, and answers specific questions of fact, this is called a:
general verdict with special interrogatories.
Invasion of the right to privacy
i) appropriation by defendant of plaintiffs picture or name for defendant's commercial advantage; (unauthorized) ii) intrusion upon plaintiff's affairs or seclusion iii) publication by defendant of facts placing plaintiff in false light; iv) public disclosures of private facts about plaintiff.
For claim preclusion to apply, the following requirements must be met:
i) earlier judgment valid and final on the merits; ii) case brought by the same plaintiff against the same defendant iii) same course of action involved in later lawsuit.
In a party's pleadings, performance of conditions precedent and denial of performance must be alleged:
performance of conditions precedent may be alleged generally but denial of performance or occurrence must be alleged specifically and with particularity.
Nuisance
private nuisance is a substantial, unreasonable interference with another's use or enjoyment of property. Must be substantial; offensive, inconvenient, or annoying to the average person in the community. The injury must outweigh utility.
Slander
spoken defamation. Special damages usually required, damages not presumed UNLESS slander per se: i. business or profession ii. loathsome disease iii. crime involving moral turpitude unchastity of woman.
A principal will be vicariously liable for tortious acts of an independent contractor if:
the independent contractor is engaged in inherently dangerous activities or duty because of public policy considerations (can't simply delegate certain duties)
Once a final judgment on the merits has been rendered on a particular cause of action, the claimant is prevented from asserting the same cause of action in a later lawsuit by the doctrine of claim preclusion (res judicata). When the claimant won the earlier lawsuit, the claim is said to be: response - incorrect A Barred by the prior judgment B Either merged into or barred by the prior judgment C Merged into the prior judgment
C Although both merger and bar are used to indicate that claim preclusion (res judicata) is in effect, where the claimant won the earlier lawsuit, the claim is said to be merged into the prior judgment. Where the defendant won the earlier lawsuit, the claim is said to be barred by the prior judgment. Because the question specifically states that the claimant won the earlier
What judgments must state and federal courts recognize?
Generally must recognize judgments of sister states and federal courts generally must recognize judgments of state courts.
Attractive Nuisance Doctrine
Ordinary care to avoid reasonably foreseeable risk of harm to children caused by artificial conditions on property: 1) dangerous condition on land which owner is or SHOULD BE aware; 2) owner knows or should know young persons frequent vicinity of this dangerous condition; 3) condition likely to cause injury or dangerous because of child's inability tp appreciate risk 4) expense of remedying slight compared with magnitude of risk.
What kind of motions may a federal court extend time period for filing and which ones may it not?
May extend for motion of summary judgment, may not for motion for new trial, motion to amend judgment, or renewed motion for judgment as a matter of law; all must be made within 28 days after entry of judgment.
Generally, amendments to pleadings relate back to the date the original pleading was filed if:
the amendment concerns the same T/O that was set forth in the original pleading.
Products Liability: Proving a Defect
for manufacturing defects, plaintiff will prevail of product dangerous beyond expectation of ordinary consumer. For design defects, plaintiff must show reasonable alternative design, that there is a less dangerous modification or that the alternative was economically feasible. Compliance with government safety standard will not preempt product liability on defective warnings.
General Venue Rules
i. Judicial district in which any defendant resides, if all defendants in same state in which district is located; ii. district in which substantial part of events or omissions giving rise to claim occurred, or substantial part of property subject to the action is situated; or iii. if no distrct which satisifes i or ii, any district in which ANY defendant subjectt o PJ.
If a landlord's breach of duty renders the premises unsuitable for occupancy, the tenant may vacate the premises, terminate the lease, and sue for damages. Under the doctrine of constructive eviction, if the landlord's breach (i.e., doing an act or failing to provide some service that he has a legal duty to provide) makes the premises untenantable, the tenant may terminate the lease and also may seek damages if the following conditions are met:
(i) The breach must be by the landlord or by persons acting for him. (ii) The breach must substantially and materially deprive the tenant of her use and enjoyment of the premises (e.g., flooding, absence of heat in winter). (iii) The tenant must give the landlord notice and a reasonable time to repair. (iv) The tenant must vacate the premises within a reasonable time. Because a tenant cannot claim a constructive eviction unless and until she vacates the premises, her remedies do not include remaining in possession of the premises and refusing to pay rent until the interference ceases or continuing to pay rent and suing for damages. The tenant is not limited to suing for breach only if the lease contained an express covenant for quiet enjoyment. Every lease contains an implied covenant that neither the landlord nor someone with paramount title will interfere with the tenant's quiet enjoyment and possession of the premises. If a landlord does so, the tenant has the remedies discussed above.
An advertising agency specializing in aerial banners and skywriting signed a contract with a film production company that was premiering a new blockbuster film. The contract provided that the agency would advertise the film by flying over the city towing a giant streamer belonging to the film company heralding the film's catch phrase and title in large letters. This contract specified that the flight was to be conducted on the first Saturday in June at noon (the day of the local premier), and the film company was to pay the advertising agency $500 for the flight. On the designated Saturday, the advertising agency was unable to fly because of a defective fuel pump. The defective condition was entirely unforeseeable and did not occur through any negligence or fault of the agency. The film company did not pay the agency, and each of the parties has sued the other for damages. Which of the following best states the rights and liabilities of the parties? A The film company is entitled to recover damages from the advertising agency on account of the agency's failure to fly. B The advertising agency is entitled to recover from the film company the $500 contract price, as the incapacity of the airplane was not the agency's fault. C Neither party is entitled to recover against the other, because the advertising agency's duty to fly was discharged by impossibility, and the film company's duty to pay was contingent on the agency's flight. D Neither party is entitled to recover against the other, because the film company's offer to pay $500 for the flight was in effect an offer for an act, and because the act was not performed, there was no valid acceptance.
A. The film company will be able to recover damages from the advertising agency because the agency's failure to fly constituted a breach of contract. The parties entered into a bilateral contract—the agency promised to fly with the streamer and the film company promised to pay for the flight. The agency breached the contract by failing to fly on the designated Saturday. Its duty to fly was not discharged by impossibility. A contractual duty to perform may be discharged by objective impossibility (i.e., no one could have performed), but subjective impossibility (defendant could not perform) is insufficient. Here, the defect in the plane constituted only subjective impossibility (if it amounted to impossibility at all) because the agency could have obtained another plane to pull the streamer. If the agency had been unable to fly the plane because of weather (e.g., a severe ice storm), its performance would have been objectively impossible, and the agency would have been discharged. However, under these facts, the film company is entitled to damages for the agency's breach. (B) is incorrect because the film company's duty to perform (pay $500) was subject to the condition precedent of the agency's performance (flying), and, as discussed above, the agency breached the contract by failing to fly. Therefore, the film company's duty to pay never arose. The fact that the engine problem was not the agency's fault does not change things. The agency's inability to perform, even if it were due to impossibility, would merely discharge the contract, and each party would be excused from performance; the film company would not have to pay the $500. (C) is incorrect because, as determined above, the agency's duty was not discharged because performance was still possible. (If there had been objective impossibility, (C) would have been the correct choice.) (D) is incorrect because it suggests that the contract was a unilateral one (the offer to pay could be accepted only by completion of performance). This interpretation is clearly contrary to the facts. Although the film company offered to pay $500 for the flight, the agency accepted that offer by signing the contract. A promise to pay was given in exchange for a promise to fly. Thus, there was a contract to which both parties were bound.
After the failure of a state bill granting gay and lesbian couples the right to marry, 30 students from a local university marched on the state capitol to protest, carrying signs with slogans such as "let gays marry" and "no religious tyranny." As they marched, about 15 people began following them, shouting anti-gay remarks. At the capitol building, they were met by 50 officers in full riot gear. The leader of the students addressed his followers in the park across the street, vigorously denouncing the legislature's actions, which caused the counter-protestors to become more vocal. The officer in charge told the leader that he must end his speech because a riot was about to start. The leader refused and was arrested and convicted of disorderly conduct. If the leader appeals his conviction on constitutional grounds, will the conviction be reversed? A Yes, because the leader's arrest constituted an interference with his First Amendment right to free speech. B Yes, because the leader's arrest constituted undue interference with the students' right to peaceably assemble under the First Amendment. C No, because the leader's speech caused an immediate and substantial threat to public order. D No, because the leader had a duty to obey the police officer since other avenues of communication likely were available.
A. The speaker's conviction will be reversed. A park is a public forum. The government can limit rights of speech in such a forum only when there is a serious and imminent threat to the public order. It can restrict the speech of a speaker because of an unruly audience only in the rare case when the police are absolutely unable to control the crowd. [See Feiner v. New York (1951)] In this question, the conditions under which the police can prevent a speaker from continuing because of an unruly crowd have not been met. There were 50 police officers who would have been able to restrain or subdue anyone who appeared to be intent on committing violence. Hence, (C) is incorrect. (B) is incorrect. No one in the audience has raised any constitutional argument. The speaker probably cannot raise the audience members' First Amendment rights in this situation. (D) is incorrect. The state may not limit access to a public forum on the sole basis that there are other times and places where the right of free speech can be exercised. The state must show a more substantial reason.
Which of the following is true under the preemption doctrine? A Federal laws can occupy an entire field and prevent any state laws in the field, whether conflicting or complementary B A state law will not be held invalid unless it interferes with achievement of a federal objective C A state law that does not interfere with achievement of a federal objective will be upheld unless a federal law expressly prohibits state legislation D A preemption clause in a federal law will be construed broadly by the federal courts
A. Under the Supremacy Clause, it is true that federal laws can occupy an entire field and prevent any state laws in the field, whether conflicting or complementary. The statement that a state law will not be held invalid unless it interferes with achievement of a federal objective is not true; it is too narrow. As discussed above, a state or local law may fail under the Supremacy Clause, even if it does not conflict with federally regulated conduct or objectives, if it appears that Congress intended to "occupy" the entire field, thus precluding any state or local regulation. The statement that a state law that does not interfere with achievement of a federal objective will be upheld unless a federal law expressly prohibits state legislation is not correct. Preemption may be express or implied. The statement that a preemption clause in a federal law will be construed broadly by the federal courts is not correct. The opposite is true. An express preemption clause will be construed narrowly.
A plaintiff filed a civil action based on negligence against a defendant in federal district court, alleging that the defendant negligently ran a red light at an intersection and collided with the plaintiff's vehicle, causing the plaintiff's injuries. A week after the close of discovery, the plaintiff filed a motion for summary judgment on the issue of whether the defendant was negligent. With the motion, the plaintiff filed (i) his own sworn affidavit, which stated that the traffic signal was green as he entered the intersection; (ii) an affidavit of a witness who was driving the car behind him, which stated that the witness saw the entire incident and that the plaintiff's traffic signal was green as he approached and entered the intersection; and (iii) an affidavit of another witness, which stated that she saw the entire incident and that the defendant's signal had been red for several seconds before the defendant entered the intersection and was still red when the defendant entered the intersection. In response to the motion, the defendant filed his own affidavit which stated that he does not recall seeing the traffic signal before entering the intersection, but believes that it was not red. He also filed the affidavit of a pedestrian on the scene. The pedestrian stated that she did not see the traffic signal prior to the accident, but that another passerby told her that the traffic signal was yellow for both drivers. How should the court rule on the plaintiff's motion? A Grant the motion, because the plaintiff has supported his motion with substantial evidence and the defendant has failed to produce admissible evidence that contradicts the plaintiff's evidence. B Grant the motion, because the defendant's conduct is still negligent even if the defendant's traffic signal was yellow. C Deny the motion, because the defendant has presented evidence that contradicts the evidence presented by the plaintiff. D Deny the motion, because a party may not obtain summary judgment on an issue on which that party has the burden of proof.
A. The court should grant the motion. Summary judgment may be granted if, from the pleadings, affidavits, and discovery materials, it appears that there is no genuine dispute of material fact and the moving party is entitled to judgment as a matter of law. The court may not decide disputed fact issues on a motion for summary judgment; if there is a genuinely disputed material fact (meaning a dispute backed by evidence on both sides of the issue), the case must go to trial. Here, the plaintiff's evidence that the light was green when he entered the intersection has not been contradicted by admissible evidence. The defendant's affidavit essentially does not dispute the color of the light, as the defendant cannot remember what color it was. The pedestrian's testimony constitutes inadmissible hearsay. Thus, there is no material fact in dispute, and the motion should be granted. (Alternatively, the court could delay in ruling on the motion to give the defendant an opportunity to find the unavailable witness who spoke with the pedestrian. However, that is not an answer choice, and it is unclear from the facts whether that witness could even be found.) (B) is not correct, as it would become a matter for the jury to decide if running a yellow light is negligent. (C) is incorrect because the defendant has not produced admissible evidence that contradicts the plaintiff's position. (D) is an incorrect statement of the law. Summary judgment may be granted on an issue as to which the party has the burden of proof.
In a property settlement after a divorce, the wife was awarded all personal property that had been accumulated during the marriage, including the husband's classic 19-inch black-and-white TV set. In order to get his prized TV set back, the husband lied to his friend, telling him that the wife took the TV set in violation of the property settlement. The friend remembered that the wife gave the friend's wife a key to her new home, and he volunteered to go with the husband to get the TV back while the wife was at work. The husband and the friend went to the wife's house, but, unbeknownst to them, the wife had taken the day off work. After the friend noisily opened the back door with his wife's key, the wife called the police, who quickly arrived and arrested the husband and the friend. As to a charge of common law conspiracy to commit larceny, how should the friend be found? A Not guilty, because he did not intend to steal. B Not guilty, because he did not have a corrupt motive. C Guilty, because there was an agreement, and the opening of the locked door was sufficient for the overt act. D Guilty, because good motives are not a defense to criminal liability.
A. The friend should be found not guilty because he did not intend to steal. At common law, conspiracy consists of (i) an agreement between two or more persons, (ii) an intent to enter into an agreement, and (iii) an intent to achieve the objective of the agreement. The object of the agreement must be something unlawful. Here, the friend did not intend to achieve the objective of the conspiracy—to permanently deprive the owner of her property—because the friend thought the husband was the true owner of the TV. (C) is incorrect because there must be an agreement to reach an unlawful objective. Because the friend thought he was achieving a lawful objective, he did not have the intent required for conspiracy. (B) is incorrect because a "corrupt motive" is not an element of a crime. A person could be found guilty of a crime even if he did not have a corrupt motive, assuming all required elements for a crime are present. (D) is incorrect for a similar reason. "Good motive" is largely irrelevant; the intent, or lack thereof, is what is important.
The defendant was on trial for murder. The defendant called a witness to testify to an alibi. On cross-examination of the witness, the prosecutor asked, "Weren't you on the jury that acquitted the defendant of another criminal charge?" What is the best reason for sustaining an objection to this question? A The question goes beyond the scope of direct examination. B The probative value of the answer would be substantially outweighed by its tendency to mislead. C The question is a leading question. D Prior jury service in a case involving a party renders the witness incompetent.
B This question raises several different issues: competency of witnesses, use of leading questions on cross-examination, the proper scope of cross-examination, and the probative value/prejudicial impact balancing test. Through a process of elimination, (B) emerges as the correct answer. (D) is incorrect. Under the Federal Rules, virtually all witnesses with personal knowledge are competent to testify. [Fed. R. Evid. 601] A witness is not rendered incompetent simply by having served on a jury in a prior case involving a party to the current suit. Such prior jury service might render the witness's testimony unpersuasive, but it would not make it inadmissible. (C) is incorrect because ordinarily, leading questions are permitted on cross-examination. [Fed. R. Evid. 611(c)] The prosecutor's question is a leading question, but that is perfectly permissible, especially in a case like this, where the alibi witness is not "friendly" toward the prosecution. (A) is incorrect because cross-examination is generally limited in scope to the subject matter of the direct examination and matters affecting the credibility of the witness [Fed. R. Evid. 611(b)], and the prosecutor's question is, in a roundabout way, an attempt to impeach the witness's credibility. The implication behind the question is that if the witness had served on a jury that acquitted the defendant of another criminal charge, the witness would be inclined to think the defendant innocent of the pending charge. Alternatively, the implication behind the question could be that the witness is the kind of person who is "soft on crime" and for that reason is not a credible witness. In either event, because the question is an attempt to impeach the witness's testimony, it is within the proper scope of cross-examination. This leaves (B) as the remaining correct answer.
Comparative negligence
"Partial" comparative negligence: plaintiff barred if his negligence more serious than that of the defendant. If multiple defendants, combined comparison (total neg. of all the defendants) "Pure comparative negligence: recovery no matter how great plaintiffs negligence (if P 90% at fault, and D 10%, P can still recover on the 10%). Pure comparative negligence is the applicable rule unless stated otherwise.
Question 18 At a trial in which a pedestrian is suing a driver, a hospital record was admitted into evidence that included the following statement: "The pedestrian's leg was run over by a car driven by a driver who blew through a red light while the pedestrian was crossing in a crosswalk." The driver's attorney now wishes to admit the other portion of the hospital record, which says, "The pedestrian stepped off the curb without first looking both ways for traffic." How should the court rule? (A) Admit the statement on fairness grounds because the plaintiff has the other portion of the record. (B) Admit the statement as a past recollection recorded. (C) Exclude it because it is hearsay not within any exception. (D) Exclude it because it is self-serving.
(A) The court should admit the statement. Federal Rule 106 provides that, when a statement or part of a statement is introduced, the adverse party may introduce any other statement or part of the statement which ought, in fairness, to be considered at the same time. (B) is incorrect because the statement in the hospital record bears none of the required indicia for a past recollection recorded. There is no indication that the pedestrian's memory has failed or that the report was made under his direction to accurately reflect his memory of the incident. Moreover, the statement is admissible under Federal Rule 106 because it ought, in fairness, be considered to counterbalance the evidence already improperly admitted, even if it is hearsay. (C) is incorrect because Federal Rule 106 applies even if both statements are inadmissible hearsay. (D) is incorrect. The fact that evidence is self-serving is often just another way of saying that it is hearsay. However, the rule stated in Rule 106 applies even when the remaining part of the document contains inadmissible hearsay
Question 4 A landowner properly executed a warranty deed conveying her land to an animal shelter "so long as the premises are used for animal shelter purposes." The animal shelter promptly and properly recorded the deed. A few years later, the landowner died intestate, with her husband as her only heir. The following year, the husband conveyed by means of a quitclaim deed "all of my interest" in the animal shelter land to his daughter from a previous marriage. The daughter promptly and properly recorded the deed. Last month, the animal shelter closed due to lack of funding. A domestic violence shelter wishes to use the land and petitions the court to allow it to take the animal shelter's place. If there is no applicable statute, the court should hold that title to the land is in which party? (A) The daughter, because the land is no longer used for animal shelter purposes. (B) The husband, because the interest he holds in the land is not transferable inter vivos. (C) The animal shelter, because no party has taken action to terminate its interest in the land. (D) The domestic violence shelter, because the court will likely apply cy pres.
(A) The daughter has title to the land because it is no longer used for animal shelter purposes. A fee simple determinable is an estate that automatically terminates on the happening of a stated event and goes back to the grantor. The interest that is left in a grantor who conveys a fee simple determinable is a possibility of reverter, which in almost all jurisdictions is transferable, devisable, and descendible. Here, the animal shelter has a fee simple that is subject to automatic termination if the land is no longer used for animal shelter purposes. The landowner's possibility of reverter descended on her death to her husband, who subsequently transferred it by inter vivos conveyance to his daughter. Thus, when the land ceased to be used for animal shelter purposes, it automatically reverted to the daughter. (B) is incorrect because, as explained above, a possibility of reverter is transferable inter vivos in most jurisdictions. (C) is incorrect because a fee simple determinable automatically terminates on the happening of the stated event. On the other hand, the grantor of a fee simple subject to a condition subsequent (e.g., "to A, provided that the land is used for animal shelter purposes, and in the event that it is not so used, then O or her heirs may enter and terminate the estate") must take affirmative steps to terminate the estate of the grantee if the stated event occurs. (D) is incorrect because the cy pres doctrine allows a court to redirect a trust to a different charitable purpose that is as near as may be to the settlor's original intent, but here a trust is not present.
The police department of a small city has jurisdiction within the city limits and over a defined portion of the surrounding rural communities within the county. A farmer lives in one of the rural communities receiving police protection from the city. The farmer does not pay any tax to the city directly, but a portion of the farmer's county property tax is turned over by the county to the city in order to support the city's police department. The farmer's property was vandalized several times over the past several months, and the farmer became unhappy with the police protection that the city was providing. After his complaints to the police department and city hall did not improve the situation, the farmer wanted to vote against the mayor in the next election, but a city ordinance provides that only residents of the city may vote in city elections. If the farmer brings a suit to compel the city to allow him to vote in the city's mayoral election, is he likely to prevail? A No, because the resident voting limitation appears to be rationally related to a legitimate government interest. B No, but only if the city can prove that the resident voting limitation, which affects a fundamental right, is necessary to a compelling interest. C Yes, because the resident voting limitation violates the Privileges or Immunities Clause of the Fourteenth Amendment. D Yes, because the resident voting limitation constitutes an instance of taxation without representation.
(A) The farmer is not likely to prevail because the rational relationship test applies. Although the right to vote is a fundamental right, laws prohibiting nonresidents from voting are generally valid, provided that they meet the minimal scrutiny, or "rational basis," standard. Under this standard, a law will be upheld as long as it is rationally related to a legitimate government interest. Limiting the voters in a city's mayoral election to residents of the city serves the interests of efficiency and prevents persons with little personal interest in the city from voting. Thus, a court would likely uphold the ordinance. Answer (B) is incorrect because it sets out the strict scrutiny standard. While voting is a fundamental right and governmental action affecting fundamental rights generally is reviewed under strict scrutiny, that is not the case with laws limiting voting to residents. Answer (C) is incorrect. The Privileges or Immunities Clause of the Fourteenth Amendment prohibits a state from denying its citizens certain privileges or immunities of national citizenship. The voting restriction here does not affect any national right, except, perhaps, the right to travel. Even if that right is impacted here, as discussed above, the Supreme Court has approved limiting the right to vote to residents. Answer (D) is incorrect. While "no taxation without representation" was a rallying cry for the war of independence, it is broader than the constitutional doctrines applicable here.
Eighty-five percent of tobacco products sold in the United States originate in the tobacco fields of one state. Its legislature passed a law prohibiting cultivation of tobacco in the state, citing severe health risk concerns. In response, Congress enacted legislation specifically authorizing and encouraging cultivation of tobacco by providing tax incentives and federal subsidies for tobacco growers. If a tobacco grower and state resident is prosecuted under the state statute, will the federal law compel the state court to dismiss the action? (A) Yes, because the commerce power authorizes Congress to regulate activities of national economic significance and the state statute is inconsistent with the federal legislation. (B) Yes, because the General Welfare Clause empowers Congress to enact legislation it believes to be beneficial to the people of the United States, making the inconsistent state statute invalid. (C) No, because the power to regulate to promote the general welfare is reserved to the states by the Tenth Amendment. (D) No, because the state has a compelling interest in protecting the health of its residents.
(A) The federal law will compel the state court to dismiss the action. Congress's broad power to regulate interstate commerce includes the power to control production of items having a substantial impact on such commerce. (B) is not the best answer because there is no federal general welfare power except the taxing and spending power, and the challenged legislation does more than spend. (C) is irrelevant. An otherwise proper state enactment may still be invalid if inconsistent with a valid federal law regardless of the Tenth Amendment. (D) is also irrelevant. Given the sweeping nature of the commands of the Supremacy Clause, the fact that a state interest is compelling has no bearing on whether state actions predicated on that interest must fall in the face of a valid federal action
A purchasing agent of a steel company invited bids from several coal companies for 10 lots of metallurgical coal. The coal company with the winning bid offered $750,000, which was substantially under the market price for metallurgical coal. The agent immediately made a written agreement with the coal company to carry out the purchase, but several weeks before the coal was to be delivered, the coal company notified the agent that it had made an arithmetical error in pricing the coal and that the price should have been $1 million. The coal company requested a price increase to $825,000, which the agent approved in writing. If the agent had no reason to believe that the coal company had made an error in its initial calculations, is the modification by which the steel company agreed to pay $825,000 enforceable? (A) Yes, because the request for the modification was made in good faith. (B) Yes, only if the coal company can establish that enforcement of the contract at the $750,000 price would be unconscionable. (C) No, because the coal company was under a preexisting duty to sell at $750,000. (D) No, unless the arithmetical error is tantamount to changed circumstances.
(A) The modification is enforceable because it was made in good faith. Article 2 of the U.C.C. applies because coal is a movable good. Under section 2-209 of the U.C.C., consideration is not required for an enforceable modification; however, any modification will be subject to the general Code requirement of good faith and fair dealing, which requires honesty in fact and conformity with reasonable commercial standards. It appears that the coal company was being honest in informing the agent of the clerical error and that both parties made the modification freely. There is no indication that the modified price is commercially unreasonable, despite the required assumption that the disparity was not sufficient to indicate any mistake to the agent. (B) is incorrect because it is not true that the law will only support modifications of otherwise unconscionable contracts. (C) is incorrect because U.C.C. Article 2 abolishes the preexisting duty rule and the requirement of additional consideration to support a modification. (D) is incorrect because Article 2 does not require changed circumstances in order to support a valid modification. All that is required is good faith.
An elderly grandfather who wanted to ensure that his property would remain in the family after his death included the following clause in his will: "I give my house in the city to my son, but if he ever tries to sell it while he is alive, I want it taken away from him and given to my grandson." The grandfather's will was properly executed. When the grandfather later died, what interests did the son and grandson take in the property? (A) The son took a fee simple. (B) The grandson took a fee simple. (C) The son took a fee simple subject to an executory interest, and the grandson took an executory interest. (D) The son took a fee simple determinable, and the grandson took a contingent remainder.
(A) The son received a fee simple in the property. The grandfather attempted to give his son a fee simple, but placed a restraint on alienation. Direct restraints on alienation of a fee simple are void. The grant is simply read as if it had been "O to A in fee simple." The grandson gets nothing. Thus, (A) is correct. (B) is incorrect because the grandson could not have gotten a fee simple in this devise even if there had not been the problem of restraints on alienation. He would, even by the terms of the will, take only if the son tried to sell the property. (C) is incorrect because of the rule prohibiting restraints on alienation of a fee simple. Absent the rule on restraints, these would be the interests taken by the son and grandson. (D) is incorrect because the form of the language does not support a fee simple determinable in the son (no "so long as" type of language), and the grandson could not have a remainder because a remainder can never follow a fee simple
Question 17 A pedestrian is suing a driver after being run over by the driver. At trial, a copy of a hospital record is offered into evidence. The record indicates that hospital personnel took the pedestrian's statement 30 minutes after the accident, and includes the following statement: "The pedestrian's leg was run over by a car." The driver's counsel objects to the admission of the pedestrian's statement in the hospital record. Should the court admit the pedestrian's statement in the record? (A) Yes, even though hearsay within hearsay, because there is an applicable exception to each level of hearsay. (B) Yes, as an admission by a party. (C) No, as hearsay not within any exception. (D) No, because of the physician-patient privilege.
(A) The statement in the record is admissible. Hearsay within hearsay is admissible if each level of the hearsay—here (i) the pedestrian's statement to the admitting officer, and (ii) the admitting officer's statement in the record—comes within a hearsay exception. The business records exception to the hearsay rule, under Federal Rule 803(6), admits into evidence those records kept in the course of the regular conduct of any business, organization, occupation, or calling. Therefore, the statement about the pedestrian's medical condition would be admissible. The record itself (level (ii)) is kept in the ordinary course of the hospital's business of treating patients, and the pedestrian's statement about his medical condition (level (i)), while not made by someone with a duty to keep a record, is admissible both as a statement of present physical condition under Federal Rule 803(3), and as a statement made for purposes of medical treatment under Federal Rule 803(4). Thus, (C) is incorrect. (B) is incorrect because a party cannot introduce his own statement as an admission; it only qualifies as an admission when it is offered by an opposing party. (D) is incorrect because the physician-patient privilege is waived by the patient when he introduces his medical record at trial
A father gave his daughter marketable title to a five-acre parcel of undeveloped land that adjoined 200 acres of uninhabited forest owned by a neighbor. When she visited her property with her father, he mistakenly pointed out the boundary line. She subsequently staked out the boundaries and built a log fence along what she thought was the boundary line. Approximately an acre of the neighbor's land was inside her fence. The daughter built a cabin and lived in it for 30 years until she had to sell the property for medical reasons. The daughter entered into a contract to sell the land. In accordance with the contract, the purchaser had a survey of the land done, which revealed the boundary discrepancy in the legal description. The purchaser contacted the neighbor, who said he knew nothing of the matter and did not consent to the daughter's placement of the fence on his property. The purchaser then refused to proceed with the purchase. The jurisdiction in which the parcel was located had a 20-year period of occupation to satisfy the requirements of adverse possession. If the daughter sues for specific performance of the land sale contract, will she prevail? (A) Yes, because the daughter satisfied all elements required to make out adverse possession of the portion of the neighbor's property within her fence. (B) Yes, because the land sale contract was unaffected by the minor discrepancy in the legal description. (C) No, because the daughter's title to the land is not marketable. (D) No, because the daughter's fencing in of the neighbor's property was not hostile, even if the neighbor had no knowledge of her actions.
(C) The daughter will not prevail because her title is not marketable. The daughter did satisfy all six requirements of adverse possession of the portion of the neighbor's property. She possessed it exclusively for more than the statutory period, her possession was continuous and adverse to the rights of the true owner, and her possession was visible for all to see. But, even though she did acquire title by adverse possession, that title is not marketable until a judicial action is taken to quiet title. The law does not require purchasers to have to go to court to clearly establish title. Because the daughter had not done this, the title she contracted to convey was not, in fact, marketable. Thus, (C) is correct. (A) is incorrect because mere satisfaction of the elements of adverse possession is not enough to make title marketable; the adverse possessor must quiet title in a court action. (B) is incorrect. While the doctrine of marketable title merely requires a title that a reasonable person would accept, a discrepancy of one acre in a tract this small could in no way be considered "minor." (D) is incorrect. The daughter's fencing in of the neighbor's acre was hostile because she neither owned it nor had any right to fence it in. The true owner's knowledge (or lack of it) of an adverse possessor's actions is irrelevant to adverse possession.
A parks and recreation association placed an order for 100 12-inch softballs with a softball manufacturer. The order stated that the association would pay the current market price for the softballs and requested prompt shipment. The manufacturer promptly shipped 100 16-inch softballs to the association because the manufacturer's shipping director negligently misread the size listed on the shipping instructions. Which of the following best describes the association's rights and duties upon arrival of the 16-inch softballs? (A) A contract was formed by the manufacturer's prompt shipment of the 16-inch softballs and the association must accept the softballs and offset its damages against the purchase price. (B) A contract was formed by the manufacturer's prompt shipment of the 16-inch softballs, but the association is not required to accept the softballs and may sue the manufacturer for its damages. (C) The manufacturer's prompt shipment was a counteroffer and the association can accept the softballs or reship them to the manufacturer because it has a duty to mitigate damages. (D) The manufacturer's prompt shipment was a counteroffer, and the association can refuse to accept the softballs, but has no cause of action for damages.
(B) The association may reject the softballs and sue for damages. Softballs obviously are movable goods; therefore, Article 2 of the U.C.C. applies. According to section 2-206(1)(b), an order for "prompt shipment" may be accepted by shipment of either conforming or nonconforming goods, and a contract is created upon such shipment. This alters the traditional rule that a shipment of nonconforming goods was a counteroffer which the buyer accepted upon taking delivery. The association may reject the nonconforming shipment [U.C.C. §2-601] and sue for damages. It may also, if it wishes, accept the shipment [U.C.C. §2-601], notify the seller of the breach [U.C.C. §2-607(3)], pay the contract price [U.C.C. §2-607(1)], and seek damages even after accepting the nonconforming goods [U.C.C. §2-7l4]. (A) is incorrect because, while a contract was indeed formed by shipment under U.C.C. section 2-206(1)(b), the association is not required to accept shipment, but may reject it. [U.C.C. §2-601] (C) is incorrect because the shipment constitutes an acceptance, not a counteroffer, as described above. Additionally, even where a buyer rightfully rejects a nonconforming tender, the buyer is under no obligation to reship prior to receipt of reasonable instructions from the seller. [U.C.C. §2-603] (D) is incorrect because, as discussed above, the shipment constitutes an acceptance, not a counteroffer.
A buyer purchased a tract home in a new development, putting up 10% of the purchase price as a down payment and financing the rest through a mortgage with a bank. After four years, the buyer put her house on the market, continuing to make all mortgage payments promptly. The buyer eventually sold the house to a third party, who took subject to the mortgage. After the third party took possession, the bank received no further mortgage payments from either the buyer or the third party. In most states, which of the following best describes the remedy or remedies available to the bank? (A) The bank may foreclose on the land, but may not sue either the buyer or the third party on the underlying debt. (B) The bank may foreclose on the land, or it may sue the buyer on the underlying debt. (C) The bank may foreclose on the land, or it may sue the third party on the underlying debt. (D) The bank may foreclose on the land, or it may elect to sue either the buyer or the third party on the underlying debt.
(B) The bank may foreclose or sue the buyer on the debt. In the majority of jurisdictions, when a mortgagor transfers title to another, and the transferee takes "subject to" the mortgage, that means that the transferee will not be liable to the mortgagee on the promise underlying the mortgage. So the third party cannot be sued on the debt. But the mortgage follows the property and if the transferee does not make the payments, the mortgagee may foreclose on the mortgage. In any event, the mortgagor, the buyer, remains liable on the mortgage after the transfer and she can be sued on the debt. If the facts showed that the third party had "assumed" the mortgage, rather than merely taking "subject to" it, then he could have been sued on the debt. (A) is incorrect. The buyer signed the mortgage and promised to pay the debt. She will always continue to be liable for her promise regardless of whether the third party is liable. And, because the mortgage follows the land, the mortgagee may always foreclose. (C) is incorrect. The third party did not "assume" the mortgage. He can thus not be held liable on the underlying debt, although if he does not make the payments, the mortgagee may foreclose, since the mortgage always follows the land. (D) is incorrect. Only the buyer is liable on the underlying debt since the third party only took "subject to" the mortgage. Because the mortgage follows the land, the mortgagee may always foreclose if the mortgage is not paid.
A landowner owned several dozen acres of mountain land near a national forest. A plaintiff who was injured by a condition on the owner's land brought an action for personal injury against the landowner. In a jurisdiction that applies the traditional rules for landowners and possessors of land, which of the following plaintiffs is most likely to win? (A) A 10-year-old trespasser who was swept onto some rocks while attempting to cross a swiftly flowing river. (B) A five-year-old trespasser who fell into a mineshaft from which the owner had removed all warning signs, but the plaintiff was not attracted onto the owner's land because of the mineshaft. (C) A five-year-old trespasser who inadvertently stepped into a badger hole that was obscured in the undergrowth. (D) A 10-year-old niece visiting the landowner who stepped into a badger hole that the landowner did not know was present but that could have been discovered by inspection.
(B) The child who fell into the mineshaft is most likely to win. Under the attractive nuisance doctrine,a child trespasser who is injured by a dangerous artificial condition need not have been attracted onto the property by the condition. (A) is wrong because generally there is no obligation for a landowner to warn trespassers, whether they are children or adults, of dangerous natural conditions. (C) is wrong for the same reason. (D) is wrong because, as a licensee, the plaintiff need only be warned of dangerous natural conditions of which the landowner is in fact aware and which are unknown to the licensee or unlikely to be discovered by her, and here the landowner did not know of the hole
Question 5 A lunarian society believed that intelligent life existed on the moon, but that the government was not interested in searching for it. A wealthy political contributor who was a member of this society successfully lobbied Congress to pass a $100 million bill to investigate this hypothesis, even though virtually all reputable scientists rejected it. The President vetoed the bill, calling it a waste of money, but Congress overrode the veto, and the appropriation was authorized. If the constitutional validity of the expenditure is challenged in federal court, is the court likely to find it valid? (A) Yes, because the spending power of Congress is limited only by the political process. (B) Yes, because Congress could believe that the expenditure is a reasonable measure to advance the general welfare. (C) No, because the expenditure is inconsistent with the exclusive authority of the President over foreign affairs. (D) No, because the spending does not directly affect interstate or foreign commerce.
(B) The court is likely to find the expenditure constitutional. Congress can spend federal monies for the general welfare [art. I, §8] provided the spending is reasonably related to a legitimate federal interest. [South Dakota v. Dole (1987)] Here, Congress could believe that it is in the national interest to determine conclusively whether life exists on the moon. (A) is incorrect because Congress cannot spend for unconstitutional purposes, e.g., to promote racial discrimination. (C) is too broad a statement. While the President has broad authority over foreign affairs, the authority is not "exclusive," as Congress also plays a major role in this area. (D) is incorrect because the Spending Clause is not limited by its relationship to commerce.
Question 18 A state law provided that only citizens of the United States may be hired by any governmental unit within the state. A citizen of the Philippines, who had been a legal resident of the state for five years, was awarded a medical degree from the state university and was licensed by the state to practice medicine. The doctor applied for a job opening in the state government, but despite the fact that she was fully qualified, she was rejected solely because of the statute. If she files suit in federal court to enjoin enforcement of the statute, what is her best constitutional argument? (A) The Ex Post Facto Clause. (B) The Equal Protection Clause of the Fourteenth Amendment. (C) The Due Process Clause of the Fourteenth Amendment. (D) The Privileges or Immunities Clause of the Fourteenth Amendment.
(B) The doctor's best constitutional argument is the Equal Protection Clause. The Supreme Court has ruled in a handful of cases that a state may require citizenship for important state jobs that directly affect the political process, provided the state has a rational basis for such discrimination. [See, e.g., Foley v. Connelie (1978)—state troopers; Ambach v. Norwick (1979)—primary and secondary school teachers; Cabell v. Chavez-Salido (1982)—probation officer] This is an exception to the usual rule that state classifications of aliens are "suspect" and subject to strict scrutiny. Serving as a doctor is unlikely to fit this exception. (A) is wrong because the Ex Post Facto Clause applies only to retroactive criminal laws. (C) is wrong because the doctor does not have a liberty or property interest in the possible government job. Because there is not a deprivation of life, liberty, or property, the Due Process Clause is inapplicable. (D) is wrong because the Privileges or Immunities Clause merely forbids a state from infringing on rights of national citizenship and has essentially been a dead letter since the Slaughterhouse Cases (1873). Thus, (B)—the Equal Protection Clause—is the basis for the doctor's strongest argument.
Question 21 A driver on a 3,000-mile cross-country trip in his new car tried to drive the entire trip without stopping, but fell asleep at the wheel, causing the car to strike a bridge abutment and roll over. The driver was seriously injured by the rollover, and suffered additional injuries when the turn signal rod broke off and punctured his lung. The driver had purchased the car from his local auto dealer. The car was manufactured by a local manufacturer, and the turn signal rod was manufactured by a subcontractor whom the manufacturer had used for many years. Tests after the accident established that the turn signal rod was defective and that the defect was the reason it broke off. The defect was not discoverable through reasonable inspection and the manufacturer had had no prior indication of any defects. The driver brought a strict liability action against the manufacturer in a jurisdiction that does not apply its comparative negligence rules to strict liability actions. What is the likely result of the driver's action? (A) The driver will be awarded damages for all injuries incurred as a result of the accident. (B) The driver will be awarded damages for injuries incurred because the turn signal rod was defective, but he will not recover for other injuries incurred in the accident. (C) The manufacturer will prevail, because the accident was caused by the driver's negligence. (D) The manufacturer will prevail, because the driver cannot show that the manufacturer knew or should have known that the turn signal rod was defective.
(B) The driver will be awarded damages for injuries from the turn signal rod. Even though the defective turn signal rod was not the proximate cause of the accident, the driver can recover from the manufacturer for the additional injuries suffered due to the defective turn signal rod, because those are identifiable as caused by that defect. The manufacturer is strictly liable under a products liability theory for injuries caused by that defect, even though a subcontractor manufactured the rod. Thus, (B) is the correct answer. (D) can be eliminated because it deals with a negligence theory, and the suit is being filed under a strict liability theory. Because liability attached only for the damages actually caused by the defective turn signal, (A) is incorrect. (C) is incorrect. In a jurisdiction that does not apply its comparative negligence rules to strict liability actions, ordinary contributory negligence is not a defense. The driver's falling asleep at the wheel was negligent and a cause of the accident, but his punctured lung was also caused by the defective turn signal.
Question 12 A defendant on trial for robbery took the stand in his own defense and testified that the robber was his neighbor. The jury acquitted the defendant based on this testimony. The neighbor was then indicted and brought to trial for the robbery. At that trial the prosecution called the defendant from the first trial to the stand, expecting him to incriminate the neighbor. Surprisingly, the defendant testified: "My neighbor didn't have anything to do with that robbery, but I know who did! I committed the robbery myself." When asked about the testimony he gave at his own trial, the defendant insisted he didn't remember anything about it. Finding her case in shambles, the prosecutor calls a juror from the first trial to the stand as a witness, who is prepared to testify that the defendant said at the first trial that the neighbor committed the robbery. On objection by the defense, should the court admit the juror's testimony? (A) Yes, to impeach the first defendant's credibility as a witness, but not as substantive evidence of the neighbor's guilt. (B) Yes, to impeach the first defendant's credibility as a witness, and as substantive evidence of the neighbor's guilt. (C) No, because former jurors are not competent to testify concerning cases upon which they sat as jurors. (D) No, because a transcript of the first defendant's testimony at his robbery trial is the best evidence.
(B) The juror's testimony is admissible to impeach the first defendant's credibility and as substantive evidence of the neighbor's guilt. As long as the witness is given an opportunity to explain or deny the statement, extrinsic proof of a prior inconsistent statement is admissible to impeach the witness's testimony. If the witness is subject to cross-examination and the prior inconsistent statement was made under penalty of perjury at a prior trial or proceeding, or in a deposition, it is admissible nonhearsay; i.e., it is admissible as substantive evidence. A witness is generally "subject to cross-examination" if he responds willingly to questions (even if he claims a lack of memory). In this case, the prior inconsistent statement was made under oath at the first defendant's trial and thus is admissible for its substance as well as for impeachment. (A) is incorrect because, as discussed above, the juror's testimony is admissible as substantive evidence of the MPQ 205 workshop evidence S.indd 21 12/5/2018 1:57:06 PM 22. EVIDENCE ANSWERS neighbor's guilt. (C) is incorrect because jurors are incompetent to testify only (i) before the jury on which they are sitting, and (ii) in post-verdict proceedings as to certain matters occurring during jury deliberations. Since the juror is not testifying before the jury on which she was sitting and is not testifying about jury deliberations, she is a competent witness. (D) is incorrect because the best evidence rule does not apply to this situation. The juror is not being called to prove the terms of a writing or to testify about knowledge she gained from reading a writing. The facts she is testifying to exist independently of any writing; thus, the best evidence rule does not apply
A shoe retailer ordered by telephone 12 pairs of dress shoes from a shoe manufacturer at its list price of $500 per dozen; delivery in 20 days. The manufacturer orally accepted the offer, and immediately faxed to the retailer this signed memo: "Confirming our agreement today for your purchase of a dozen pairs of dress shoes for $500, the shipment will be delivered in 20 days." Although the retailer received and read the manufacturer's message, he rejected the conforming shipment when it timely arrived. On learning of the rejection, does the manufacturer have a cause of action against the retailer for breach of contract? (A) Yes, because the shoes were identified to the contract and tendered to the retailer. (B) Yes, because the manufacturer's faxed memo to the retailer was sufficient to make the agreement enforceable. (C) No, because the agreed price was $500 and the retailer never signed a writing evidencing a contract with the manufacturer. (D) No, because the retailer neither paid for nor accepted any of the goods tendered.
(B) The manufacturer has a cause of action for breach of contract. Because the contract was for goods priced at $500, the Statute of Frauds must be satisfied under U.C.C. section 2-201. Because both the retailer and the manufacturer are merchants, a memorandum of the terms of the sale sent by the manufacturer to the retailer satisfies the Statute of Frauds unless the retailer objects within 10 days, which she did not do. (A) is incorrect because identifying the goods to the contract and tendering the goods does not satisfy the Statute of Frauds. (C) is incorrect because, while the price makes the Statute of Frauds applicable, it is satisfied by the memorandum written by the manufacturer to the retailer, which was never rejected. (D) is incorrect because neither payment for nor acceptance of goods is necessary to make a contract enforceable, which it is here because the Statute of Frauds was satisfied by the memorandum sent by the manufacturer to the retailer.
An investor filed a civil action against her broker in a State A state court seeking $30,000 for violation of federal securities statutes. The investor is a citizen of State B and the broker is a citizen of State A. May the broker remove the action to federal district court? (A) No, because the amount in controversy is too small for federal subject matter jurisdiction. (B) No, because the defendant is a citizen of the state from whose court the action is removed. (C) Yes, the broker may remove the action to federal district court on the basis of federal question jurisdiction. (D) Yes, the broker may remove the action to federal district court on the basis of federal diversity of citizenship jurisdiction.
(C) The broker may remove the action on the basis of federal question jurisdiction. A defendant may remove an action that could have originally been brought in the federal courts, either on the basis of a federal question being presented or on the basis of diversity of citizenship. Federal question jurisdiction is available when the plaintiff, in his well-pleaded complaint, alleges a claim that arises under federal law. In the instant case, the cause of action is based on a violation of federal securities law. Thus, a federal question has been presented, making the case removable on the basis of federal question jurisdiction. (A) is incorrect. Although diversity of citizenship jurisdiction has an amount in controversy requirement of more than $75,000, there is no amount in controversy requirement for federal question jurisdiction. (B) is also incorrect. When jurisdiction of the federal court is based solely on diversity, and one of the defendants is a citizen of the state in which the action was brought, the action is not removable. There is no similar restriction on removal when removal is based on federal question jurisdiction. (D) is incorrect because the amount in controversy ($30,000) is not sufficient to invoke diversity jurisdiction (i.e., it does not exceed $75,000).
A taxi crashed into a telephone pole. The taxi's passenger commenced a negligence action against the taxi driver, properly serving the taxi driver with the summons and complaint. The taxi driver, being an independent contractor, turned the summons and complaint over to his insurance company. After trial, the jury returned a verdict in favor of the passenger, finding that the taxi driver's negligence was the sole cause of the crash. The passenger then commenced a second action, this time against the owner of the taxi. The passenger alleged that the taxi driver's negligence was the sole cause of the crash and that the owner was vicariously liable. The passenger immediately moved for partial summary judgment against the owner based on the finding of the taxi driver's negligence in the prior action. If the court denies the motion, what is the most likely reason? A Claim preclusion bars the passenger's claim against the owner. B The taxi owner is not precluded from litigating the issue of whether the taxi driver was negligent. C The passenger waived her right to sue the owner by not joining the owner as a defendant in the first action. D The taxi driver is a necessary and indispensable party to the second action.
(B) The taxi owner is not precluded from litigating that issue. For a party to be bound by issue preclusion, (i) there must have been a final judgment; (ii) the issue must have been actually litigated and determined; (iii) the issue must have been essential to the judgment; and (iv) the party to be bound by the prior judgment must have been a party to the prior action or in privity with a party to the prior action. Here, it is the last element that is missing. The taxi owner was not a party to the prior action, and there is no indication that the taxi owner was in privity with the taxi driver. (A) is incorrect for a similar reason. Claim preclusion applies when the earlier case and the later case are brought by the same claimant against the same defendant (or where a party in the later case is in privity with a party in the earlier case). Here, the claim is against a different defendant who was not in privity with the previous defendant. (C) and (D) are incorrect because there generally is no requirement to join joint tortfeasors in a single action. (The exception being if state law requires it.) The risk the plaintiff runs in such a situation is that she loses the first case and thereafter might be bound by issue preclusion in subsequent actions.QUESTION ID: MJ151
A tenant entered into a two-year apartment lease with a landlord on July 1. Rent was specified in the lease to be $850 per month, payable on the first of each month. On June 15, near the end of the two-year term, the landlord asked the tenant if he wanted to renew the lease for an additional term. The tenant said he would like to think about it, and the landlord agreed, but added that the rent for the new lease would be $975 per month. The landlord heard nothing from the tenant. On July 10, the landlord found that the tenant was still in the apartment and told him that he was imposing a new tenancy on the tenant for the period allowed by the law under these circumstances. What tenancy can be imposed, and at what rent? (A) The landlord can impose a new periodic tenancy of month-to-month and the rent will be $850 per month. (B) The landlord can impose a new periodic tenancy of month-to-month and the rent will be $975 per month. (C) The landlord can impose a new periodic tenancy of year-to-year and the rent will be $850 per month. (D) The landlord can impose a new periodic tenancy of year-to-year and the rent will be $975 per month.
(B) The tenancy will be month-to-month at $975 per month. When a tenant wrongfully holds over after the expiration of a lease, the landlord has two choices: either treat the tenant as a trespasser and sue for damages and possession or impose a new periodic tenancy on the hold-over tenant. If the landlord chooses, as this landlord has done here, to impose the new periodic tenancy, most courts in residential situations would impose a month-to-month tenancy. While the rent (as well as other terms) of the new tenancy will generally be the same as the old tenancy, there is an exception when the landlord has told the tenant of a future higher rent and that notification came before the expiration of the old lease. In that event, the landlord can impose the higher rent in the new periodic tenancy. Exam Tip: The facts in this type of question may vary—be sure that the landlord told the tenant of the higher rent before the expiration of the old lease; otherwise, the old, lower rent applies. Because the landlord told the tenant that the tenant could stay on at $975 and this occurred on June 15, prior to the expiration of the two-year lease, then if the tenant held over and the landlord elected to hold the tenant to a new periodic tenancy, it would be at the higher rent. (A) is incorrect. While the new tenancy is month-to-month, the rent will be at $975, not $850. (C) is incorrect in both assertions. The landlord can impose a periodic tenancy of month-to-month and it will be at $975 per month, not $850. (D) is incorrect. While the rent will be $975, the new tenancy is month-to-month, not year-to-year
A tenant invited a friend over for dinner. On his arrival, the friend stepped on a split board on the front steps and the board broke, causing him to lose his balance and break his ankle. If the friend sues the tenant for his injuries and does not prevail in a jurisdiction that applies the traditional rules for landowners and possessors of land, what is the most likely explanation? (A) In the lease, the landlord had undertaken the duty to discover and repair dangerous conditions on the premises. (B) The friend arrived an hour earlier than his invitation specified. (C) The friend should have noticed the dangerous condition himself. (D) The tenant had stayed beyond the lease term and she no longer had the legal right to occupy the premises.
(C) If the friend does not prevail, it will be because he should have noticed the dangerous condition himself. In jurisdictions applying the traditional rules for landowners and possessors of land, the nature of the duty owed by an owner or occupier of land to those on the premises for dangerous conditions on the land depends on the legal status of the plaintiff in regard to the property, i.e., trespasser, licensee, or invitee. A licensee is one who enters on the land with permission for his own purpose or business and includes social guests. The owner or occupier owes a licensee a duty to warn of or make safe a dangerous condition known to the owner or occupier that creates an unreasonable risk of harm to the licensee and that the licensee is unlikely to discover. The owner or occupier does not have a duty to inspect for defects or to repair known defects. Here, the friend had been invited for dinner, making him a licensee. The facts do not indicate whether the tenant knew of the split board and neglected to alert the friend or simply was not aware of it, but the duty to warn does not extend to dangerous conditions that the licensee should reasonably have discovered. Hence, (C) presents the best basis for the friend not prevailing. (A) is incorrect because that fact would not make a difference to the tenant's liability. The tenant remains liable to the friend for dangerous conditions on the premises as the occupier of the land, regardless of the landlord's obligation to inspect and repair. (B) is incorrect because the friend still qualifies as a licensee even though he arrived sooner than his invitation specified. It is true that a person may lose invitee status and become a licensee by being on the premises at a time outside the scope of his invitation. However, there is no similar principle applicable here. The fact that the friend arrived an hour early does not make him a trespasser rather than a licensee. Hence, the fact in (B) would not affect the tenant's liability. (D) is incorrect because it also is irrelevant. Even if the tenant had no legal right to occupy the land, she still would be the possessor of the land as to the friend, and she owed the friend the duties owed to a licensee.
A tenant's apartment was without hot water for over a week because of a broken water heater, even though the landlord had been notified right away and the lease provided that the landlord would make repairs promptly. The tenant heated a large pot of water on the stove and started to carry it to the bathroom so she could warm up her bath. Her young nephew, who was visiting for a few days, came around the corner suddenly and collided with her. The hot water spilled on the nephew, burning him. Because the nephew had a rare blood disorder, the burns resulted in several of the nephew's toes requiring amputation. The nephew's guardian brought a negligence action against the landlord in a jurisdiction that follows the traditional rules for landowner liability. If the jury finds in favor of the landlord, what is the most likely reason? (A) The nephew, as a social guest of the tenant, was not owed a duty by the landlord. (B) The tenant's conduct was the actual cause of the nephew's injuries. (C) The landlord's conduct was not the proximate cause of the nephew's injuries. (D) The nephew's injuries were not foreseeable.
(C) Proximate cause is not established. To establish proximate cause in indirect cause cases, where an intervening force combines with the defendant's conduct to cause the plaintiff's injury, the plaintiff must show that the defendant's negligence caused a foreseeable harm or caused a foreseeable reaction from a foreseeable intervening force. Intervening forces that produce a harm outside of the scope of what would normally be anticipated from the defendant's negligence are generally deemed unforeseeable and superseding. Such a superseding event will break the chain of causation and relieve the defendant of liability. Here, it is ultimately a question for the jury whether the landlord's failure to fix the water heater was a proximate cause of the nephew's injury. However, the jury could very well find that the landlord's failure to do so, even if negligent, is not a proximate cause of the nephew's burn injuries because the conduct of the nephew and the tenant are superseding forces. Given the invalidity of the other choices, lack of proximate cause is the most likely reason for the landlord to prevail. (A) is incorrect. Not only the tenant but also the landlord owes a duty to the tenant's nephew. The landlord's duty to maintain hot water in the tenant's apartment extends to guests of the tenant as well. (B) is incorrect. The tenant's conduct was not the only actual cause of the nephew's injuries under the "but for" test for actual cause. The landlord's failure to repair the water heater promptly was also an actual cause, because but for his failure to do so, the tenant would not have been carrying a pot of hot water to the bathroom. (D) is incorrect. Under the rule that a tortfeasor takes his victim as he finds him, it is irrelevant that the extent or severity of the plaintiff's injuries was unforeseeable. If the landlord were liable to the nephew, he would be liable for all of his injuries even though some were not foreseeable.
A dealer in oriental rugs acquired an antique rug measuring 24 feet by 36 feet. A banker inspected the rug and orally agreed to buy it for the asking price of $65,000, provided he was successful in purchasing the house he was trying to buy, because it had a living room large enough to accommodate the rug. The sale agreement was later reduced to writing, but the provision concerning the purchase of the house was not included in the written agreement. If the banker is unsuccessful in acquiring the house he wants because the owner decided not to sell, and the dealer sues the banker for the purchase price, what is the most likely result? (A) The dealer will prevail because the original oral agreement need not be in writing to be enforceable. (B) The dealer will prevail because of the parol evidence rule. (C) The banker will prevail because he was unable to acquire the house he wanted. (D) The banker will prevail because the dealer is not entitled to specific performance.
(C) The banker will prevail because he could not acquire the house. In general, the parol evidence rule bars oral evidence contradicting a written agreement which was intended to be a final and exclusive embodiment of the parties' agreement. However, one exception to this general rule provides that parol evidence is admissible to show a condition precedent to the existence of a contract. Here, the contract between the banker and the dealer for the sale and purchase of the rug was only to be effective if the banker acquired the house he wanted. This condition precedent may be shown by the banker despite the fact that it was not reduced to writing. (A) is incorrect. This contract must be in writing because it is for the sale of goods of more than $500 in value. However, there is a written agreement here and the question is whether the written agreement precludes proof of the oral agreement. Under the parol evidence rule, a written agreement can prevent proof of an oral agreement even if the agreement does not have to be in writing. (B) is incorrect because a condition precedent to a written agreement's enforceability may be shown by parol evidence. (D) is incorrect because specific performance is not the issue here; whether the contract is enforceable at all is the issue. If this contract is enforceable, specific performance may be available because the rug might be considered unique.
A high-volume pleasure-boat retailer entered into a written contract to sell a customer a power boat for $120,000. The retailer could obtain from the manufacturer, for $90,500, as many of these boats as it could sell. As the contract provided, the customer paid the retailer $40,000 in advance and promised to pay the full balance on delivery of the boat. The contract contained no provision for liquidated damages. Prior to the agreed delivery date, the customer notified the retailer that he would be financially unable to conclude the purchase; the retailer thereafter resold the boat that the customer had ordered to a third person for $120,000 cash. If the customer sues the retailer for restitution of the $40,000 advance payment, which of the following should the court decide? (A) The customer's claim should be denied, because, as the party in default, he is deemed to have lost any right to restitution of a benefit conferred on the retailer. (B) The customer's claim should be denied, because, but for his repudiation, the retailer would have made a profit on two boat sales instead of one. (C) The customer's claim should be upheld in the amount of $40,000 minus the amount of the retailer's lost profit under its contract with the customer. (D) The customer's claims should be upheld in the amount of $39,500 ($40,000 minus $500 as statutory damages under the UCC).
(C) The customer should recover $40,000 minus the retailer's lost profit. The correct measure of damages is the lost profits of the retailer. That amount should be deducted from the deposit and the balance returned. The U.C.C. authorizes four different measures of damages when a buyer defaults after putting down a deposit. First, the Code allows the deposit to serve as liquidated damages if the contract so provides, within certain limits. However, this rule is inapplicable in this case because a liquidated damages clause was not included in the contract. Second, the Code provides in section 2-718(2) that when a deposit is given and there is no liquidated damages provision, the seller may keep 20% of the contract price or $500, whichever is less. In this case, the $500 limitation would control. However, if actual damages are more, as they are in this case, the seller may collect actual damages rather than the statutory amount. In section 2-708, the Code provides two measures of actual damages when the buyer defaults. The first is the traditional expectancy measure of damages, resale price minus contract price. In a case such as this, there are no damages under this measure because the boat was resold for the contract price. The second alternative under section 2-708 applies here. When the seller is a dealer and the traditional measure of damages is inadequate to put him in as good a position as he would be in if the sale went through, then the measure of damages is lost profit. Here, since the dealer could have ordered as many boats as there were buyers, he actually lost his profit on this sale. (A) is incorrect because even a defaulting party has the right to recover a portion of the deposit back in restitution if the amount of the deposit exceeds the contract damages to which the non-defaulting party is entitled. (B) is incorrect. While it is true that the retailer would have made profits on two boat sales instead of one if the customer had not defaulted, it did make the profit on the second sale, and cannot collect this from the customer. The lost profit on the first sale amounts to $29,500, less than the $40,000 advance. Therefore, the customer is entitled to $10,500, the difference between the amount of advance and the profit actually lost. (D) is incorrect because, as discussed above, actual damages allowed under U.C.C. section 2-708 exceed the $500 allowed under U.C.C. section 2-718(2), so the higher amount of damages would be awarded.
A woman owned 400 acres of land, half of which was densely wooded and the other half of which was almost entirely occupied by a large gravel pit which supplied gravel for her small landscaping business. A small house located on the edge of the pit was in very poor condition and had been vacant for many years. The woman transferred the 400 acres to her son for life, with the remainder going to a local charity on the son's death. Now the son wants to increase gravel production and expand the pit by tearing down the house. He also wants to cut the trees on the wooded half and sell them for profit. The local charity, holder of the remainder, sues to enjoin the son from doing any of these things. How will the court likely rule? (A) The charity can stop both the gravel mining and the tree cutting and can block the destruction of the house. (B) The charity can stop neither the gravel mining nor the tree cutting but can block the destruction of the house. (C) The charity can stop the tree cutting but not the gravel mining or the destruction of the house. (D) The charity can stop the gravel mining and the tree cutting but not the destruction of the house because it is dangerous and unfit for use.
(C) The charity can stop only the tree cutting; life tenant can tear down house and mine. The charity would be suing the life tenant on a theory of waste. Both the gravel mining and the tree cutting could be voluntary waste. Generally, a life tenant can only maintain the property and not sell off any of the natural resources, such as trees and gravel. But there is an exception for existing exploitation of these resources. Because the gravel mine was operating prior to the son's taking of the life estate, that preexisting use is protected and the son is not liable for continuing the mining of gravel. But the trees do not fall under this exception. Because the property had not been in use as a tree farm, the general rule applies and the cutting of the trees would be waste. The destruction of the house might be considered ameliorative waste—the destruction of improvements on the life estate that increase the value of the property (i.e., they make the gravel mining more profitable). Generally, a life tenant cannot tear down improvements simply because the life tenant wants to make a more profitable use of the land. But an exception exists when changed conditions have made the destruction of the improvement reasonably necessary. Here, the house was on the edge of the pit, vacant because of its very poor condition. Changed conditions (the encroachment of the pit) have made the house reasonably unsuitable, so the life tenant can tear it down. (A) is incorrect because the gravel mining cannot be stopped and the house can be destroyed. (B) is incorrect because the tree cutting can be stopped but the house can be destroyed. (D) is incorrect because the tree cutting is the only thing that can be stopped
A camper at a state park built a campfire within a fire ring on a calm day according to approved procedures. Just as a sudden strong wind arrived and blew some embers onto the grass, a large bear came out of the woods and charged at the camper. The camper ran to his car, which was some distance away, with the bear in close pursuit. By the time the bear left and the camper was able to exit the car and summon assistance, the embers in the grass had started a brush fire. The fire destroyed another camper's equipment and automobile at a nearby campsite before it could be extinguished. The other camper sued the camper who started the fire. At trial, the parties stipulated to the above facts. The plaintiff introduced into evidence a state statute that prohibited leaving any campfires unattended and required them to be extinguished immediately if any embers were blown out of the fire ring. At the conclusion of the proofs, both parties moved for a directed verdict. How should the court rule on the motions? (A) Grant the plaintiff's motion, because the statute was intended to prevent the type of harm that occurred, making the statutory standard applicable. (B) Grant the plaintiff's motion, because a brush fire caused by a campfire does not ordinarily happen in the absence of negligence by the camper. (C) Grant the defendant's motion, because the plaintiff has not established a prima facie case of negligence. (D) Deny both motions, because the jury should make the factual determination of whether the defendant was negligent.
(C) The court should grant the defendant's motion because the plaintiff has not offered sufficient evidence of negligence on the defendant's part to go to the jury. The standard of care in a negligence case may be established by proving that a statute imposing a specific duty applies instead of the more general common law duty of care. However, violation of the statute may be excused where compliance would cause more danger than violation or where compliance would be beyond the defendant's control. Here, the statute regulating campfires is applicable because (i) the plaintiff, a fellow camper, is in the class intended to be protected by the statute, (ii) the statute was designed to prevent the escape of a campfire, which is what occurred here, and (iii) the statute clearly specifies what is required. However, even though the statute would apply to the defendant's conduct and the defendant violated the statute, the violation will be excused here because he was fleeing for his life from a bear and had to take refuge in his car. Hence, the defendant will not be held to the statutory standard of care here. Because the plaintiff has offered no other evidence that the defendant was negligent, the defendant's motion should be granted. (A) is incorrect because, as discussed above, even though the statute was intended to prevent the harm that occurred, the defendant's violation of the statute will be excused. (B) is incorrect because the plaintiff cannot rely on an inference of negligence here to establish breach of duty. Res ipsa loquitur permits the trier of fact to infer breach of duty where the facts strongly indicate that the plaintiff's injuries resulted from the defendant's negligence, but here the undisputed facts as to how the brush fire was caused are inconsistent with a finding of negligence. (D) is incorrect because there is neither a reasonable inference of negligence nor evidence of negligence, given that the violation of the statute will be excused here. Hence, the case should not be submitted to the jury because the plaintiff has not established a prima facie case.
In a property dispute, a granddaughter claims that her grandfather gave her a deed to his home just before he died. The grandfather's son claims that the property is rightfully his by a previously executed will. At issue is the authenticity and content of the deed. The granddaughter begins to testify as to the content of the deed, but the son's attorney objects. Should the court sustain the objection? (A) Yes, because only the original deed itself is admissible to prove its contents. (B) Yes, unless the court is satisfied that the granddaughter is accurately testifying as to its contents. (C) No, if the judge is satisfied that the deed could not be found after a reasonable search. (D) No, if the deed is recorded, because the court can take judicial notice of the recorded deed.
(C) The court should rule that the testimony is admissible if the deed could not be found after a reasonable search. When a proponent is attempting to prove the contents of a document, the best evidence rule applies and the original must be accounted for in order to introduce secondary evidence as to its contents. A proper foundation for the admissibility of secondary evidence may be laid by a showing that the original has been lost and cannot be found despite diligent search. Hence, (A) is incorrect. (D) is incorrect because the court may take judicial notice of court records, but the proponent of a recorded document must produce the document or record she desires introduced into evidence. A court has no power to take judicial notice of public records. (B) makes no sense because the court has no way of making such a determination.
An owner conveyed her property to a buyer, who put the deed in his suitcase and took off for a five-month tour of the world. The owner, knowing that the buyer had left the country, sold the property to an investor who was not aware of the previous transaction. The investor did not record her deed. When the buyer returned from his trip, he recorded his deed. A month later, the investor conveyed the property to a developer. The developer knew that the buyer held a deed to the property but completed the transaction anyway. Instead of recording, however, the developer immediately filed an appropriate action against the buyer and against the investor to determine ownership of the property. The property is situated in a jurisdiction containing the following statute: "Any conveyance of an interest in land, other than a lease for less than one year, shall not be valid against any subsequent purchaser for value, without notice, unless the conveyance is recorded." How should the court rule? (A) The buyer has rights superior to those of both the investor and the developer. (B) The investor has rights superior to those of both the buyer and the developer. (C) The developer has rights superior to those of both the buyer and the investor. (D) The developer has rights superior to those of the investor, but the buyer has rights superior to those of the developer.
(C) The court will rule that the developer has rights superior to those of the other parties. Under a notice statute, which is the statute applicable here, the general rule is that a subsequent bona fide purchaser ("BFP") will prevail over a prior grantee who failed to record before the BFP's purchase; if the prior grantee has previously recorded, the subsequent purchaser ordinarily will be deemed to have record notice of the prior conveyance and will not be a BFP. Here, the owner resold the property to the investor before the buyer recorded his deed to the property; thus, the investor had no record notice of the transfer. Because the investor also had no actual knowledge or inquiry notice (the buyer was out of the country), she qualifies as a BFP. Finally, under the shelter rule, a person who takes from a BFP will prevail against any interest that the transferor-BFP would have prevailed against. Thus, even though the developer knew that the buyer held a deed to the property, the developer prevails against the buyer because the investor would have prevailed against the buyer. (A) is incorrect because, as discussed above, the buyer's recording came too late under the notice statute. If this jurisdiction instead had a race-notice statute, the buyer would prevail because he recorded before another BFP recorded. (B) is incorrect because, while the investor would have superior rights to the buyer, the investor does not have superior rights to the developer because the investor conveyed the property to the developer. (D) is incorrect because, as discussed above, the developer can rely on the shelter rule to prevail against the buyer even though the developer was aware of the buyer's interest in the property.
A defendant charged with criminal battery testified on his own behalf at trial and asserted that he acted in self-defense. Which of the following offered for impeaching the defendant's credibility is most likely to be barred, if objected to by the defendant? (A) A public record showing that the defendant had been convicted of the felony of aggravated battery two years ago. (B) A public record showing that the defendant had been convicted of the misdemeanor of filing a false police report eight years ago. (C) Testimony from a competent witness that the defendant regularly cheats at cards. (D) Testimony from a competent witness that, in the opinion of the witness, the defendant is a habitual liar.
(C) The defendant's cheating at card games is most likely to be barred. This question involves the use of three different forms of evidence (reputation or opinion testimony, evidence of specific conduct, and prior criminal record) to impeach the testimony of a witness by showing that the witness has a propensity to be dishonest. Because the question asks which item of evidence is least likely to be admissible to impeach the defendant's testimony, turn the question around and ask which of the items of evidence would be admissible to impeach the defendant's testimony. The evidence in (B) is a public record of the defendant's misdemeanor conviction for filing a false police report eight years ago. Although only a misdemeanor, this conviction is admissible under Federal Rule 609(a)(2) because it is for a crime involving dishonesty. It does not matter that the conviction is eight years old, because Rule 609(b)'s limitation on the admissibility of criminal convictions applies only if 10 years have passed since the conviction or the end of the convict's incarceration. Nor does it matter that the prejudicial impact of the criminal offense involving dishonesty is offered to impeach a witness's testimony. Thus, the evidence in (B) would be admissible and so (B) is wrong. The evidence in (D) is testimony from a competent witness that, in the opinion of the witness, the defendant is a habitual liar. Federal Rule 608(a) allows such testimony, as long as the witness is competent (i.e., able to base the opinion on personal knowledge). The testimony clearly relates to the defendant's character for truthfulness or lack of truthfulness. Thus, the evidence in (D) is wrong. The evidence in (A), a public record of the defendant's felony conviction for aggravated battery two years ago, is possibly admissible. Federal Rule 609(a)(1) permits evidence of a witness's past conviction for a serious crime (one punishable by death or imprisonment of more than one year) to be used to impeach the witness's testimony. Such evidence, however, is subject to a probative value/prejudicial impact balancing test. It is possible that evidence of the defendant's felony conviction would not be admissible because of the prejudicial impact it might have on his defense, particularly since the crime for which he was convicted (aggravated battery) is virtually identical to the crime with which he is now charged. However, evidence of a criminal conviction for committing a felony, particularly a recent conviction, has traditionally been freely admitted to impeach the testimony of a witness. Thus, the evidence in (A) might be admitted. The evidence in (C), testimony from a competent witness that the defendant regularly cheats at cards, is almost certainly inadmissible. This is evidence (here, the testimony from the witness) to prove specific conduct of another witness (the defendant) bearing on his character for dishonesty. Specific acts of the defendant's dishonesty may be brought out on cross-examination, but not through the use of extrinsic evidence. Thus, (C) is correct because it is a better answer than (A); it is the least likely to be admissible for the purpose of impeaching the defendant's credibility.
Question 9 The father of a high school senior who had poor grades his junior year offered to pay his son $1,000 if he quit playing video games for the rest of the school year. The son agreed to quit that very day. However, shortly thereafter the son began to have doubts as to whether his father would really pay him the $1,000 if he kept his end of the bargain. The son told his mother about the conversation and she told him, "Go ahead and quit—if your father won't pay, I'll see that you get the money he promised you from my own account." The son quit playing video games for the rest of the school year, and he made the honor roll that year. Unfortunately, his parents did not live to see him complete this feat—they were killed in a skydiving accident two weeks before the end of the school year and one month before his 18th birthday. At the appropriate time, the son submitted a claim for $1,000 to the executor of his father's estate and was refused payment because the estate was insolvent after paying numerous secured creditors. The son then filed a claim against his mother's estate for the $1,000 based on the promise his mother made to pay that amount if his father refused to pay it. What is the best argument for the probate court's rejecting this claim against the mother's estate? (A) The contract between the mother and the son was illusory. (B) The son has not been damaged by any breach because the only effect—that he quit playing video games—was salutary. (C) The contract between the mother and the son was oral. (D) The son did not turn 18 years of age until two weeks after he completed performance and, thus, the contract was void because it lacked mutuality of obligation.
(C) The estate's best defense is that the contract was oral. Generally, contracts need not be in writing to be enforceable; however, under the Statute of Frauds, certain contracts must be evidenced by a writing signed by the party to be charged to be enforceable. Contracts in which one party promises to pay the debt of another, such as this contract, must be in writing. The mother promised to pay her son the amount owed to him by his father if the father did not pay him. Thus, there was a suretyship promise that had to be in writing under the Statute of Frauds, and this one was not. Therefore, (C) is correct. (A) is incorrect because the promise was not illusory. A promise is illusory when there is not consideration on both sides of the contract. Here, the son will receive $1,000 if he performs (i.e., refrains from playing video games for a specified time period), and the mother will receive the son's detriment of not doing something that he has a right to do, which is valid consideration (the benefit to the promisor need not have economic value). The son's performance is valid consideration even though he has already promised his father that he would give up video games (i.e., it is not a preexisting duty), because the son was not bound by his promise to his father. The father's offer was for a unilateral contract (i.e., one seeking performance rather than a promise to perform), and so could be accepted only by performance. The son had not yet performed when his mother made her promise to pay him if his father did not, so the son had not yet accepted his father's offer and was not bound by his promise to quit playing video games. Therefore, he was not under a preexisting duty, and his mother's promise served as additional consideration for the son's performance. Note also that a surety such as the mother will be bound by her promise to pay another's debt as long as she makes her promise before the creditor (the son) performs or promises to perform; the surety need not receive any separate consideration. (B) is incorrect because the son's giving up what he had a legal right to do—even if it is detrimental—is sufficient consideration to support a contract, so the mother is bound to pay even though the contract was beneficial to the son. (D) is incorrect because a contract between an infant (i.e., a person under the age of 18) and an adult is binding on the adult, even though it is voidable by the infant. Courts abide by the legal fiction of mutuality of consideration in cases involving infant-adult contracts for reasons of public policy, so such contracts may be made and acted on.
Question 11 The President issued an executive order prohibiting exportation to certain countries of specific computer software that, although not usable directly to develop nuclear weapons, would facilitate nuclear weapons technology. Congress had previously passed a law authorizing the issuance of such orders. Prior to the issuance of the executive order, a computer software company contracted with one of those countries for software that is now banned for sale and distribution. What effect does the executive order have on this contract? (A) The executive order unconstitutionally impairs the obligation of the company's preexisting contract, which was lawful when made. (B) The executive order unconstitutionally denies the company a valuable property interest without due process, because it is not limited to computer software used directly to produce nuclear weapons and, therefore, is not necessary to vindicate a compelling national need. (C) The executive order is constitutional because Congress has plenary powers to regulate commerce with foreign nations and has used that power to authorize such orders. (D) The executive order is constitutional, because the inherent power of the President to conduct foreign affairs is plenary.
(C) The executive order is constitutional because it was authorized by Congress. For all practical purposes, the power to regulate foreign commerce lies exclusively with Congress, and Congress may delegate this power to the President. By authorizing the issuance of the Presidential executive order, Congress has constitutionally exercised this power. (D) is incorrect. The President's power is limited by the Constitution, which grants Congress the power to regulate foreign commerce. (A) is incorrect because the impairment of the Contract Clause applies to state governments only, not to the federal. (B) is incorrect because the company's contract right would not qualify as a property interest under the Fifth Amendment.
A mass marketer contracted with a political campaign to send out mass mailings to voters for $100,000. It subcontracted with a printer to print brochures for $20,000 over a period of several weeks. The printer would be paid on a weekly basis. After providing $15,000 of printing services the first few weeks, the printer unjustifiably refused to perform any additional work for the marketer. The marketer had paid the printer $10,000 to that point, and had to pay another printer $12,000 to print the balance of the brochures. The marketer sued the printer for breach of contract, and the printer counterclaimed for the reasonable value of the benefits conferred on the marketer and not paid. What will be the outcome of this litigation? (A) The printer should recover $5,000, the benefit conferred on the marketer for which the printer has not been paid. (B) The printer should recover $3,000, the benefit conferred on the marketer less the $2,000 in damages suffered by the marketer. (C) The marketer should recover $2,000, the excess it had to pay over the contract price to get the performance the printer had promised. (D) Neither party should recover anything, because the printer was in breach of contract and the marketer received $15,000 in printing services from the printer for $10,000.
(C) The marketer should recover $2,000. The primary objective of contract damages is to put the nonbreaching party in the same position that he would have been in had the contract been performed. The normal measure of damages is expectation damages. The marketer has a legally enforceable right to have the work under the printer contract performed for $20,000. Because the marketer paid the printer $10,000 and needed to spend $12,000 to have the printing completed by a third party, the marketer has spent $22,000. This establishes the marketer's right to seek $2,000 from the printer for its breach. (A) and (B) are incorrect because the purpose of contract damages is to put the nonbreaching party where he would have been had the promise been performed. For the printer to recover anything, he would have to prove that he is entitled to restitution. The breaching party will prevail in a restitution action only if the nonbreaching party seeks to keep the value of the benefit conferred without paying and, therefore, is unjustly enriched. The marketer had to pay $2,000 more than the price stated in the contract with the printer. The marketer was not unjustly enriched and, in fact, had damages of $2,000 from the breach. The printer's costs are not relevant. The printer is not entitled to recover anything. (D) is incorrect. To provide no remedy to either party on the theory stated in (D) is to make the marketer pay a total of $22,000 for the performance that was promised by the printer for $20,000. The primary objective of contract damages is to put the nonbreaching party in the same position that he would have been in had the contract been performed. Therefore, this theory fails.
A mother whose young son was riding on a roller coaster by himself for the first time walked some distance away to get a snack. She heard a commotion by the ride and saw a crowd gathered. When she came closer, she heard someone close to the scene say that a young boy had fallen off and was killed. She was very distraught but could not see through the crowd. In fact, it was not her son but another boy who had fallen off. That boy had struck her son while falling, resulting in minor injuries to the son.The mother, who was pregnant, ultimately suffered a miscarriage as a result of accident-related stress. In a previous suit by the parents of the boy who was killed, the ride operator was found liable for negligence in operating the ride. Can the mother recover damages for her distress and resulting miscarriage in an action against the ride operator for negligent infliction of emotional distress? (A) Yes, because her son was injured as a result of the operator's negligence. (B) Yes, because she was closely related to someone in the zone of danger from the operator's negligence. (C) No, because she was not within the zone of danger from the operator's negligence. (D) No, because her son was not the boy who was killed.
(C) The mother cannot recover. A duty to avoid negligent infliction of emotional distress may be breached when the defendant creates a foreseeable risk of physical injury to the plaintiff. For a bystander who is outside the zone of danger from the risk of physical injury but who suffers emotional distress from seeing the defendant negligently injure another, most states allow recovery if: (i) the plaintiff and the person injured by the defendant are closely related; (ii) the plaintiff was present at the scene of the injury; and (iii) the plaintiff personally observed or perceived the event. Here, the mother was some distance away and not in the zone of danger, so her distress was not caused by any perceived danger to her. Nor can she recover under the bystander rules. While she is related to her son, who was injured in part by the ride operator's negligence, she was not present at the scene of the injury and did not personally observe or perceive the event. Rather, her distress was due primarily to hearing someone say that a young boy had been killed and not being able to get close enough to the ride to see where her son was. Hence, she cannot recover damages for negligent infliction of emotional distress. (A) is incorrect. The mother's distress was not caused by the injuries her son suffered, but rather at her fear that he was the one killed. While she may be able to recover damages for his injuries, those would be distinct from any damages for her emotional distress. (B) is incorrect. As discussed above, the plaintiff's close relationship with the injured person is only one of the requirements for a plaintiff outside the zone of danger to recover emotional distress damages. Because the mother was not present at the scene and did not observe the event, she cannot recover. (D) is incorrect. While her son was not killed, he was injured by the boy striking him while falling. Had the mother been present and observed the boy falling and hitting her son, she could recover damages for her emotional distress. Conversely, even if the boy who was killed were her son, her damages would be compensable instead through a wrongful death action.
The owner of land conveyed it "to my best friend, and upon her death to my daughter." The best friend took possession of the land and lived there for four years. She then conveyed "my interest in the land" to her longtime neighbor. Since the neighbor took up residence on the land, he has been sent two county property tax bills, which he has refused to pay. The county is now threatening to force a sale of the property to satisfy the tax lien. The daughter filed an appropriate suit, asking the court to evict the neighbor from the land and to compel him to pay the taxes for his period of occupancy. How should the court rule? (A) The neighbor has a life estate in the land and he must pay the taxes on the property. (B) The neighbor has a life estate in the land for the period of the friend's life and the neighbor does not have to pay the taxes on the property because taxes are the responsibility of the holder of the future interest. (C) The neighbor has a life estate in the land for the period of the friend's life and the neighbor must pay the taxes on the property. (D) The daughter owns the land, because the friend could not convey her interest to her neighbor since the daughter held the future interest.
(C) The neighbor has a life estate for the life of the friend and must pay taxes. When the best friend transferred her life estate to the neighbor, he took an estate that will terminate when the best friend dies. That is all that the best friend owned and all that she could convey. In addition, the rule is that the life tenant is responsible for paying taxes on the property (had there been a mortgage, the life tenant would have had to pay the interest on the mortgage but not the principal). (A) is incorrect. While it is true that the neighbor had a life estate and he does have to pay taxes, this is not a full answer because it does not recognize that his life estate is not an ordinary one; it is one measured by the best friend's life, not his own. (B) is incorrect. The neighbor does have a life estate measured by the best friend's life, but he must pay the taxes. Taxes are not a responsibility of the holder of the future interest, but are an obligation of the life tenant. (D) is incorrect. A life tenant can transfer the life estate to anyone, and the holder of the future interest has no say in the matter. Of course, if the new tenant abuses the property, the holder of the future interest has the ability to sue to stop waste, but cannot stop the transfer itself.
Restrictions on Use of Supplemental Jurisdiction in Diversity Cases
1) claims by plaintiffs against impleader, compulsory joinder, permissive joinder, or intervention parties
Rights of plaintiff to recover for IIED when defendant commits harm to another?
1) close friend or relative 2) plaintiff present 3) defendant knows of relationship Exception: plaintiff does not need to be present or close relative if defendants goal was to harm plaintiff.
The owner of a house purchased a new home but decided to keep his old residence for a few months until the real estate market improved. He rented it to a tenant with the understanding that the tenant might have to move out in a few months if the house was sold. The tenant paid the owner the agreed rent of $100 per week every Friday. During the next few months, the owner's business suffered serious setbacks so he decided to sell his new home and move back into his old one. He informed the tenant that he would have to vacate the old home, but the tenant refused to vacate and tendered the $100 rental payment the following Friday, which the owner refused to accept. The owner immediately filed suit to eject the tenant. The jurisdiction requires that a statutory written notice be served on any tenant whose term is for less than month-to-month or is not for a fixed term at least three days prior to commencement of eviction proceedings. No written notice of any kind was given to the tenant. How should the owner characterize the tenant to gain immediate possession of his home? (A) As a tenant from month to month. (B) As a tenant from week to week. (C) As a licensee. (D) As a trespasser ab initio.
(C) The owner's best argument is that the tenant is a licensee. A license is a privilege to enter onto another's property. It may be revoked at any time merely by a manifestation of the licensor's intent to end it. (A) is wrong because a month-to-month tenant would benefit from the common law right to a period of notice equal to the tenancy, i.e., one month. (B) is wrong because a tenant at sufferance would be entitled to the statutory notice. (D) is wrong because the tenant is clearly not a trespasser ab initio since he entered onto the premises with permission.
To combat the spread of a dangerous and highly contagious disease, the state passed legislation authorizing testing clinics be set up in major cities. One of the cities, with a population of 25,000 people, had an army base in the city employing 2,500 people and housing 5,000 soldiers. The mayor and the state director of public health determined that the best location for the clinic would be on the base, because that is where the disease had the greatest risk of spreading. When the proposal was made to the base commander he refused to allow it, even though there was ample space available. Can the state compel the base commander to allow the testing clinic on base? (A) Yes, because the concentration of service personnel and civilian base employees in one location poses a significant health threat to the citizens of the state if they are not tested. (B) Yes, because the area of public health has not been preempted by the federal government. (C) No, because the state has no right under its police power to impose this burden on the federal government. (D) No, because the state could easily construct the clinic elsewhere.
(C) The state cannot compel the commander to allow the clinic. State governments may not regulate federal government activities. States thus lack the authority to impose regulations on federal military bases. The Supremacy Clause prevents such state control of the federal government. (A) is incorrect because the state lacks constitutional authority to impose regulations on federal military bases. (B) misses the issue because the question is not whether the public health regulation is preempted by the federal government, but rather the immunity of federal instrumentalities from state government regulations. (D) is incorrect because regardless of the ease (or even hardship) of constructing other facilities, states may not regulate federal military bases.
A manufacturer and a buyer entered into a written contract for the manufacturer to produce and sell to the buyer 2,000 widgets at a price of $20 per widget. The contract expressly provided that the buyer shall have no liability under the contract unless 2,000 widgets are delivered to the buyer at his place of business no later than July 1. On July 1, 1,800 widgets meeting the buyer's specifications were tendered by the manufacturer. The remaining 200 widgets were tendered on July 5. The buyer refused to accept any of the widgets. In an action by the manufacturer against the buyer, which of the following would best support the manufacturer's case, assuming it can be proven? (A) The buyer had orally agreed, just prior to the time the written contract was executed, to accept and pay for partial deliveries of the widgets. (B) Widgets are a unique product produced only by the manufacturer and in a size and tolerance that varies with the needs of each purchaser. (C) Delivery of the 200 widgets on July 1 was delayed by a storm which disrupted the shipper's activities, and was not the manufacturer's fault. (D) A drop in the buyer's credit rating from good to fair had caused the manufacturer not to produce and tender the full 2,000 widgets on or before July 1.
(C) The storm delay would best support the manufacturer's case. The storm may have made delivery on time impossible, which may excuse performance under either the doctrine of impracticability of performance or the doctrine of impossibility of performance. These defenses can be used not only to excuse performance totally, but also to excuse the delay in performance. (A) is incorrect because an oral agreement made prior to a written agreement on the same subject will not be admissible to alter the written agreement under the parol evidence rule. Therefore, the oral agreement does not modify the terms of the written contract and this contention will not help the manufacturer's case. (B) is incorrect. The fact that goods are specially made creates an exception to the Statute of Frauds, but it does not excuse the obligation to tender conforming goods in the correct quantity at or before the time specified for delivery. The perfect tender rule is incorporated MPQ 203 workshop contracts into every contract for the sale of goods except where the contract specifies different terms or where there is an explicit installment contract. (D) is incorrect. If the buyer became insolvent, the manufacturer could, under the U.C.C., require that the buyer pay cash upon delivery or give assurances of payment. Here, however, the manufacturer has not requested that payments be made in cash or that the buyer give assurances. Moreover, the buyer only suffered a decrease in his credit rating; he is not insolvent. Therefore, the manufacturer would have had no basis for making either request. The manufacturer is not excused from performance by the buyer's decreased credit rating
Due to violence erupting against picketers advocating automatic deportation of foreign persons accused of a crime, a state enacted a law prohibiting all picketing "carried out for the purpose of deterring others from exercising their constitutional rights." The strongest constitutional defense that can be asserted by those charged with violating this statute is which of the following? (A) The Fifth Amendment right to due process of law, because the statute is so vague that reasonable persons cannot ascertain its scope. (B) Equal protection of the laws, because the statute does not prohibit picketing for purposes other than those specified. (C) First and Fourteenth Amendment rights of free expression and assembly, because the statute excessively restricts the marketplace of ideas. (D) Article IV privileges and immunities of state citizenship, because picketing is a fundamental right.
(C) The strongest defense is based on free expression and assembly grounds. It is presumptively unconstitutional for the government to place burdens on speech because of its content. Because the statute regulates content of the speech rather than conduct, the state would need to establish a compelling interest behind the legislation for the law to be valid. This puts the burden on the state to prove the need for the legislation. (A) is incorrect because the Due Process Clause of the Fifth Amendment applies to the federal government, not to the states. (B) is incorrect because any discrimination that may arise from the statute is merely an effect of facially neutral legislation and not subject to the Equal Protection Clause. (D) is incorrect because Article IV prohibits discrimination by a state in favor of its own citizens, which is not the case here.
Question 15 In support of a charity fundraising luncheon, three volunteers independently brought to the event a casserole dish made with ground beef. Each of them had prepared her dish in her own kitchen. Another volunteer combined the dishes onto one large serving platter, from which guests at the luncheon served themselves. One of the guests became seriously ill with what the health department later determined to be a bacterial infection from undercooked beef that was in the combined casserole. The guest brought an action against the three volunteers who made the casserole dishes, alleging negligent preparation of the ground beef. Assuming that the guest can establish only the above facts and his injuries, who is likely to prevail in the action? (A) The guest, because, under the doctrine of res ipsa loquitur, he has established an inference of negligence. (B) The guest, because he can require each of the volunteers to prove that she was not the actual cause of the injury. (C) The volunteers, because the guest cannot establish which of the volunteers breached her duty of care. (D) The volunteers, because they all were donating their time and food to the event.
(C) The volunteers are likely to prevail. The elements of the prima facie case for negligence are (i) a duty owed to the plaintiff, (ii) breach of that duty, (iii) actual and proximate cause, and (iv) damages. Here, the volunteers each owed a duty of care to anyone consuming the food they prepared, including the guest. The facts indicate that at least one of the volunteers breached that duty by improperly preparing or cooking the ground beef. That breach of duty caused the guest to become seriously ill. However, he cannot establish which of the volunteers breached the duty of care and was the actual cause of his injury. Absent additional evidence, the guest will not prevail. (A) is incorrect. Res ipsa loquitur does not apply because more than one person supplied the casserole dish. The res ipsa loquitur doctrine enables a plaintiff to establish breach of duty just from the fact that an injury occurred that would not ordinarily occur unless someone was negligent. However, the plaintiff must establish evidence connecting a particular defendant with the negligence to support a finding of liability against that defendant. When more than one person was in control of the instrumentality that caused the injury, such as here, res ipsa loquitur generally may not be used. The doctrine sometimes has been applied to multiple parties involved in a joint venture, but that does not apply in this case. Each person volunteered independently to make the casserole dish, and each of them worked individually with their own recipes in their own homes, and another person combined the three batches into one casserole dish. (B) is incorrect. The alternative liability approach of Summers v. Tice applies when two or more persons have been negligent but it cannot be determined which one caused the plaintiff's injury. The court will shift the burden of proof to each of the negligent defendants to show that his negligence was not the actual cause of the injury. Here, however, there is no evidence that all of the volunteers were negligent; most likely, just one of them was. Hence, the volunteers will not be required to prove that they did not cause the guest's injury. (D) is incorrect. Regardless of the fact that they were just donating their time and food, each of the volunteers owed a duty of care to anyone consuming the food they prepared, as stated above. If the guest could prove that a particular volunteer undercooked the ground beef in her casserole, he could recover damages for his illness that was caused by the undercooked beef.
The plaintiff is suing the defendant for serious, life-threatening injuries she suffered when the defendant's car collided with the plaintiff's at a busy intersection. At trial, the plaintiff testified that the light was green when she entered the intersection. She was not cross-examined. She also testified that she lost consciousness after the collision but awoke to find a witness comforting her as she lay on the ground. The plaintiff calls the witness to testify that, when she awoke, she said, "Help, I'm dying! Why did [the defendant] enter the intersection when the light was green for me?" Is the witness's testimony concerning the plaintiff's statement admissible? (A) No, because it is hearsay not within any exception. (B) No, because a party is not permitted to introduce her own out-of-court statement that is consistent with her testimony on the witness stand. (C) Yes, as an excited utterance. (D) Yes, as a statement under belief of impending death.
(C) The witness's testimony is admissible as an excited utterance. For a statement to be admissible as an excited utterance, it must relate to a startling event and be made while the declarant was under the stress of excitement caused by that event. These conditions are met in this fact pattern. The statement was made immediately after the plaintiff regained consciousness after the accident, and appears to have been made while the plaintiff was under the excitement caused by the accident. Moreover, the other choices can clearly be eliminated. (A) is incorrect because, while the statement is clearly hearsay, it reasonably could come within the excited utterance exception under Federal Rule 803(2). For a statement to be admissible as an excited utterance, it must relate to a startling event and be made while the declarant was under the stress of excitement caused by that event. These conditions are fulfilled in this fact pattern. The statement was made immediately after the plaintiff gained consciousness after the accident, and appears to be made while the plaintiff was under the excitement caused by the accident. (B) is incorrect because the statement is too broad. A party is permitted to introduce her own consistent out-of-court statement as long as that statement is relevant and is not inadmissible hearsay. In this case, the statement is clearly relevant and could reasonably come within the excited utterance exception to the hearsay rule under Federal Rule 803(2). (D) is incorrect. The apparent dying declaration is a favorite trick on the MBE. Remember that if the victim does not die, she must be unavailable to testify before the dying declaration is admissible on those grounds. Although the statement of the plaintiff was made at a time when she believed that her death was imminent and related to what would have been the cause of death, her statement is not admissible under Federal Rule 804(b) (4). That exception, while not requiring that the declarant actually die of the cause described in the statement, does require that the declarant be unavailable. In this case, the declarant is actually testifying and therefore not unavailable.
A woman who owned a parcel of land had direct access to the main road by an unpaved driveway. However, she decided that it would be more convenient to use a paved driveway on an adjoining parcel, so she began doing so. The man who owned the adjoining parcel also used the driveway, but he did not discover that the woman was using the driveway until two years later. At that time, he wrote a letter to the woman protesting the use of the driveway, but the woman continued to use the road. After she had used the road for a total of 20 years, she filed an action for declaratory judgment, claiming a prescriptive right to use the driveway on the west parcel. The time limit necessary to obtain an easement by prescription in this jurisdiction is 20 years. Which of the following is the most accurate statement with regard to the woman's use of the driveway now? (A) The woman could not acquire a prescriptive easement in the west parcel because her original use was wholly trespassory and without any claim of right. (B) The prescriptive period began to run when the woman first began using the driveway because the woman was in open and notorious possession, despite the fact that the man did not know of it. (C) The prescriptive period began to run when the woman first began using the driveway despite the fact that, at that time, she was sharing her use with the man. (D) The prescriptive period did not begin to run until two years after the woman began using the driveway, at the time her use of the driveway was contested.
(C) The woman's first use of the driveway started the prescriptive period running. There is no requirement of exclusive use for the woman to obtain an easement by prescription. Exclusivity is only required in cases of adverse possession. Adverse possession is not at issue here because the woman has not "possessed" any part of the west parcel, and does not require title to continue her use of the property, which is all she seeks. (A) is incorrect because the only way in which a prescriptive easement can be obtained is for a person to trespass on land and to continue to trespass for the statutory period. The fact that the use was wholly trespassory in fact gives the man the cause of action and starts the clock running on the statute of limitations. Whether the use was undertaken under a claim of right is wholly immaterial to the analysis. The statute of limitations runs whether or not the trespasser thinks the use is lawful. (B) is incorrect. This choice correctly states that the woman's conduct when she began using the driveway started the prescriptive period and correctly states that the man's lack of knowledge is irrelevant. It is incorrect because it does not correctly describe the conduct of the woman. She was not in open and notorious possession; she was merely using the man's driveway. The distinction is important because exclusive possession for the statutory period leads to title by adverse possession, while adverse use will only result in a prescriptive easement. The woman does not need to and could not live up to the more exacting standard of adverse possession in this case. (D) is incorrect because the prescriptive period began to run when the woman began using the driveway. The fact that the man was not aware of it and did not contest it is irrelevant, because her initial use of the driveway was without permissio
Question 5 Owen owned Blackacre in fee simple. He executed a will leaving the property to his sister, Sarah, for life, with remainder to the children of his niece, Norah. At the time of the execution of the will, Norah had one daughter, Donna. The next year, Norah had a son, Sam. A year later, Owen died. Shortly after Owen's death, Donna died in a plane crash, leaving all of her estate to her husband, Harold. One year after Owen's death, Norah had another daughter, Debbi. Shortly thereafter, Sarah, Owen's sister, died. Sixteen months after that, Norah had a second son, Sid. Now who owns Blackacre? (A) Harold, Sam, Debbi, and Sid in equal shares. (B) Harold and Sam in equal shares. (C) Sam, Debbi, and Sid in equal shares. (D) Harold, Sam, and Debbi in equal shares.
(D) Harold, Sam, and Debbi take Blackacre in equal shares. At Owen's death, Norah had a daughter and a son, making their interests at that time vested remainders in fee simple subject to open to make room for any future children of Norah. When Donna died, her vested remainder passed to Harold. When Debbi was born, she joined Sam and Harold, all three holding vested remainders in fee simple. When Owen's sister Sarah died, her life estate ended, giving the present possessory estate to those three holders of the future interest. Why is Sid left out? Because of the class closing rule: Whenever any member of the class is entitled to a distribution, the class closes and the distribution is made then. Any later arrivals to the class lose out entirely. Because Sam, Harold, and Debbi are eligible for a distribution the moment the life tenant (Sarah) dies, they take, and Sid, not yet born, loses out. (A) is incorrect because Sid does not take, since he was born after a member of the class was entitled to a distribution. (B) is incorrect because Debbi also takes, because she was born before the class closed at the end of the preceding life estate. (C) is incorrect. Harold should be included because vested remainders, including vested remainders subject to open, are fully devisable and descendible. (C) is also incorrect because Sid should not be included for the reason discussed above
Question 19 A music fan sued a well-known groupie of a rock band, alleging that she was assaulted by the groupie during a melee at a concert. The fan's attorney calls the custodian of records for the hospital that treated her. He wishes to introduce a portion of the record by the emergency room physician, who is now deceased, reporting that the fan said she was assaulted by the groupie. Assuming the custodian testifies that the record is an original, kept in the ordinary course of hospital business, is that portion of the hospital record admissible? (A) Yes, as hearsay within the exception for records of regularly conducted activity. (B) Yes, as a statement made for the purpose of medical diagnosis or treatment. (C) No, because the physician who made the record is not available for cross-examination. (D) No, as hearsay not within any exception.
(D) That portion of the record is inadmissible. The statement is an assertive out-of-court statement by the music fan which does not qualify under any exception to the hearsay rule. For a business record to be admissible as an exception to the hearsay rule, the declarant must either have personal knowledge of the fact stated or must have received the information from someone with personal knowledge who transmitted it in the ordinary course of business. In this case, the music fan's statement does not qualify for the business record exception because the emergency room physician had no personal knowledge of the attack and the fan did not transmit it in the ordinary course of business. It likewise does not qualify as a statement made for the purpose of medical diagnosis or treatment. The statement is not offered to describe the injuries which the music fan suffered so that she could be treated for them, but rather to identify the assailant. The exception for statements made for the purpose of medical treatment under Federal Rule 803(4) is inapplicable here. Therefore, (A) and (B) are incorrect. (C) is incorrect because the presence of the physician is irrelevant under the business record exception. That exception does not require the presence of the author of the record, only a custodian of the record.
Question 18 Three drivers were in a traffic accident in State A. The three drivers were citizens of State A, State B, and State C, respectively. Only the driver from State C sustained any injuries and damage. The driver from State C filed a tort action against the State A driver, seeking $100,000 in damages. The State A driver believes that he was not at fault in any way and that the driver from State B was the sole cause of the accident. Assume State A does not recognize any claim of contribution among joint tortfeasors. May the State A driver assert a third-party claim against the State B driver in the pending action, alleging that the State B driver was the party at fault and should pay for the State C driver's injuries? (A) Yes, because third-party claims are permitted as long as they arise from the same transaction or occurrence as the original action. (B) Yes, because the State A driver's third party claim is derivative of the State C driver's original claim. (C) Yes, provided the State A driver obtains leave of the court. (D) No, the State A driver may not maintain the third-party claim.
(D) The State A driver may not maintain the third-party claim. Third-party claims may be maintained only if the defendant/third-party plaintiff alleges that the third-party defendant is liable to the defendant/third-party plaintiff for all or part of the defendant/third-party plaintiff's liability to the plaintiff. Here, the defendant/third-party plaintiff (the State A driver) can make no such claim because state law does not recognize contribution among joint tortfeasors. In other words, the State A driver may raise the State B driver's negligence as a defense, alleging that the State B driver was the sole cause of the action, but he may not implead the State B driver based on joint contribution. (A) is incorrect because the answer provides the wrong standard for third-party claims. A defending party may implead a nonparty, but only if the nonparty is or may be liable to her for any part of a judgment that the plaintiff may recover against her. This is not the case here, as discussed above. (B) is incorrect because it provides the wrong standard for a third-party impleader claim. The correct standard is stated above. (C) is also incorrect for the reasons stated above. Additionally, leave of court is required only if the third-party complaint is filed more than 14 days after service of the defendant's original answer.
Question 14 State A passed a law requiring at least 30% of energy produced in State A to be from sustainable sources such as wind or solar. To help achieve this, State A created an agency to build wind farms and sell the energy to consumers. Some of the wind farms were located near State A's border with State B. Although there is a shortage of available electricity in the area of State B that borders State A, the State A agency decided that it would not sell electricity to consumers in State B unless State B also adopted a law requiring at least 30% of State B energy to be produced from sustainable sources. To date, State B has not adopted such a law. State B consumers who wish to purchase electricity from State A brought an action against the state in an appropriate State A court, claiming State A's refusal to sell them electricity violates the Commerce Clause. Assume there are no relevant federal statutes. Are the State B consumers likely to prevail? (A) Yes, because State A is discriminating against out-of-state residents. (B) Yes, because State A's decision is not necessary to protect an important state interest. (C) No, because State A has a compelling interest in protecting the environment. (D) No, because State A is acting as a market participant.
(D) The State B consumers are not likely to prevail. The Commerce Clause does not prevent a state from preferring its own citizens when the state is acting as a market participant. Here, State A is acting as a market participant because an agency of the state built the wind farm and is selling the energy. Therefore, State A can favor its own citizens in selling the energy, free of scrutiny under the Commerce Clause. (C) is wrong because State A does not need a compelling interest. Because it is acting as a market participant, it can favor its own citizens. (A) is wrong because, although it is true that State A is discriminating against out-of-state residents, it is allowed to do so as a market participant. (B) is wrong because it is irrelevant. As discussed, it does not matter whether State A has an important interest.
After a difficult divorce, a mother wrote to her son and daughter the following: In consideration of your emotional support for me during that trying time and your love and affection for me, I promise to divide my estate between you in equal shares. You know you can count on your mother's word. The daughter thereafter continued her usual practice of calling her mother once a week and visiting her at Christmas and on her birthday until her mother died three years later. Shortly after the funeral, the daughter learned that the mother's will made the son the sole legatee. If the daughter sues the executor of the mother's estate for one-half of that estate, based on the mother's letter to her, will she win? (A) Yes, because she relied to her detriment on her mother's promise by visiting her mother. (B) Yes, because her emotional support of her mother during the divorce proceedings constituted valid consideration for her mother's promise. (C) Yes, because even though her mother's letter is a promise to make a gift in the future, the promise is in writing and intended by her to be enforceable and therefore needs no consideration. (D) No, because the mother's promise was not supported by consideration.
(D) The daughter will lose because there is no consideration to support the promise. Promises to make gifts in the future are unenforceable even if they are in writing and are intended by the promisor to be enforceable. Hence, (C) is incorrect. A promise to make a gift does not involve a bargained for exchange, and the requirement of consideration is not fulfilled. (A) is incorrect. This answer states the principle of promissory estoppel. Under the doctrine of promissory estoppel, where there is substantial detrimental reliance by a party on a promise of the promisor, the promise will be enforceable even absent consideration. It is unlikely that the daughter's continuation of her calling and visiting her mother would constitute substantial detrimental reliance. Absent consideration or a substitute, the mother's original promise would not be enforceable, and her daughter would be unsuccessful in her suit against the executor. (B) is incorrect because the emotional support given to the mother by her daughter was not bargained-for consideration. It was voluntarily given before there was any promise to leave property by will, and therefore does not make the promise enforceable. In order to be part of the bargain, the element of consideration must be part of the bargained-for exchange. Thus, if the mother had said to her daughter, "If you will give me emotional support, I will leave you half my estate," the emotional support thereafter given by the daughter would have been bargained for by the mother in exchange for part of her estate. Because the daughter gave her support gratuitously before the promise, the mother's promise did not induce the legal detriment, and was therefore not supported by consideration.
A citizen of State A filed a breach of contract action against a citizen of State B in a State A state trial court. The State B defendant timely and properly removed the action to the United States District Court for the District of State A. The defendant then filed a motion to dismiss the action based on insufficient service of process. Following a hearing, the court found that service was proper and denied the motion. The defendant then filed her answer, responding to the merits of the complaint and asserting that the case should be dismissed on the grounds that another action was pending between the same parties for the same cause in a State B state court. The State A Rules of Civil Procedure provide that a party waives the right to seek dismissal on that ground if the party files a pre-answer motion to dismiss and does not assert that ground in the motion. Should the federal court hold that the defendant has waived the right to seek dismissal based on the pendency of the same cause in another court? (A) Yes, because this action was commenced in a State A state court, so the federal court must apply State A law. (B) Yes, because this action is in a federal court in State A on the basis of the court's diversity of citizenship jurisdiction, so the federal court must apply State A law. (C) No, because this issue is unlikely to affect the outcome of the action or to create forum-shopping, so the federal court should apply federal law, under which the defendant has not waived this defense. (D) No, because this issue is governed by the Federal Rules of Civil Procedure, under which the defendant has not waived this defense.
(D) The defendant has not waived this defense. Unlike the state rule here, Federal Rule of Civil Procedure 12 does not require the defendant to raise the defense that another action is pending between the parties in the first responsive pleading. While the federal court exercising diversity of citizenship subject matter jurisdiction must apply state substantive law (here, State A law), applicable Federal Rules of Civil Procedure supersede state law in federal court as long as the rule comports with the requirements of the Rules Enabling Act (i.e., the rule governs practice and procedure and does not modify or abridge substantive rights). (A) and (B) are incorrect because they are too broad. The federal court need not apply State A procedural law. (C) is incorrect because, if the rule in question is on point and clearly procedural, it is simply applied. There is no need to determine if its application would affect the outcome of the case.
A woman owned two adjacent parcels. The east parcel fronts on a poor unpaved public road, while the west parcel fronts on a major highway. Fifteen years ago, the woman conveyed the east parcel to her son "together with a right-of-way 25 feet wide over the north side of the west parcel to the highway." At that time, the east parcel was improved with a 10-unit motel. Ten years ago, the woman died. Her will devised the west parcel "to my son for life, remainder to my daughter." Five years ago, the son executed a deed purporting to convey the east and west parcels to his friend in fee simple. The friend then enlarged the motel to 12 units. Six months ago, the son died and the daughter took possession of the west parcel. She brought an appropriate action to enjoin the friend from using the right-of-way. Who should prevail? (A) The daughter, because merger extinguished the easement. (B) The daughter, because the friend has overburdened the easement. (C) The friend, because he has an easement by necessity. (D) The friend, because he has the easement granted by the woman to her son.
(D) The friend should prevail. The deed from the woman to her son granted the son an express easement. That easement was never terminated by merger and the friend, the owner of the motel property on the east parcel, still has a valid easement over the daughter's property, the west parcel. (A) is incorrect because, for there to be a merger which will extinguish an easement, the duration of the servient estate must be equal to or longer than the duration of the dominant estate (and therefore the easement). In this case, the son owned the east parcel in fee simple but owned only a life estate in the servient estate, the west parcel. Therefore, because the duration of the servient estate was shorter, there was no merger of the two estates and the easement was not extinguished by the son's partial ownership of the servient estate. The friend likewise never owned the west parcel in fee simple because all that was owned by the son, his grantor, was a life estate in the west parcel. (B) is incorrect. The right of way over the west parcel at the time it was granted MPQ 206 workshop real property serviced a 10-unit motel. The expansion of the motel by an additional two units will cause a minimal increase of the use of the way and does not overburden it. (C) is incorrect because the friend does not have an easement by necessity. The east parcel fronts on a poorly paved public road. At the time the woman conveyed the east parcel to the friend's predecessor in title, the son, there was access to his property from a public way and there is no absolute necessity which would have created an easement by necessity. The friend therefore never received the property benefited by an easement by necessity
A large producer of bread wrote to a distributor of flour, asking, "How much will you charge to supply my needs for flour for the next year?" The distributor replied in writing that it could supply the producer with all the flour it would need next year at a specified price per pound. The producer wrote back, "Your offer to supply me with flour is hereby accepted, provided that you agree to a 10% discount if payment is made within 10 days from date of billing." What should the producer's reply concerning a 10% discount be characterized as? (A) An acceptance with a proposal for an additional term. (B) An acceptance with a term that, absent an objection, is part of the contract. (C) A proposed modification of the contract. (D) A rejection of the distributor's offer.
(D) The producer's reply is a conditional acceptance, which is a rejection of the offer. This question deals with the "battle of the forms" provision of the UCC. Under section 2-207 of the UCC, an acceptance containing additional or different terms is effective as between merchants unless the offeree expressly makes his acceptance conditional on assent by the offeror to the additional terms. When an acceptance is made expressly conditional on the acceptance of new terms, it is a rejection of the offer. The conditional acceptance is essentially a new offer, and the original offeror may form a contract by expressly assenting to the new terms. Here, both parties are merchants, but the producer made his acceptance conditional on the distributor's assent to the 10% discount. Thus, his communication is a rejection of the distributor's offer. (A) is incorrect because this is not an acceptance. The rule stated in (A) is the one used for a nonmerchant when an acceptance contains an additional term. (B) would have been correct had the acceptance not been conditional. Because the parties are merchants, an acceptance with an additional term would result in the term becoming part of the contract unless (i) it materially altered the original terms of the offer; (ii) the offer had expressly limited acceptance to the terms of the offer; or (iii) the offeror objected to the terms within a reasonable time after notice of it. (C) is incorrect because at this point there is no contract to be modified; there is no acceptance of the offer
A first-time home buyer financed the purchase of a house with a $100,000 mortgage she took out with a bank. The mortgage was recorded. A few years later she borrowed $5,000 from a finance company to pay for a foreign trip, using her house as security. The finance company promptly and properly recorded its mortgage on the property. One year after that, she borrowed $40,000 from an equity company to pay for an addition on the house. The equity company promptly and properly recorded the mortgage it took on the property. Shortly thereafter, she lost her job and was unable to make payments on either the finance company's or the equity company's mortgages, but she was able to make payments on the bank's mortgage. The finance company filed foreclosure of its mortgage and included the equity company in the action, and a purchaser bought the property at the foreclosure sale. What is the purchaser's obligation regarding the bank's mortgage and the equity company's mortgage? (A) The purchaser takes the property subject to both mortgages. (B) The purchaser takes the property subject to neither mortgage. (C) The purchaser takes the property subject to the equity company's mortgage, but not subject to the bank's mortgage. (D) The purchaser takes the property subject to the bank's mortgage, but not subject to the equity company's mortgage.
(D) The purchaser takes the property subject only to the bank's mortgage. A foreclosure sale wipes out all junior mortgages (those that came later in time than the mortgage that was foreclosed) but does not wipe out senior mortgages (those that came earlier). Because the bank's mortgage preceded the finance company's, it is senior and is not wiped out. The purchaser takes subject to this mortgage. Thus, (B) is wrong. Although the purchaser is not personally liable on this debt (he did not sign the note, only the original buyer did), he must pay the mortgage or face foreclosure by the bank. Because the equity company's mortgage came later than the finance company's, it is a junior interest that is wiped out as long as the holder of the junior interest is included in the foreclosure action (which occurred here). Thus, (A) and (C) are wrong. If, after paying the cost of the foreclosure and paying off the finance company's mortgage, there is money left over from the sale, it will first go to paying off the equity company's mortgage. But regardless of whether the equity company's mortgage is paid off, the purchaser takes completely free of this obligation.
Question 15 Several decades ago, a square tract of 640 acres was subdivided into lots, streets were constructed, and utilities were installed. As the developer sold off each of the lots, each lot's deed contained a restriction limiting the lots to residential use only. The deeds were all recorded. Over the years, houses were constructed on all of the lots. The property to the south, east, and west of the subdivision was initially forest, but gradually the city expanded to surround the development. Unfortunately, the city expansion was mostly industrial and some commercial property, but no residential property. The subdivision is now bounded on all sides by many industries that operate 24 hours a day. The combination of noise, dirt, fumes, and other pollution has made many of the houses in the subdivision unfit for residential use, yet each deed still stipulates "residential use only." Can the restriction be voided under the doctrine of changed neighborhood conditions? (A) Yes, the restriction can be voided on all lots in the subdivision because changed conditions have made many of the houses unusable as residences. (B) Yes, the restriction can be voided under these circumstances, but only as to the lots that have actually been rendered unfit for residential purposes. (C) No, the restriction cannot be voided because in this circumstance the proper remedy is the tort action of nuisance. (D) No, the restriction cannot be voided unless the entire subdivision is so seriously affected by the pollution that enforcement of the restriction would be inequitable.
(D) The restriction cannot be voided under the doctrine of changed neighborhood conditions. Restrictive covenants on all lots in a subdivision such as this can be voided if changed conditions have made the property unusable for the specified use, and this means that the entire subdivision must have changed so significantly that enforcement of the restriction would be inequitable. If some houses in the center of the subdivision are not affected by the pollution, then none of the restrictions can be voided; if all lots are affected, then all restrictions are voided. (A) is incorrect because it does not make clear that to void all the restrictions the entire subdivision must be seriously affected. (B) is incorrect. Courts will not lift a restriction on the parcels affected by the change if doing so would change the conditions for nearby lots that were previously unaffected. To avoid this domino effect, the condition of the subdivision as a whole must be considered. (C) is incorrect. This situation may indeed give rise to tort liability in nuisance, but that is not the only remedy. If homeowners want to sell out to nonresidential users, or engage in nonresidential use themselves, they must first get out from under the restriction in their deeds. This avenue exists independently from any right to sue in nuisance.
Question 16 The issue at trial is whether the sun was shining in a particular city on a particular day at 11 a.m. A witness testified that he was lying beside his wife on the beach in that city at 11 a.m. that day and she said to him, "Now that the sun is out, I'll be able to get my great tan!" Is the statement admissible? (A) No, as hearsay not within any exception. (B) No, because the witness does not have firsthand knowledge. (C) Yes, but only if the witness's wife is unavailable. (D) Yes, whether or not the witness's wife is unavailable
(D) The statement is admissible whether or not the witness's wife is unavailable. The statement by the witness's wife is hearsay, an out-of-court statement offered to prove the truth of the matter stated, that the sun was out at a specific time. However, it is admissible as a present sense impression under Federal Rule 803(1) because she was observing the condition of the sun at the time that she made the statement. Unavailability is not required for this exception. Therefore, (A) and (C) are incorrect. (B) is incorrect on the law and the facts. The present sense impression exception to the hearsay rule, which is applicable in this case, only requires that the witness actually hear the declarant make the statement at the time that the declarant is observing the event. It does not require that the witness also observe the event. Moreover, as a factual matter, although it is not legally relevant, the witness would have first-hand knowledge of the event described because he was on the beach, too.
Negligence (Prima Facie)
1) existence of duty to conform to specific standard of conduct; 2) breach of that duty 3) breach actual and proximate cause of plaintiff's injury 4) damage
If one party files in federal court and another in state, federal court is prohibited from enjoining state court proceedings unless:
1) expressly authorized by statute; 2) necessary in aid of its jurisdiction or to protect or effectuate its judgments.
Personal Jurisdiction Test
1) Need statute and due process. 2) Due process: contact, relatedness and fairness. i) Contact (personal availment) of forms, such minimum contacts foreseeable that defendant could be hauled into court in forum. ii) Relatedness of claim to conduct: In state activity OR "at home" in the state (general jurisdiction) iii) Fairness (traditional notions of fair play and substantial justice). Look to: convenience (hard standard to meet, money not really an issue, must be so gravely difficult and inconvenient that a party is unfairly put at a severe disadvantage in comparison to his opponent.) Forum state's interest in providing redress for residents,
Duty Regarding Negligent Infliction of Emotional Distress
1) Plaintiff must be within "Zone of Danger" 2) Must suffer physical symptoms from distress Special Situations: 1) bystander not in Zone of Danger: plaintiff and person injured closely related; plaintiff present at scene of injury; person personally observed or perceived event.
Specific Standards of Care
1) professionals: required to possess and exercise knowledge and skill of member of profession or occupation; required to disclose risks of treatment. 2) Children: like age, education, intelligence and experience UNLESS engaged in adult activities (operating motorized vehicle) 3) common carriers and inn keepers
The essential terms of a land sale contract under the Statute of Frauds are:
1. A description of the property; 2. An identification of the parties to the contract; and 3. A price and manner of payment, if agreed upon. The Statute also requires the contract to be in writing and signed by the party to be charged.
Pre-requisites for res ipsa loquitur
1.) injury not attributable to plaintiff or third person; 2) evidence connecting defendant with negligence; 3) accident of the type that would not normally occur unless someone was negligent. Do not need to show defendant had actual possession.
A woman sued her employer for sexual harassment. At the close of the trial, the employer made a motion for judgment as a matter of law, arguing that the woman's evidence was insufficient to establish the elements of her claim. The court denied the motion. When the jury returned a verdict in favor of the woman, the employer renewed its motion for judgment as a matter of law. In addition to the renewed motion for judgment as a matter of law, the employer also moved for a new trial, asserting that the verdict was against the weight of the evidence. The court denied both motions. If the employer appeals the denials of both the renewed motion for a judgment as a matter of law and the motion for a new trial, what is the appropriate standard of review? A De novo for the renewed motion for judgment as a matter of law and abuse of discretion for the new trial motion. B Clearly erroneous for the renewed motion for judgment as a matter of law and de novo for the new trial motion. C Abuse of discretion for both. D De novo for both.
A (A) When an appellate court reviews a trial court's ruling on a post-trial motion for judgment as a matter of law (including a renewed one), it employs a de novo standard. It does so because the issue is one of law. In contrast, when an appellate court reviews a trial court's denial of a motion for new trial, it employs a more deferential standard, reversing the trial court's denial only when there is a clear showing of an abuse of discretion.
A city council passed an ordinance providing: "No person may contribute more than $100 annually to any group organized for the specific purpose of supporting or opposing referenda to be voted on by the city electorate or regularly engaging in such activities." If the ordinance is challenged in federal court, how should the court rule on the constitutionality of this ordinance? A Strike it down, because it violates First Amendment rights of free speech and freedom of association. B Strike it down as a violation of due process, because no hearing mechanism has been provided for. C Uphold it, because the city council has a legitimate interest in controlling such contributions. D Dismiss the case, because it involves a political question and is thus a n
A The federal court should strike the ordinance for violating the First Amendment. While the government may limit the amount of contributions that an individual can contribute to a candidate's campaign (to avoid corruption or the appearance of corruption), the government may not limit the contributions to a political committee that supports or opposes a ballot referendum, because such a law does not serve a sufficiently important interest to outweigh the restraints that it puts on the First Amendment freedoms of speech and association. (B) is incorrect because the Due Process Clause does not require that every law provide for a hearing, but rather only those laws involving the deprivation of life, liberty, or property of an individual. The law here does not involve a deprivation of life or property, and liberty is not being denied to individuals on a judicial basis (i.e., according to the facts of each case), but rather is being denied to all persons on a legislative basis. In such a case, individual hearings are not required to satisfy due process; as long as the law was lawfully adopted (e.g., with notice to all interested parties), the Due Process Clause has been satisfied. (C) is incorrect because a legitimate interest in controlling contributions to a political committee for ballot referendum is not enough. The statute must be "closely drawn" to match a "sufficiently important interest," which is an intermediate scrutiny standard, and the Supreme Court has invalidated limitations on contributions to influence referendum elections. (D) is incorrect because political questions, which are nonjusticiable, arise when the issue is committed to another branch of the government by the Constitution or is incapable of resolution and enforcement by the judiciary. Determining whether a law is valid is within the realm of the judiciary and certainly is capable of resolution (i.e., the law could be invalidated). Thus, there is no political question here.
A pedestrian walking on the sidewalk was struck by a car backing out of a driveway. The driver did not see the pedestrian because her neighbor's bushes obscured her view of the sidewalk. The pedestrian was seriously injured and brought suit against the driver and the neighbor. The pedestrian also included the city in his lawsuit, alleging that the city failed to enforce its ordinance requiring homeowners to provide a clear view of sidewalks where they intersect with driveways. The trier of fact determined that the driver was 60% at fault, the neighbor was 30% at fault, and the city was 10% at fault. The jurisdiction has adopted comparative contribution in cases applying joint and several liability. Which of the following is a correct statement regarding liability? A The city is liable to the pedestrian for the full amount of the damage award. B Both the driver and the neighbor are liable to the pedestrian for 90% of the damage award. C Each of the three defendants are liable to the pedestrian for one-third of the damage award. D The driver is liable to the pedestrian for 60% of the damage award, the neighbor is liable for 30% of the damage award, and the city is liable for 10% of the damage award.
A The city is liable to the pedestrian for the full amount of the damage award. The city is liable to the pedestrian for the full amount of the damage award. Under joint and several liability, each defendant found by the trier of fact to be at fault for an indivisible injury is liable to the plaintiff for the entire amount of damages incurred, not just a portion of it. (Of course, multiple recovery is not allowed.) Thus, because the city has been found to be at fault for the accident, the pedestrian could recover the full amount of the damage award from the city. C) and (D) are incorrect because they are not applying joint and several liability. Under a joint and several liability system, contribution allows a defendant who pays more than his share of damages to recover the excess from the other jointly liable parties; responsibility for the total damages is thus apportioned among those at fault. Traditional contribution rules require all defendants to pay equal shares regardless of their respective degrees of fault (choice (C)), while states with a comparative contribution system impose contribution in proportion to the relative fault of the various defendants (choice (D)). Nevertheless, this simply means that the city (assuming it paid the judgment award to the plaintiff) has contribution rights against the other defendants (i.e., it can recover from the others for damages paid in excess of the amount proportionate to its relative fault). This does not, however, mean that the defendants' liability to the plaintiff is based on their relative fault. In fact, if one defendant were judgment-proof, the others would still be responsible for the full amount despite the fact that the judgment-proof defendant was mostly at fault.
Restraint on alienation and encumbrance
A provision in a deed or will that attempts to restrict the sale or transfer of the property forever or for an extremely long period of time -- for example, selling your house to your daughter with the provision that it never be sold to anyone outside the family. These provisions are usually unenforceable on the grounds that a present owner should not be allowed to tie the hands of future generations. The maximum period of time for limiting transfer is generally "lives in being, plus 21 years." (This is known as the rule against perpetuities.) Restraints on alienation (restrictive covenants) based on race ("only Caucasians may hold title") were declared unconstitutional in 1949.
Congress passed a statute requiring energy consumption be reduced by a specified percentage, to be set by a presidential executive order. The statute provided specific standards that the President must use in setting the percentage, and detailed procedures to be followed. Is this statute likely constitutional? (A) Yes, because it creates a limited administrative power to implement the statute. (B) Yes, because inherent executive powers permit such action even without statutory authorization. (C) No, because it is an undue delegation of legislative power to the executive branch. (D) No, because it violates the Due Process Clause of the Fifth Amendment.
A) The statute is likely constitutional. Congress has broad discretion to delegate its legislative power to executive officers or administrative agencies. Delegations of legislative power to the executive branch of government are virtually always upheld. The Supreme Court says that there must be some standards to guide the exercise of discretion, but the requirement is very minimal. Here, the statute "sets forth specific standards" and thus is sure to be upheld. (B) is incorrect because, even if the President had such inherent powers, that would not make the legislation constitutional. The question asks about the constitutionality of a statute; an answer about the President's inherent powers is nonresponsive. (C) is incorrect for the reasons explained above: Since the New Deal era, virtually no laws have been declared unconstitutional as an excessive delegation of legislative power, and this statute has sufficiently specific standards that it would not be invalidated on that basis. (D) is incorrect because due process claims require that an individual be denied life, liberty, or property without due process of law. The facts of the question do not indicate any such deprivations.
The state of Blue enacted a statute to protect its faltering lobster industry. The statute provides that no lobster shall be taken from lobster beds lying within three miles of the state shoreline unless the lobster is at least one pound in weight. The statute's one-pound limitation is intended to enable young lobsters to reproduce before being caught. At the same time, Congress enacted a lobster conservation act that provides $5 million for research funds to develop and improve breeding grounds for lobsters. The federal act imposes a special excise tax of $1,000 on each lobster caught in violation of state law if later shipped in interstate commerce. A lobsterman who lives in the state of Green, which is just south of the state of Blue, crossed over into waters lying within three miles of the Blue coastline. He was arrested by state Blue law enforcement officers for taking lobsters that weighed less than one pound. The man defended the charge by challenging the constitutionality of the state Blue statute. Which of the following results is most likely? A The statute will be upheld because it does not violate the Constitution in any way. B The statute will be upheld because the Commerce Clause does not apply when a state is seeking to protect natural resources. C The statute will be held invalid because it violates the Interstate Privileges and Immunities Clause of Article IV, Section 2. D The statute will be held invalid because it is preempted by the federal lobster conservation act.
A. The statute will be upheld because it does not discriminate against out-of-state economic interests and it is not unduly burdensome. A state or local government may regulate local aspects of interstate commerce if such regulation: (i) does not discriminate against out-of-state competition to benefit local economic interests; and (ii) is not unduly burdensome (i.e., the incidental burden on interstate commerce does not outweigh the legitimate local benefits produced by the regulation). The state of Blue statute does not discriminate against out-of-state elements of the lobster fishing industry. The statute is designed to maintain the lobster population by allowing lobsters to reproduce. By maintaining the lobster population, the state is attempting to further the legitimate interest of reviving its faltering lobster fishing industry, rather than trying to protect a local business against interstate competition. In addition, the statute is applied evenhandedly (i.e., it does not merely regulate the activities of out-of-state lobster fishers while exempting in-state fishers from those same regulations). Therefore, the statute does not discriminate against interstate commerce. The statute may impose some incidental burden on interstate commerce by requiring out-of-state lobster fishers to refrain from taking lobsters weighing less than one pound from lobster beds within three miles of the state shoreline. However, this burden should not result in any great difficulty for out-of-state lobster fishers who wish to catch lobsters in state Blue waters. There is nothing especially burdensome or restrictive about the statute's provisions, and certainly the incidental burden that does exist does not outweigh the state's legitimate interest in maintaining its lobster population and the vitality of its lobster fishing industry. Consequently, the statute is not unduly burdensome. Because the statute is not unduly burdensome and is nondiscriminatory against out-of-state competition, it does not violate the Commerce Clause. (B) is incorrect because it is overbroad. The Commerce Clause does apply even when a state seeks to protect natural resources
Which of the following interests in property are subject to the Rule Against Perpetuities? A Contingent remainders, executory interests, and vested remainders subject to open B Reversions, options, and class gifts C Executory interests, rights of entry, and powers of appointment D Contingent remainders, possibilities of reverter, and rights of first refusal
A.
Which of the following statements regarding specific performance of a land sale contract is true? A Both the buyer and the seller generally are entitled to specific performance B If the seller cannot convey marketable title, the buyer may not obtain specific performance C Specific performance is available only to the buyer D Specific performance is available only to the seller
A.
The police obtained a valid arrest warrant for a drug dealer. A reliable informant told the police that the drug dealer was staying at a friend's house until "the heat was off." Without having obtained a search warrant, the police went to the friend's house, knocked on the door, and asked the friend if the drug dealer was there. The friend replied that the drug dealer had been staying at the house for a few days but had left a few hours ago. The police pushed open the door and began searching for the drug dealer. They found him hiding in a closet along with two five-pound bricks of marijuana. They arrested both the drug dealer and the friend. Before his trial for possession of marijuana, the friend moved to suppress the marijuana found in the closet. Should the court grant the motion to suppress? A Yes, because a search warrant was required. B Yes, because the police may not execute an arrest warrant at the third party's home. C No, because the police had probable cause to believe that the drug dealer was staying at the friend's home. D No, because the police had a valid arrest warrant and the marijuana was found incident to the arrest.
A. The court should grant the motion to suppress because a search warrant was required. Absent exigent circumstances, the police executing an arrest warrant may not search for the subject of the warrant in the home of a third party without first obtaining a separate search warrant for the home. If the police do execute an arrest warrant at the home of a third party without obtaining a search warrant for the home, the arrest is still valid but evidence of any crime found in the home cannot be used against the owner of the home because it is the fruit of an unconstitutional search. Thus, (A) is correct and (D) is incorrect. (B) is incorrect because it is too broad. A person can be arrested at the home of a third party, but the police generally cannot enter the third party's home without consent unless they have a search warrant for the home. (C) is incorrect because, as discussed above, a search warrant is required absent exigent circumstances, which are not present in this case. Here, the probable cause established by the informant's disclosure would have enabled the police to obtain a search warrant.
For which type of security interest in land does the debtor transfer title to a third party acting on behalf of the lender? A Deed of trust B Installment land contract C Absolute deed D Equitable mortgage
A. A deed of trust is a security interest in land by which the debtor (i.e., the trustor) transfers title to the land to a third party (i.e., the trustee), such as the lender's lawyer or a title insurance company, acting on behalf of the lender (i.e., the beneficiary). In the event of default, the lender instructs the trustee to foreclose the deed of trust by selling the property. An equitable mortgage exists if a court concludes that a grantor transferred an absolute deed to serve as security for an obligation. If the court so determines, the grantee must foreclose by judicial action, as with any other mortgage. The court will consider: (i) The existence of a debt or promise of payment by the grantor; (ii) The grantee's promise to return the land if the debt is paid; (iii) Whether the amount advanced to the grantor was much lower than the value of the property; (iv) The degree of the grantor's financial distress; and (v) The parties' prior negotiations. An installment land contract is a security interest in land in which the debtor (i.e., the buyer) contracts with the seller to pay for the land in regular installments until the full contract price has been paid, plus interest. Only then will the seller transfer legal title to the buyer. The contract may contain a forfeiture clause providing that the seller may cancel the contract upon default, retain all money paid, and retake possession of the land.
When may a mortgagor redeem her land in equity? A Before the foreclosure sale. B Only during the foreclosure sale. C After the foreclosure sale. D At any time, unless waived in the mortgage itself.
A. A mortgagor may redeem her land in equity before the foreclosure sale. At any time prior to the foreclosure sale—i.e., this right does not exist only during the foreclosure sale—the mortgagor has the right to redeem the land or free it of the mortgage by paying off the amount due, plus interest. If the mortgage or note contained an acceleration clause, which permits the mortgagee to declare the full balance due in the event of default, the full balance must be paid in order to redeem. A mortgagor may not redeem her land in equity after the foreclosure sale. However, about half the states provide a statutory right to redeem—distinct from the equitable right discussed above—for some fixed period after the foreclosure sale has occurred (e.g., six months or one year). While a mortgagor may redeem her land in equity at any time before the foreclosure sale, this right cannot be waived in the mortgage itself. Doing so is known as "clogging the equity of redemption" and is prohibited. However, the right can be waived later, for consideration.
A witness subpoenaed to testify before a grand jury has no right to: A Counsel in the courtroom, Miranda warnings, or warnings that she is a potential defendant B Warnings that she is a potential defendant, but she must be provided counsel in the courtroom and Miranda warnings C Counsel in the courtroom or Miranda warnings, but she must be warned if she is a potential defendant D Counsel in the courtroom, but she must be provided Miranda warnings and warned if she is a potential defendant
A. A witness subpoenaed to testify before a grand jury has no right to receive Miranda warnings. A grand jury witness also has no right to have an attorney present, but she may consult with an attorney outside the grand jury room. A witness who is under investigation and may well become a defendant has no right to a warning that she is a "potential defendant" when called to testify before the grand jury.
On April 15, a wholesaler of tulip bulbs telephoned a local nursery and offered to sell to the nursery 80 gross of tulip bulbs for $8,000, not including delivery charges. The nursery accepted immediately. On April 17, the nursery sent the wholesaler an email confirming the deal for the sale of 80 gross of tulip bulbs for $8,000, and stating that it anticipated a waiver of the delivery charges because of the size of the order. On May 3, the wholesaler telephoned the nursery and stated that, due to a poor growing season for tulips, it would not be able to supply any tulip bulbs to the nursery. If the nursery brings suit against the wholesaler and the wholesaler asserts the Statute of Frauds as a defense, will the nursery prevail? A Yes, because its April 17 email contained the quantity term. B Yes, because its April 17 email contained the price term. C No, because the nursery's April 17 email varied the terms of the wholesaler's offer. D No, because the wholesaler is the party to be charged and has signed nothing.
A. Because the quantity was stated in the April 17 email, the Statute of Frauds is satisfied and the nursery may prevail. This contract is for the purchase and sale of goods; thus, the UCC applies. The Statute of Frauds requires that a contract for the sale of goods for $500 or more be evidenced by a writing signed by the party to be charged. This writing must contain the essential elements of the agreement. The quantity term is the key to the sufficiency of a memorandum, and here the writing includes the quantity term. Thus, the writing complied with the Statute of Frauds. (B) is wrong because all other terms (including price) may be proved by parol evidence. The UCC requires only that the memorandum contain (i) quantity, (ii) the signature of the party to be charged, and (iii) a writing sufficient to indicate that a contract was formed. [UCC §2-201] (C) is wrong because it does not bear on the Statute of Frauds issue, but rather on the issue of the additional terms, which will not prevent a contract from being formed between merchants. (D) is wrong because UCC section 2-201(2) provides that, in a deal between merchants, a writing confirming the deal sent by one party will bind both parties, unless the other party objects in writing within 10 days. Here, the wholesaler did not object within 10 days, and so the nursery's email confirmed the deal. Thus, the wholesaler can be charged even though the wholesaler has not signed the memorandum.
A homeowner borrowed $50,000 from a bank, secured by a mortgage on his home. Shortly thereafter, the homeowner sold his home to a buyer for $70,000 by a deed containing a recital signed by both parties that title passed "subject to" the bank's mortgage, "which obligation grantee expressly assumes." The buyer paid the homeowner $20,000, took possession of the house, and began making monthly payments of principal and interest to the bank. A few years later, a chemical manufacturing firm built a huge sulfur processing plant just down the road from the home, which caused the house to immediately decline in value to $35,000. Subsequently, the buyer stopped making the monthly payments to the bank. The bank exercised its contractual right of nonjudicial foreclosure and sold the house at a public auction for $34,000. The bank then brought suit against the homeowner and the buyer for $14,000, the difference between the proceeds of the foreclosure sale and the $48,000 principal remaining due on the original loan to the homeowner. The jurisdiction does not bar deficiency judgments. Against whom should the bank be granted a judgment for $14,000? A Both the homeowner and the buyer. B Only the homeowner. C Only the buyer. D No one.
A. Both the homeowner and the buyer are liable for the deficiency. If a sale of foreclosed property does not bring enough to satisfy the mortgage debt, the mortgagee/lender can bring a personal action against the mortgagor/debtor for the deficiency (as long as the jurisdiction does not bar deficiency judgments). When the mortgagor sells the mortgaged property and gives a deed, the grantee takes subject to the mortgage, which remains on the land. If the grantee does not sign an agreement to assume the mortgage, he does not become personally liable on the loan, and the original mortgagor remains personally liable. If the grantee does sign an assumption agreement, however, the lender is considered a third-party beneficiary of the agreement, and hence may recover from the assuming grantee, who is primarily liable, or the original mortgagor, who is secondarily liable. Here, the buyer signed the recital providing for the assumption, so she will be personally liable on the loan. Therefore, (A) is correct, and (B) is incorrect. (C) is incorrect because the homeowner, the original mortgagor, did not extinguish his own personal liability on the loan by obtaining the assumption agreement from the buyer. He remains secondarily liable as a surety. Thus, the bank may sue the homeowner on the original mortgage agreement. (Note that while the bank may obtain a judgment against both of them, its maximum recovery will be the $14,000 deficiency.) (D) is incorrect because the facts indicate that the jurisdiction does not bar deficiency judgments.
Which of the following acts will terminate an easement? A Condemnation of the servient estate. B Use of the easement beyond its legal scope. C Nonuse of the easement for the statutory period. D Voluntary destruction of the servient estate.
A. Condemnation of the servient estate will terminate an easement. The easement holder may be entitled to compensation for the value lost. Use of the easement beyond its legal scope will not terminate an easement. Instead, the easement is surcharged, and the servient owner may sue to enjoin the use. Nonuse of the easement for the statutory period will not terminate an easement. An easement can be extinguished by the easement holder's physical act of abandonment (e.g., erection of a permanent structure over the easement). However, mere nonuse, even for a long period of time, is insufficient to constitute an abandonment of the easement. To terminate the easement, the nonuse must be combined with other evidence of intent to abandon it. Voluntary destruction of the servient estate (e.g., tearing down a building to erect a new one) will not terminate an easement. On the other hand, involuntary destruction of the servient estate (e.g., by fire or flood) will extinguish the easement.
Which of the following is NOT required for the benefit of a real covenant to run to successors in interest? A There is horizontal privity between the original covenanting parties. B There is vertical privity between the covenantee and her successor in interest. C The covenant touches and concerns the land. D The covenanting parties intended that successors in interest be benefitted by the covenant.
A. Horizontal privity between the original covenanting parties is not required for the benefit of a real covenant to run to successors in interest. A real covenant is a written promise to do or not to do something on the land. The benefit of the covenant will run to successors in interest if: 1. The covenanting parties intended that successors in interest be benefitted by the covenant; 2. There is vertical privity between the covenantee and her successor n interest; and 3. The covenant touches and concerns the land (i.e., it benefits the covenantee and her successor in their use and enjoyment of the benefited land). For the burden of a real covenant to run to successors in interest, vertical privity requires that the successor in interest to the covenanting party hold the entire durational interest held by the covenantor at the time he made the covenant. On the other hand, vertical privity for the running of the benefit is satisfied when the successor in interest holds any possessory estate, even a lesser estate.
An accountant employed by the Federal Communications Commission was offended by various jokes and cartoons that employees would post in the office cafeteria. The Commission did not have any rules regarding what employees could post in the cafeteria, and none of the cartoons were pornographic or harassing. Nevertheless, the accountant lodged a number of complaints with his supervisor that went unheeded. Finally, the accountant posted his own notice chastising the hypocrisy and immorality of the agency for allowing such cartoons when it was charged with ensuring a standard of decency on the public airwaves. The notice prompted a great deal of debate among employees and a great deal of displeasure on the part of the accountant's supervisor, particularly after it was posted on another employee's blog and received some media attention. A labor contract between the agency and the clerical workers' union contained a policy for providing for termination of union employees only for certain specified grounds, but the accountant was not a member of the union and was not covered by the policy or any other employment agreement. Which of the following statements is most accurate regarding the agency's right to dismiss the accountant? A The accountant has a liberty interest in the exercise of his First Amendment rights that entitles him to a hearing to contest the grounds of his dismissal. B The accountant has a property interest as a public employee that precludes him from being fired without notice and an opportunity to respond. C The accountant has no right to a hearing because his statements were not an expression of views on public issues. D The accountant has both a liberty interest and a property interest that entitles him to a pretermination evidentiary hearing.
A. If the accountant is fired, he has a right to a hearing to determine whether his First Amendment rights were violated by his dismissal. Under the Due Process Clause of the Fifth Amendment, a person has a liberty interest in the exercise of specific rights provided by the Constitution, including freedom of speech. A government employee may not be fired for expressing his views regarding public issues, but can be fired for speech that disrupts the employer's policies or undermines the employer's authority. Under the Court's expansive interpretation of what a public issue is in this context [see Rankin v. McPherson (1987)], the accountant's statement would probably qualify. At the very least, he can make enough of a showing that his termination violates his free speech rights to be entitled to a hearing on the issue under procedural due process principles. [See Givhan v. Western Line Consolidated School District (1979)] (B) is wrong because the accountant does not appear to have a property interest in his job. A public employee who is subject to removal only for "cause" has a property interest in his job and must be given notice of the charges against him that are to be the basis for his job termination, and a pretermination opportunity to respond to those charges. Here, however, the accountant did not have a property interest in his job. He could have been dismissed for no reason at all. He was not covered by the labor contract between the agency and its clerical workers, and there appears to be no other basis for him to claim an entitlement to continued employment. (C) is wrong because the accountant is entitled to a hearing as long as he can raise a prima facie claim that his speech, which was regarding an issue important to the perception of his agency, was on a public issue and therefore protected by the First Amendment. (D) is wrong for two reasons: As discussed above, the accountant does not have a property interest in his job. Also, due process does not necessarily entitle him to a pretermination evidentiary hearing; a post-termination evidentiary hearing is probably sufficient. [See Cleveland Board of Education v. Loudermill (1985)]
A landowner and a purchaser orally agreed that the landowner would convey 20 acres of his 160-acre farm to the purchaser. At the time of their agreement, the landowner wrote on the back of an envelope, "I hereby promise to convey the northern 20 acres of my farm to [the purchaser] for $10,000." One month later, the purchaser tendered $10,000 to the landowner, but the landowner refused to convey the 20 acres. If the purchaser sues the landowner to convey the land and the landowner prevails, what will be the most likely reason? A The writing was not signed by the landowner. B The writing was not signed by the purchaser. C The writing did not describe the property with specificity. D The writing was on the back of an envelope.
A. If the landowner prevails, it will be because the writing was not signed by the landowner. Under the Statute of Frauds, to be enforceable a contract for the sale of land must be evidenced by a writing signed by the party sought to be charged. Here, the landowner is the party that the purchaser is seeking to charge, so his signature is required on the writing. (B) is wrong because the purchaser's signature is not required to bind the landowner. (C) is wrong because the contract need only reasonably describe the subject matter; great specificity, such as a legal description, is not required. Nothing in the facts suggests that "the northern 20 acres of my farm" is not an adequate description, which makes (C) a less certain reason than (A) why the landowner would prevail. (D) is wrong because it does not matter on what substance the writing is made.
In determining whether government action is valid under the Establishment Clause, courts will usually consider all of the following except whether: A The action is one of general applicability B The action has a secular purpose C The action has a primary effect that neither advances nor inhibits religion D The action does not produce excessive government entanglement with religion
A. In determining whether government action is valid under the Establishment Clause, courts will usually not consider whether the action is an action of general applicability—that is more related to a free exercise case. In Establishment Clause cases, the courts usually use the three-part Lemon test, which is reflected in the other answer choices: 1. The action has a secular purpose, 2. The action has a primary effect that neither advances nor inhibits religion, and 3. The action does not produce excessive government entanglement with religion.
Under the Commerce Clause: A Congress can adopt laws discriminating against interstate commerce and so can states if authorized by federal law B Neither Congress nor the states may adopt laws discriminating against interstate commerce C Congress can adopt laws discriminating against interstate commerce, but states cannot D The states can adopt laws discriminating against interstate commerce but Congress cannot
A. It is true that Congress can adopt laws discriminating against interstate commerce and so can states if authorized by federal law. Congress's power over interstate commerce is plenary; i.e., Congress can adopt a law discriminating against interstate commerce as long as the law does not violate some other constitutional provision. And although the states generally may not adopt laws that discriminate against interstate commerce, they may do so if authorized by federal law. From the above, it follows that it is incorrect that neither Congress nor the states may adopt laws discriminating against interstate commerce. Congress may freely do so and the states may do so with federal authorization. It is incorrect that Congress can adopt laws discriminating against interstate commerce, but states cannot because this statement fails to recognize the fact that the states may adopt laws discriminating against interstate commerce with federal authorization. Finally, it is not true that the states can adopt laws discriminating against interstate commerce but Congress cannot. The opposite generally is true: generally Congress can adopt laws discriminating against interstate commerce but states cannot.
What does the Fourteenth Amendment Privileges or Immunities Clause protect? A The rights of national citizenship B The rights of corporations C The rights of aliens D The rights of legal residents
A. The Fourteenth Amendment Privileges or Immunities Clause prohibits states from denying their citizens the rights of national citizenship, such as the right to petition Congress for redress of grievances, the right to vote for federal officers, the right to enter public lands, the right to interstate travel, and any other right flowing from the distinct relation of a citizen to the United States Government. Corporations, aliens, and legal residents are not citizens of the United States and are not protected by the Fourteenth Amendment Privileges or Immunities Clause.
Three drivers were in an automobile accident in a city in State A. The drivers were citizens of State A, State B, and State C. The State B driver filed a tort action against the other two in a State A state court, seeking $300,000 for her severe injuries. The State C driver wants to remove the action to a federal district court. Is the action removable? A No, because one of the defendants is a citizen of State A. B No, because an action may be removed from state court only if it "arises under" federal law. C Yes, because there is complete diversity of citizenship and the amount in controversy exceeds $75,000. D Yes, because one of the defendants is a citizen of a state other than State A.
A. The action is not removable. Under 28 U.S.C. section 1441, a defendant may remove an action that could have originally been brought in the federal courts. (In other words, subject matter jurisdiction based on either a federal question being presented or on diversity of citizenship would have been present had the case been filed in federal court.) However, a case may not be removed on the basis of diversity jurisdiction if a defendant is a citizen of the state in which the action was filed. Here, the action was filed in State A against a State A defendant; thus, the case may not be removed, even though it is a State C defendant who is seeking removal. This fact also makes (D) incorrect. (B) is incorrect because a case may be removed based on diversity, with the restriction that removal is not available if one of the defendants is a citizen of the forum state. (C) is incorrect. Even though the case satisfies the requirements of diversity jurisdiction, the "in-state defendant" restriction prevents removal.
A buyer bought a home from a real estate developer for $700,000. The buyer paid $100,000 of the purchase price herself. The buyer's employer provided $100,000 of the purchase price by giving the buyer a loan and taking a mortgage. The developer loaned $500,000 to the buyer to finance the remainder of the purchase price, and in return took a mortgage on the property. One week later, a bank obtained a judgment against the buyer for a delinquent credit card balance. The bank properly recorded its judgment as a lien against the property.Another month after that, the buyer incurred some extraordinary medical expenses, and asked the employer for another $100,000, which the employer provided and added onto the principal balance the buyer owed on the loan. Finally, six months later, the buyer asked the developer to change the terms of the loan, so that the buyer would have more time to pay. The developer and the buyer agreed that the buyer could have an additional five years to pay the balance of the loan in exchange for an increase in the principal of the loan. Shortly thereafter, the buyer lost his job and defaulted on all of his payments. The employer brought an action to foreclose its mortgage. All mortgages and liens were promptly and properly recorded. Regarding the distribution of the proceeds of an eventual sheriff's sale of the property, which of the following statements is true? A The bank is paid in full before the developer is paid in full. B The employer is paid in full before the bank receives any proceeds. C The developer is paid in full before the employer receives any proceeds. D The developer is paid in full before the employer is paid in full.
A. The bank will be paid in full before the developer is paid in full. Generally, the priority of mortgages is chronological. A number of other factors, however, may affect priority. Where a seller of property receives a mortgage as part of the purchase price, a purchase money mortgage results. Purchase money mortgages may also arise when a third party lends money to the buyer for the purchase of property and takes a mortgage on the property in return. In general, the seller's purchase money mortgage will take priority over the third-party purchase money mortgage. Purchase money mortgages, however, are subject to later liens by virtue of recording acts. In the case where a mortgage is modified by agreement between the parties, any increase in the debt resulting from the modification will be subject to a junior lien, even if the original mortgage itself had priority over the junior lien. In the same way, an optional (as opposed to an obligatory) advance that is made after the junior lien will have a lower priority than the junior lien. Again, this is the case even if the original mortgage is first in priority. Therefore, the distribution of sale proceeds in this case would be: (i) the original amount of the employer's purchase money mortgage, (ii) the bank's judgment, (iii) the $100,000 advance by the employer, and finally, (iv) the amount of the increase in the debt to the developer due to the agreed modification of the principal of the original loan. The original unmodified purchase money mortgage of the developer would remain on the land because it was senior to the mortgage being foreclosed (the employer's). (B) is incorrect because the employer would not be paid in full before the bank received payment. (C) is incorrect because the developer would initially receive only the increased amount of the debt according to the modified loan terms. Also, the developer would have to wait until the bank judgment was satisfied and the employer was paid in full before the developer's claim for the modification amount could be paid. (D) is incorrect because the developer would not be paid in full—its original purchase money mortgage would remain on the property after the foreclosure.
A plumber working for a company providing plumbing services to commercial and industrial establishments was required to be "on call" for emergency plumbing services 24 hours a day, and was required to drive his company van home each night so he would have all of his tools and equipment at hand for any calls. However, he was not permitted to use the company van for personal errands. On his way home one afternoon, he took a detour toward a supermarket a few blocks away to pick up some items for dinner. While entering the supermarket parking lot, he drove negligently and struck a pedestrian, seriously injuring him. The pedestrian filed suit against the plumber's company in a jurisdiction that maintains traditional common law rules regarding contribution and indemnity, and the jury awarded him $100,000 in damages, which the company paid. If the company sues the plumber to recoup its loss in the lawsuit, which party will prevail? A The company can recover 100% of the judgment as an indemnity, because the plumber was negligent, not the company. B The company will prevail, because the company had a rule against using company vehicles for personal errands. C The company will not prevail, because the company has already been found liable under principles of vicarious liability in the lawsuit by the pedestrian. D The company will not prevail, because the company required the plumber to be "on call" 24 hours a day
A. The company can recover 100% of the judgment under common law indemnity rules. The principle of indemnity permits a shifting between the tortfeasors of the entire loss (i.e., the payment made to satisfy plaintiff's judgment). This is in contrast to contribution, which apportions the loss among those who are at fault. Indemnity is available in vicarious liability situations, where one party is held liable for damages caused by another simply because of his relationship to that person. Hence, an employer such as the plumber's company that has been held vicariously liable under the doctrine of respondeat superior can obtain indemnification from the employee (the plumber) whose conduct actually caused the damage. (B) is incorrect because the company need not show that the plumber breached a company rule before it can obtain indemnity. The fact that the plumber's negligence caused the injury and that the company was liable for the judgment solely because of its relationship to the plumber permits indemnification here. (C) is incorrect because vicarious liability is one of the most common areas where indemnity is available. (D) is incorrect because the company's requirement that the plumber be on call 24 hours a day merely establishes that the company will be vicariously liable for the plumber's negligence; it does not bar the company from recovering from the plumber because the plumber's negligence actually caused the damage.
A pedestrian was injured in a car accident involving two cars. The pedestrian filed a negligence action in federal district court against the first driver, seeking $100,000 in damages. The pedestrian is a citizen of State A and the first driver is a citizen of State B. The first driver then filed a third-party claim against the second driver, claiming that the second driver is responsible for half of the harm caused to the pedestrian and seeking to recover half of any liability the first driver is found to have to the pedestrian. The second driver is a citizen of State A. Does the federal court have subject matter jurisdiction over the third-party claim asserted by the first driver against the second driver? A Yes, because the court has supplemental jurisdiction over the third-party claim. B Yes, because the court has diversity of citizenship jurisdiction over the third-party claim. C No, because the amount in controversy in the third-party claim is too small. D No, because complete diversity of citizenship is lacking.
A. The court has supplemental jurisdiction over the third-party claim. Diversity of citizenship jurisdiction is available when (i) there is complete diversity of citizenship, meaning that each plaintiff must be a citizen of a different state from every defendant; and (ii) the amount in controversy exceeds $75,000. In the instant case, the case is properly in federal court because diversity jurisdiction exists for the underlying claim (i.e., the claim by the pedestrian against the first driver), given that the pedestrian is from State A, the first driver is from State B, and the amount in controversy is $100,000. The third-party indemnity claim, however, cannot invoke diversity jurisdiction, even though the first driver is from State B and the second driver is from State A, because the amount claimed is $50,000. This makes (B) an incorrect answer choice. When the federal court has subject matter jurisdiction over one claim, it has discretion to exercise supplemental jurisdiction over related claims that derive from the same common nucleus of fact and are such that a plaintiff would ordinarily be expected to try them in a single judicial proceeding. (Essentially, this means that the supplemental claim must arise from the same common nucleus of operative fact as the claim invoking federal subject matter jurisdiction.) In the instant case, the claims of pedestrian (the underlying claim) vs. first driver and of first driver vs. second driver (the indemnity claim) are derived from the same accident, and thus both are derived from the same common nucleus of operative fact. There are restrictions on the use of supplemental jurisdiction when the use of supplemental jurisdiction would be contrary to diversity jurisdiction. In terms of third-party practice, claims by a plaintiff against an impleaded party may not use supplemental jurisdiction to circumvent the diversity statute. However, claims by a defendant are not listed among the restrictions; thus, supplemental jurisdiction is available to a defendant (third-party plaintiff) against a third-party defendant. Therefore, the first driver may use supplemental jurisdiction to have his claim against the second driver heard in federal court. (This would be true even if they were from the same state, so long as the claim was a true indemnity claim.) As a result, (A) is correct, and (C) and (D) incorrect.
To help alleviate discrimination in private contracts, Congress passed a bill providing: "It shall be unlawful to discriminate against minority race members in the making and enforcement of any public or private contract, of every kind whatsoever. Any person whose rights under this statute are violated may bring a cause of action against the party that has so violated the person's rights in the federal district court for the district in which he resides, seeking treble damages or $1,000, whichever is greater." Several large banks that have been accused of discriminatory loan practices challenge the federal statute. If the court finds that Congress had the power to enact the statute, the court most likely will find that the power arose from which of the following? A The Thirteenth Amendment. B The Contract Clause. C The Fourteenth Amendment. D The Commerce Clause.
A. The court most likely will find that Congress had the power to enact the legislation under the Thirteenth Amendment. The Thirteenth Amendment simply provides that neither slavery nor involuntary servitude shall exist within the United States and gives Congress the power to adopt appropriate legislation to enforce the proscription. Since the amendment is not limited to proscribing state action, Congress may adopt legislation regulating private parties. Under the amendment, the Supreme Court has allowed Congress to prohibit any private conduct that Congress deems to be a "badge" or "incident" of slavery, and has upheld statutes regulating private contracts. [See, e.g., Runyon v. McCrary (1967)] (B) is not a good basis for the statute because the Contract Clause is a limitation on states' rights to modify contracts retroactively; it is unrelated to Congress's power to regulate private contracts. (C)—the Fourteenth Amendment—is incorrect. The Fourteenth Amendment prohibits states from discriminating on the basis of race; it does not extend to private conduct. [See United States v. Morrison (2000)] (D)—the Commerce Clause—might also be a basis for the legislation here, but it is not as good an answer as (B) because the commerce power is limited to transactions that either in themselves or in combination with other activities have a substantial economic effect on interstate commerce, and by its terms the legislation here can reach wholly intrastate transactions. The interstate commerce requirement is a limit on congressional legislation and no such limit is present under the Thirteenth Amendment. Therefore, the Thirteenth Amendment is a better basis for the legislation here.
The police obtained a valid arrest warrant for a drug dealer. A reliable informant told the police that the drug dealer was staying at a friend's house until "the heat was off." Without having obtained a search warrant, the police went to the friend's house, knocked on the door, and asked the friend if the drug dealer was there. The friend replied that the drug dealer had been staying at the house for a few days but had left a few hours ago. The police pushed open the door and began searching for the drug dealer. They found him hiding in a closet along with two five-pound bricks of marijuana. They arrested both the drug dealer and the friend. Before his trial for possession of marijuana, the friend moved to suppress the marijuana found in the closet. Should the court grant the motion to suppress? A Yes, because a search warrant was required. B Yes, because the police may not execute an arrest warrant at the third party's home. C No, because the police had probable cause to believe that the drug dealer was staying at the friend's home. D No, because the police had a valid arrest warrant and the marijuana was found incident to the arrest.
A. The court should grant the motion to suppress because a search warrant was required. Absent exigent circumstances, the police executing an arrest warrant may not search for the subject of the warrant in the home of a third party without first obtaining a separate search warrant for the home. If the police do execute an arrest warrant at the home of a third party without obtaining a search warrant for the home, the arrest is still valid but evidence of any crime found in the home cannot be used against the owner of the home because it is the fruit of an unconstitutional search. Thus, (A) is correct and (D) is incorrect. (B) is incorrect because it is too broad. A person can be arrested at the home of a third party, but the police generally cannot enter the third party's home without consent unless they have a search warrant for the home. (C) is incorrect because, as discussed above, a search warrant is required absent exigent circumstances, which are not present in this case. Here, the probable cause established by the informant's disclosure would have enabled the police to obtain a search warran
On August 1, the plaintiff, a resident of State A, sued two defendants in State A for personal injuries arising out of an automobile accident. One defendant is a citizen of State A while the other is a citizen of State B. The lawsuit claimed damages of $500,000. The plaintiff quickly reached a settlement agreement with the defendant from State A, and the court dismissed that defendant by order on August 16. The order is served on the remaining defendant on August 20. On September 18, the remaining defendant files a notice of removal with the court, which the plaintiff opposes. How should the court rule on the defendant's notice of removal? A For the remaining defendant, because she filed her notice of removal within 30 days after she discovered the case had become removable. B For the remaining defendant, because there are no time restrictions on removing a case to federal court. C For the plaintiff, because a plaintiff has the right to choose his own forum. D For the plaintiff, because more than 30 days have passed since the case became removable.
A. The court should rule for the remaining defendant. There are essentially two time restrictions on removal of a diversity case to federal court: (1) a case based on diversity must be removed within 30 days of the defendant's receipt of a copy of the paper (order, motion, etc.) that makes the case removable; but (2) in no event may the case be removed more than one year after it was commenced in state court. [28 U.S.C. §1446] Here, it is the 30-day time limit that is in question; i.e., specifically, whether the 30-day clock starts to tick on August 16 (when the case became removable) or on August 20 (when the defendant learned that the case became removable). As stated above, it is the latter. The clock starts to tick when the defendant learns by service of any paper that the case has become removable. Because she requested removal within this 30-day period, the court should rule in favor of the remaining defendant.
The defendant is charged with the battery of a bouncer at a local tavern. At the trial, the prosecutor introduces evidence that while the bouncer was attempting to question the defendant about her intoxicated demeanor, the defendant committed a battery on the bouncer. The defendant attempts to defend against the charge on the basis of self-defense, insisting that the bouncer used excessive force in stopping her from entering the tavern. The defendant attempts to introduce into evidence an authenticated copy of the tavern records that show that three patrons had written complaints against the bouncer within the past six months for the use of excessive force. The prosecutor objects on the grounds that the records are inadmissible character evidence. Should the court sustain the objection? A Yes, because the character of a victim can be established only by reputation or opinion evidence. B Yes, because there is no evidence that the incidents involving the three patrons were based on the same facts as the defendant's claim. C No, because the records were authenticated. D No, because the character trait of a victim may be established by opinion evidence, reputation evidence, or specific acts of misconduct.
A. The court should sustain the objection because the records are evidence of specific bad acts. The Federal Rules permit a defendant to introduce evidence of a bad character trait of the alleged victim if it is relevant to the charge or the defense, but limit it to reputation and opinion evidence. Evidence of specific acts of the person in question that demonstrates that person's character is permitted only in a few instances, such as if the acts are relevant to some issue other than disposition to commit the crime charged. Here, no issue is raised by this evidence other than the bouncer's propensity to use excessive force. (A) is therefore correct and (D) is wrong. (B) is wrong because the facts do not have to be identical if evidence of bad acts were otherwise admissible. (C) is wrong; documentary evidence, even if fully authenticated and relevant, may be excluded if it violates a rule of competency, such as the rule for character evidence. Here, the objection should be sustained because the document is improper evidence of a specific bad act.
The defendant and the victim got into a minor verbal altercation, concluding with the defendant lightly shoving the victim. The victim lost his balance and struck his head on the pavement, causing serious bodily injury. The defendant was charged with battery, which is defined in the jurisdiction as "purposely or knowingly causing serious bodily injury to another." Should the defendant be convicted of battery? A No, because the defendant did not know that the victim would be seriously injured. B No, because the defendant did not strike a serious blow to the victim. C Yes, because the defendant purposely shoved the victim. D Yes, because the victim suffered serious bodily injury.
A. The defendant should not be convicted of battery. Under the statute's fault standards, a defendant must have acted purposely (i.e., with conscious intent to cause the result) or knowingly (i.e., with knowledge that his conduct will necessarily or very likely cause the result) as to the harmful result. The apparent inference to be drawn from the facts is that the defendant did not consciously desire, nor contemplate to a practical certainty, the serious injury to the victim that actually occurred. Had the defendant intended to cause such severe harm, he no doubt would have dealt the victim a strong blow rather than simply giving the victim a light shove. Therefore, as to the nature of the result, the defendant did not act with "purpose" or "knowledge" as those terms are defined in the Model Penal Code and modern criminal codes. (B), while close, is not as good an answer as (A) because it does not address the state of mind issue in the problem. A light shove might be sufficient for a battery as defined under a different set of facts (e.g., if the defendant believes that the victim would fall down stairs with a light shove). (C) is incorrect because it addresses the act but not the result. As defined in this question, battery must not only be committed by a purposeful act, but also be done with a "purposeful" or "knowing" state of mind as to the result. (D) is incorrect for much of the same reason—the state of mind requirement also applies to the result, as discussed above. The injury must have been purposely or knowingly caused, and that concept is not contained within choice (D).
Using his cellphone, a witness recorded a speeding driver hitting a pedestrian. The witness sold the recording to the driver. The driver then gave the recording to his attorney. After the pedestrian filed suit against the driver, the pedestrian sent a discovery request to the driver requesting that he produce "all items that show or describe the accident." Which of the following best describes whether the driver must provide the video, or a copy of it? A The driver must provide the video because it is relevant to the pedestrian's claim. B The driver need not provide the video because, although relevant to the pedestrian's claim, it would be damaging to the driver's defense. C The driver need not provide the video because he paid money for it, entitling him to possess it. D The driver need not provide the video unless the pedestrian can show substantial need and the inability to obtain the equivalent without undue hardship, because the video constitutes work product.
A. The driver must provide the video. Parties are entitled to discovery that fits under Rule 26(b) (1), which includes "any nonprivileged matter that is relevant to any party's claim or defense." In other words, even if the matter is only relevant to the opposing party, it would still be covered. Additionally, Rule 34 requires a party to produce relevant physical material, including electronically stored information, such as the recording here. There is no exception to relevance for matter that is damaging to a party, so (B) is incorrect. (Being damaging to the defendant's defense might be reason not to disclose it as an initial disclosure because the defendant would not use the recording to support his defense, but the recording would have to be disclosed on a proper request.) (C) is also incorrect. There is no exception from the scope of discovery for items that were purchased; discovery encompasses all items in a party's possession or control. Furthermore, although the recording is in the attorney's possession, this should be interpreted as under the party's control. (D) is incorrect because, while it states the rule for when work product must be provided, there is no work product involved. Mere possession by an attorney of factual information does not constitute work product that is exempt from discovery. Rather, work product is material created by a party or a representative (such as an attorney) prepared in anticipation of litigation. This recording was not created by the party or any representative of the party, and therefore it is not work product.
A man and a woman were traveling in the man's car when they were stopped by the police for running a red light. Before the police came up to the car, the man told the woman, "You owe me a favor. Keep this package for me," and gave the woman a small foil package. The woman put the package in her backpack, saying, "O.K., but don't tell me what's in it." Before the police even began to question the occupants, the man blurted out, "I'm clean, man, but she has a stash," pointing at the woman. The officers searched the backpack that the woman was holding and found the foil package, which contained heroin. The woman was arrested, but the man was not. Is the evidence found on the woman admissible? A Yes, under the automobile exception. B Yes, because due process imputes knowledge where there is willful ignorance. C No, because due process forbids granting of immunity to the more culpable defendant. D No, because the woman did not know that the package contained heroin.
A. The evidence is admissible because the search was valid. Even though the police have validly stopped an automobile, they cannot search the vehicle without meeting the requirements of one of the exceptions to the warrant requirement, such as the automobile exception (which requires probable cause) or consent. The automobile exception comes into play when the police have probable cause to believe that the vehicle contains evidence of a crime. Under the exception, the police may search anywhere in the vehicle in which the item for which they have cause to search may be hidden, including packages in the vehicle. The statement of the man to the police officers gave them probable cause to believe that the car contained evidence of a crime (i.e., that the woman had drugs somewhere in the car). Thus, the requirement for application of the automobile exception was present, providing validity for the warrantless search conducted by the police. Because the search was valid, the evidence found on the woman is admissible. Besides being an incorrect statement of law, (B) is incorrect because it focuses on the woman's knowledge of the contents of the package. Whether the woman knew that heroin (or some other illegal substance) was in the package is irrelevant to the admissibility of the heroin. Even assuming that the woman knew of the contents, the search would not be valid unless there was a ground for the warrantless search
For many years, civil service rules have provided that any member of the city's police department must serve a one-year probationary period before he or she will be considered a permanent employee. However, because the rules were enacted before the city's police academy was established, a prospective police officer now spends six months in the academy before being hired by the city. A graduate of the police academy was with the city police department for eight months after graduation when she was terminated. There were no city ordinances or state laws that required that she be given either a reason for the termination or a hearing, and she was given neither. The graduate brought suit against the city in state court because of the termination of her employment, alleging a violation of her due process rights. Which of the following would most likely give her a constitutional basis to require the city to give her a statement of reasons for the termination and an opportunity for a hearing? A No police officer had ever been terminated during probation except where there was actual cause. B The six months she spent in the academy must be considered as part of her probationary period. C The budget of the police department was recently increased to allow for the hiring of additional officers. D She was the only female police officer on probation and the only officer not given permanent employment.
A. The fact that no police officer has been terminated during probation except for cause may be enough for the graduate to show that she has a right to a hearing. Continued public employment may be a protected property interest if there is a clear practice or mutual understanding that an employee can be terminated only for "cause." If the graduate can establish this, she will be able to force the city to give her a reason for her termination and a hearing. (C) is incorrect because a general increase in police officer positions does not establish a specific right to employment for the graduate. (B) is wrong because there is nothing in the Constitution that requires that a city follow any particular method of employment practice, as long as the method chosen by a city does not violate some constitutional prohibition. The fact that the city's civil service law was not changed to reflect the additional period of time a police officer spends in the police academy does not create a right protected by the Fourteenth Amendment. (D) states facts that could give rise to an equal protection, rather than due process, claim
A mining company closed down operations at an isolated mine it owned and informed the electric company that electricity in the power poles that led to the mine should be cut off. However, the electric company, following its standard policy, left the power running in the line to deter thieves from stealing valuable transformers and cables. The mining company was unaware that the power was left on. A hitchhiker who was passing by the entrance to the mine saw that it was closed, so he went onto the property and climbed up a power pole to steal a transformer. He received an electric shock and fell from the pole, suffering serious injuries. If the hitchhiker sues the mining company, which of the following is the mining company's strongest defense? A The hitchhiker was a trespasser. B The hitchhiker was a thief. C The mining company asked the utility company to turn off the power. D The mining company was unaware that the utility company had not turned off the power.
A. The hitchhiker's status as a trespasser is the mining company's strongest defense because it means that the mining company owed no duty to the hitchhiker, thereby completely relieving the mining company of any liability for his injuries. An owner or occupier of land owes no duty to an undiscovered trespasser. However, with regard to a discovered trespasser, the owner or occupier must warn of or make safe artificial conditions known to the landowner that involve a risk of death or serious bodily harm and that the trespasser is unlikely to discover. The hitchhiker, having come onto the land owned by the mining company without permission or privilege, is a trespasser. Because the mining company had no notice of the hitchhiker's presence on the property, the hitchhiker is deemed to be an undiscovered trespasser. Consequently, the mining company owes no duty to the hitchhiker with regard to the injuries incurred on its property. (C) and (D) each present factors that would be helpful to the mining company, but they are not as strong as (A). The fact that the mining company asked the power company to turn off the power, as well as the mining company's being unaware that it had not turned off the power, would be indicative of the exercise of due care on the part of the mining company (i.e., it took every reasonable step to see that the power was not left running in the abandoned mine, and could not reasonably have known that in fact the power was still on). However, if it is shown that no duty of care extended from the mining company to the hitchhiker, then the first element of a prima facie case for negligence is absent, thus eliminating any need for the mining company to attempt to show that it acted with ordinary, reasonable care. As a result, (A) is a much stronger defense than (C) or (D). (B) is incorrect because the fact that the hitchhiker tried to commit theft of the power company's transformers is not relevant to any duty that may have been owed to him by the mining company. It is relevant that the hitchhiker was a trespasser, because this means that the mining company owed him no duty. However, the hitchhiker's status as a thief is of no significance.
The "market participant" exception, which allows a state to favor its own residents in interstate commerce when the state itself acts as a participant in the market, is an exception to: A The Dormant Commerce Clause B The Privileges and Immunities Clause C Both the Dormant Commerce Clause and Privileges and Immunities Clause D Neither the Dormant Commerce Clause nor Privileges and Immunities Clause
A. The market participant exception is an exception to the Dormant Commerce Clause. When a state acts as a market participant, it generally is not restricted by the Dormant Commerce Clause; it may favor its own citizens, such as by distributing state-owned resources only to residents or paying residents more for something than it would pay a nonresident. While a state or local government does not violate the Dormant Commerce Clause by preferring its own citizens while acting as a market participant, there is no market participant exception to the Interstate Privileges and Immunities Clause. Thus, a regulation that interferes with private sector employment may violate the Privileges and Immunities Clause unless the regulating entity can show a substantial justification for the regulation.
A developer owned a 30-acre tract of farmland. As required by law, the developer filed a plat with the county planning board, but did not record it. The plat divided the parcel into 87 one-third-acre residential lots. A one-acre strip on the eastern edge of the parcel that abutted a busy highway was set aside for commercial development. The plat restricted each lot to a single residence and banned all "nonconforming detracting structures or appurtenances," including "free-standing flagpoles more than six feet in height, television antennas and receiving equipment of excessive size and obtrusiveness, and windmills." The restrictive clause was put into the deeds of all the residential lots in the subdivision, except for the deeds to lots 23, 24, and 25. This oversight was due to an error by the developer's secretary. All the other lots had deeds stating that the restriction applied "to the grantee and his or her heirs and assigns." A homeowner purchased lot 24 and duly recorded her deed in the office of the county recorder of deeds. The developer's salesperson had orally informed the homeowner of the general restrictions applicable to lots in the subdivision. A year later, a sports bar purchased the one-acre commercial strip and installed a large satellite dish. Two years later, the homeowner sold her property to a buyer. The homeowner never mentioned any of the restrictions to the buyer. The buyer put a satellite dish on top of his house. His dish was not as large as the bar's dish, but it was obviously bigger than any of his neighbors' modest antennas. The owners of 15 lots in the subdivision sue the buyer, demanding that he remove the dish. If the court finds for the buyer, what is the likely reason? A The buyer is not charged with record notice based on other deeds given by a common grantor. B The buyer's predecessor in interest, the homeowner, was not bound by the oral restriction told to her by the developer's salesperson. C The property owners suing the buyer all purchased their lots prior to the homeowner's purchase of lot 24. D The existence of a satellite dish on the eastern end of the original parcel indicates that neighborhood conditions have changed to the point where it would be inequitable to enforce the restrictions.
A. The most likely reason to find for the buyer is that the court is not charging him with record notice of deeds to other lots given by the developer. When a developer subdivides land into several parcels and some of the deeds contain negative covenants but some do not, negative covenants or equitable servitudes binding all the parcels in the subdivision may be implied under the doctrine of "reciprocal negative servitudes." Two requirements must be met before reciprocal negative servitudes will be implied: (i) a common scheme for development, and (ii) notice of the covenants. The second requirement may be satisfied by actual notice, record notice, or inquiry notice. Here, the buyer has not been given actual notice, and the antenna restriction is not so obvious that the appearance of the neighborhood would provide the buyer with inquiry notice. Finally, the buyer has no record of the restriction in his chain of title to establish record notice. If the buyer had been the first purchaser of the lot, some courts might require him to read all deeds given by a common grantor, but the better view does not require such a search. In any case, the buyer's grantor here is the homeowner, and the restriction was not contained in her deed; the buyer thus does not have record notice of it and is not bound. (B) is incorrect because the restriction could have been enforced against the homeowner as an equitable servitude even in the absence of an express restriction (oral or written). A common scheme for development existed and the developer's salesperson gave the homeowner actual notice of the restriction. (C) is incorrect because courts will allow prior purchasers to enforce the restriction against a subsequent purchaser even if the original grantor made no covenant in the deeds that all subsequent parcels would be subject to the restriction. One theory courts use is that an implied reciprocal servitude attached to the common grantor's retained land at the time the first lots were deeded to the prior purchasers, and the prior purchasers are merely enforcing this implied servitude against the purchaser of a subsequent lot. Hence, if the buyer were deemed to have had notice of the restriction, a court would allow prior purchasers to enforce it. (D) is incorrect. While "changed neighborhood conditions" is an equitable defense to enforcement of a servitude, the strip on the eastern edge of the parcel was always earmarked for commercial uses; the presence of a satellite dish on that property is not sufficient to bar enforcement of the restriction against the residential parcels.
A pedestrian was injured in an auto accident caused by a driver. The pedestrian's injuries included a broken nose and a broken toe. Not sure of the strength of her case, she sued the driver only for the injuries to her nose. She was awarded $15,000 in damages. Encouraged by this success, she now wishes to sue the driver for the injuries to her toe. May she sue the driver again? A No, because all related claims "merged" with the final decision in the first case. B No, because she will be collaterally estopped from pursuing the claim. C Yes, and she may use the prior decision against the driver. D Yes, but she will have to relitigate the driver's liability.
A. The pedestrian may not sue the driver again because of merger. Merger occurs when the plaintiff wins; her cause of action is said to "merge" into the judgment such that she cannot relitigate the cause of action later. The court would hold that all of the personal injuries received by the pedestrian in one accident constitute a single cause of action, and that claim preclusion principles forbid relitigation. (B) is incorrect. Issue preclusion, also called "collateral estoppel," applies to issues, not entire cases. (C) and (D) are incorrect for the reasons stated above.
A professional baseball player visited a sick boy in the hospital. The player told the boy that in consideration of the boy's courage, he would hit a home run for him in his next game. As the player was leaving the hospital, the boy's father stopped the player and told him how important the home run could be in improving his son's spirits and health. The father told the player he would pay him $5,000 if he did hit a home run in his next game. The player agreed and took extra batting practice before his next game to improve his chances. In his next game, the player hit two home runs. The player's contract with his ball club does not forbid him from accepting money from fans for good performance. The player has now asked the father for the $5,000. If the father refuses to pay and the baseball player brings an action against him for damages, which of the following is correct under the prevailing modern rule in contract law? A The player can recover the $5,000 because the preexisting duty rule does not apply where the duty is owed to a third person. B The player can recover the $5,000 if he can prove that the value of the home run to the boy is at least $5,000. C The player cannot recover from the father because the player had a preexisting duty to use his best efforts to hit home runs. D The player cannot recover from the father because, even under the modern trend, moral consideration is not valid.
A. The player can recover because, under the prevailing modern rule, the preexisting duty rule does not apply if the duty is owed to a third person. Generally, contracts must be supported by consideration. A promise to perform is valid consideration, but if a person already owes a duty to perform, traditionally that performance cannot be used as consideration for another promise. Thus, under the traditional rule, the player could not enforce the father's promise to pay the player $5,000 if he hit a home run because the player gave no valid consideration in exchange for the father's promise, since the player owed a preexisting duty to his ball club to exert his best efforts to hit home runs. However, under the modern view as formulated in Restatement (Second) of Contracts, section 73, and followed by a majority of courts, a duty is a preexisting duty only if it is owed to the promisee. Thus, a promise to perform a duty is valid consideration as long as the duty of performance is not already owed to the promisee. In other words, if the duty is owed to a third party, a promise to perform given to another is valid consideration as long as it was bargained for. (B) is incorrect because there is no exception to the preexisting duty rule—modern or otherwise—that allows the promisor to recover merely because his performance benefited a third party. The player can recover under the modern approach because his promise to the father was bargained for. Conversely, the player does not have to prove that the value of his home run to the boy was at least $5,000, because courts generally will not inquire into the adequacy of consideration. (C) would be correct under the traditional rule, but, under the modern trend, the promise here is valid consideration because the duty to hit home runs was owed to a third party (the ball club) rather than to the promisee (the father). (D) is incorrect because while it is true that moral consideration is not good consideration, the father did not rely on moral consideration, but rather exchanged a promise to pay $5,000 for the player's performance.
A property owner sued a developer in federal court alleging breach of an oral contract. According to the property owner, only one other person of questionable credibility heard the conversation in which the contract was allegedly made. That person was listed in the pretrial conference order. The next day, five days before the scheduled trial, the property owner's attorney discovered that a disinterested person also heard the conversation, and he wanted to call this witness at trial as well. He immediately notified the developer of the witness and his change in plans. May the property owner call the additional witness? A Yes, if the court modifies the pretrial order. B Yes, because the trial has not yet begun. C No, because the final pretrial order controls the subsequent course of the trial. D No, if the jury has already been selected.
A. The property owner may call the additional witness only if the court modifies the pretrial order, since a pretrial order controls the subsequent course of an action unless modified. The order will be modified "only to prevent manifest injustice." [Fed. R. Civ. P. 16(e)] (B) is incorrect. Although a court will consider the timing of the request in determining whether to modify the pretrial order, there is nothing to prevent a judge from modifying the order, even if the trial has begun. (C) is incorrect because, although the pretrial order does control the subsequent course of the trial, it can be modified, as discussed above. Here, because the additional witness is disinterested, having her testify could greatly affect the outcome of the case. (D) is incorrect. The fact that the jury was selected would not prevent the court from modifying the pretrial order.
A downtown department store engaged an electrician to service all electrical appliances sold by the store for a flat fee of $5,000 per month. Under a written contract signed by both parties, the store was responsible for pickup and delivery of the appliances to be repaired and the billing for the work. By its terms, the contract would continue until either party gave 180 days' written notice of its intent to terminate. Several months ago the electrician informed the store that he was losing money on the deal and was in financial trouble. He requested in good faith that the fee for the next three months be increased by $1,000 and that this increase be paid to a local bank to help pay off a loan that the bank had made to the electrician. The store orally agreed to so modify the original contract. However, the store did not pay the bank and now the bank is suing the store for $3,000. Who will prevail? A The store, because there was no consideration to support the promise to pay the bank. B The store, because the bank is only an incidental beneficiary of the modified contract between the store and the electrician. C The bank, because it is an intended creditor beneficiary of the modified contract between the store and the electrician. D The bank, because the electrician exercised good faith in requesting the modification regarding the payment to the bank.
A. The store will prevail, because there was no consideration to support its promise to pay the bank the additional $1,000 per month. This question looks like it concerns third-party beneficiaries, but it actually presents a consideration issue. Generally, there must be consideration for modification of a contract, and a promise to perform an act that a party is already obliged to do is not sufficient consideration (the "preexisting legal duty" rule). Here, the electrician is promising to do exactly what he was obliged to do under his original contract with the store; thus, there is no consideration to support the promise to increase the fee. Note that the modern view permits modification without consideration if it is fair and equitable in view of unanticipated circumstances. That is not applicable here. This exception contemplates an unanticipated circumstance arising in performance of the contract that makes performance more difficult or expensive. (B) is wrong because the bank is an intended beneficiary, not an incidental beneficiary. An intended beneficiary is one who is clearly intended to benefit from the agreement. Here, the bank was named in the agreement and performance was to be made directly to it, and so it is clearly an intended beneficiary. (C) is wrong even though it is true that the bank is an intended creditor beneficiary. Despite this status, the bank will not recover because there was no consideration to support the modification of the contract. The status of creditor beneficiary does not give the bank any more rights than the electrician would have had to enforce the agreement, and the electrician could not enforce the agreement for the additional money because there was no consideration. (D) is wrong because it is based on the rule of UCC section 2-209, which states that an agreement subject to the UCC does not need consideration to be binding. However, the UCC governs only in cases of the sale of goods, and this question presents a contract for services. Thus, the UCC does not apply and the common law rule requiring consideration controls.
Felony murder generally requires that: A The killing be committed during the course of the felony, the felony must be independent of the killing, and the death must have been a foreseeable result of the felony B The killing be committed during the course of the felony, the felony must be independent of the killing, and the defendant must have been convicted of the underlying felony. C The killing be committed during the course of the felony, the death must have been a foreseeable result of the felony, and the defendant must have been convicted of the underlying felony D The death must have been a foreseeable result of the felony, the felony must be independent of the killing, and the defendant must have been convicted of the underlying felony
A. To convict a defendant of felony murder, the prosecution must prove, beyond a reasonable doubt, that the defendant committed a felony (i.e., he is factually guilty of the felony). However, the defendant need not actually be convicted of the underlying felony if the statute of limitations for the felony has expired. The killing must take place while the felony is being committed. When the defendant reaches "a place of temporary safety," the felony is deemed terminated. The felony must be independent of the killing (e.g., the felony of manslaughter cannot be the underlying felony for felony murder). Finally, most states require that the death must be a foreseeable result of the commission of the felony.
A plaintiff sued an auto manufacturer for negligence after a car accident involving the plaintiff's car that was made by the auto manufacturer. Sixty days after service of the complaint and 40 days after service of the manufacturer's answer that contained no counterclaim, the plaintiff filed a motion seeking to file an amended complaint adding a claim for strict products liability against the auto manufacturer stemming from the same incident. The statute of limitations for strict products liability claims expired one week before the motion was filed. How should the court rule on the plaintiff's motion? A Grant the motion, because every party is entitled to amend once as a matter of course. B Grant the motion, because the amended complaint relates back. C Deny the motion, because, while the motion is timely, the proposed claim is futile because the statute of limitations has run. D Deny the motion, because it is not timely.
A. The court should grant the motion. Federal Rule of Civil Procedure 15 states that leave of court (to grant motions to amend) is to be "freely given when justice so requires." The rule does not provide any clear date when amendments are no longer permissible, although later amendments obviously would be less fair and less likely to be considered in the interest of justice. Additionally, for statute of limitations purposes, proposed claims may be considered to "relate back" to the date of the original pleading in which the claim was made under Rule 15(c). That is critical here because, although the statute of limitations for the proposed products liability claim had expired at the time the motion was made, the amended complaint asserting a products liability claim relates back to the original filing because that claim stems from the same facts alleged in the original complaint, and the statute of limitations had not expired at the time of the original filing. (A) is incorrect because a plaintiff may amend the complaint once as a matter of course (i.e., without court intervention) not later than 21 days after service of the auto manufacturer's answer. Here, 40 days have passed since the auto manufacturer filed its answer, so amending as a matter of course is not permitted. (C) is incorrect. Although the proposed claim would be futile if filed in an original complaint, here, because the claim can relate back to a complaint filed at a time when the proposed claim was still viable, the proposed claim is not considered futile. (D) is incorrect by suggesting that the motion is not timely. Although plaintiff can no longer amend as a matter of course, there is no absolute date pursuant to which a motion to amend is untimely.
A woman sued her ex-husband for installments due under their divorce property settlement agreement. The ex-husband defended the suit on the ground that the ex-wife was in breach of the agreement. However, the court granted judgment for the ex-wife. Now, the ex-wife is suing her ex-husband for other installments due under the agreement. The ex-husband raises the defense that the agreement was void and illegal because of fraud perpetrated by his ex-wife at the time the agreement was signed. The ex-wife moves to strike her ex-husband's defense, claiming issue preclusion. Should the court grant the motion? A No, because the issue was never litigated. B No, because judgment on one installment does not bar subsequent actions on other installments. C Yes, because the husband should have raised the defense in the first action. D Yes, because the issue of fraud is never waived.
A. The court should not grant the motion. Generally, issues actually litigated between the parties are binding on them in subsequent actions concerning the same claim. If the second suit involves a different claim, the first judgment may be invoked as to all matters actually litigated and determined in the first action, provided that the findings were essential to the first judgment. Here, the fraud issue was not actually litigated in the first action, and thus it can be raised with respect to the later installments. (B) is incorrect. The general principle recited in this answer is true for claim preclusion (unless there is an acceleration clause). However, even if claim preclusion does not apply, if the defendant raises the same issues as in the first case, he may be prevented by issue preclusion (collateral estoppel) from re-litigating those issues in that subsequent case. (C) is incorrect. Claim preclusion (res judicata) would bar claims arising out of the same transaction or occurrence. Here, claim preclusion does not apply because a defense, not a claim, is involved, and the second installment is a separate transaction or occurrence. (D) is incorrect because there is no such rule.
An owner of two abutting lots on a street deeded the north parcel to a buyer and inserted in the deed the following language: "Grantee, his heirs and assigns shall not plant any shrubbery within 10 feet of the boundary line." The buyer recorded. The buyer later deeded the north parcel to a friend and did not include the language about the shrubbery in the deed. The friend planted a row of shrubbery within five feet of the common boundary. If the owner sues the buyer's friend to require him to remove the shrubbery, what is the likely result? (A) The owner will prevail because there is a covenant running with the land. (B) The owner will prevail because the deed from the buyer to the friend carried with it an implied promise not to plant shrubbery within 10 feet of the common boundary of the two parcels. (C) The friend will prevail because the deed from the buyer to the friend created only a personal contract between them. (D) The friend will prevail because there was no writing signed by the buyer, the friend's predecessor in title.
A. (A) The owner will prevail because the covenant runs with the land. The covenant in the original deed from the owner to the buyer will be enforceable against the buyer's successor, the friend, only if the covenant runs with the land, either at law or as an equitable servitude. With the facts presented, the owner would prevail under either theory. This covenant was created by a signed writing. The buyer's acceptance and recording of the deed containing the covenant is the legal equivalent of his signature. Therefore, the covenant is enforceable between the original parties. Moreover, the covenant was intended to run with the land and bind the grantee's successors. The provision states that it binds the "grantee, his heirs and assigns." The covenant also clearly touches and concerns the land, because it controls the grantee's land use (for the benefit of the grantor's adjoining land). There is horizontal privity because the covenant was created in the deed from the owner to the buyer, which also created the buyer's estate. Also, the friend is in vertical privity with the buyer, because the friend purchased from the buyer. Therefore, the covenant runs at law to the buyer's successor, the friend. The fact that the covenant was recorded in the friend's chain of title would also give him constructive notice of the covenant such that it could be enforced against him as an equitable servitude. Under either theory, the owner will prevail. (B) is incorrect because reliance on an "implied promise," which does not accurately reflect any recognized rule of law, is misplaced. This obligation is enforceable as a covenant running with the land. A successor to land subject to a covenant which runs with it would be bound by the covenant even if the original contracting party and the successor specifically disavowed the contractual obligation. (C) is incorrect. The covenant in the deed from the owner to the buyer did create an enforceable contract between the two. However, that mere contract became a covenant running with the land enforceable at law or as an equitable servitude against the buyer's successors in title, so the owner will prevail. (D) is incorrect because the covenant was created by a signed writing. While the Statute of Frauds normally requires a writing signed by the party to be charged for a contract relating to land to be enforceable, the buyer's acceptance and recording of the deed containing the covenant is the legal equivalent of his signature. Therefore, the covenant is clearly enforceable between the original parties, and as a covenant running with the land is also binding on their successors, including the friend.
While land sale contracts must be memorialized in writing and signed by the party to be charged to be enforceable under the Statute of Frauds, courts in most states will enforce an oral contract in equity under the doctrine of part performance if the buyer has performed at least two of the following acts:
AT LEAST TWO OF: 1. Taken possession of the land; 2. Made substantial improvements to the land; and/or 3. Paid all or part of the purchase price. Some courts will accept as part performance additional acts showing the buyer's detrimental reliance.
"Common Question" class action suits
All members of a class will be bound by judgment if they do not opt out. Personal jurisdiction over absent class members not required. Notice: notice to all members of class of pendency of class action: Notice must state :1)nature of action, 2)definition of the class 3) class claims, issues or defenses 4) binding effect of class judgment
What is an equitable servitude and how does it run with the land at law?
An equitable servitude is a covenant (i.e., a promise to do or not to do something on the land) that, regardless of whether it runs with the land at law, can be enforced in equity against assignees of the burdened land who have notice of the covenant. The burden of an equitable servitude will run to successors in interest if: 1. The covenanting parties intended that successors in interest be bound by the covenant; 2. The successor in interest has notice of the covenant (if she has given value); and 3. The covenant touches and concerns the land (i.e., it benefits the covenantor and his successor in their use and enjoyment of the burdened land). Horizontal privity between the original covenanting parties and vertical privity between the covenantor and his successor in interest are not required.
A party to an action may set forth as many claims or defenses as he may have in his pleadings:
And the claims or defenses NEED NOT BE CONSISTENT WITH EACHOTHER
A landlord employed his friend as the on-site manager of one of his apartment buildings despite being aware that he had previously been arrested for criminal battery, disorderly conduct, and driving while intoxicated. The manager did a good job dealing with the general maintenance of the apartment building, although the landlord was aware that he continued to drink heavily. One night the manager, who was extremely intoxicated, attempted to swat an insect on the ceiling of his apartment and could not do so after several attempts. Enraged, he took a pistol from his drawer and shot at the insect. The bullet missed the insect and passed through the ceiling of his apartment into the apartment above, lodging in the leg of a tenant's social guest. Does the guest have a viable cause of action against the landlord? A Yes, because the guest had been invited onto the property by the tenant. B Yes, because the landlord was aware of the manager's habitual drunkenness and propensity for violence. C No, because the landlord cannot be held liable for the manager's intentional torts. D No, because shooting an insect was outside the scope of the manager's employment.
B Because the landlord knew about the manager's continued heavy drinking and tendencies toward violence, the guest has a cause of action for negligence in the landlord's hiring of the manager. An employer owes a duty to all those who may foreseeably come into contact with his employee to exercise due care in the hiring, supervision, and retention of the employee, and the landlord's retention of the manager under these circumstances may be a breach of that duty. (A) is incorrect because the landlord's liability here is based on negligent hiring rather than the guest's status on the property. (C) is also incorrect. An employer can be held directly liable for the intentional tort of an employee if it was foreseeable and the employer was negligent in hiring or retaining the employee. (D) is a true statement that would be relevant for vicarious liability purposes. However, it does not preclude the landlord from being liable for his own negligence based on the foreseeability of his employee acting violently.
Which of the following factors is not required in order for an otherwise nonappealable interlocutory order to be appealable under the Interlocutory Appeals Act? A The controlling question of law is one as to which there is a substantial ground for a difference of opinion B The amount in controversy exceeds $75,000. C At least two appellate court judges agree to allow an appeal D An immediate appeal of the order may materially advance the ultimate termination of the litigation
B Review under the Interlocutory Appeals Act is discretionary with the court and may be available when: (i) the trial judge certifies that the order involves a controlling question of law as to which there is substantial ground for a difference of opinion and an appeal would materially advance the conclusion of the case, and (ii) at least two appellate court judges agree to hear the appeal.
During the course of his trial for assault, the defendant placed his neighbor on the stand. The neighbor testified that the defendant had a good reputation in their community for being a peaceable man. On cross-examination, the prosecutor asked the neighbor if she filed a false income tax return last year. The neighbor has in fact been convicted of filing a false tax return; however, the defense immediately objected. Should the court permit the question? A Yes, because the neighbor has been convicted of filing a false tax return. B Yes, because the question is relevant to the truthfulness and credibility of the witness. C No, because specific instances of conduct are inadmissible. D No, because the question does not go to a relevant character trait.
B The court should require the neighbor to answer the question because it is relevant to her truthfulness and credibility as a witness. Under the Federal Rules, a witness may be impeached on cross-examination with her prior specific acts of misconduct that are probative of truthfulness. Specific "bad acts" that show the witness unworthy of belief (i.e., acts of deceit or lying) are probative of truthfulness. Filing a false income tax return reflects on the neighbor's veracity and, thus, her credibility. Therefore, the neighbor should be required to respond. (A) is incorrect because inquiry into bad acts to impeach a witness's credibility is permitted even if the witness was never convicted of a crime relating to the act. Even though the neighbor could have been impeached by evidence of her conviction for the crime, here she is just being asked about her conduct rather than the criminal conviction. (C) is incorrect because Federal Rule 608 permits inquiry about specific acts of misconduct, within the discretion of the court, if they are probative of truthfulness. An important limitation is that extrinsic evidence is not admissible to prove such acts; here, however, the impeachment was properly limited to inquiry on cross-examination. (D) is incorrect because the question relates to truthfulness, and a witness's credibility is always relevant.
A witness testified against a defendant in a contract action. The defendant then called the witness's neighbor to the stand, who testified that the witness had a bad reputation for truth and veracity. The defendant then also called the witness's employee to testify that the witness once perpetrated a hoax on an insurance company. The witness had in fact been convicted for perpetrating the hoax 20 years ago. Is the employee's testimony admissible? A No, because it is merely cumulative impeachment. B No, because it is extrinsic evidence of a specific instance of misconduct. C Yes, because the hoax resulted in a conviction of the witness. D Yes, because a hoax involves untruthfulness.
B The testimony is inadmissible because it is not a permitted way to impeach a witness. A witness may be impeached by cross-examining her about specific criminal or immoral acts, but extrinsic evidence is not permitted. A specific act of misconduct offered to attack the witness's character for truthfulness can be elicited only on cross-examination of the witness. If the witness denies it, the cross-examiner cannot refute the answer by calling other witnesses or producing other evidence. Thus, the witness could be asked on cross-examination about the hoax, but her employee cannot properly be called to testify about it. (A) is incorrect because there is no specific rule limiting cumulative impeachment. (C) is incorrect. A witness may also be impeached by introducing evidence that the witness was convicted of a crime if the conviction required proof or admission of an act of dishonesty or false statements, or if the crime was a felony. However, the prior conviction will generally not be admitted if more than 10 years have passed since the date of conviction or release from confinement, whichever is the later date (although the judge has discretion to admit older convictions in extraordinary circumstances). Here the witness was convicted 20 years ago and the conviction is likely too remote. Additionally, the defendant did not attempt to introduce evidence of the witness's conviction; the proposed testimony only concerns the commission of the misconduct. (D) is incorrect because even though a hoax would impair a witness's credibility, it cannot be proven by extrinsic evidence, as discussed above.
A shopkeeper loaned a long-time employee $1,500 from his personal bank account because a family illness was causing the employee unexpected financial difficulties. Because the employee had proved himself to be trustworthy, there was no writing evidencing the loan and no payback date established; it was understood that the employee would repay the loan when he was able to do so. Sometime later, the shopkeeper's nephew asked him if he could help fund a business that he was starting up. Because most of the shopkeeper's assets were currently tied up, he asked his employee if he would be in a position to repay the $1,500 loan. The employee promised to repay the loan on the following Monday, so the shopkeeper told the employee to pay the $1,500 directly to his nephew. Immediately thereafter, the shopkeeper informed the nephew to expect $1,500 from the employee on the following Monday. When Monday came, the employee decided he would rather tender the money to the shopkeeper than to someone he did not know, and the shopkeeper accepted the money. If the nephew never receives any money from the shopkeeper, will he succeed in an action against the employee for the $1,500? (A) Yes, because the shopkeeper effectively assigned his right to collect the $1,500 to the nephew. (B) No, because the shopkeeper's acceptance of the $1,500 from the employee revoked the shopkeeper's gift to the nephew. (C) No, because the assignment was unsupported by consideration and therefore never effective. (D) No, because the employee's tender of the $1,500 to the shopkeeper and the shopkeeper's acceptance of it constituted a novation.
B) The shopkeeper validly assigned his right to receive the money to his nephew. However, this assignment was revocable, and it was revoked when the shopkeeper accepted the money from the employee. A creditor's right to receive money due from a debtor is a right that can be assigned, regardless of whether the debt is evidenced by a writing. By telling the employee to pay the money to the nephew, the shopkeeper manifested an intent to transfer his rights completely and immediately to the nephew. Neither a writing nor consideration was required for this assignment to be valid. However, these factors do not affect revocability. This assignment was not given for value. Such a gratuitous assignment is generally revocable. An exception to this rule arises when the assignor is estopped from revoking because he should reasonably foresee that the assignee will change his position in reliance on the assignment and such detrimental reliance occurs. Here, there is no indication that the nephew in fact changed his position detrimentally in reliance on the assignment. Consequently, the general rule of revocability of a gratuitous assignment applies. One way in which a gratuitous revocable assignment may be terminated is by the assignor taking performance directly from the obligor. By accepting the money from the employee, the shopkeeper (the assignor) took direct performance from the obligor, thereby revoking the assignment. As a result, the nephew has no right to the money. (A) is incorrect because it fails to account for the fact that, although the shopkeeper effectively assigned his right, he later revoked this assignment. (C) is incorrect because, as discussed in (B) above, a gratuitous assignment (i.e., an assignment not supported by consideration) is effective, although revocable except under certain circumstances not applicable here. (D) is incorrect because the facts do not indicate that there has been a novation. There is a novation when a new contract substitutes a new party to receive benefits and assume duties that had originally belonged to one of the original parties under the terms of the old contract. Here, the original agreement was between the employee and the shopkeeper. The employee's payment of the money to the shopkeeper and the shopkeeper's acceptance thereof did not substitute any new parties or extinguish contractual duties as between the original contracting parties. Thus, there was no novation
An infant was injured in an automobile accident when the vehicle, driven by the infant's mother, left the roadway and rolled over down an embankment. At the time of the accident, the infant was buckled into an infant carrier car seat. The carrier was designed to snap into a base that was secured in the back seat by the rear center seat belt. Prior to driving, the mother had snapped the car carrier onto the base and pulled up on the car carrier's handle to ensure that the carrier was indeed secured in the base. When the rollover occurred, however, the carrier came loose from the base and was thrown about the inside of the vehicle, causing injuries to the infant's neck and face. The mother brought a products liability action on behalf of the child against the manufacturer of the car carrier, alleging that the manufacturer was negligent in the design of the base and seat combination. If the mother establishes at trial that the force of the rollover was enough to knock the seat loose, and that a reasonable, economically feasible alternative design existed, which of the following, if true, would be most helpful to the manufacturer's defense? (A) The mother violated a statute by traveling too fast for conditions, which caused the rollover accident. (B) No one had reported a car carrier coming loose in a rollover prior to this accident. (C) The car seat conformed with federal labeling requirements. (D) The retailer who sold the car seat was negligent in failing to notice the defect.
B) The most helpful fact is that no one had reported this type of problem previously. The mother is alleging that the manufacturer's negligence led to the supplying of a defective product. To establish this, the plaintiff must show that those designing the product knew or should have known of enough facts to put a reasonable manufacturer on notice about the dangers of marketing the product as designed. Negligent design is not shown, however, if the danger of the product becomes apparent only after the product reaches the public. Hence, the absence of any previous complaints about this problem would be most helpful to the manufacturer. (C) is less helpful than (B). Although compliance with government safety standards, such as labeling, is evidence that the product is not defective, it is not conclusive evidence, and federal labeling requirements do not preempt state products liability laws on defective warnings. (A) will not support the manufacturer's defense. The answer choice suggests that the mother was contributorily negligent; however, the contributory negligence of a parent is not imputed to the child. (D) does not help the manufacturer. An intermediary's negligent failure to notice a defect does not relieve the liability of a manufacturer whose original negligence was the cause of the defect.
In most states, murder is of the second degree unless __________, in which case the murder is of the first degree. A deliberation and premeditation can be shown, or the defendant acted "with a depraved heart" B deliberation and premeditation can be shown, or the killing occurred during an enumerated felony C the killing occurred during an enumerated felony, or the defendant acted "with a depraved heart" D "a depraved heart" can be shown
B.
____________ is an element of __________. response - correct A Obtaining title; larceny by trick B Intent to defraud; false pretenses C Trespass; false pretenses D Trespass; embezzlement
B. Intent to defraud is an element of false pretenses. The offense of false pretenses generally consists of: (i) obtaining title; (ii) to the property of another; (iii) by an intentional (or, in some states, knowing) false statement of past or existing fact; (iv) with intent to defraud the other. Trespass is not an element of false pretenses or embezzlement. Embezzlement is the fraudulent conversion of the property of another by a person in lawful possession of the property. Obtaining title is not an element of larceny by trick; rather with larceny by trick, the defendant merely obtains possession, not title.
While walking on patrol in a commercial district in the early evening, a police officer noticed that a light was on in the back window of a machine shop. Curious about what was going on inside, the officer went to the back of the building and tried to look through the window of the shop, but it had been painted on the inside so that only a strip about three inches at the top, eight feet above ground level, was still transparent. The officer quietly brought two trash cans from a neighboring business over to the window, stood on them and saw, through the strip of unpainted window, that the shop owner's son was inside with a friend, sucking white powder into his nose through a rolled-up tube of paper from off a small mirror. Recognizing that the shop owner's son was snorting cocaine, the officer knocked at the front door to the shop, and the son let him in. The officer immediately arrested the owner's son and his friend. In the back room of the shop through whose window he had peered, the officer found and seized several grams of cocaine, a razor blade, and a mirror. In the subsequent prosecution of the shop owner's son for possession of cocaine, the owner's son seeks to bar introduction of the cocaine, mirror, and razor blade into evidence. Will his motion be granted? A Yes, because the officer could not have known that the owner's son was snorting cocaine absent a chemical test of the substance being snorted. B Yes, because the officer violated the owner's son's reasonable expectation of privacy. C No, because the search was incident to a valid arrest. D No, because the owner's son consented to the officer's entry into the shop.
B. The shop owner's son had a reasonable expectation of privacy, as evidenced by the obscuring of the window so that neighbors could not see into the shop. Hence, the officer's search would have to be based on a valid warrant or qualify under one of the exceptions to the warrant requirement. Putting trash cans next to a back window to climb upon and peering into the window through a narrow opening eight feet above the ground would be considered a violation of the owner's son's Fourth Amendment rights and not a "plain view" of criminal activity. Because the seizure of the cocaine, mirror, and razor was based on the illegal search, the evidence could not be used by the state. (A) is wrong; absolute certainty of illegal activity is not required for a valid search. A reasonable belief is required. (C) is wrong. The arrest itself is probably invalid, and in any event a search of the next room would not be an area within the immediate control of the defendant. (D) is wrong. Consent to enter the shop is not a consent to search the back room.
A conspirator can be convicted of a crime committed by another conspirator if: A The crimes were committed in furtherance of the objectives of the conspiracy. B The crimes were committed in furtherance of the objectives of the conspiracy, and the crimes were foreseeable. C The crimes were committed in furtherance of the objectives of the conspiracy, or the crimes were foreseeable. D The crimes were foreseeable.
B. A conspirator can be convicted of a crime committed by another conspirator if the crimes were committed in furtherance of the objectives of the conspiracy AND the crimes were foreseeable.
When a contractor is under a contractual duty to construct a building and the building is destroyed by an act of nature while it is still a work in progress, the destruction __________. A will discharge the contractor's duty to perform B will not discharge the contractor's duty to perform, but will extend the date of performance C will discharge the contractor's duty to perform if rebuilding cannot be reasonably completed by the date of performance D will neither discharge the contractor's duty to perform nor extend the date of performance
B. A contractor's duty to construct a building is not discharged by destruction of the work in progress. However, if the destruction was not caused by the contractor, such as by an act of nature, most courts will extend the date of performance beyond the original deadline.
When a party that offers a rare or unique service has breached a service contract, the court may grant __________ to the nonbreaching party. A punitive damages B injunctive relief C restitution D specific performance
B. A court may grant injunctive relief to enjoin a breaching party from working for a competitor throughout the duration of the contract if the services contracted for are rare or unique. Specific performance is not available for breach of a contract to provide services, even if the services are rare or unique because it is difficult to supervise and most courts find that it is tantamount to involuntary servitude. Punitive damages are generally not awarded in contract cases. Restitution is available as an alternative to contract damages in a non-contract situation to prevent unjust enrichment. Here there is a contract and there are no facts indicating unjust enrichment.
A person whose interest in land gives him the right to use someone else's land independent of his ownership or possession of his own tract holds: A An easement appurtenant B An easement in gross C A license D A servient tenement
B. A person whose interest in land gives him the right to use someone else's land independent of his ownership or possession of his own tract holds an easement in gross. An easement gives the holder the right to use a tract of land but no right to possess it. The land burdened by the easement right is called the servient tenement. An easement appurtenant, by contrast, benefits its holder in his physical use or enjoyment of his own tract of land. The land benefitted by the easement is called the dominant tenement. Unlike an easement, a license is not an interest in land, but is merely a privilege to go upon another's land.
Which of the following acts will NOT sever a joint tenancy? A Inter vivos conveyance by one joint tenant. B Testamentary disposition by one joint tenant. C In a title theory state, the execution of a mortgage by one joint tenant. D Suit for partition by one joint tenant.
B. A testamentary disposition by one joint tenant will not sever a joint tenancy. A will devising a joint tenant's interest to another is inoperative as to joint tenancy property because when the testator dies ( i.e., when the will becomes effective), his rights in the joint tenancy property are extinguished, and the will has no effect on them. A suit for partition by one joint tenant will sever a joint tenancy. The court will either divide the tract into parcels (partition in kind) or sell the property and divide the proceeds among the joint tenants in accordance with their ownership interests (partition by sale). An inter vivos conveyance by one joint tenant will sever a joint tenancy. The transferee takes the interest as a tenant in common and not as a joint tenant. In a title theory state, the execution of a mortgage by one joint tenant will sever a joint tenancy. In states following the lien theory, a mortgage is regarded as a lien on title, and one joint tenant's execution of a mortgage on her interest does not by itself cause a severance. But in states following the title theory, a mortgage is regarded as a transfer of title, and the transfer destroys the unity of title and severs the joint tenancy.
To buy a house, an investor secured a $10,000 mortgage from a bank. The bank promptly and properly recorded its mortgage. Subsequently, the investor financed certain improvements to the house with a $2,000 mortgage on the land from a finance company. The finance company promptly and properly recorded its mortgage. Before the investor made a payment on either mortgage, the federal government announced that it would begin storing nuclear waste products in the area. The value of property, including the investor's house, plummeted. The investor did not pay either the bank or the finance company, and the bank brought a proper action to foreclose, notifying both the investor and the finance company. A buyer bought the house at the foreclosure sale for $6,000, which was its fair market value. There are no special statutes in the jurisdiction regarding deficiency judgments. What does the investor owe? A $5,000 to the bank and $1,000 to the finance company. B $4,000 to the bank and $2,000 to the finance company. C Nothing to the bank and $2,000 to the finance company. D $4,000 to the bank and nothing to the finance company.
B. Absent any anti-deficiency statutes, the investor remains personally liable to pay for any shortfall arising from the foreclosure sale. Proceeds from the sale are used to satisfy the loan that was foreclosed first. Hence, all of the proceeds ($6,000) went to the bank. Thus, the investor must pay the balance still due the bank ($4,000) and the entire amount of the finance company's mortgage ($2,000), which is terminated by the foreclosure of the senior mortgage. (A) is wrong because foreclosure sales are not allotted proportionally between senior and junior interests. (C) is wrong because foreclosure does not extinguish the underlying debt. (D) is wrong because the finance company's mortgage does not remain on the land after foreclosure of the senior mortgage; hence, the investor is liable for that debt as well.
The owner of an exclusive clothing salon entered into a written agreement with a customer to sell the customer a certain full-length fake fur coat for $12,000, with delivery by December 7. On December 6, the customer went to the salon at 5:30 p.m., and the salon owner told her that her coat was ready and she could take it home with her. The customer inspected the coat and discovered that a button was missing. She told the salon owner that she would not accept the coat without the missing button. He informed her that the tailor had gone home for the day at 5 p.m. and would not be back at the salon until 8:30 a.m. on December 8, because Congress had enacted a law declaring December 7 to be Pearl Harbor Day, a new federal holiday, and the tailor had the day off. The salon owner assured the customer that the coat could be ready with the button sewn on by 9:15 a.m. on December 8. Which of the following best states the customer's position? A The customer may reject the coat, because the salon owner failed to provide perfect tender. B The customer may reject the coat, but she must give the salon owner an opportunity to cure. C The customer must accept the coat, because its value is not substantially impaired by the missing button. D The customer must accept the coat, because the defect can be easily cured.
B. Although the customer is entitled to reject the coat for even a minor defect such as one button being missing, she is required to give the seller an opportunity to cure this defect. Because this is a contract for the sale of goods, the Uniform Commercial Code applies. Pursuant to the UCC, if goods or any tender fail in any respect to conform to the contract, the buyer may reject the goods. This rule of perfect tender allows rejection for any defect, and does not require material breach. However, the perfect tender rule is softened by the rules allowing the seller to cure the defect by giving reasonable notice of an intention to cure and making a new tender of conforming goods within the time originally provided for performance. Also, where the buyer rejects a tender that the seller reasonably believed would be acceptable with or without money allowance, the seller, upon reasonable notification to the buyer, has a further reasonable time beyond the original contract time within which to make a conforming tender. Here, one button missing on a $12,000 coat is a very minor defect. However, pursuant to the perfect tender rule, the customer has the right to reject the coat even for this defect. In turn, the salon owner is entitled to cure the defect by notifying the customer of his intention to do so and by making a conforming tender. The salon owner has told the customer that the tailor will sew on the button, which will result in the coat's conforming to the contract. Although the tailor, due to the holiday, will not be able to sew on the button by the agreed-upon date of delivery, being able to do so early in the morning of the day after falls within a further reasonable time beyond the original contract time within which to make a conforming tender. In any event, at this point the customer must give the salon owner an opportunity to cure. (A) is incorrect because it fails to add that the salon owner must be allowed an opportunity to cure, as discussed above. (C) and (D) are incorrect because with the failure of perfect tender, the customer is not required to accept the coat. This right of rejection is not affected by the minor nature of the defect, as (C) suggests. Similarly, the buyer's right of rejection is not impaired by the fact that the defect is easily curable, as (D) states.
Under the Federal Rules, a party may be held vicariously responsible for the statement of someone with any of the following relationships to the party: A Partner; co-conspirator; co-party; principal-agent B Authorized spokesperson; partner; co-conspirator; principal-agent C Authorized spokesperson; partner; co-conspirator; co-party D Co-conspirator; co-party; principal-agent; authorized spokesperson
B. An admission (i.e., statement attributable to an opposing party) is frequently not the statement or act of the party against whom the admission is offered at trial. A party can be held vicariously liable for statements made by people with the following relationships to the party: Authorized Spokesperson The statement of a person authorized by a party to speak on its behalf ( e.g., statement by company's press agent) can be admitted against the party as an admission. Principal-Agent Statements by an agent concerning any matter within the scope of her agency or employment, made during the existence of the agency or employment relationship, are admissible against the principal. Therefore, if a truck driver-employee has an accident while on the job and admits that she was negligent, this admission may be introduced against her employer even if she was not authorized to speak for the employer. Partners After a partnership is shown to exist, an admission of one partner, relating to matters within the scope of the partnership business, is binding upon her co-partners since, as to such matters, each partner is deemed the agent of the others. Co-Conspirators The Supreme Court has held that admissions of one conspirator, made to a third party in furtherance of a conspiracy to commit a crime or a civil wrong, at a time when the declarant was participating in the conspiracy, are admissible against co-conspirators. In contrast, statements of a party are not receivable against her co-plaintiffs or co-defendants merely because they happen to be joined as parties to the action.
For claim preclusion (res judicata) to apply, it is not necessary that the judgment be: A Valid B Satisfied C On the merits D Final .
B. Answer Discussion - Incorrect For claim preclusion (res judicata) to apply, it is not necessary that the judgment be satisfied. Before merger or bar apply, it must be shown that (i) the earlier judgment is a valid, final judgment "on the merits"; (ii) the cases are brought by the same claimant against the same defendant; and (iii) the same "cause of action" (or "claim") is involved in the later lawsuit
How can one avoid the preexisting legal duty rule? response - incorrect A By full performance of the duty B By modifying the original consideration slightly C By making a brand-new identical promise D By beginning performance
B. Courts are anxious to avoid the preexisting duty rule, which states that the promise to perform, or the performance of, an existing legal duty is not consideration. Thus modifying the original consideration, even slightly, is generally enough to avoid the rule. Making a brand-new identical promise is not sufficient because there is no consideration for the new promise. There must be new consideration or the consideration that is different in some way, such as by accelerating performance, to avoid the preexisting duty rule. Beginning performance does not avoid the preexisting legal duty rule. Even full performance of a preexisting legal duty is not sufficient consideration. There must be some new or different obligation.
The difference between the crimes of larceny by false pretenses and larceny "by trick" is that: A For larceny by false pretenses, the victim must actually rely on the misrepresentation in giving up her property, but for larceny by trick, the misrepresentation need not induce the victim to give up her property. B For larceny by false pretenses, the victim intends to convey title, but for larceny by trick, the victim intends to convey only custody. C For larceny by false pretenses, the victim intends to convey only possession, but for larceny by trick, the victim intends to convey title. D For larceny by false pretenses, the misrepresentation need not induce the victim to give up her property, but for larceny by trick, the victim must actually rely on the misrepresentation in giving up her property.
B. False pretenses differs from larceny by trick in that title is obtained through false pretenses, but only custody of the property is obtained through larceny by trick. What is obtained depends upon what the victim intended to convey to the defendant. For both crimes, the victim must actually be deceived by, or act in reliance on, the misrepresentation, and this must be a major factor (or the sole cause) of the victim passing title (larceny by false pretenses) or custody (larceny by trick) to the defendant.
Miranda warnings are required as a prerequisite to the admissibility of confessions resulting from custodial police interrogation. Which of the following statements is true about the custody requirement under Miranda? A In considering whether a person was in custody during an interrogation, a court will consider the person's experience with the criminal justice system. B For Miranda purposes, a person may be in custody in his own home. C For Miranda purposes, a voluntary detention may constitute custody. D Whether a person is in custody depends on the subjective views of the person and the interrog
B. For Miranda purposes, a person may be in custody in his own home. Whether a person is in custody depends on whether his freedom of action is denied in a significant way. This is an objective test—whether a reasonable person under the circumstances would think that he is in custody. For example, if police officers handcuff a suspect in his own home, he is clearly in custody. Similarly, if they surround a person in the middle of the night in his own bedroom, awaken him, and begin questioning him, custody will be found. It is not true that custody depends on the subjective views of the person and the interrogator. As discussed above, the test is objective—what a reasonable person would think. Similarly, the courts will not consider a person's experience with the criminal justice system in determining custody. This is a subjective factor. Neither is it true that a voluntary detention may constitute custody. If a person consented to the detention, he has not been denied the requisite freedom of action; he is there by choice.
What does it mean for a grantee to assume a mortgage? A The grantee becomes a surety for the original mortgagor. B The grantee becomes primarily liable to the lender. C The grantee institutes foreclosure proceedings. D The grantee takes out an additional mortgage on the property.
B. For a grantee to assume a mortgage means the grantee becomes primarily liable to the lender. When a mortgagor conveys mortgaged property, the grantee takes the land subject to the mortgage. A grantee who signs an assumption agreement promises to pay the mortgage loan, thus becoming personally and primarily liable to the lender. The original mortgagor becomes secondarily liable as a surety. Assumption of a mortgage does not mean the grantee becomes a surety for the original mortgagor. The assuming grantee becomes primarily liable to the lender, and the original mortgagor becomes secondarily liable as a surety. Assumption of a mortgage does not mean the grantee institutes foreclosure proceedings. Foreclosure is a process that terminates the mortgagor's interest in the property. Generally, the property is sold in a foreclosure sale to satisfy the mortgage debt. The grantee who assumes a mortgage promises to pay the mortgage loan; thus, if the grantee defaults, foreclosure proceedings may be brought against him. Assumption of a mortgage does not mean the grantee takes out an additional mortgage on the property. As explained above, a grantee who signs an assumption agreement promises to pay the original mortgage loan, thus becoming primarily liable to the lender.
Under the plain view exception to the warrant requirement, which of the following is not required? A The item must be evidence, contraband, or a fruit or instrumentality of a crime. B The police must have inadvertently discovered the item. C The police must be legitimately on the premises where the item is found. D It must be immediately apparent that the item is evidence, contraband, or a fruit or instrumentality of a crime.
B. For the warrantless seizure of an item under the plain view exception, it is not required that the item be inadvertently discovered. Formerly, this exception to the warrant requirement applied only if the item was inadvertently discovered. Inadvertence, however, is no longer a requirement. To make a warrantless seizure, the police (1) must be legitimately on the premises where the item is found; (2) the item must be evidence, contraband, or a fruit or instrumentality of a crime; (3) the item must be in plain view; and (4) it must be immediately apparent (i.e., probable cause) that the item is evidence, contraband, or a fruit or instrumentality of a crime.
A landowner owned a large parcel of land in a rural area. He built his home on the northern half of the property, and developed a large orchard of fruit trees on the southern portion. A county road ran in front of the northern portion. To service his orchard, the landowner built a driveway directly from the county road across the northern portion of the property to the orchard. To provide electricity to his house, the landowner ran an overhead power line across the orchard property to hook up to the only available electric power pole located on the far southern side of the property. Subsequently, the landowner conveyed the northern parcel to his brother and the southern parcel to his daughter, who said that she did not mind having the power line on the property. Recently, the brother has begun parking his car on the driveway, thus blocking the daughter's access to the southern parcel. Finding no recorded document granting an easement for the power line, the daughter has decided to remove it. If the brother is successful in preventing the daughter from removing the power line, what is the likely reason? A The daughter knew that the power line ran across the land when she accepted the deed from the landowner. B The brother's alternative access to power is much less convenient and would cost 100 times as much. C The daughter told the landowner that she did not mind having the power line on the property. D The daughter is acting in retaliation against the brother for blocking the driveway, and not in any good faith belief that she has the right to remove the power line.
B. If the brother is successful in preventing the daughter from removing the power lines, it will be because the brother's alternative access to power is much less convenient and would cost 100 times as much as the current arrangement. This helps to prove that there was an easement implied by operation of law ("quasi-easement"). An easement may be implied if, prior to the time the tract is divided, a use exists on the "servient part" that is reasonably necessary for the enjoyment of the "dominant part," and a court determines that the parties intended the use to continue after division of the property. To give rise to an easement, a use must be apparent and continuous at the time the tract is divided. In this case, the landowner used the servient part of his property (the southern parcel) to run an overhead power line to the dominant part of his property (the northern parcel). Overhead wires are clearly visible and would be readily discoverable on reasonable inspection. The lines are, therefore, apparent. The use must also be reasonably necessary. Whether a use is reasonably necessary depends on many factors, including the cost and difficulty of the alternatives. This use was reasonably necessary to the enjoyment of the dominant parcel because electricity is important to the enjoyment of the property, and the cost (100 times as much) and difficulty of the alternatives are excessive. Thus, the fact that the use of the southern parcel is reasonably necessary would bolster the brother's case. (A) is wrong because the daughter's actual knowledge is irrelevant. The daughter need not be aware of the use; it need only be shown that the use was apparent (see above). (C) is similarly wrong. Oral statements made to the grantor after the northern parcel had been conveyed have little effect. They show the daughter's knowledge, but as discussed above, that has little relevance with respect to an implied easement. (D) is wrong because the daughter's motive for removing the power line is also irrelevant. If no easement is established, the daughter may remove the lines for whatever reason she likes. If, however, the requirements for an implied easement are satisfied, the daughter may not remove the lines regardless of how good her reasons are.
Under the UCC, in the case of a single delivery contract, if the buyer finds that the goods are defective at the time of delivery: A The buyer must accept the defective goods, but must allow the seller to cure the defects within the original time for performance of the contract B The buyer can reject the goods, but the seller then has a right to cure within the original time for performance of the contract C The buyer must accept the defective goods, but may recover from the seller whatever damages she has suffered as a result of the defects D The buyer can reject the goods outright, thus placing the seller in immediate breach with no right to cure
B. In a single delivery contract, a buyer can reject defective goods, but the UCC grants the seller the right to cure the defective shipment within the original time for performance of the contract. A seller cures by giving reasonable notice of his intention to do so and making a new tender of conforming goods within the time originally provided for performance in the contract. A buyer is not required to accept the defective goods. But if the seller properly cures the defect, the buyer must accept the cure or will be in breach of contract.
For a governmental regulation of speech in a public or designated public forum to be valid, which of the following is not necessary? A The regulation must be narrowly tailored to serve an important government interest. B The regulation must be the least restrictive method of achieving the governmental interest. C The regulation must be content neutral. D The regulation must leave open alternative methods of communication.
B. It is not necessary that the regulation be the least restrictive method of achieving the governmental interest in order for a court to uphold a regulation affecting speech in a public or designated public forum. It is enough that the regulation is narrowly tailored to serve the interest at issue. The regulation must be content neutral, the regulation must be narrowly tailored to serve an important government interest, and the regulation must leave open alternative methods of communication in order to be valid. Keep in mind, however, that even if these requirements are met, the regulation may be invalidated for other reasons, such as for being vague or overbroad.
The following are elements of larceny: A Taking of personal property; passing of title; trespass B Taking of personal property; asportation; trespass C Taking of personal property; asportation; passing of title D Asportation; passing of title; trespass
B. Larceny consists of: (i) A taking; (ii) And carrying away (asportation); (iii) Of tangible personal property; (iv) Of another; (v) By trespass; (vi) With intent to permanently (or for an unreasonable time) deprive the person of his interest in the property. The passing of title is not an element of larceny. However, it is an element of false pretenses.
Regarding the delivery and acceptance of a deed, which of the following will courts generally NOT presume? A A grantee accepts a deed if the conveyance would benefit her. B A grantee's return of a deed is a reconveyance to the grantor. C A grantee accepts a deed if she is a minor. D A grantee's possession of a deed means it has been delivered.
B. Regarding the delivery and acceptance of a deed, courts will not presume that a grantee's return of a deed is a reconveyance to the grantor. A deed will not transfer an interest in land unless it has been delivered by the grantor and accepted by the grantee. Title passes to the grantee upon effective delivery, and returning the deed to the grantor has no effect. To effect a reconveyance, the grantee must execute and deliver a new deed. Courts generally will presume that a grantee's possession of a deed means it has been delivered. Unless the grantor clearly expressed his intent that title pass to the grantee without physical delivery, the grantor's continued possession of the deed raises a presumption of nondelivery. Conversely, the grantee's possession of a properly executed deed raises a presumption of delivery. Regarding acceptance of a deed, courts generally will presume that a grantee accepts a deed if the conveyance would benefit her. In most states, acceptance is presumed if the conveyance is beneficial to the grantee, regardless of whether the grantee knows of it. In other states, acceptance is presumed only where the grantee knows of the grant and fails to reject it. Courts also will presume that a grantee accepts a deed if she is a minor. In all states, acceptance is presumed if the grantee is an infant or an incompetent.
Which of the following are specific intent crimes? A False imprisonment, kidnapping, and battery B Solicitation, assault, and burglary C Assault, battery, and first degree murder D Larceny, robbery, and arson
B. Solicitation, assault (attempted battery type) and burglary are specific intent crimes. Solicitation requires the intent to have the person solicited commit the crime. The attempted battery type of assault, like all attempt crimes, requires the specific intent to commit the crime attempted. Burglary requires the specific intent to commit a felony at the time of entry into the dwelling of another. Assault (attempted battery type) is a specific intent crime. First degree murder is a specific intent crime, in that it requires the intent to kill after premeditation and deliberation. However, battery is a general intent crime. False imprisonment, kidnapping, and battery are all general intent crimes. Larceny and robbery are both specific intent crimes that require the intent to permanently deprive another person of his interest in the property taken. However, arson is a malice crime.
The owner of a boat took two friends out on a lake near his home. One of his friends was driving the boat when it struck a partially submerged rock that the owner of the boat had forgotten to tell him about. The owner of the boat and the other passenger were injured; the driver of the boat was not hurt. In a jurisdiction that applies joint and several liability with comparative contribution, the passenger brought suit against both the boat owner and the driver, and the boat owner also sued the driver. The jury determined that the boat owner was 55% at fault and suffered $10,000 in damages, the driver of the boat was 45% at fault, and the injured passenger suffered $100,000 in damages. After entry of judgment, the boat owner paid the passenger her total damages of $100,000, while the driver of the boat has paid nothing. How much, if anything, can the boat owner recover from the driver? A $45,000, because the driver was 45% at fault. B $49,500, because the driver was 45% at fault and the boat owner suffered $10,000 in damages. C $50,000, because the boat owner and the driver are jointly liable. D Nothing, because the boat owner was more at fault than the driver.
B. The boat owner can recover $45,000 through comparative contribution for the passenger's claim and $4,500 on his own claim against the driver of the boat. Most comparative negligence states have adopted a comparative contribution system based on the relative fault of the various tortfeasors. Nonpaying tortfeasors who are jointly and severally liable are required to contribute only in proportion to their relative fault. Here, because the jurisdiction retained joint and several liability, the boat owner had to pay the passenger all of her damages. Under comparative contribution rules, the boat owner can obtain contribution from the driver for 45% of that amount, because the driver was 45% at fault. In addition, the boat owner has a direct claim against the driver for his own damages of $10,000, reduced by 55%, the amount of his fault. Thus, the total amount that the boat owner can recover from the driver is $49,500, making (B) correct and (A) incorrect. (C) is incorrect because it reflects traditional contribution rules, in which all tortfeasors were required to pay equal shares regardless of their respective degrees of fault. (D) is incorrect because a tortfeasor who was jointly and severally liable is not precluded from recovering contribution merely because he was more at fault than the other tortfeasors.
On April 1, a graduate student who owned an antique dictionary agreed to sell it to a buyer for $1,500. The written contract between the seller and the buyer provided that the dictionary would not be delivered to the buyer until April 20. Late on April 15, a fire swept through the seller's apartment building, through no fault of the seller, and the dictionary was destroyed. Fortunately for the seller, he had insurance that covered all of his damages, including compensation for the destroyed dictionary. On April 20, the seller told the buyer of the fire, but still demanded payment, claiming that the buyer was the equitable owner of the dictionary when it was destroyed, and told her that she could have obtained insurance on the dictionary had she wanted to, because she had an insurable interest in the dictionary as soon as the contract was made. The buyer refused to pay. The seller brings an action against the buyer for the $1,500. Who will prevail? A The buyer, because the seller was fully compensated for his dictionary and making the buyer pay would therefore result in unjust enrichment. B The buyer, because destruction of the dictionary avoids the contract and discharges her duty to pay. C The seller, because when he contracted with the buyer, the risk of loss passed to her. D The seller, because of the doctrine of equitable conversion.
B. The buyer will prevail because complete destruction of the dictionary results in avoidance of the contract and discharge of her duty to pay, since the seller still had the risk of loss. Because the contract here is for the sale of goods, it is governed by the Uniform Commercial Code ("UCC"). Under the UCC, if a contract requires for its performance particular goods identified when the contract is made, and, before risk of loss passes to the buyer, the goods are destroyed without the fault of either party, the contract is avoided. [UCC §2-613] All of the elements of section 2-613 are present here. The contract required the seller's particular dictionary, which was identified at the time the contract was made. The risk of loss had not yet passed to the buyer because, in a sale by a nonmerchant such as the seller, risk of loss does not pass to the buyer until tender [UCC §2-509], and the seller never tendered the dictionary here (there was no actual tender and delivery was not due until April 20). Finally, the goods were destroyed by a fire and without the fault of either party. Thus, the contract is avoided. (The same conclusion would result under the common law doctrine of impossibility—all executory duties are discharged when the subsequent destruction of the subject matter of a contract renders performance impossible.) (A) is wrong because the UCC contains no such rule. The only UCC remedy that depends on an injured party's insurance involves the risk of loss after the buyer's revocation of acceptance or wrongful repudiation under section 2-510. Here, the buyer does not have to pay because the destruction of the dictionary discharged her duty to do so. (C) is wrong because, as explained above, the risk of loss had not yet passed to the buyer. (D) is wrong because the UCC does not follow the doctrine of equitable conversion; rather, the Code contains very specific risk of loss rules, as detailed above.
The state passed a law stating that "only persons living with their parents or guardians who are bona fide residents of the state shall be entitled to free public education; all others who wish to attend public schools within the state may do so, but they must pay tuition of $3,000 per semester." A 15-year-old girl moved in with her friend so that she could attend the public schools in the state, and the state legislature passed the tuition statute just as she completed her junior year. The girl wants to complete her senior year in the state high school, but cannot afford to pay tuition. If the girl sues in federal court to strike down the tuition statute, is the court likely to rule that the statute is constitutional? A Yes, provided that the state can show that the statute is necessary to promote a compelling state interest. B Yes, unless the girl can show that the statute is not rationally related to a proper state interest. C No, because it infringes on the girl's fundamental right to an education. D No, because it interferes with the girl's fundamental right to interstate travel.
B. The court is likely to rule that the statute is constitutional. A bona fide residence requirement, such as this statute, that is not based on a suspect classification and does not limit the exercise of a fundamental right, is judged by the rational basis test. Thus, (A) is incorrect. The statute provides free education for all children who are bona fide residents of the state. Thus, it uniformly furthers the state interest in assuring that services provided for its residents are enjoyed only by residents. (C) is incorrect because education is not a fundamental right. (D) is incorrect because this statute does not impair the right of interstate travel. Any person is free to move to the state and establish residence there. This statute does not deter people from moving into the state.
One night when a man was very drunk, he took one of his rifles, loaded it, and fired a bullet through his front door. Unbeknownst to him, at the time he fired the rifle, someone was driving by the house. The bullet went through the front door, through the window of the car, and killed the driver. The shooter was convicted of murder and appeals. He contends that there was insufficient evidence to support a finding of murder. How should the court of appeals rule? A That the evidence is sufficient to prove that the killing was intentional. B That the evidence is sufficient to prove that the killing was done with malice aforethought. C That the evidence is insufficient, because the shooter did not know that the driver was driving by his house and therefore he could not have acted intentionally. D That the evidence is insufficient, because at most the shooter's conduct constituted gross negligence and involuntary manslaughter.
B. The court of appeals should rule that the evidence is sufficient to prove that the killing was done with malice aforethought. Under the facts of this case, to support a finding of murder, the trial court would have to find that the shooter acted either intentionally or with malice aforethought. The facts clearly indicate that the shooter did not know of the car, so it cannot be said that he shot at it intentionally, and therefore (A) is not correct. "Malice aforethought" can mean that the defendant is acting with reckless indifference to an unjustifiably high risk to human life. There is little question that shooting a rifle through a front door shows reckless indifference to an unjustifiably high risk to human life. Thus, the question is whether the shooter's intoxication was sufficient to negate this state of mind. Voluntary intoxication is not a defense to crimes requiring malice, recklessness, or negligence. In the case of recklessness, if a defendant's lack of awareness results from voluntary intoxication, his conduct will nevertheless be deemed reckless
A thief was passing by a house under construction when he noticed that the ladder being used by workers on the roof had copper braces supporting the rungs. After making sure that the workers on the roof could not see him, the thief used pliers that he had in his pocket to remove all of the copper braces that he could reach from the ground. A short time later, a worker climbed down the ladder and it collapsed. He fell to the ground and severely injured his back. The thief was apprehended a few hours later trying to sell the copper for scrap. A statute in the jurisdiction makes it a felony for "maliciously causing serious physical injury to another." The thief was charged with malicious injury under the statute and was also charged with larceny. After a jury trial in which the above facts were presented, he was convicted of both charges. If he appeals the conviction for the malicious injury charge on grounds of insufficient evidence, how should the court rule? A Affirm the conviction, because the thief was engaged in criminal conduct at the time of the act that resulted in the injury. B Affirm the conviction, because the jury could have found that the thief acted with malice. C Reverse the conviction, because there was no evidence that the thief intended to injure anyone. D Reverse the conviction, because there was no evidence that the thief bore any malice towards the workers on the roof.
B. The court should affirm the thief's conviction. Crimes imposing a mens rea of malice generally do not require the proof of intent that specific intent crimes require. It is sufficient if the defendant recklessly disregarded an obvious or high risk that the particular harmful result would occur. Here, the facts presented were sufficient to allow the jury to conclude that the thief knew of the probability that the ladder would collapse without the braces when someone climbed down it, and acted in reckless disregard of that risk by removing the braces. (A) is incorrect because the fact that the thief was committing larceny when he removed the braces does not establish malice for purposes of the malicious injury charge. Even if his conduct were otherwise legal, he could be liable for that charge if he acted with reckless disregard of the high risk of injury. (C) is incorrect because, as discussed above, it is generally not necessary to show an intent to injure for a crime requiring a mens rea of malice; reckless disregard of an obvious risk will usually suffice. (D) is incorrect because crimes requiring a mens rea of malice do not refer to malice in the dictionary sense; a showing of ill will or hatred of the victim is not required.
A federal statute designed to stop organized crime enumerated certain activities as crimes and provided that, in addition to charging these activities as the crimes they constitute, the activities would also constitute the criminal act of intentional furtherance of the goals of organized crime. Among the enumerated activities was the interstate distribution of cocaine. The statute's constitutionality has been upheld by the Supreme Court. The defendant was arrested by federal agents after having driven a truck containing cocaine from Florida to Illinois, where he delivered his illicit cargo as directed. At trial, the defendant is convicted of interstate distribution of cocaine in violation of federal law, and convicted of a violation of the federal statute above. How may the defendant be sentenced? A Under either statute, but not both. B Under both statutes. C Only under the statute that carries a lesser maximum sentence. D Only under the statute that carries a greater maximum sentence.
B. The defendant may be sentenced under both statutes. Double jeopardy does not prohibit the imposition of cumulative sentences for two or more statutorily defined offenses specifically intended by the legislature to carry separate punishments, even though constituting the "same" crime under the Blockburger test (i.e., each offense does not require proof of some additional fact that the other does not) when the punishments are imposed at a single trial. Absent a clear intention, it is presumed that multiple punishments are not intended for offenses constituting the same crime under Blockburger. Here, it is clear that Congress, in enacting the statute, intended that certain offenses, such as interstate distribution of cocaine, be subject to separate punishments. (B) is the only alternative that expresses the view that the defendant may be sentenced under both statutes. Thus, it is the correct answer, and (A), (C), and (D) are incorrect.
A defendant is on trial for manslaughter after he hit a victim in a bar, causing the victim to fall and hit his head on the marble bar top. The defendant claims that he hit the victim in self-defense after the victim lunged at him with a knife. During the prosecution's case, a witness testifies that she heard the victim's friend shout at the defendant, "You just killed a helpless man!" A defense witness is called to testify that he was there and does not remember hearing the victim's friend say anything. Should the defense witness's testimony be admitted? A No, it is irrelevant to any issue in the case. B Yes, it is proper impeachment of the prosecution's witness. C No, it is improper impeachment of the prosecution's witness because it relates to a collateral matter. D No, it is improper impeachment because it does not positively controvert the prosecution witness's testimony, as the defense witness merely says he does not remember.
B. The defense witness's testimony should be admitted as proper impeachment of the prosecution's witness. Impeachment is the casting of an adverse reflection on the veracity of a witness. A witness may be impeached by either cross-examination or extrinsic evidence, such as by putting other witnesses on the stand who contradict the witness's testimony. Here, the defense is using the testimony of its witness to impeach the prosecution witness's testimony as to what the victim's friend said. This is proper. (A) is wrong because a witness's credibility is always relevant. Furthermore, the defense witness's testimony relates to a crucial issue in the case; i.e., whether the victim was armed with a knife or was "helpless." Thus, the testimony is relevant. (C) is wrong because this is not a collateral matter. Impeachment on a collateral matter is prohibited, but a collateral matter is one that arises when a witness makes a statement not directly relevant to the issues in the case. The prosecution witness's statement about the victim being "helpless" is directly relevant to the issue of the defendant's self-defense claim and, thus, is a proper subject of impeachment. (D) is wrong because impeachment evidence need not positively controvert the prior testimony; it need only tend to discredit the credibility of the prior witness.
A landowner conveyed her parcel of land to "my brother and my sister jointly, with right of survivorship." Shortly thereafter, the brother was in an automobile accident. The driver of the other vehicle sued the brother on a theory of negligence, and obtained a judgment in the amount of $250,000. Because the brother did not have insurance or enough cash to satisfy the judgment, the driver levied on the brother's interest in the land. What interest will the driver most likely take? A None, because the brother's interest in the land cannot be partitioned. B An undivided one-half interest, regardless of whether the brother and the sister's title to the land is construed as a joint tenancy or a tenancy in common. C An undivided one-half interest, assuming the brother and the sister's interest is construed as a tenancy in common and not a joint tenancy. D A contingent right of survivorship that will vest if the brother survives the sister.
B. The driver will get an undivided one-half interest in the land regardless of the status of the brother and the sister's title. A joint tenancy is a concurrent estate with a right of survivorship, while a tenancy in common does not have a right of survivorship. At common law, the conveyance here would qualify as a joint tenancy because the unities of time, title, interest, and possession are present in the conveyance. Although under modern law a joint tenancy must be created with specific language or else it will be presumed to be a tenancy in common, the conveyance here still would probably qualify as a joint tenancy, even though it did not use the words "joint tenancy," because it contained the "right of survivorship" language. However, regardless of whether the estate is characterized as a joint tenancy or a tenancy in common, one tenant's interest may be transferred without the consent of the other tenant, and a creditor may levy on the interest. In most jurisdictions, a lien against one joint tenant's interest does not sever the joint tenancy until the lien holder proceeds to enforce it by foreclosure. At that point, the purchaser at the foreclosure sale will hold the property as a tenant in common with the other tenant, but will still have an undivided one-half interest in the property unless and until he brings an action to partition the estate. (A) is incorrect because both joint tenancies and tenancies in common may be subject to partition. (In contrast, tenancies by the entirety cannot be terminated by involuntary partition.) (C) is incorrect because, as discussed above, a joint tenant may validly convey or encumber his interest in the property. (D) is incorrect because the driver does not have a contingent interest; she has a present lien on the brother's interest that can be enforced immediately by foreclosure, which would sever the joint tenancy.
A state statute has detailed classifications of civil servants for both state and city positions. It provides that all civil servants who have been employed for over 18 months may be dismissed only for "misconduct" and also requires that state and city agencies comply with all procedures set forth in any personnel handbook issued by that agency. The personnel handbook of the state tollway authority sets forth detailed procedures for dismissal of civil servant employees. The handbook provides that written notice of the grounds for dismissal must be given to the employee prior to dismissal, and that the employee must, on request, be granted a post-dismissal hearing within three months after the dismissal takes effect. An employee is entitled to present witnesses and evidence at the post-dismissal hearing, and is entitled to reinstatement and back pay if the hearing board decides that the employer has not shown by a preponderance of the evidence that the dismissal was justified. A state tollway employee who had been employed for three years recently was fired. After an investigation by state auditors, the employee was notified by registered letter that he was being dismissed because of evidence that he took bribes from construction firms in exchange for steering contracts to them. He was informed of his right to a hearing and requested one as soon as possible. Three weeks after his dismissal, the state personnel board conducted a hearing at which the employee denied the charges and presented witnesses to attest to his honesty. At the conclusion of the hearing, the board upheld his dismissal, finding that it was supported by a preponderance of the evidence. If the employee files suit in federal court challenging his dismissal on constitutional grounds, will he be likely to prevail? A Yes, because the employee had a right to a pretermination hearing at which he could present witnesses to support his side of the story. B Yes, because the employee had a right to have an opportunity to respond to the charges prior to his dismissal. C No, because the state may establish the required procedures for terminating an interest that it created by statute. D No, because the procedures taken for termination of the employee's job satisfied due process requirements.
B. The employee will likely prevail because the procedures taken to terminate his employment did not satisfy due process. Under the Due Process Clause of the Fourteenth Amendment, a public employee who is subject to removal only for "cause" under a statute, ordinance, or personnel document has a property interest in continued employment that cannot be taken away without due process of law. The Court has held that such an employee generally must be given notice of the charges and a pretermination opportunity to respond to those charges. The employee must also be given a subsequent evidentiary hearing regarding the termination (with reinstatement if the employee prevails). [Cleveland Board of Education v. Loudermill (1985)] Here, the employee was notified of the charges but was not given any opportunity to respond to the charges until after his termination. Hence, his termination did not satisfy due process requirements. (A) is incorrect because the employee does not have to be given a full, formal hearing before his termination, as long as he is given oral or written notice of the charges, an explanation of the employer's evidence, and an opportunity to tell his side of the story. (C) is incorrect because the fact that the state created the employee's property interest in his job does not permit the state to define what procedures may be used to terminate the interest. The procedures to which the employee was entitled are determined by independent constitutional standards. (D) is incorrect because, as discussed above, the procedures followed here did not satisfy due process standards. The Supreme Court has held that an employee can be suspended from his job without a prior hearing if the government has a significant reason for removing the employee from the job and providing him with only a post-termination hearing. [Gilbert v. Homar (1997)—campus police officer suspended after being arrested and charged with felony drug offense] Here, however, there is no substantial reason why the employee could not have been given the opportunity to respond to the charges prior to dismissal.
A father purchased a tract of land, financing a large part of the purchase price by a loan from a bank that was secured by a mortgage on the land. A provision in the mortgage agreement, which had an acceleration clause, provided that a defaulting borrower waives his right to redeem once foreclosure proceedings have started. The bank properly recorded its mortgage. Several years later, the father needed money to send his twin daughters to college, so he obtained a loan from a credit union, also secured by a mortgage on the land. The credit union properly recorded its mortgage. The following year, the father became ill and was unable to make payments to either the bank or the credit union due to his high medical bills. The balance on the loan from the bank was $75,000, and the balance on the credit union loan was $25,000. The bank instituted foreclosure proceedings in a jurisdiction that provides a statutory right of redemption. The day before the judicial sale, the father inherited $100,000 from his aunt. He quickly contacted the bank and offered to pay off both loans in full. The bank refused because it was hoping to buy the now valuable property at the judicial sale. If the father seeks to force the bank to accept his offer, will he likely prevail? A Yes, because the jurisdiction has a statutory right of redemption. B Yes, because equity requires a creditor to accept such an offer. C No, because his agreement with the bank waived his right to redeem once foreclosure proceedings started. D No, because he lost all of his rights in the property when he defaulted on the loan.
B. The father will win because he is exercising his equity of redemption rights. The equity of redemption gives the borrower the right to free the land of the mortgage by paying off the amount due, plus any accrued interest, at any time prior to the foreclosure sale. If the borrower has defaulted on a mortgage with an acceleration clause, he must pay the full balance in order to redeem. Here, the father's offer to pay both the bank's and the credit union's loans is adequate to redeem the land. (A) is incorrect because a statutory right of redemption, recognized in about half the states, gives the borrower a right to redeem for the foreclosure price after the foreclosure sale. (C) is incorrect because this right to redeem cannot be waived in the agreement establishing the security interest. This would be "clogging the equity of redemption." However, the right can be waived later for consideration. (D) is similarly incorrect because a defaulting debtor does not lose the equity of redemption.
Two robbers planned to commit armed robberies targeting older victims. However, when the time came to actually commit the robbery, one of the robbers, thinking that the potential victim looked too much like his grandmother, backed out and told his cohort that he was going home. The second robber went ahead with the plan and robbed the elderly victim, who died of a heart attack due to the stress of the robbery. The second robber was arrested and implicated the first robber. What crime(s) has the first robber committed? A No crimes. B Conspiracy. C Murder. D Murder and conspiracy.
B. The first robber is guilty of conspiracy but not murder. Conspiracy consists of: (i) an agreement between two or more persons; (ii) an intent to enter into an agreement; and (iii) an intent to achieve the objective of the agreement. In addition, most states require an overt act in furtherance of the conspiracy, although an act of mere preparation will usually suffice. The conspiracy was complete when the robbers agreed to commit the robbery and targeted their first victim. The first robber's withdrawal is no defense to the conspiracy charge because a conspiracy is complete as soon as an agreement is made and an overt act is committed. Hence, (A) is incorrect
In which of the following situations is there sufficient intent to constitute larceny? A Taking goods with the belief that the taker is entitled to them as repayment for a debt of the owner. B Taking goods that are not for sale with the intent to pay the owner. C Taking goods with the intent to return them and receive a reward. D Taking goods with the intent to return them within a reasonable time.
B. Taking goods that are not for sale with the intent to pay the owner constitutes larceny. If the goods taken are not for sale, the fact that the defendant intends to pay the owner for them does not negate the larceny. (If the goods are for sale and the defendant has a specific and realistic intent to repay the owner, the taking is not larceny.) Taking goods with the intent to return them within a reasonable time does NOT constitute larceny if at the time of the taking the defendant has a substantial ability to do so. However, many states make it a crime to borrow a motor vehicle, even when the borrower fully intends to return it ("joyriding"). Taking goods with the honest belief that the taker is entitled to them as repayment for a debt of the owner is NOT larceny, but the goods must not be worth more than the amount of the debt. In these situations, the defendant believes the property is hers and therefore lacks an intent to deprive someone else of his property. Taking goods, intending to return them and receive a reward is NOT larceny . However, if the defendant takes them, not intending to return them unless she is assured of a reward, she has committed larceny, because she has created a substantial risk of loss.
The owner of a garage and one of his mechanics had a dispute over the amount of wages due for work that the mechanic had done on cars. The owner alleged that he paid the mechanic for work that the mechanic did not perform, while the mechanic contended that she did the work. After arguing over the matter for over an hour, the mechanic quit. The owner did not allow the mechanic to take her tools with her, stating that he was keeping her tools until she repaid the money that he had overpaid her. The mechanic met with a friend and discussed the situation with him. After hearing what had happened, the friend, believing that the owner was unlawfully retaining the mechanic's tools, suggested that he could pose as a garage client and retrieve the tools for the mechanic, and the mechanic agreed. The plan succeeded, except for the fact that the friend took tools that belonged to a new employee. The owner immediately realized what had happened, and he gave the friend's license number to the police. The friend was arrested before he could show the tools to the mechanic. Of the following, which is the best argument for the friend as a defense to a charge of larceny in a common law jurisdiction? A He had the consent of the tools' owner. B He thought the tools belonged to the mechanic. C He intended to return the tools to the mechanic rather than keep them. D He was apprehended by the police before the mechanic could inspect the tools
B. The friend's best argument is that he thought the tools belonged to the mechanic. Larceny is the taking and carrying away of the tangible personal property of another by trespass with the intent to permanently (or for an unreasonable time) deprive the owner (or person in possession) of his interest in the property. Thus, larceny is a specific intent crime, in that the defendant must intend to permanently deprive the person of his interest in the property. Importantly, this intent generally must exist at the time of taking (save for the continuing trespass doctrine). In this case, the friend believed that the mechanic was being unlawfully deprived of his property, and his intent was to return the tools to their rightful owner. Thus, when he took the tools, he lacked the intent to permanently deprive another of his interest in the property. This would be true even if the garage owner factually had a lawful right to retain the tools, as the mistake of law would negate the friend's intent to permanently deprive another of his interest in the property. (A) is wrong, of course, because he did not have the consent of the owner of the tools he took. (C) is wrong because it is not a defense to a charge of larceny that the defendant did not intend to keep the item taken for his own use. (D) is wrong because it prematurely raises the continuing trespass doctrine. Under the continuing trespass doctrine, a defendant may be convicted of larceny when he initially takes property with a wrongful state of mind, but without the intent to permanently deprive the owner of the property, and then later forms the intent to permanently deprive the owner of the property. Here, the friend had not yet had the opportunity to form that later intent (even assuming the initial taking was with a wrongful state of mind). Thus, (D) does not address the correct issue.
The owner of a corner lot allowed a hedge on his property to become overgrown, obstructing the view of motorists at that corner. Two motorists were driving inattentively and each ran a stop sign at the intersection bordering the lot. Their cars collided in the intersection and one of the motorists was injured. She sued the owner of the lot. The jury determined that the lot owner was 10% at fault and each of the motorists was 45% at fault. Will the injured motorist recover damages from the lot owner? A Yes, because she was not more than 50% at fault. B Yes, because it was foreseeable that motorists could be injured if the hedge was not cut back. C No, because the other motorist's negligence was a superseding cause of her injuries. D No, because the lot owner's fault was slight compared with the motorist's fault.
B. The injured motorist may recover damages from the lot owner because the jury found that the lot owner should have foreseen that motorists could be injured if the hedge was not cut back. The lot owner owes the duty of an owner and occupier of land to those off the premises for unreasonably dangerous artificial conditions. In contrast to overgrown weeds, which are a natural condition for which no duty is owed absent a statute, a hedge is considered an artificial condition, analogous to a fence. Hence, by letting the hedge become so large that it created a foreseeable danger to motorists by obstructing their vision, the lot owner has breached his duty to the motorist. The other elements of the motorist's negligence action (besides a duty and a breach of the duty) are actual and proximate cause, and damages. The motorist can establish actual cause by showing that, although she failed to notice the stop sign, she would have noticed another car traveling on a collision course with hers; i.e., but for the overgrown hedge, the motorist would have been able to avoid the accident. Proximate cause in an indirect case such as this can be established by showing that any intervening forces were foreseeable and not superseding. The other motorist's negligent failure to stop may also have been caused in part by the overgrown hedge and is a foreseeable intervening force that does not break the chain of causation. Hence, (C) is incorrect. Under pure comparative negligence, the injured motorist can recover 10% of her damages even though her fault was greater than that of the lot owner. (A) is incorrect because a pure comparative negligence jurisdiction allows a plaintiff to recover no matter how great her negligence is. Thus, even though the motorists are both 45% at fault and the lot owner is only 10% at fault, the motorist can recover 10% of her damages from the lot owner. (D) is incorrect because, as discussed above, the lot owner is still liable for some damages in a pure comparative negligence jurisdiction even if his fault was only slight.
A plaintiff sued a defendant for negligence when the tractor that he was driving at a construction site collided with the plaintiff's car. The plaintiff alleged that she was driving in a proper lane when the tractor collided with her car. The plaintiff's counsel called the responding police officer to testify that the defendant's employee made a statement to the police officer, in the defendant's presence, that the defendant "accidentally went too far into traffic," and the defendant did not say anything. Should the trial judge rule that this evidence is admissible? A Yes, because the employee's statement is a statement against interest. B Yes, because silence may be deemed an admission. C No, because the employee's statement was hearsay, and the defendant's silence is also hearsay. D No, because the employee has not yet testified to his statement.
B. The judge should rule the evidence admissible as an admission. Admissions—statements by or attributable to a party and offered against that party—are not considered hearsay under the Federal Rules. If a party fails to respond to accusatory statements where a reasonable person would have spoken up, his silence may be considered an implied admission. Although it is arguable that a person who may be liable for negligence would reasonably remain silent when in the presence of a police officer, this is the best answer. (D) is wrong because there need be no foundation evidence. (C) is wrong; the silence is deemed the adoption of the employee's statement. Thus, both the employee's statement and the defendant's silence would be admissible. (A) is wrong because the statement against interest exception does not apply here. To fall within the exception, a statement must be against the declarant's own interest when made—here, the employee's statement was against the defendant's interest, not his own. Furthermore, the exception requires the unavailability of the declarant (i.e., the employee), and there is no indication that the employee is unavailable.
A landlord rented an art studio to an artist. Under the terms of the signed, written, two-year lease, the artist agreed to pay the landlord $1,000 per month and to assume responsibility for all necessary repairs. After the first year of the lease, the artist assigned the balance of his lease to a sculptor. The landlord approved the sculptor as a tenant and accepted two rent payments from her, and then the landlord sold the building to an investor. The sculptor had made two payments to the investor when an electrical fire broke out in the studio, injuring the sculptor. The fire was caused by faulty wiring. The landlord was aware that there was a dangerous wiring problem when he leased the property to the artist. But when the landlord discovered how costly repairs would be, he decided it would be more profitable to sell the property than to repair it. The problem was not easily discoverable by anyone other than an expert electrician, and the landlord did not tell the artist, the sculptor, or the investor about the problem. The sculptor sues to recover damages for her injuries. From whom can the sculptor recover? A The investor, because she breached the implied warranty of habitability. B The landlord, because he failed to disclose a latent defect. C The artist, because the artist is considered the sculptor's landlord. D No one, because the covenant to repair runs with the land, and the sculptor is bound by it.
B. The landlord is liable for the sculptor's injuries because he failed to disclose a latent defect. If, at the time the lease is entered into, the landlord knows of a dangerous condition that the tenant could not discover upon reasonable inspection, the landlord has a duty to disclose the dangerous condition. Failure to disclose the information about the condition results in liability for any injury resulting from the condition. Because the landlord knew of the dangerous electrical problem at the time he leased the premises to the artist and did not disclose it to either the artist or the sculptor, he is liable for any injuries resulting from that condition. (A) is wrong for two reasons: (i) the implied warranty of habitability does not apply to commercial leases; and (ii) even if this were a residential lease, it is doubtful that the investor would be liable for a condition of which she had no knowledge or notice. (C) is wrong because it describes the relationship between the artist and the sculptor as though there had been a sublease, when the facts clearly state that the artist assigned the balance of the lease to the sculptor. If a tenant sublets the premises (i.e., the tenant retains part of the remaining term), the tenant is the landlord of the sublessee. The sublessee cannot sue or be sued by the landlord. However, if there has been an assignment (i.e., the tenant makes a complete transfer of the entire term remaining), the assignee is substituted for the original tenant and can sue or be sued by the landlord. The original tenant's relationship to the assignee is at most that of a surety. Here, because the artist transferred the balance of his lease to the sculptor, there was an assignment and thus the artist cannot be considered the sculptor's landlord. (D) is wrong because, as stated above, the sculptor can recover from the landlord. The statement that the covenant to repair runs with the land and binds the sculptor is true, but the landlord's failure to disclose a dangerous preexisting condition renders the landlord liable for the sculptor's injuries despite the covenant; i.e., the sculptor's covenant does not relieve the landlord of his tort liability.
A retiree purchased a rustic cabin on a small plot of land near the center of a landowner's large parcel of land. The deed to the land, which the landowner delivered to the retiree for fair consideration, did not specifically grant an easement over the landowner's property to reach the public highway bordering her land. There were two means of access to the cabin from the public roads: a driveway from the county road on the south, and a private road from the highway on the east. The landowner told the retiree that he could use the private road from the highway. Twice during his first two years at the cabin, the retiree took the driveway from the county road instead; at all other times he used the private road. At the end of his second year at the cabin, the retiree began reading tarot cards to supplement his retirement income. He had a steady stream of clients coming to his home at all hours of the day and night. Most of the clients came in on the driveway from the county road, which ran close to the landowner's home. The landowner objected, and told the retiree that neither he nor his clients had any right to use that driveway and that they must use the private road from the highway. The retiree refused, and he and his clients continued to use the driveway from the county road for three years. Finally, the landowner began blocking off the driveway from the county road. The retiree brought suit to enjoin this practice. The prescriptive period in this jurisdiction is five years. Who will most likely prevail? A The landowner, because the tarot business has changed the nature of the use of the easement by necessity. B The landowner, because she may select the location of the easement. C The retiree, because he has a valid easement by necessity in the driveway from the county road. D The retiree, because he has acquired an easement by prescription in the driveway from the county road.
B. The landowner will prevail in a suit because she, as the holder of the servient estate, has the right to choose the location of an easement by necessity. An easement by necessity arises when the owner of a tract of land sells a part of the tract and by this division deprives one lot of access to a public road or utility line. The owner of the servient parcel has the right to locate the easement, provided the location is reasonably convenient. The landowner has chosen the private road from the highway; thus, the retiree has no right to use the driveway from the county road. Both (A) and (C) are incorrect because the retiree has no easement by necessity in the driveway. As stated above, the owner of the servient parcel (the landowner) has located the easement in the private road; thus, no easement in the driveway exists. When the owner of an easement uses it in a way that exceeds its legal scope (i.e., the easement is surcharged), the servient landowner may enjoin the excess use and possibly collect damages. If the easement by necessity had been located in the driveway, the excess use from the tarot business could have been the basis for the court's ruling in the landowner's favor. However, as stated above, the easement is in the private road from the highway. (D) is incorrect because the retiree's use has not been continuous for the five-year period. To acquire an easement by prescription, the use must be: (i) open and notorious, (ii) adverse, and (iii) continuous and uninterrupted for the statutory period. Continuous adverse use does not mean constant use. Periodic acts that put the owner on notice of the claimed easement fulfill the requirement. In this case, however, two uses in the first two years would not be sufficient to put the landowner on notice that the retiree intended to claim an easement in the driveway. Therefore, the retiree has not acquired a prescriptive easement in the driveway from the county road.
The state legislature of state A enacted legislation prohibiting the use of tractor-trailer rigs weighing more than 100,000 pounds gross, on the basis that superheavy trucks rapidly degrade the state's roadways and pose a greater safety danger than smaller trucks. A trucking firm that frequently uses state A's highways for trips between state B and state C purchased several tractor-trailer rigs weighing over 100,000 pounds when loaded. The trucking firm brings an action for declaratory relief in federal court in state B, seeking to have the state A legislation declared unconstitutional. It presents expert testimony that the heavier trucks are no less safe than smaller models. State A produces no evidence, but asserts that the legislation is justified as an exercise of its police power. How should the trial court rule? A That the legislation is an unconstitutional violation of the trucking firm's Fourteenth Amendment rights to due process of law. B That the legislation is unconstitutional because it violates the Commerce Clause. C That the legislation is a valid exercise of the state's police power to regulate highway safety. D That the evidence of the damage done to the state's highways by the superheavy trucks is sufficient to uphold the legislation independently of the safety argument.
B. The legislation violates the Commerce Clause. As a general matter, a state may regulate in ways that impact on interstate commerce as long as the regulation does so only indirectly and the benefits outweigh the burdens imposed by compliance with the regulation. [Kassel v. Consolidated Freightways Corp. (1981)] When, as here, only a bare assertion that the regulation would increase safety is involved, a court will generally find that the regulation is invalid. This does not mean that a state could not prevail if it proved that the benefits of the regulation do in fact outweigh the burdens. Indeed, the Court intimated that such would be the case in Bibb v. Navajo Freight Lines, Inc. (1959). But state A has not made a sufficient showing here, and the trucking firm has presented colorable "expert" evidence to the contrary. (A) is incorrect because economic and social regulations are tested at the rational basis level for due process purposes, and even the minimal showing here would suffice for the state. (C) states a correct premise (i.e., that the state is regulating for highway safety), but an incorrect result, and is wrong. (D) is wrong because the state has in fact made no such showing before the court. Thus, while there is a normal presumption of constitutionality, the state here has not met its burden in defending the measure in the face of contrary, expert evidence.
The plaintiff, an electrical contractor, sued the defendant homeowner for refusal to pay for extensive wiring repairs performed on his home by the plaintiff's employee. The plaintiff called the employee to the stand. The employee, under oath, testified that he did not perform any work at the defendant's home. The employee also denied writing a letter to a friend telling the friend that the employee was going to do electrical work on the home. Without releasing the employee as a witness, the plaintiff offers into evidence the letter written by the employee to his friend. If the employee's letter to his friend is properly authenticated, should the court admit the letter? A Yes, for impeachment purposes only. B Yes, as both substantive and impeachment evidence. C No, because a party may not impeach his own witness. D No, because it is inadmissible hearsay.
B. The letter is admissible as substantive evidence as well as for impeachment purposes. For the purpose of impeaching the credibility of a witness, a party may show that the witness has, on another occasion, made statements that are inconsistent with some material part of his present testimony. This may be done by first questioning the witness as to the prior inconsistent statement that he has made. If the witness denies having made the statement or fails to remember it, the making of the statement may be proved by extrinsic evidence. A proper foundation must be laid by giving the witness an opportunity to explain or deny the statement, and it must be relevant to some issue in the case. Here, the plaintiff's employee has denied that he wrote the letter to his friend. The plaintiff can then impeach the employee by offering the letter into evidence. Because the employee has not been released as a witness, he will have an opportunity to explain or deny the statement, and it is relevant to whether any work was done at the home. Because prior inconsistent statements are generally hearsay, they often are admissible only for purposes of impeachment. The Federal Rules do categorize a testifying witness's prior inconsistent statement as nonhearsay if it was made under penalty of perjury at a prior trial, hearing, or proceeding, or in a deposition. Here, of course, the employee's letter to the friend was not made under oath, so it is hearsay. However, it is still admissible as substantive evidence because it falls within an exception to the hearsay rule. Under Rule 803(3), a statement of a declarant's then-existing state of mind is admissible as a basis for a circumstantial inference that the declarant acted in accordance with his state of mind. [See also Mutual Life Insurance Co. v. Hillmon (1892)] The employee's statement that he was going to do electrical work on the home is admissible as circumstantial evidence tending to show that he followed through with his plans and did the electrical work, which is what the statement is being offered to establish. In this case, therefore, the letter should be admissible as both substantive and impeachment evidence, making (B) correct and (A) incorrect. (C) is incorrect because the Federal Rules provide that the credibility of a witness may be attacked by any party, including the party calling him. [Fed. R. Evid. 607] (D) is incorrect. The letter is hearsay because it is being offered to prove the truth of the matter asserted—that the employee was going to do electrical work on the home—as a basis for inferring that the employee did do the work. Additionally, it is not categorized as nonhearsay under the Federal Rules because it was not made under oath. However, as discussed above, it falls within the "present state of mind" exception to the hearsay rule.
Two robbers planned to rob a local convenience store, with one using a gun to force the clerk to turn over all of the money in the cash register while the other stood lookout near the door. The robbery did not go as planned. Instead of turning over any cash, the store clerk tried to disarm the gunman. During their struggle for the gun, the lookout decided that her best course of action was to grab what she could and flee the scene. The lookout took a newspaper and a bag of potato chips and ran out of the store. On her way out, she heard a gunshot. Later that day, she learned from news accounts that the gun accidentally discharged, killing the gunman. After an investigation, the lookout was arrested. If the lookout is charged with felony murder, what would be her most promising defense? A She did not intend for the gunman to get killed. B The only person killed was the gunman. C The killing occurred after the robbery was over. D The robbery was not a felony because the items that the lookout took had only minimal value.
B. The lookout's best defense is that the gunman was the only person killed. Under the felony murder doctrine, a killing committed during the course of a felony is murder, malice being implied from the intent to commit the underlying felony. However, under the majority view, criminal liability for murder cannot be based on the death of a co-felon from resistance by the victim or police pursuit. Thus, given that the gunman's death resulted from an act by the clerk, the victim of the robbery, the lookout cannot be found guilty of the felony murder of the gunman, a co-felon. (A) is incorrect because any desire or lack of desire by the lookout to see her co-felon harmed is irrelevant to liability for felony murder. The only mens rea required is the intent to commit the underlying felony. Here, the lookout had the intent to commit robbery, the underlying felony. From this intent, the malice required for murder is implied. (C) is incorrect because the fact that the felony was technically completed before the gunman's death does not prevent the killing from being felony murder. A death caused while fleeing from the crime is considered to have been caused during the commission of the felony. (D) is incorrect because robbery is a felony regardless of the value of the property that is taken.
A state statute provided for the suspension or revocation of a retail business license where there have been "repeated violations" of specified regulatory statutes by any one individual or business. A store owner had already been fined three times under the statute. Several days later, the store owner received a notice from the regulatory agency that was created by the statute stating that his business license will be suspended for 21 days for violation of the statute. If the store owner files suit to set aside the suspension, will the owner most likely prevail? A Yes, because the statute fails to set forth the exact number of violations that a business would have to commit before its business license is revoked. B Yes, because the store owner has not been given a hearing before the agency. C No, because the agency could by its rules construe the term "repeated violations" in such a manner as to comport with due process requirements. D No, because the agency is merely a regulatory agency, and therefore the store owner has no right to a hearing.
B. The most obvious problem in this situation is that the store owner has not been afforded the right to a hearing either before or after his license was suspended. A business license is a valid property right, and procedural due process under the Fourteenth Amendment requires notice and an opportunity to be heard before the government may deprive a person of property. Thus, the store owner should be granted a hearing before his license can be suspended. Thus, (C) is wrong, because regardless of how the agency construes violations of the statute, revoking a business license still involves interference with a property right, and a hearing is thus required before this can be done. The agency obviously had enforcement powers; therefore, (D) is wrong. (A) is an arguable complaint by the store owner, but, because there appears to be a regulatory agency that can make rules regarding the enforcement of the statute, the agency can construe "repeated violations" so as to comport with due process requirements.
A defendant is being tried for the murder of a bank teller, which occurred during the robbery of a bank. At trial, a witness, who knew the defendant, is called to testify that on the day after the robbery he saw the defendant buying some groceries, and when the defendant removed a large roll of money, the witness had asked, "You didn't steal that from someone, did you?" The defendant nodded. This evidence is: A Admissible, as an excited utterance. B Admissible, because it is not hearsay. C Inadmissible, because it is hearsay not within any exception. D Inadmissible, because the defendant had no reason to respond to this statement.
B. The nod constitutes nonverbal conduct intended as an assertion and would thus be considered a "statement" for purposes of the hearsay rule. However, this statement constitutes a statement of a party-opponent and hence is not hearsay under Federal Rule 801. (C) is therefore incorrect. Likewise, (A) is incorrect, because (i) an excited utterance is an exception to the hearsay rule and this is not hearsay, and (ii) even if it were hearsay, this does not constitute an excited utterance because the statement was not made during or soon after the startling event and under the stress of that event. (D) is incorrect. The defendant responded to the question with a nod. If the defendant had failed to respond and the prosecutor wished to introduce his silence as an adoptive admission, then it would be necessary to determine whether there was a reason to respond.
A tenant agreed in writing to lease a retail site in a shopping mall from the owner of the property. The term of the tenancy was two years, and rent was payable in monthly installments at the beginning of each month. At the end of the second year, there had been no discussions between the tenant and the owner regarding renewal or termination. The tenant did not vacate the premises at the end of the term; instead, she sent a check for the next month's rent to the owner. The owner cashed the check and then informed the tenant that he was holding her to a new tenancy and a rent increase of 10%. What is the status of the tenancy that the owner created? A A month-to-month tenancy for the original rent amount. B A year-to-year tenancy for the original rent amount. C A month-to-month tenancy for the increased rent amount. D A tenancy at will, terminable at any time, for the increased rent amount.
B. The owner can hold the tenant to a year-to-year tenancy for the original amount. When a tenant continues in possession after the termination of her right to possession, the landlord has two choices of action: He may treat the hold-over tenant as a trespasser and evict her under an unlawful detainer statute, or he may, in his sole discretion, bind the tenant to a new periodic tenancy, in which case the terms and conditions of the expired tenancy apply to the new tenancy. Unless a residential lease is involved, a year-to-year tenancy results from holding over if the original lease term was for a year or more. The new tenancy has the same terms as the original tenancy unless the landlord notified the tenant before termination of the original tenancy that occupancy after termination will be at an increased rent. Here, the original lease was a commercial lease for a two-year term, so the owner's decision to hold the tenant to a new tenancy makes it a year-to-year tenancy. However, because the owner did not notify the tenant of the rent increase prior to the end of the term, the new tenancy is at the original amount of rent. (A) is wrong because the lease here is not a residential lease; thus, the periodic tenancy created is a year-to-year tenancy rather than a month-to-month tenancy. (C) is wrong for the same reason that (A) is wrong and also because the new tenancy is at the original amount of rent, as discussed above. (D) is wrong because when a landlord elects to bind a hold-over tenant to a new tenancy, it will be a periodic tenancy rather than a tenancy at will.
The search incident to arrest exception to the warrant requirement applies: A Only if the police fear for their safety B After any constitutional arrest C After any arrest, even if the arrest itself is unconstitutional D Only if the arrest is for a felony crime
B. The police may conduct a search incident to arrest after any constitutional arrest. The Fourth Amendment prohibits unreasonable searches and seizures. Generally, for a search to be valid, it must be pursuant to a warrant issued by a neutral and detached magistrate and based on probable cause to believe that seizable evidence or fruits of a crime will be found on the premises to be searched. However, searches incident to arrest are an exception to the general rule. The choice that this search can be made after any arrest, even if it is unconstitutional is wrong. The police may conduct a search incident to an arrest whenever they arrest a person, even if the arrest is invalid under state law, as long as the arrest was constitutionally valid (e.g., reasonable and based on probable cause). Although the exception is based on safety concerns, the Supreme Court has held that it is not necessary that the police fear for their safety for a warrantless search to be conducted following an arrest. The search may be conducted incident to arrest in any case, as long as it is conducted contemporaneously with the arrest. It is not true that the exception applies only in the case of felony arrests. As discussed above, the exception applies after any lawful arrest.
A man was driving very erratically when he was stopped by state troopers and arrested for drunk driving. He was advised of his constitutional rights and elected to remain silent. At trial for his drunk driving charge, the man testified in his own defense, stating that he had just left his doctor's office and had been administered medication without being told that it would seriously and immediately hamper his coordination. On cross-examination, the prosecutor asked whether the defendant just made up this medication story after the fact to evade legitimate liability for driving while intoxicated and the man said he had not. The prosecutor then asked why the defendant had not told the arresting officer about the medication, and defense counsel objects. The trial court should rule that the question is: A Improper, because to require the defense to inform the prosecution of defendant's testimony prior to trial would be unconstitutional pretrial discovery. B Improper, because use of defendant's post-arrest silence violates his right to due process of law. C Proper, because defendant's silence was not used as direct evidence but only for impeachment on cross-examination. D Proper, because defendant's post-arrest silence is a prior inconsistent statement which is admissible to show recent fabrication.
B. The question was improper because it effectively comments on the defendant's post-arrest silence and thus violates his right to due process of law. A prosecutor may not comment on the defendant's silence after being arrested and receiving Miranda warnings. The warnings carry an implicit assurance that silence will carry no penalty. Thus, the defendant's invocation of his right to remain silent cannot be used as evidence against him at trial, nor can he be questioned or cross-examined about his decision to remain silent, even if he testifies at trial. Thus, (B) is the correct answer, and (C) is wrong. (Distinguish: If the defendant fails to disclose potential exculpatory evidence to the police after waiving his right to remain silent, he may be cross-examined at trial on that failure in an effort to show that it is a recent fabrication. In this case, however, the defendant invoked his right to remain silent.) (A) is wrong; the question is improper, but not because it would be unconstitutional pretrial discovery. (D) is also wrong. Although statements made without proper Miranda warnings can sometimes be used to impeach, the defendant's silence would not be classified as a prior statement.
A defendant was charged with arson (a felony) of an antique shop. Only one corner of the shop was damaged before the fire was extinguished. Under a plea agreement, the defendant pled guilty and received a suspended sentence. Because the owner of the shop had not yet insured a recently acquired 400-year-old refectory table that was destroyed by the fire, he sued the defendant for damages. At trial, the owner offers the properly authenticated record of the defendant's conviction for arson. Should the record be admitted into evidence? A Yes, as proof of the defendant's character in order to infer liability. B Yes, as proof that the defendant set the fire. C No, because the conviction was not the result of a trial. D No, because it is hearsay not within any exception.
B. The record of the defendant's conviction should be admitted to prove that the defendant set the fire. The record of the conviction is hearsay; i.e., it is a statement, other than one made by the declarant while testifying at the trial or hearing, offered to prove the truth of the matter asserted. Under the Federal Rules, however, such judgments fall within the hearsay exception for records of felony convictions. Under the Federal Rules, judgments of felony convictions are admissible in both criminal and civil actions to prove any fact essential to the judgment, whether the judgment arose after trial or upon a plea of guilty. [Fed. R. Evid. 803(22)] For purposes of this Rule, a felony is any crime punishable by death or imprisonment in excess of one year. Arson is a felony. Consequently, a properly authenticated copy of the defendant's conviction of this crime is admissible to prove the fact that the fire that destroyed the table was set by the defendant, a fact essential to the judgment of conviction. Note that the actual plea of guilty is also admissible as a statement of a party-opponent (commonly called an admission). This type of judicial admission is not conclusive, and the defendant may explain the circumstances of the plea. The plea, being an admission, is nonhearsay under the Federal Rules. (A) is incorrect because, in a civil case, evidence of character to prove the conduct of a person in the litigated event is generally not admissible. Circumstantial use of prior behavior patterns for the purpose of inferring that, at the time and place in question, a person probably acted in accord with such patterns creates a danger of prejudice and distraction from the main issues. Therefore, the record of the conviction cannot be used to infer liability by showing the defendant's character. (C) is incorrect because, as noted above, a judgment of a felony conviction is admissible under Federal Rule 803(22) regardless of whether the conviction resulted from a trial or a guilty plea. (D) is incorrect because, as discussed above, the judgment is within the exception to the hearsay rule for records of felony convictions.
Under the Privileges or Immunities Clause of the Fourteenth Amendment, a state may not infringe on its citizens': A Right to earn a living B Right to travel C Right to an attorney D Freedom of speech
B. The right to travel is a right of national citizenship and is protected against infringement by the Privileges or Immunities Clause of the Fourteenth Amendment. The freedom of speech is not a right of national citizenship under the Privileges or Immunities Clause of the Fourteenth Amendment. In The Slaughterhouse Cases, the Supreme Court held that rights of national citizenship do not include the Bill of Rights, although rights under the Bill of Rights may be held applicable to the states as incidents of due process. The right to earn a living is not a right of national citizenship under the Privileges or Immunities Clause of the Fourteenth Amendment, although the right is protected under the Privileges and Immunities Clause of Article IV, which limits states from discriminating against nonresidents. The right to an attorney is not a right of national citizenship under the Privileges or Immunities Clause of the Fourteenth Amendment. Although the right is included in the Bill of Rights under the Sixth Amendment, as discussed above, in The Slaughterhouse Cases, the Supreme Court held that rights of national citizenship do not include the Bill of Rights.e
A man was a permanent resident alien of the United States who was awaiting an opportunity to become a citizen. He filed an application to become an instructor in the local public high school but was denied the position solely on the ground that he was not a citizen. The man now brings suit, alleging that his status as a resident alien was not a proper ground for denying him a position as an instructor. May the state deny a permanent resident alien employment as an instructor in the public high school? A Yes, because employment by the state is a privilege, not a right. B Yes, because citizenship bears some rational relationship to the interest that is being protected. C No, to do so would be a denial of equal protection. D No, because the evidence presented was uncontroverted that he was awaiting the opportunity to become a citizen.
B. The state's action would be reviewed under the rational basis standard. Although state classifications based on alienage are generally suspect, a state may reserve a government position for citizens if it is related to self-governance, involves policymaking, or requires exercise of important discretionary power over citizens. In these cases, only a rationality test is used. A public school teacher at the primary and secondary school level performs an important governmental function (e.g., he influences students' attitudes about government, the political process, citizenship, etc.), and therefore the exclusion of aliens is rationally related to the state's interest in furthering educational goals. [Ambach v. Norwick (1979)] (C) is, accordingly, incorrect. The principle articulated in (A) is correct, but has no bearing here, where the question is whether a distinction based on alienage is permissible. (D) is true, but irrelevant; the state may deny the man's application regardless of his ultimate intentions, so long as he remains an alien.
A landlord leased an apartment to a tenant for five years. The lease provided that the landlord will: (i) keep the apartment building at a comfortable temperature 24 hours per day, and (ii) have the carpets cleaned once a year. Two years later, the landlord began turning off the air conditioning at 10 p.m. The tenant's apartment became hot and stuffy, and she demanded that the landlord honor the covenant. The landlord refused. The following month, the pipes burst in the tenant's only bathroom, rendering it unusable. The resultant flooding soiled some of the carpeting, which had not been cleaned in the past 12 months. The tenant reported the problems to the landlord, who did not return the tenant's phone calls. Which of the following are valid reasons for the tenant to terminate the lease? A Only that the landlord did not keep the apartment building at a comfortable temperature 24 hours per day. B Only that the landlord did not fix the bathroom pipes. C That the landlord did not keep the apartment building at a comfortable temperature 24 hours per day and did not fix the bathroom pipes. D That the landlord did not keep the apartment building at a comfortable temperature 24 hours per day, did not have the carpets cleaned, and did not fix the bathroom pipes.
B. The tenant will be successful in terminating the lease because the landlord breached the implied warranty of habitability by failing to fix the bathroom pipes. The general rule at common law was that the landlord was not liable to the tenant for damages caused by the landlord's failure to maintain the premises during the period of the leasehold. Today, however, a majority of jurisdictions, usually by statute, provide for an implied warranty of habitability for residential tenancies. In the absence of a local housing code, the standard applied is whether the conditions are reasonably suitable for human residence. If the landlord breaches the implied warranty, the tenant may: (i) terminate the lease, (ii) make repairs and offset their cost against future rent, (iii) abate rent, or (iv) seek damages. Here, a court is likely to consider the lack of a functioning bathroom as making the premises unsuitable for human residence, allowing the tenant to terminate the lease. (A) is therefore incorrect. (C) would be a stronger answer if the tenant had vacated the premises within a reasonable time. The doctrine of constructive eviction provides that where a landlord does an act or fails to perform some service that he has a legal duty to provide, and thereby makes the property uninhabitable, the tenant may terminate the lease and seek damages. However, a tenant cannot claim a constructive eviction unless: (i) the injurious acts were caused by the landlord, (ii) the premises are uninhabitable, and (iii) the tenant vacates the premises within a reasonable time. Here, the landlord's failing to keep the apartment building at a comfortable temperature 24 hours per day meets conditions (i) and perhaps (ii), but the tenant remains in possession. Therefore, the tenant cannot claim constructive eviction and (C) is incorrect. (D) is incorrect for the same reason.
A plaintiff sued a defendant for damages suffered when a load of bricks fell off the defendant's truck directly in front of the plaintiff while she was driving on a highway. The plaintiff charged that the defendant was negligent in supplying his truck with a defective load chain clamp, which helped tie the load to the bed of the truck, and in failing to secure the load properly on the truck. The plaintiff calls a witness who testifies that he was formerly employed as a truck driver and is an acquaintance of the defendant. The witness further testifies that immediately prior to the accident he had coffee with the defendant at a cafe, and mentioned to the defendant that the tie chains holding the load of bricks looked kind of loose. Assuming proper objection by the defendant's attorney, how should the court rule on the admissibility of such testimony? A Admissible under an exception to the hearsay rule. B Admissible nonhearsay. C Inadmissible hearsay. D Inadmissible opinion evidence.
B. The witness's testimony is admissible nonhearsay. The statement by the witness is not being offered to prove the truth of the matter asserted therein and thus is not hearsay. Hearsay is a statement made out of court by the declarant, offered in evidence to prove the truth of the matter asserted. [Fed. R. Evid. 801(c)] Although hearsay is inadmissible (unless an exception to the hearsay rule is applicable), a statement that would be inadmissible hearsay to prove the truth thereof may be admitted to show the statement's effect on the listener or reader. Thus, in a negligence case, where knowledge of a danger is at issue, a person's warning statement is admissible for the limited purpose of showing knowledge or notice on the part of a listener. Here, one of the theories of recovery underlying the plaintiff's lawsuit is that the defendant negligently failed to secure the load. Therefore, the plaintiff must show that the defendant either knew or should have known that the load was not properly secured. Consequently, the witness's statement that the chains looked loose is admissible to show that the defendant had notice of the possible danger. If this same out-of-court statement were offered to show that its contents were true (i.e., that the chains were in fact loose), then it would constitute hearsay, but because the statement is offered to show notice to the defendant of a possible danger, it is nonhearsay and (C) is incorrect. (A) is incorrect because the admissibility of the statement arises from its status as nonhearsay. If a statement is nonhearsay, then there is no need to refer to hearsay exceptions in determining the statement's admissibility. (D) is incorrect for two reasons: First, the statement is not being offered to show the witness's opinion that the chains were loose. Rather, the statement is offered to show that the defendant had notice of a possible danger involving the chains. Because the testimony simply relates this statement made to the defendant, such testimony cannot be characterized as opinion testimony. Second, (D) incorrectly implies that opinion evidence is inadmissible. Even opinions of lay witnesses are admissible when they are: (i) rationally based on the perception of the witness; (ii) helpful to a clear understanding of the witness's testimony or to the determination of a fact in issue; and (iii) not based on scientific, technical, or other specialized knowledge. [Fed. R. Evid. 701] Certainly, the witness would be permitted to testify that the chains looked loose at the time he observed them, because such an opinion would be based on personal observation, would be helpful to the determination of a fact in issue (i.e., whether the load was properly secured), and would not be based on technical knowledge.
A woman whose child attended a charter school learned that the children of the woman's neighbor who attended a parochial school received a hot lunch paid for, in part, through federal expenditures enacted under Congress's spending power. The charter school received no funding from the federal government. The woman challenged this federal expenditure as a violation of the Establishment Clause. For her to bring the suit, at the very least what must the woman allege? A She is exempt from paying federal income taxes and has children who are of school age. B She pays federal income taxes and the use of federal funds in this manner is an improper taxing and spending method. C She pays federal income taxes and has children who are of school age. D She pays federal income taxes and has children who are of school age, and the use of federal funds in this manner is an improper taxing and spending method.
B. The woman must allege that she pays federal income taxes and that the use of federal funds in this manner is improper under Congress's taxing and spending power. In general, a taxpayer has no standing to challenge the expenditure of federal funds. The major exception to this rule is where the taxpayer alleges that the expenditure was enacted under Congress's taxing and spending power, and exceeds some specific limitation on that power, in particular the Establishment Clause. Here, by providing federal tax money to parochial schools, there may be excessive entanglement with religion and thus a violation of the Establishment Clause. Thus, the woman would have standing to contest this federal expenditure. The woman would not have to have children to make this challenge. However, she would have to be a taxpayer. Thus, (A), (C), and (D) are wrong.
A former construction worker became intoxicated one night and decided to move some heavy construction equipment that was parked at a construction site. Ignoring "no trespassing" signs, the worker jumped the fence and climbed into a large dump truck and started it up. However, even though he knew how to operate the truck, he quickly lost control of it due to his intoxication. It rumbled a short distance and crashed into a trailer housing the main office of the construction site. The worker is prosecuted for recklessly damaging property. A separate statute in the jurisdiction prohibited the unauthorized operation of construction equipment. Should the worker be found guilty of recklessly damaging property? A Yes, because his actions constituted an unlawful operation of the construction equipment. B Yes, because he was intoxicated while attempting to move the construction equipment. C No, because at most he could be found guilty of criminal negligence. D No, because he must have been aware that his conduct would cause the damage to the trailer in order to be found guilty of reckless damage.
B. The worker should be convicted because he was intoxicated when he damaged the trailer. The worker is being charged with reckless damage to property. A person acts recklessly when he consciously disregards a substantial and unjustifiable risk that a prohibited result will follow, and this disregard constitutes a gross deviation from the standard of reasonable care. Attempting to move a large piece of construction equipment while intoxicated should be considered reckless conduct because of the great potential for destruction arising from the size and destructive power of the construction equipment. Therefore, (B) is correct. (A) is incorrect because merely operating the equipment in violation of the law would not necessarily be reckless. For instance, here, the statute prohibiting unauthorized operation of the equipment likely was enacted to prevent untrained persons from driving dangerous equipment, but the worker was trained to operate the truck in question; thus, if not for the fact that he was drunk, his action would not necessarily have been reckless. Violating the statute may be evidence of negligence, but negligence is insufficient to establish recklessness. (C) is incorrect for the same reason that (B) is correct—driving the equipment while intoxicated constitutes reckless conduct. Although voluntary intoxication is a defense to a crime that requires purpose or knowledge, it is no defense to crimes involving recklessness. Even though the worker's condition may in fact have precluded him from being consciously aware of the risk, one who is not consciously aware of a risk only because he was voluntarily intoxicated will be deemed to have acted recklessly with regard to the risk. (D) is incorrect because it states the mental state for knowing conduct—if the defendant is aware that his conduct will necessarily or very likely cause a certain result, he acts knowingly with respect to that result. Recklessness is a lesser standard of fault.
The police of a resort town discovered that a well-known cat burglar was currently living in town under an assumed name. To try to catch her in the act of burglary, an undercover officer approached the burglar with a plan for a burglary. The undercover officer told the burglar that he knew who she was and that he had a plan to steal jewels from someone staying in one of the town's resorts. The burglar initially refused the offer; however, after lengthy cajoling, she finally agreed to the plan. As the time for the burglary drew near, the burglar had second thoughts. Three hours before the theft was scheduled to take place, the burglar called the police and told them of the plan. She told them that she was not going to show up, but that her cohort (the undercover officer) would be there, and told them how to recognize the undercover officer. Is the burglar guilty of conspiracy at common law? A Yes, because the burglar made an agreement with the undercover officer to commit the theft. B No, because there was no agreement. C No, because the intended crime was never completed. D No, because the burglar effectively withdrew.
B. There was an insufficient agreement for conspiracy liability at common law. Conspiracy consists of: (i) an agreement between two or more persons; (ii) an intent to enter into an agreement; and (iii) an intent to achieve the objective of the agreement. In addition, most states require an act in furtherance of the conspiracy, although an act of mere preparation will usually suffice. The agreement requirement means that the parties must agree to accomplish the same objective by mutual action. There must be a meeting of at least two "guilty minds"; i.e., between two or more persons who are actually committing themselves to the scheme. If one person in a two-party conspiracy is only feigning agreement, the other person cannot be convicted of conspiracy under the common law bilateral approach. Here, the officer, in his undercover capacity, was simply trying to set up a situation in which the burglar would be caught in the act. Thus, the undercover officer merely pretended to reach an agreement with the burglar to commit a burglary. At no time did the undercover officer actually commit himself to the burglary.
Generally speaking, the promise to perform an existing legal duty is __________. A Past consideration B Not consideration C Sufficient consideration D Valuable consideration
B. Traditionally the promise to perform, or the performance of, an existing legal duty is not consideration. A promise to perform an existing legal duty is not valuable consideration, unless an exception to the preexisting legal duty rule applies, e.g., new or different consideration is promised, or a minor's ratification of a voidable contract upon reaching the age of majority. Past consideration, which is also not sufficient consideration, is based on something already given or performed, not a promise to perform based on a preexisting legal duty.
Felony murder generally requires that: A The killing be committed during the course of the felony, the death must have been a foreseeable result of the felony, and the defendant must be convicted of the underlying felony B The killing be committed during the course of the felony, the felony must be independent of the killing, and the death must have been a foreseeable result of the felony C The killing be committed during the course of the felony, the felony must be independent of the killing, and the defendant must be convicted of the underlying felony D The death must have been a foreseeable result of the felony, the felony must be independent of the killing, and the defendant must have been convicted of the underlying felony
B. felony (i.e., he is factually guilty of the felony). However, the defendant need not actually be convicted of the underlying felony if the statute of limitations for the felony has expired. The killing must take place while the felony is being committed. When the defendant reaches "a place of temporary safety," the felony is deemed terminated. The felony must be independent of the killing (e.g., the felony of manslaughter cannot be the underlying felony for felony murder). Finally, most states require that the death must be a foreseeable result of the commission of the felony.
A deed generally must contain which of the following in order to be valid? A The grantor's acknowledgment. B The grantor's words of intent. C The grantee's signature. D The metes and bounds of the land.
B. A deed generally must contain the grantor's words of intent in order to be valid. A deed must demonstrate that the grantor intends to transfer realty (e.g., by using the word "grant"). However, no particular technical phrasing is necessary. A deed generally need not contain the grantor's acknowledgment in order to be valid. Before a deed can be recorded under most recording statutes, it must be acknowledged by the grantor before a notary public. However, the grantor's signature, without an acknowledgement, is sufficient for the deed itself to be valid. A deed generally need not contain the metes and bounds of the land in order to be valid. While a deed must identify the land, a metes-and-bounds description is only one of many ways property may be described. A description is sufficient if it provides enough information to identify the property in question (e.g., a street address, or a reference to a lot in a recorded subdivision plat). A deed generally need not contain the grantee's signature in order to be valid. Even if the deed contains covenants on the grantee's part, her acceptance of the deed is sufficient to make those covenants enforceable.
A distributor of electric toy trains and a hobby shop owner entered into a written contract providing that the distributor will tender to the shop owner four dozen of a popular electric train set at a price of $100 apiece, to be delivered no later than October 31, to take advantage of the holiday shopping season. The shop owner chose to order from this distributor because its price for the train set was lower than that of other distributors. Shortly after the shop owner placed his order, the distributor raised its prices due to a sudden surge in popularity of that train set. Because the distributor did not have enough train sets to accommodate everyone due to the surge of orders, it decided to deliver train sets only to those buyers who had ordered them at the increased price. The distributor notified the shop owner that it would not deliver the train sets it ordered. The shop owner filed an action to force the distributor to deliver the train sets at the agreed-upon price. Will the court compel the distributor to deliver the train sets to the shop owner? A No, because a contract for the sale of goods is not subject to specific performance. B No, because the shop owner can buy them from another distributor. C Yes, because the shop owner will not be able to buy them from another source at the contract price. D Yes, because time is of the essence.
B. Because the shop owner can cover (i.e., buy the train sets from another source), a court will not grant specific performance. If the seller fails to deliver goods under a valid contract, the buyer has a number of remedies available, including the right to cover and the right to obtain specific performance if appropriate. A buyer may obtain specific performance of a contract for the sale of goods if the goods are unique or in short supply, but that does not appear to be the case here because the other distributors carried that train set. Thus, the shop owner can buy the train sets from another distributor and get the difference between the cost of the substitute goods and the contract price. Thus, (B) is correct. (A) is incorrect because, as discussed above, under certain circumstances a seller of goods may be subject to specific performance. (C) is incorrect because, as discussed above, the shop owner can buy the train sets from another distributor and then sue for damages for the difference in cost. Thus, specific performance is not the appropriate remedy. (D) is irrelevant to whether specific performance is granted and is unsupported by the facts.
In which of the following cases would strict scrutiny of a government action be applied to determine whether the action violates equal protection? A A federal law denies government benefits to an individual based on alienage. B A state law denies government benefits to an individual based on alienage. C A state law penalizes an individual who is an undocumented alien. D A state law makes aliens ineligible for public employment in positions that directly affect the self-government process.
B. A STATE LAW that denies government benefits to an individual based on alienage is subject to strict scrutiny and will be upheld only if the state proves the discrimination is necessary to achieve a compelling government interest. Generally, alienage classifications made by states are subject to strict scrutiny. (However, there is an exception where the law is related to participation in the self-government process. Such laws are subject only to the rational basis test.) A FEDERAL LAW that denies government benefits to an individual based on alienage is not subject to strict scrutiny. Because the Constitution gives Congress plenary power over aliens, federal classifications based on alienage generally are tested under the rational basis test (i.e., valid unless the challenger can prove that the classifications are not rationally related to a legitimate government interest). A STATE LAW that makes aliens ineligible for public employment in positions that directly affect the self-governing process are not subject to strict scrutiny like other state alienage classifications. Laws relating to the self-governing process are subject only to a rational basis review. A STATE LAW that penalizes an individual who is an undocumented alien also is not subject to strict scrutiny, because the Supreme Court has not held that undocumented aliens is a suspect classification.
The Model Penal Code defines acting purposefully as when: A The defendant is aware that his conduct is of that nature or that certain circumstances exist B It is the defendant's conscious object to engage in certain conduct or cause a certain result C The defendant consciously disregards a substantial and unjustifiable risk that circumstances exist or that a prohibited result will follow, and this disregard constitutes a gross deviation from the standard of care that a reasonable person would exercise in the situation D The defendant fails to be aware of a substantial and unjustifiable risk that circumstances exist or a result will follow, and such failure constitutes a substantial deviation from the standard of care that a reasonable person would exercise under the circumstances
B. A person acts purposefully when it is his conscious object to engage in certain conduct or cause a certain result. A person acts negligently when he fails to be aware of a substantial and unjustifiable risk that circumstances exist or a result will follow, and such failure constitutes a substantial deviation from the standard of care that a reasonable person would exercise under the circumstances. A person acts knowingly when he is aware that his conduct is of that nature or that certain circumstances exist. A person acts recklessly when he consciously disregards a substantial and unjustifiable risk that circumstances exist or that a prohibited result will follow, and this disregard constitutes a gross deviation from the standard of care that a reasonable person would exercise in the situation. Recklessness is one of the four categories into which the Model Penal Code classifies the mental component of a criminal offense (i.e., the element of fault)
If no federal question is involved and diversity does not exist when a case is commenced, removal will: A Be permitted if the nondiverse parties are thereafter dismissed from the action and the requirements for diversity jurisdiction are then present B Be permitted if the nondiverse parties are thereafter dismissed from the action, the requirements for diversity jurisdiction are then present, and not more than one year has passed since the case was commenced in state court C Not be permitted under any circumstances D Be permitted, because subject matter jurisdiction is not required for removal
B. If no federal question is involved and diversity does not exist because a party is a co-citizen of an opposing party (but the amount in controversy is satisfied), removal will be permitted if the nondiverse parties are thereafter dismissed from the action, the requirements for diversity jurisdiction are then present, and not more than one year has passed since the case was commenced in state court. This rule is subject to certain limitations.
A state bans the use of disposable diapers to reduce the volume of nonbiodegradable material in its landfills. The ban was a boon for diaper services within the state, but many parents of young children were displeased with the use of conventional diapers. With support from retail establishments that lost business from the disposable diaper ban, a grass roots coalition formed to fight the ban funded a study showing that the trucks and cleaning supplies used by diaper services within the state harmed the environment more than disposable diapers. The coalition and retailers then filed suit seeking to have the ban on disposable diapers declared unconstitutional. If the court strikes down the statute, on which of the following constitutional provisions would its decision most likely be based? A The Equal Protection Clause of the Fourteenth Amendment. B The Due Process Clause. C The Impairment of Contracts Clause. D The Privileges or Immunities Clause of the Fourteenth Amendment.
B. Of the choices presented, the only likely basis to strike down the statute is under the Due Process Clause as a violation of substantive due process. Substantive due process tests the reasonableness of a statute; it prohibits arbitrary governmental action. Under substantive due process, when government action limits a fundamental right, the government must prove that the action is necessary to promote a compelling interest. If a fundamental right is not involved, the challenging party must prove that the act is not rationally related to any legitimate government interest. The retail sale of diapers is not a fundamental right, and so a challenger must prove that there is no rational basis for the statute. Almost any law can be justified under the rational basis standard. The law need not be the best law for accomplishing the government's goal. Thus, even if it is true that the disposable diaper ban causes more pollution than it prevents, because the ban is rationally related to reducing the volume of trash in landfills, the challenge is unlikely to succeed. Nevertheless, none of the other choices states a viable ground for invalidating the statute, and so (B) is the best choice. (A) is wrong because equal protection applies where a statute or governmental action treats similar people in a dissimilar manner (i.e., classifies people), and here there is no classification—under the statute no one can sell disposable diapers for use within the state. Thus, an equal protection argument is not applicable. (C) is wrong because the Impairment of Contracts Clause prohibits only the substantial impairment of existing contracts (and there are exceptions even where there is substantial impairment), and nothing in the facts indicates that forbidding the retail sale of disposable diapers would substantially impair any existing contract. (D) is wrong because the privileges and immunities covered by the Fourteenth Amendment are those attributes peculiar to United States citizenship (e.g., the right to petition Congress for redress or the right to vote for federal officers). The statute here does not affect such rights.
A man and a woman were arrested and charged with a series of armed robberies. Each suspect was given Miranda warnings, and different interrogation teams questioned each suspect separately. Upon being questioned, the man told the police, "I'm not going to talk until I see a lawyer." An officer responded, "You might want to reconsider, because your partner has already confessed, and she's implicated you in the crimes." The man then told the police that he wanted to talk to the woman privately. The police escorted the man to the woman's cell, locked him in with her, and left. Unbeknownst to either of them, the police had bugged the woman's cell and recorded both the man and the woman making self-incriminating statements during their meeting. The man made no further statements to the police on advice of counsel, whom he called immediately after his conversation with the woman. The man was put on trial first, and the prosecution sought to introduce into evidence tapes of the bugged conversation between the man and the woman. The defense made a motion to suppress the evidence. Should the court grant the motion to suppress? A Yes, because the evidence is the fruit of a wiretap that violated the Fourth Amendment. B Yes, because the police created a situation likely to induce the defendant to make an incriminating statement. C No, because there is no expectation of privacy in a jail cell. D No, because the conversation constituted a waiver of the man's Miranda rights.
B. The conversation should be suppressed because the police conduct violated the man's Sixth Amendment right to counsel. The Sixth Amendment provides that in all criminal prosecutions a defendant has a right to the assistance of counsel at all critical stages after formal proceedings have begun. For Sixth Amendment purposes, a criminal prosecution begins when adversary judicial proceedings have commenced, such as the filing of formal charges in this case. Because custodial interrogation is a critical stage of prosecution, the Sixth Amendment is violated by post-charge interrogation unless the defendant has waived his right to counsel. Interrogation includes not only direct questioning, but also any other conduct by the police intended to elicit a response. The police conduct here (telling the man that the woman had implicated him and then bugging the conversation) constitutes prohibited interrogation. [See Maine v. Moulton (1985)] (A) is incorrect because the wiretap was not an illegal search under the Fourth Amendment. Wiretapping and other forms of electronic surveillance are subject to the Fourth Amendment prohibition of unreasonable searches and seizures. However, to have a Fourth Amendment right, a person must have a reasonable expectation of privacy with respect to the place searched or the item seized. In a different context, the Supreme Court has held that prisoners have no reasonable expectation of privacy in their cells or in any personal property that they have in their cells. [Hudson v. Palmer (1984)] Hence, neither defendant can assert a Fourth Amendment claim based on the wiretap, because they had no reasonable expectation of privacy in the jail cell. The fact that there was no expectation of privacy does not make choice (C) correct, however. Even though he probably cannot claim that the bugging was an unreasonable search under the Fourth Amendment, the man can claim that it was an interrogation in violation of his Sixth Amendment right to counsel, as discussed above. (D) is incorrect because it is irrelevant. The facts probably would not give rise to a Miranda violation in light of the Court's ruling in Illinois v. Perkins (1990) that Miranda does not apply unless interrogation is by someone known to be a police officer (on the rationale that Miranda is merely a prophylactic rule designed to offset the coercive nature of a custodial interrogation by a police officer). In any case, Miranda rights and Sixth Amendment rights to counsel can only be waived knowingly, and so the man's ignorance of the fact that the cell was bugged precludes a finding of waiver here.
Some constitutional rights applicable in criminal cases are offense specific (i.e., when invoked, they apply only to the specific charge for which they were invoked), and some are not offense specific (i.e., once invoked, they apply to all charges against the defendant and not just the charge for which they were invoked). Which of the following statements is correct regarding whether the Fifth and Sixth Amendments are offense specific? A Neither the Fifth Amendment nor the Sixth Amendment is offense specific. B The Fifth Amendment is not offense specific, but the Sixth Amendment is offense specific. C The Fifth Amendment is offense specific, but the Sixth Amendment is not offense specific. D Both the Fifth Amendment and the Sixth Amendment are offense specific.
B. 5th amend not offense specific, but the Sixth Amendment is offense specific. Thus, if a defendant invokes his right to counsel, under the Fifth Amendment the police cannot interrogate the defendant about any charge without counsel. But under the Sixth Amendment, the defendant can be interrogated regarding a different charge. The difference is significant when a defendant requests counsel after being charged and is put into a cell with an informer. The Sixth Amendment right to counsel applies to any post-charge interrogation—whether or not the defendant knows he is being interrogated by a government agent. The Fifth Amendment right to counsel, on the other hand, applies only when the defendant knows that he is being interrogated by a government agent. Thus, an undercover informer in the defendant's cell can question the defendant about anything without violating his Fifth Amendment right to counsel, and under the Sixth Amendment, the informer can question the defendant about any crime but the one with which he is charged. Therefore, if the cellmate informer questions the defendant and obtains information regarding a crime different from the one with which the defendant was charged, the questioning violates neither the Fifth nor the Sixth Amendment.
Several members of a small terrorist group are on trial in federal court for conspiring to bomb a military installation. The prosecution would like to introduce the testimony of a military guard at one of the installation's gates. The guard had been present when a bomb that was being planted by a member of the group had exploded prematurely. The guard will testify that she ran over to administer first aid to the member, who in great pain told her that his group was in the process of planting three other bombs in other areas of the military installation and was going to detonate them all at the same time to get publicity for their cause. The guard will also testify that the member disclosed the locations of the other bombs and the names of two other members of the group. The authorities were able to prevent the other bombings and arrest the other members of the group. The member died from his injuries. What is the best basis for allowing the guard to testify as to the member's statements? A As a vicarious admission of a co-conspirator. B As a statement against interest. C As a statement of present state of mind. D As a dying declaration.
B. The member's statements are admissible as a statement against interest. Under the Federal Rules, statements of a person, now unavailable as a witness, against that person's pecuniary, proprietary, or penal interest when made are admissible as an exception to the hearsay rule. Here, the member's statements implicating himself in the bombing conspiracy were against his penal interest when he made them; hence, they are probably admissible under that exception. (A) is wrong because for a statement to qualify under the Federal Rules as a vicarious admission of another member of the conspiracy, the admission must have been in furtherance of the conspiracy by a participant in it. Here, the member's statements were not made in furtherance of the conspiracy but instead served to thwart its success. (C) is wrong because the member's statements are not being used to show his then-existing state of mind but rather the scope of the conspiracy and the defendants' participation in it. (D) is wrong because, even assuming that the member made the statements while believing his death was imminent (which the facts do not clearly establish), dying declarations are admissible under the Federal Rules only in a prosecution for homicide or in a civil action, and this case was neither of those.
To be valid, a time, place, and manner regulation of a limited public forum must be: A Viewpoint neutral and narrowly tailored to achieve an important government interest B Viewpoint neutral and rationally related to a legitimate government purpose C Content neutral and rationally related to a legitimate government purpose D Content neutral and narrowly tailored to achieve an important government interest
B. To be valid, a time, place, and manner regulation of a limited public forum must be viewpoint neutral and rationally related to a legitimate government purpose. "Viewpoint neutral and narrowly tailored to achieve an important government interest" is incorrect because regulations of limited public forums need not be narrowly tailored to achieve an important government interest—the standard is lower (rationally related to a legitimate government purpose) as indicated above. "Content neutral and rationally related to a legitimate government purpose" is incorrect because regulation of a limited public forum can be based on content, but it must not be based on viewpoint.
On July 1, a cattle rancher offered to sell his ranch to a dairy farmer for $150,000. The dairy farmer paid the cattle rancher $1,000 to hold the offer open for a period of 30 days. On July 10, the dairy farmer wrote to the cattle rancher, telling him that he could not pay more than $100,000 for the ranch, and that if he would not agree to accept that amount, he would not go through with the deal. The dairy farmer received no reply from the cattle rancher. On July 29, the dairy farmer mailed a letter to the cattle rancher telling him that he accepted his offer to sell the ranch and enclosed a check for $150,000. The cattle rancher received this letter on August 1. Has a contract been formed between the parties for the sale of the ranch? A No, because the dairy farmer's letter of July 10 terminated the cattle rancher's offer. B No, because the cattle rancher did not accept the dairy farmer's counteroffer of $100,000. C No, because the cattle rancher did not receive the dairy farmer's acceptance within 30 days. D Yes, because the dairy farmer dispatched his acceptance of the cattle rancher's offer prior to the expiration of 30 days.
C No contract was formed because the cattle rancher did not receive the dairy farmer's acceptance within 30 days. Under the mailbox rule, acceptance by mail or similar means creates a contract at the moment of dispatch. However, the mailbox rule does not apply to option contracts. An acceptance under an option contract is effective only upon receipt. [Restatement (Second) of Contracts �63] Here, an option contract existed because the dairy farmer paid the cattle rancher $1,000 to hold the offer open for 30 days. The dairy farmer mailed his acceptance within 30 days but it was not received by the cattle rancher within the 30-day period, so the acceptance was not effective. The option specified the period of time during which the offer would remain open, after which the offer terminated. Thus, (C) is correct, and (D) is wrong. (A) and (B) are wrong because an option contract is irrevocable for the time period stated. Thus, not even the dairy farmer himself could revoke the offer within the 30-day period.
The plaintiff sued a local restaurant, claiming that she injured her teeth, gums, and mouth when she bit into a hamburger that contained a large, jagged piece of glass. The plaintiff called to the stand a waiter for the restaurant, who testified that, when he heard the plaintiff scream, he looked in her direction and saw her remove a piece of glass from her bleeding mouth. On cross-examination, the defense asked the waiter, "Isn't it a fact that three months ago you were fired by the restaurant for serving drinks to your friends and not charging for them?" The waiter responded, "Yes, but I wasn't trying to steal anything. I just forgot to charge them." The defense then asked, "Isn't it a fact that last month you threw a rock through the plate glass window at the restaurant?" The waiter replied, "That's not true; I was there but I didn't throw the rock." The defense then offered the testimony of a witness who was prepared to testify that she saw the waiter throw the rock through the restaurant's window. Assuming that there have been no criminal charges filed as a result of the broken window, is the witness's testimony admissible? A No, because specific acts of misconduct that did not result in a conviction cannot be used to impeach a witness, either on cross-examination or through extrinsic evidence. B No, because specific acts of misconduct that did not result in a conviction cannot be established through extrinsic evidence. C Yes, as evidence of bias. D Yes, to establish that the waiter lied under oath.
C The witness's testimony is admissible to show bias. A witness can be impeached, either on cross-examination or by extrinsic evidence, with evidence that suggests a bias on the part of the witness, because it tends to show that the witness has a motive to lie. Evidence that the witness disliked the party he is testifying against would qualify as evidence of bias. The witness could testify that she saw the waiter throw the rock through the restaurant's window, because such evidence would help establish the waiter's bias against the restaurant. (A) is incorrect for two reasons: Federal Rule 608 provides that, if offered to impeach, prior bad acts may not be proved through extrinsic evidence but may be inquired into during cross-examination. Furthermore, if the prior bad act also helps establish bias, the courts have held that extrinsic evidence also will be admissible. (B) is incorrect as well for this latter reason. (D) is too broad a statement. In a broad sense, the evidence is offered to impeach the credibility of the waiter and to suggest to the jury that he may be lying under oath. However, the defense does not need to establish that the waiter lied; the reason the evidence is relevant and does not constitute impeachment on a collateral matter is because it is offered to show bias, making (C) the better answer.
In contrast to products liability cases based on negligence, those based on strict liability do NOT: A Impose liability when an intermediary negligently failed to discover the defect. B Prohibit recovery of solely economic losses. C Require that suppliers have an opportunity to inspect. D Require an injured bystander to be foreseeable.
C Unlike with products liability cases based on negligence, those based on strict liability do not require that suppliers have an opportunity to inspect. Thus, for a case based on the sale of a defective product, a retailer in a strict liability action may be liable for a manufacturing or design defect simply for being a commercial supplier of that defective product, even if it had no opportunity to inspect the manufacturer's product before selling it. In a negligence action, the supplier's negligence must be proved. Products liability cases based on negligence and those based on strict liability both require that an injured bystander be foreseeable. While privity is not required in these cases, and bystanders are protected and may bring a claim under either theory, they must be foreseeable. Liability under these theories applies to foreseeable plaintiffs.
When an action involves multiple claims or parties, and a judgment is entered that disposes of only some of the parties or claims, the judgment: A Generally is immediately appealable regarding the parties who were disposed B Is not immediately appealable until a final judgment is rendered as to all parties and all claims C Is immediately appealable only to the extent that the court makes an express determination that there is no just reason for delay D Generally is immediately appealable as to the claims that were disposed
C Is immediately appealable only to the extent that the court makes an express determination that there is no just reason for delay When an action involves multiple claims or parties, and the court enters a judgment as to fewer than all the claims or all the parties, it is deemed a final, appealable judgment only to the extent the court makes an express determination that there is no just reason for delay. Generally, a judgment as to only some of the claims or parties is not immediately appealable, regarding the parties who were disposed or as to the claims that were disposed. It is untrue that a judgment regarding only some of the parties or claims is not immediately appealable until a final judgment is rendered as to all claims and all parties because such judgments are appealable if the judge expressly makes a determination that there is no just reason for delay.
A man purchased a new power boat with an inboard engine from a boating supply store. The boating supply store properly inspected the boat before delivery, but did not detect a virtually invisible manufacturing defect in the boat's steering mechanism. Later that summer, the man was entertaining some friends on his boat on a lake near a dam. There were some warning pylons near the dam, warning boaters to stay clear. The man decided to show off for his friends by weaving his boat in and out of the warning pylons. As he rounded the last of them, the steering mechanism of his boat jammed, and the boat crashed into the dam. The man was severely injured. The man brings an action for damages against the boating supply store on a theory of strict liability in tort in a jurisdiction that does not apply its comparative fault rules to strict liability actions. Who will prevail? A The boating supply store, because it properly inspected the boat before selling it to the man. B The boating supply store, because the man was negligent in weaving in and out of the pylons. C The man, because the steering failed due to a defect present when the boat left the manufacturer. D The man, because the steering mechanism failed while he was operating the boat.
C The man, because the steering failed due to a defect present when the boat left the manufacturer. To recover on a theory of strict tort liability, the man must show that his injuries were caused by an unreasonably dangerous defect in the boat that existed when the boat left the boating supply store's control; (C) is the only alternative that reflects this requirement. A prima facie case in products liability based on strict tort liability consists of the following: (i) the defendant is a commercial supplier; (ii) the defendant produced or sold a defective product; (iii) the product was the actual and proximate cause of the plaintiff's injury; and (iv) the plaintiff suffered damages to person or property. Examples of commercial suppliers include manufacturers, retailers, wholesalers, and assemblers. The second element is established by proving that the product is in a defective condition unreasonably dangerous to users. A plaintiff need not prove that the defendant was at fault in selling or producing a dangerous product. To prove actual cause, a plaintiff must trace the harm suffered to a defect in the product that existed when the product left the defendant's control. Here, because the steering failed due to a defect present when the boat left the manufacturer, that defect must also have been present when the man bought the boat from the boating supply store, the retailer. This defect rendered the boat unreasonably dangerous to users such as the man. The boating supply store sold the boat in this defective condition, and the defect actually and proximately caused the man to incur severe personal injuries.
A fee simple subject to an executory interest is an estate that: A Continues after the happening of a stated event until the grantor exercises her power of termination B Continues after the happening of a stated event until the third party exercises his power of termination C Automatically divests in favor of a third party on the happening of a stated event D Automatically terminates on the happening of a stated event and reverts to the grantor
C.
A grantor executed and delivered a deed to his son conveying his land as follows: "To my son for life, but if my son dies survived by his spouse and children, then to my son's spouse for life, with the remainder in fee simple to my son's children." A year later, the son died survived by his spouse and two offspring, a girl and a boy. The boy died intestate two days after the son, leaving one child as his only heir. The common law Rule Against Perpetuities is unmodified in the jurisdiction. What are the respective interests of the spouse, the girl, and the child in the land? A The spouse has a life estate, the girl has an absolutely vested remainder, and the child has nothing. B The spouse has fee simple ownership of the land, and the girl and the child have nothing. C The spouse has a life estate, and the girl and the child have absolutely vested remainders. D The spouse has a life estate, and the girl has a vested remainder subject to open.
C.
Which of the following is required for the burden of an equitable servitude to run to a subsequent purchaser of the land? A The restriction must be recorded in the buyer's chain of title. B There must be a common scheme for development C The purchaser must have notice of the covenant D There must be horizontal privity between the original covenanting parties
C. An equitable servitude is a covenant (i.e., a promise to do or not do something on the land) that, regardless of whether it runs with the land at law, can be enforced in equity against assignees of the burdened land who have notice of it. The burden of an equitable servitude will run to a subsequent purchaser if: 1. The covenanting parties intended that successors in interest be bound by the covenant; 2. The purchaser has notice of the covenant; and 3. The covenant touches and concerns the land (i.e., it benefits the covenantor and his successor in their use and enjoyment of the burdened land). The requisite notice may be acquired through actual notice (direct knowledge of the covenants in the prior deeds); inquiry notice (the neighborhood appears to conform to common restrictions); or record notice (if the prior deeds are in the grantee's chain of title he will, under the recording acts, have constructive notice of their contents). Thus, there the restriction need not be in the buyer's record chain of title for the buyer to be burdened by it—as long as the buyer has some kind of notice. Horizontal privity between the original covenanting parties is not required. Horizontal privity means the original parties to a real covenant shared some interest in the land independent of the covenant at the time they entered it (e.g., as grantor and grantee). Horizontal privity is required to enforce the burden of a real covenant at law, but it is not required to enforce the burden of an equitable servitude. A common scheme for development is not required for the burden of a written equitable servitude to run to a subsequent purchaser. Generally, equitable servitudes are created by covenants contained in a writing that satisfies the Statute of Frauds. However, reciprocal negative servitudes may be implied absent a writing if there is a common scheme for the development of a subdivision and the grantee had actual, record, or inquiry notice of restrictions that do not appear in his deed. The common scheme exception applies only to negative covenants and equitable servitudes; affirmative covenants must be in writing.
Question 11 A defendant charged with auto theft under a theory of accomplice liability testified at trial on his own behalf that although he was in the car when the police apprehended the driver, he was unaware that the car was stolen. On cross-examination, the prosecutor asks the defendant whether he lied on an employment application three years ago when he falsely claimed to hold a college degree. If the judge allows the question, what is the most likely reason? (A) The evidence may tend to establish that the defendant is a dishonest person and therefore may have committed the crime charged. (B) The prosecutor has a right to inquire about prior bad acts during cross-examination. (C) The evidence is relevant to the issue of the credibility of the witness and the court determines that its value is not outweighed by other considerations. (D) The evidence may tend to establish the intent of the defendant to commit auto theft
C. (C) The judge will allow the evidence if it is relevant and its value is not outweighed by other considerations. Under Federal Rule 608, the trial court is given the discretion to allow counsel to inquire, during cross-examination, about specific instances of bad conduct on the part of the witness which show a lack of credibility. Therefore, (C) is a more accurate answer than (B). (A) is wrong; specific instances of previous bad conduct on the part of the accused are not admissible to prove the accused is the type of person who may have committed the crime. (D) is wrong; it is true that the prosecutor can prove previous bad acts to establish intent, but the prior behavior must show more about the defendant's intent at the time of the crime than this behavior does, because not all prior bad acts can be inquired about during cross-examination.
Which of the following is NOT a nonpossessory interest in land? A Easement B Real covenant C License D Profit
C. A license is not a nonpossessory interest in land. A license is merely a privilege to go upon another's land; it is not an interest in land. An easement is a nonpossessory interest in land. The holder of an easement has the right to use another's land, but has no right to possess and enjoy the land. A profit is a nonpossessory interest in land. The holder of a profit has the right to go upon another's land and take the soil or a substance of the soil (e.g., minerals, timber), but has no right to possess and enjoy the land. A real covenant is a nonpossessory interest in land. A real covenant is a written promise to use or not to use land in a certain manner, and does not confer a right to possess the land on the covenantee
n compliance with a federal statute that permits government agencies to sell or give away surplus government property, the Secretary of State directed that one of the State Department's surplus airplanes be given to a church. The Secretary knew that the church planned to use the plane to fly medical supplies to its missions in Third World countries. These missions provide medical assistance, but they also attempt to evangelize residents of the countries in question, and the Secretary was aware that, in addition to medical supplies, the plane might transport Bibles and religious tracts translated into local languages. Had the Secretary not ordered the plane to be given to the church, it would have been sold at a very reasonable cost to a nonprofit organization that helps teach young people the fundamentals of piloting and maintaining aircraft. Which of the following parties would be most likely to have standing to sue to prevent the Secretary of State from making the gift to the church? A A taxpayer. B A citizen of the United States. C A member of the nonprofit flying organization. D The attorney general of the state in which the airplane is located.
C. A member of the nonprofit flying association is most likely to have standing to challenge the gift. To have standing to challenge government action on constitutional grounds, a person must show that he has a concrete stake in the outcome of the litigation. This is to ensure adequate presentation of the issues. To have such a stake, the potential litigant must show that he has an injury in fact caused by the government that is more than the theoretical injury that all persons suffer when the government engages in unconstitutional acts, and that a decision in his favor will eliminate his harm. A member of the flying association can show both components here: If the gift is unconstitutional, the association has suffered more than a theoretical injury—it has lost the opportunity to purchase the airplane from the federal government at a good price, and a decision in the club's favor will eliminate the injury because it will then be able to purchase the plane. Thus, the member of the nonprofit flying organization has standing. (B) is incorrect because the only injury that a citizen would suffer here is the theoretical injury that we all suffer from the government's unconstitutional acts. People have no standing merely as "citizens" to claim that government action violates federal law or the Constitution. (A) is incorrect because a person's injury as a taxpayer is generally held to be insufficient to establish standing. There is an exception where the federal government acts under the taxing and spending power and that action allegedly violates the Establishment Clause, but the government action here falls under the Property Clause and not the Spending Clause; thus, the exception does not apply. (D) is incorrect because the state attorney general has no stake in the outcome of the litigation, and it is not sufficient even if he is deemed to represent the interests of all the citizens in the state.
Which of the following parties would be entitled to prevail against a prior transferee under "notice" and "race-notice" statutes? A A donee of the land B A judgment creditor C A mortgagee for value D One who took the land by specific bequest
C. A mortgagee for value would be entitled to prevail against a prior transferee under "notice" and "race-notice" statutes. Notice and race-notice recording acts protect bona fide purchasers ("BFPs") from prior unrecorded conveyances of the same property. A BFP is a purchaser who takes land without notice of a prior instrument and pays valuable consideration. Donees, heirs, and devisees are not BFPs because they do not give value for their interests; i.e., they are not purchasers. Thus, a donee of the land and one who took the land by specific bequest would not be entitled to protection under the recording acts. In contrast, mortgagees for value ( i.e., those who loan substantial money in return for a mortgage on the land) are treated as "purchasers," either expressly by the recording act or by judicial classification. A judgment creditor would not be entitled to prevail against a prior transferee under "notice" and "race-notice" statutes. In nearly all states, a plaintiff who obtains a money judgment can obtain, by statute, a judgment lien on the defendant's real estate. The majority of cases hold that the judgment lienor is not protected either because (i) he is not a BFP because he did not pay value for the judgment, or (ii) the judgment attaches only to property "owned" by the defendant, and not to property the defendant has previously conveyed away, even if that conveyance was not recorded.
A man and a woman met in a bar. While the two enjoyed a couple of drinks, the woman told the man that she greatly admired the diamond stickpin he had in his lapel. "Oh, this," the man laughed. "It's no diamond; it's only a piece of glass." The woman acknowledged his statement, but kept commenting on how nice it looked. After further conversation, the man orally agreed to sell the stickpin to her for $500. They agreed that in four days, the man would bring the stickpin to the same bar, and the woman would bring the $500 in cash. The woman wrote down her name and phone number on a napkin and asked the man to call her if there were any change in plans. The man duly appeared with the pin, but the woman failed to appear. The man filed suit against the woman for $500. In an action by the man against the woman for breach of contract, which of the following would be the woman's best defense? A $500 was an unconscionable amount to pay for a piece of glass. B The parties lacked capacity to contract because they were drinking alcohol. C The agreement violated the Statute of Frauds. D Neither the woman nor the man was a merchant.
C. A promise for the sale of goods priced at $500 or more is not enforceable under the Statute of Frauds unless evidenced by a writing signed by the party to be charged. Here, the woman is the party to be charged, and her promise to pay $500 was only oral. The napkin with the woman's name and number would not be sufficient to satisfy the Statute. To satisfy the Statute of Frauds, the UCC requires that the writing indicate that a contract has been made and specify the quantity term. Here, the napkin does not indicate that there is a contract. It merely contains a name and phone number. A court could not enforce a promise based on the writing. Thus, the woman's promise is unenforceable. (A) is incorrect because the concept of unconscionability allows avoidance of a contract only where the terms are so one-sided as to indicate unfair surprise or a contract of adhesion. Here, neither party had superior bargaining power, and the woman knew exactly what she was buying when she made the agreement. (B) is incorrect because just having alcoholic drinks does not mean that parties no longer have the capacity to contract. To lack capacity due to intoxication, a party must be so intoxicated that the party does not understand the nature and significance of his promise. Nothing in the facts indicates that these parties were intoxicated, let alone so intoxicated that they did not understand what they were doing. (D) is incorrect because the parties' status as nonmerchants is irrelevant. While the Code relaxes the Statute of Frauds rule in the case of a written confirmation between merchants, that exception does not apply here. There was no writing by either party that would satisfy the Statute of Frauds; thus, even if the parties were merchants, the agreement would be unenforceable.
Which of the following is not an exception to the general rule under the Commerce Clause prohibiting states from discriminating against out-of-state competition? A When the regulation is necessary to further an important, noneconomic state interest such as health or safety. B When the regulation furthers an important, noneconomic state interest such as health or safety and there are no reasonable alternatives available. C When the regulation limits access to privately owned in-state products. D When the state acts as a market participant.
C. A regulation limiting access to privately owned in-state products would violate the negative implications of the Commerce Clause; there is no exception for such regulations. When a state acts as a market participant, it generally is not restricted by the Commerce Clause; it may favor its own citizens, such as by distributing state-owned resources only to residents or paying residents more for something than it would pay an a nonresident (although such conduct might violate the Article IV Privileges and Immunities Clause). When a regulation is necessary to further an important, noneconomic state interest such as health or safety, it is excepted from the general rule of the negative implications of the Commerce Clause prohibiting discrimination against out-of-state competition. There also is an exception for when the regulation furthers an important, noneconomic state interest such as health or safety and there are no reasonable alternatives available. Actually, this is the same answer as above—if there are no reasonable alternatives, the method chosen is considered a necessary method.
When property is held in joint tenancy or tenancy in common, which of the following is not a co-tenant's right? A Mortgage her interest B Share in rents paid by third parties C Compel contribution for the cost of improvements D Possess the entire estate
C. Although a joint tenant or tenant in common may have a right to compel contribution from other co-tenants for the cost of necessary repairs, taxes, and payments due on mortgages, she does not have a right to compel contribution for the cost of improvements. Under the unity of possession, each co-tenant has a right to possess the entire estate subject to the equal right of her co-tenant. A co-tenant out of possession cannot bring a possessory action unless there has been an "ouster" (i.e., wrongful exclusion) by the co-tenant in possession. Although a co-tenant generally is not entitled to share in the rental value of the land, she does have a right to share in rents paid by third parties . A joint tenant or tenant in common may mortgage her interest. However, she may not encumber another co-tenant's interest. Note that an individual spouse may not mortgage her interest in tenancy by the entirety property.
At common law, a conveyance of property from O "to O and A as joint tenants with right of survivorship" creates a __________. A Tenancy for years B Tenancy by the entirety C Tenancy in common D Joint tenancy
C. At common law, a conveyance of property from O "to O and A as joint tenants with right of survivorship" creates a tenancy in common. There are three forms of concurrent ownership in land: joint tenancy with right of survivorship, tenancy in common, and tenancy by the entirety. In a joint tenancy, each co-tenant owns an undivided share of the property, and the surviving co-tenant has the right to the whole estate (right of survivorship). At common law, four unities are required to create a joint tenancy: (i) time (interests vested at the same time), (ii) title (interests acquired by the same instrument), (iii) interest (interests of the same type and duration), and (iv) possession (interests give identical rights to enjoyment). If these four unities are not present, a joint tenancy cannot be created at common law. Instead, a tenancy in common results. A tenancy in common is a concurrent estate with no right of survivorship. A tenancy by the entirety is a marital estate akin to a joint tenancy in that four unities (plus a fifth—marriage) are required for its creation, and the surviving spouse has the right of survivorship. A conveyance from O "to O and A" does not satisfy the unities of time and title because O acquired his interest first by another instrument. Thus, the conveyance creates a tenancy in common rather than a joint tenancy. The conveyance does not create a tenancy by the entirety at common law because, as explained above, it does not satisfy the unities of time and title, and the facts do not indicate that O and A are husband and wife. The conveyance does not create a tenancy for years at common law. A tenancy for years is a leasehold estate in land wherein the tenant has a present possessory interest in the leased premises and the landlord has a reversion. Here, O and A are not in a landlord-tenant relationship.
In order to be guilty of common law burglary, one must break and enter a dwelling with: A Recklessness about the consequences that could occur if someone was inside the home B No particular mental state; the act of breaking and entering is sufficient to establish burglary C The specific intent to commit a felony therein D Knowledge that one lacks authority to enter the dwelling
C. Burglary is a specific intent crime. A specific intent crime requires the doing of the criminal act with a specific intent or objective. The common law definition of burglary is the breaking and entering of the dwelling of another at nighttime with the intent of committing a felony therein. Because burglary requires the intent to commit a felony at the time of the entering, it is a specific intent crime.
Which of the following acts will terminate an easement? A Voluntary destruction of the servient estate. B Nonuse of the easement for the statutory period. C Condemnation of the servient estate. D Use of the easement beyond its legal scope.
C. Condemnation of the servient estate will terminate an easement. The easement holder may be entitled to compensation for the value lost. Use of the easement beyond its legal scope will not terminate an easement. Instead, the easement is surcharged, and the servient owner may sue to enjoin the use. Nonuse of the easement for the statutory period will not terminate an easement. An easement can be extinguished by the easement holder's physical act of abandonment (e.g., erection of a permanent structure over the easement). However, mere nonuse, even for a long period of time, is insufficient to constitute an abandonment of the easement. To terminate the easement, the nonuse must be combined with other evidence of intent to abandon it. Voluntary destruction of the servient estate (e.g., tearing down a building to erect a new one) will not terminate an easement. On the other hand, involuntary destruction of the servient estate (e.g., by fire or flood) will extinguish the easement.
Which of the following statements is true regarding Congress's taxing power? A By its terms, the federal power to tax is exclusive. B The federal taxing power allows Congress to tax exports. C Federal taxes must be uniform among the states. D A federal tax is permitted only if Congress otherwise has the power to regulate the subject of the tax.
C. Federal taxes must be uniform among the states. This means that a federal tax on an item or activity must be the same from state to state. By its terms, the federal power to tax is NOT exclusive. The tax provisions do not prohibit the states from taxing, and thus the states have the power to tax as well as the federal government. The federal taxing power does NOT allow Congress to tax exports. Neither Congress nor the state can tax exports to foreign countries. It is not true that a federal tax is permitted only if Congress otherwise has the power to regulate the subject of the tax. Congress may also tax to raise revenue.
Horizontal privity exists between: A A party burdened under a real covenant and any party seeking to enforce the covenant B An original party to a real covenant and her successor in interest C Parties to a real covenant who shared an independent interest in the land at the time they entered the covenant D The original parties to a real covenant, regardless of their relationship
C. Horizontal privity exists between parties to a real covenant who shared an independent interest in the land at the time they entered the covenant. A real covenant is a written promise to do or not do something on the land. The burden of the covenant will run with the land if: 1. The covenanting parties intended that successors in interest be bound by the covenant; 2. The successor in interest has notice of the covenant; 3. There is horizontal privity between the original covenanting parties; 4. There is vertical privity between the covenantor and her successor in interest; and 5. The covenant touches and concerns the land. Horizontal privity requires that the original covenanting parties shared some interest in the land independent of the covenant at the time they entered the covenant (e.g., as grantor and grantee). Thus, it does NOT exist between the original parties to a real covenant absent such a relationship, nor does it exist generally between a party burdened under a real covenant and any party seeking to enforce the covenant. In contrast with horizontal privity, vertical privity refers to the relationship between an original party to a real covenant and her successor in interest. For the burden of a covenant to run, this element is satisfied if the successor holds the entire durational interest held by the covenantor at the time she made the covenant.
If A and B own property as joint tenants, and B dies leaving a will devising her interest in the property to C, who owns the property? A C only B A and C, as joint tenants C A only D A and C, as tenants in common
C. If A and B own property as joint tenants, and B dies leaving a will devising her interest in the property to C, A only owns the property. A testamentary disposition by one joint tenant will not sever a joint tenancy. A will devising a joint tenant's interest to another is inoperative as to joint tenancy property because when the co-tenant who is the testator dies (which is when the will becomes effective), her rights in the joint tenancy property are extinguished, and the will has no effect on them. Thus, upon B's death the property is freed from her concurrent interest, leaving A the sole owner and C with no interest in the property. On the other hand, certain acts by one joint tenant will sever a joint tenancy (e.g., suit for partition, inter vivos conveyance by one joint tenant, execution of a mortgage by one joint tenant in a title theory state). Then, the transferee takes the interest as a tenant in common and not as a joint tenant. Thus, if B had successfully conveyed her interest to C by deed, A and C would own the property as tenants in common but not as joint tenants. Alternatively, if A and B had owned the property as tenants in common, B's will would have effectively conveyed her interest to C, so that A and C would own the property as tenants in common. Furthermore, C only would be incorrect in any event because B can convey no greater interest than the one-half interest she owns.
If a mortgage exists on property when a real estate contract is signed: A The contract is void B The mortgage is extinguished C Title may be marketable D Title is unmarketable
C. If a mortgage exists on property when a real estate contract is signed, title may be marketable. Every land sale contract contains an implied covenant that the seller will furnish marketable title on the date of closing. Generally, encumbrances (i.e., mortgages, liens, easements, and covenants) render title unmarketable. However, a seller has the right to satisfy a mortgage or lien at the closing with sale proceeds. Thus, if the purchase price is sufficient and this is accomplished simultaneously with the transfer of title, the buyer cannot claim that the seller's title is unmarketable. If a mortgage exists on property when a real estate contract is signed, the mortgage is NOT extinguished. Rather, the mortgage will remain on the land and will encumber the title in the hands of the buyer unless it is satisfied as explained above. If the mortgage is not timely satisfied, the seller will breach the implied covenant of marketability, for which the buyer may pursue several remedies (e.g., rescission, damages, or specific performance with abatement of the purchase price). The contract is NOT void.
If an easement is said to be surcharged, this means: A The easement holder paid valuable consideration to the servient landowner B The easement was terminated by operation of law C The easement's legal scope was exceeded D The easement's legal scope was interpreted to accommodate the holder's future reasonable needs
C. If an easement is said to be surcharged, this means the easement's legal scope was exceeded. The holder of an easement has the right to use another's land (i.e., the servient tenement), but has no right to possess the land. The scope of an easement is determined by the reasonable intent of the original parties, and when the scope has been specified, these specifics will govern. However, when an easement's scope has been set out only in general language, courts will interpret it to accommodate the holder's present and future reasonable needs. In either event, if the easement holder uses the easement in a way that exceeds its legal scope, the easement is surcharged. The servient landowner may enjoin the excess use and possibly sue for damages if the land has been harmed. However, the easement does NOT terminate by operation of law, nor does such use give the servient owner a power of termination. If an easement is said to be surcharged, this does NOT mean the easement holder paid valuable consideration to the servient landowner, but rather that he exceeded its legal scope.
A developer created an exclusive residential subdivision. In his deed to each lot, the following language appeared: Grantee agrees for himself and assigns to use this property solely as a single-family residence, to pay monthly fees as levied by the homeowners' association for upkeep and security guard services, and that the backyard of this property shall remain unfenced so that bicycle paths and walkways may run through each backyard, as per the subdivision master plan [adequately described], for use by all residents of the subdivision. The developer sold lots to an actuary, a baker, and a coroner. All deeds were recorded. The subdivision was developed without backyard fences, with bicycle paths and walkways in place in accordance with the general plan. The actuary in turn sold to an accountant by a deed that omitted any mention of the covenants above, and the accountant had no actual knowledge thereof. Shortly thereafter, the accountant started operating a tax preparation business out of his home. The baker in turn sold to a barber, who knew of, but refused to pay, the monthly fees levied by the homeowners' association. The coroner leased her property for 10 years to a chiropractor, who erected a fence around the backyard, unaware of the covenant against such fencing. According to common law principles, which of the following statements is correct? A If the developer, still owning unsold lots, sues the accountant to have him cease operating the tax preparation business, the accountant would win because there is no privity between the developer and the accountant. B If the homeowners' association sues the barber to collect the monthly fees for upkeep and security guard services, the homeowners' association would win because the covenant regarding fees is enforceable in equity against the barber. C If the barber sues the chiropractor to obtain removal of her backyard fence, the barber would win because the covenant regarding fencing is enforceable in equity against the chiropractor. D If the chiropractor sues the accountant to have him cease operating the tax preparation business, the chiropractor would win because the covenant regarding single-family use is enforceable at law against the accountant.
C. If the barber sues the chiropractor to remove her backyard fence, the barber would win because the covenant regarding fencing is enforceable against the chiropractor as an equitable servitude. An equitable servitude is a covenant that, regardless of whether it runs with the land at law, equity will enforce against the assignees of the burdened land who have notice of the covenant. The benefit of an equitable servitude runs to successors if: (i) the original parties so intended, and (ii) the servitude touches and concerns the land. The burden runs if (i) and (ii) are met and (iii) the subsequent purchaser has actual or constructive notice of the covenant. Privity of estate is not needed to enforce an equitable servitude because it is enforced not as an in personam right against the owner of the servient tenement, but as an equitable property interest in the land itself. Here, the original parties intended for the fencing covenant to be enforceable by and against assignees, as shown by the specific language of the covenant ("Grantee agrees for himself and assigns") and its purpose to provide bicycle paths and walkways running through each backyard for the use of all subdivision residents. The benefit of the covenant touches and concerns the barber's property because it increases his enjoyment thereof by providing him with such paths and walkways. Therefore, the barber is entitled to enforce the covenant. The burden of the covenant touches and concerns the land occupied by the chiropractor because it restricts the landholder in her use of the parcel (i.e., her rights in connection with the enjoyment of the land are diminished by being unable to fence in the backyard). The chiropractor will be deemed to have inquiry notice of the restriction because the subdivision is sufficiently developed in accordance with a general plan for the subdivision. Moreover, any neighbor in a subdivision can enforce a covenant contained in a subdivision deed if a general plan existed at the time he purchased his lot. As has been noted, the maintenance of access to all backyards for use as bike paths and walkways was part of such a general plan. Finally, the fact that the chiropractor did not succeed to the coroner's entire estate, but rather a leasehold interest, is irrelevant because privity is not required to enforce an equitable servitude. Therefore, all of the requirements are in place for the existence of an equitable servitude, which can be enforced by the barber against the chiropractor. (A) is incorrect because there is privity between the developer and the accountant. There was horizontal privity between the original covenanting parties because, at the time the actuary entered into the covenant with the developer, they shared an interest in the land independent of the covenant (i.e., they were in a grantor-grantee relationship). The accountant holds the entire interest held by the actuary at the time the actuary made the covenant; thus, there is vertical privity. (B) is incorrect because the remedy sought is the payment of money. Breach of a real covenant, which runs with the land at law, is remedied by an award of money damages, whereas breach of an equitable servitude is remedied by equitable relief, such as an injunction or specific performance. Because the homeowners' association seeks to obtain from the barber the payment of money, it is inaccurate to refer to this as a situation involving an equitable servitude. (D) is incorrect because, as explained above, if equitable relief is sought, the covenant must be enforced as an equitable servitude rather than a real covenant.
If the buyer of land determines that the seller's title is unmarketable, the buyer: A May sue on the implied covenant of marketable title after closing B Must take title to the land "as is" C Must notify the seller and give a reasonable time to cure the defects D May sue for damages for breach as soon as the defect is discovered
C. If the buyer of land determines that the seller's title is unmarketable, the buyer must notify the seller and give a reasonable time to cure the defects. Every land sale contract contains an implied covenant that the seller will provide marketable title at closing. Marketable title is title reasonably free from doubt, which a reasonably prudent buyer would accept. While it need not be perfect title, it must not present the buyer with an unreasonable risk of litigation. Generally, this means an unencumbered fee simple with good record title. If the buyer of land determines that the seller's title is unmarketable, the buyer may NOT sue for damages for breach as soon as the defect is discovered. As stated above, he must notify the seller and give her reasonable time to cure, even if this requires extending the closing date, and even if time is of the essence. If the seller fails to cure the defects, then the buyer may rescind the contract, sue for damages for breach, get specific performance with abatement of the purchase price, or (in some jurisdictions) require the seller to quiet title. Thus, it is not required that the buyer take title to the land "as is." The buyer may NOT sue on the implied covenant of marketable title after closing. This covenant applies at the contract stage of a land sale transaction, before the closing (i.e., exchange of purchase price and deed). The closing extinguishes the contract, which is said to merge with the deed. Then, absent fraud, the seller is no longer liable on this implied covenant; the buyer must rely on any assurances made in the deed
In most states, the reservation of an annual rent, payable monthly, in a lease with no set termination date creates a: A Tenancy at will B Tenancy for years C Year-to-year periodic tenancy D Month-to-month periodic tenancy
C. In most states, the reservation of an annual rent, payable monthly, in a lease with no set termination date creates a year-to-year periodic tenancy . A periodic tenancy is a tenancy that continues from period to period until terminated by proper notice by either the landlord or the tenant. It may be created by implication if a lease with no set termination date provides for the payment of periodic rent. The majority view is that a lease at an annual rent, payable monthly, creates a periodic tenancy from year to year, and not a month-to-month periodic tenancy. A tenancy at will is a tenancy that continues only until the landlord or the tenant gives notice and time to quit. Although a tenancy at will can arise when a lease has no set termination date, a provision requiring annual rent payments will convert it to a periodic tenancy. A tenancy for years is a tenancy that continues for a fixed period of time and then ends automatically on its termination date. A lease with no stated duration is not a tenancy for years.
Which of the following statements is true regarding executive agreements? A Executive agreements require the advice and consent of the Senate B Executive agreements may not conflict with existing state laws C Executive agreements may not conflict with existing federal statutes D The Constitution provides that executive agreements are part of the "supreme law of the land"
C. It is true that executive agreements may not conflict with existing federal statutes. Executive agreements (i.e., agreements between the President and foreign countries) have been recognized as valid by the Supreme Court, but they are not provided for in the Constitution. As such, they are not part of the "supreme law of the land" and cannot conflict with existing federal statutes, treaties, or the Constitution (which, collectively are the supreme law of the land). Executive agreements do NOT require the advice and consent of the Senate. The advice and consent of the Senate is needed to approve a treaty. Executive agreements MAY conflict with existing state laws. In such a case, the state law becomes unenforceable. The Constitution does NOT provide that executive agreements are part of the "supreme law of the land"; as indicated above, the Constitution does not provide at all for executive agreements.
May a tenant waive the implied warranty of habitability? A Yes, if the tenant covenants to repair B Yes, if the tenant accepts the premises "as is" C No, because such a waiver is against public policy D No, because waiver is prohibited by the Fair Housing Act
C. No, a tenant may not waive the implied warranty of habitability, because such a waiver is against public policy. Under the implied warranty of habitability, a landlord assures that the premises are suitable for human residence. The standard usually applied is the local housing code. One of the reasons for implying a covenant of habitability is to encourage enforcement of the housing code by tenants. Thus, even if the tenant accepts the premises "as is" or covenants to repair, the landlord's obligations under the implied warranty of habitability are usually held to be nonwaivable. The Fair Housing Act bars discrimination based on race, ethnicity, religion, national origin, gender, and disability in the sale or rental of a dwelling. It is not concerned with the implied warranty of habitability.
As part of a deal to raise the federal debt limit, Congress passed a statute by a greater than two-thirds vote in both houses giving the President authority to cancel particular spending provisions that are contained within legislation that he signs into law. The statute provided that Congress could override the President's decisions only by a three-fourths vote. As soon as the statute went into effect, a Senator who had voted against the statute filed suit in federal district court, challenging its constitutionality. Is the Senator likely to succeed in her lawsuit? A Yes, because the President does not have the constitutional power to exercise a line item veto. B Yes, because the statute requires a vote of three-fourths of Congress to override the President's decisions. C No, because the Senator lacks standing to challenge the statute. D No, because the statute passed by more than a two-thirds vote in both houses of Congress.
C. The Senator will not succeed because she lacks standing to challenge the statute. The Supreme Court has held that members of Congress lack standing to challenge a law authorizing the President to exercise a line item veto (such as the statute here), reasoning that the injury is not concrete and personal, but rather is institutional in that it is shared by all members of Congress. [Raines v. Byrd (1997)] (A) is incorrect even though it is a true statement. The Supreme Court has ruled that the President has no power to exercise a line item veto of just part of a bill because it violates the Presentation Clause of the United States Constitution; the President must either approve or reject a bill in toto. However, (A) is not the best choice here because the Senator is not a proper person to bring the challenge. (B) is incorrect. The statute itself is invalid regardless of the requirements for overriding the President's line item veto. However, as stated above, the Senator does not have standing to challenge the statute. (D) is incorrect because the success of the lawsuit challenging the statute does not depend on the strength of the vote passing the statute.
A state Occupational Health and Safety Board recently issued regulations valid under its statutory mandate requiring that all employers in the state provide ionizing air purification systems for all employee work areas. These regulations replaced previous guidelines for employee air quality that were generally not mandatory and did not specify the method of air purification used. The requirements regarding air purification systems are likely to be unconstitutional as applied to which of the following employers? A A wholly owned subsidiary of a Japanese corporation with seven retail outlets within the state. B The state supreme court, which recently completed construction of its new courthouse with a non-ionizing air purification system which the builder is contractually bound to maintain for the next three years. C A United States Armed Forces Recruiting Center located adjacent to the state capitol building. D A privately operated community service center funded by donations and constructed through use of a loan provided by the United States Veterans Administration and repayable to that agency.
C. The armed forces recruiting center is least likely to be required to comply with the new state law. A state has no power to regulate activities of the federal government unless Congress consents to the regulation. Accordingly, agents and instrumentalities of the federal government, such as the armed forces recruiting center, are immune from state regulations relating to performance of their federal functions. (D) is incorrect because, although the recreation center's construction was funded by a loan from the Veterans Administration, the center itself is privately operated and funded by donations. As a result, the center has only a tenuous connection with the federal government, so that it cannot claim the immunity afforded to a federal agency or instrumentality. Accordingly, in the same sense as is employed in the federal tax immunity cases, the agency does not "stand in the shoes" of the federal government. Thus, the application of the state regulations to the recreation center would not present constitutional problems. (A) apparently refers to the principle that the power to regulate foreign commerce lies exclusively with Congress. However, the mere fact that the regulated outlets are part of a wholly owned subsidiary of a Japanese corporation does not mean that the state regulations affect foreign commerce. The subsidiary's activities are conducted entirely within the state, and do not touch upon foreign commerce in any way. Therefore, application of the regulations so as to require the subsidiary to provide an ionizing air purification system for its employee work areas will not constitute a proscribed state regulation of foreign commerce. Thus, (A) is incorrect. (B) is more troubling, but does not offer as compelling an argument as (C). The Contract Clause limits the ability of states to enact laws that substantially impair contract rights (i.e., destroy most or all of a party's rights under an existing contract). Under the Clause, the Supreme Court will subject state actions that impair their own contracts to strict scrutiny. In any case, even if state action substantially impairs rights under an existing contract, the action still may be upheld if it: (i) serves an important and legitimate public interest; and (ii) is a reasonable and narrowly tailored means of promoting that interest. Here, the state supreme court, as an instrumentality of the state, would probably not have grounds for complaining that its rights under the contract have been impaired, but the builder might have grounds (e.g., the builder might have future economic interests during the three-year service period that will be substantially impaired if the court is required to install an ionizing system). Nevertheless, the regulation still may be valid if the state can prove that it truly serves the important public interest of protecting the health and safety of workers in the state and is narrowly tailored to promoting that interest. In any case, because it is uncertain whether the vendor's rights have been substantially impaired and, if so, whether the state can prove the worth of the regulation, (C) is a better choice.
A wealthy philanthropist owned a mansion built to his exact specifications, featuring a pipe organ built into the wall of the music room. The organ was impressive, with beautiful hand-carved wood scrollwork. The accompanying bench was made from the same wood as the organ and was carved to match the patterns on the organ. The bench was fully movable and could be slid into a niche beside the organ when not in use, although the philanthropist usually left the bench in front of the organ for its matching effect, even when the organ was not being played. The philanthropist died, and his will left all of his personal property to his daughter and all of his real property to a local charity. After the will was admitted to probate, the daughter removed all of the furniture and other movables from the mansion, including the organ bench. The daughter refused the charity's request to return the bench to the mansion. If the charity brings suit against the daughter to replevy the bench, who will prevail? A The daughter, because the bench is personalty since it was not bolted to the floor. B The daughter, because removing the bench does not damage the real property. C The charity, because the bench is integrally connected to the organ. D The charity, because removal of the bench reduces the value of the devise to the charity.
C. The charity will win because the organ is a fixture and the bench is integrally connected to the organ. Under the concept of fixtures, a chattel that has been annexed to real property is converted from personalty to realty. As an accessory to the land, it passes with ownership of the land rather than with a transfer of the personal property of an estate. The manifest intent of the annexor determines whether the chattel becomes a fixture. The factors for evaluating the annexor's intent are: (i) the relationship between the annexor and the premises, (ii) the degree of annexation, and (iii) the nature and use of the chattel. Under this analysis, the organ itself is clearly a fixture: (i) the philanthropist was the fee owner of the mansion and had the organ built to his specifications when the mansion was constructed; (ii) the organ was built into the wall of the mansion and could not be easily removed; and (iii) the appearance of the organ and how it complemented the rest of the mansion probably were more important to the philanthropist than its function. Constructive annexation occurs when an article of personal property (an "accession") becomes an integral part of the property, even though it is not physically annexed to the property, in the same sense that a fixture becomes an integral part of the realty. The doctrine is fully applicable in this case even though the accession goes with an item of property that is itself converted from personalty to realty, as the organ was here. The bench is an accession because it was created as an integral part of the organ and significantly contributes to an important aspect of the organ: its overall appearance. Removing the bench and replacing it with a bench made of different wood or carvings would damage the aesthetic value of the organ. Thus, the charity will succeed in obtaining the bench because it is not severable from the organ. (A) is incorrect because the fact that the bench was not bolted to the floor is not determinative. The bolting goes to whether the bench alone is a fixture. This is irrelevant because it is an accession to (and thus a part of) the organ, which is clearly a fixture. (B) is incorrect because the fact that removing the bench does not damage the building itself does not give the daughter the right to remove it. Removing the bench will damage the organ because the bench is an accession to the organ. The organ, as a fixture, is part of the real property; thus, removal of the bench will damage the property. (D) is incorrect because harm to the parties is not an issue in determining whether an item is a fixture. The relevant question is whether removal damages the real property. If the bench were found to be personalty, the fact that its removal would reduce the value of the charity's gift would have no impact on the daughter's right to remove it.
If a regulation of speech is found to be overbroad, but judged in relation to its plainly legitimate sweep it does not prohibit a substantial amount of protected speech, may it be enforced? A Yes, against anyone because it does not prohibit a substantial amount of unprotected speech B No, because it is overbroad, but judged in relation to its plainly legitimate sweep C Yes, but only against persons engaging in activities that are not constitutionally protected D No, because it is unconstitutional
C. The correct answer is yes, but only against persons engaging in activities that are not constitutionally protected. The First Amendment, among other things, protects against government infringement of speech. Regulations that prohibit more speech than necessary are overbroad and may be unconstitutional. However, if a regulation is not too overbroad, the Court allows it to be enforced against people whose speech or speech activities are not protected by the Constitution. On the other hand, if the regulation prohibits a substantial amount of protected speech when judged in relation to its plainly legitimate sweep, it is facially invalid and cannot be enforced against anyone. The choice indicating that the regulation may be enforced against anyone is incorrect, because, as explained above, it cannot be enforced against persons engaged in protected speech or speech activities. The choices indicating that the regulation cannot be enforced are incorrect for the reasons stated above.
One Saturday, the owner of an art gallery and her friend were discussing art after the friend had helped the owner move some furniture in her home. The friend mentioned that he was very fond of a particular artist. The gallery owner asked her friend if he would like to buy a painting by the artist, entitled "Tears of a Clown," recently consigned to the gallery. The friend said that he would love it, but he only had $2,700. The gallery owner told her friend that she would let him have the painting for that price. The friend knew that the painting was priced at $7,000. He immediately wrote out a check for $2,700 and gave it to the gallery owner, who told him to visit the gallery on Monday to pick up the painting. On Sunday, a salesperson at the gallery sold "Tears of a Clown" to a gallery customer. Neither the salesperson nor the customer knew of the agreement between the gallery owner and her friend. The customer took the painting with him on Sunday. When the friend arrived at the gallery on Monday, the painting was gone. Can the friend obtain specific performance from the gallery owner? A Yes, because there was a bargained-for exchange of promises between the friend and the gallery owner. B Yes, because the friend's assistance to the gallery owner in moving her furniture should be considered part of the quantum of adequate consideration. C No, because the painting was sold to a bona fide purchaser for value and enforcement against the gallery owner is no longer feasible. D No, because the gallery owner's promise was essentially a gift to her friend that she was free to revoke.
C. The salesperson sold the painting in good faith to a customer. Because the gallery owner no longer actually has the painting, there is no way to specifically enforce her agreement to convey it to her friend. Specific performance is granted when: (i) there is a valid contract; (ii) the legal remedy is inadequate; (iii) enforcement is feasible; and (iv) mutuality of remedy is present. The gallery owner and her friend had a contract, pursuant to which the gallery owner promised to sell her friend the painting for $2,700. Although this was an oral contract for the sale of goods for a price exceeding $500 and thus subject to the Statute of Frauds, the contract is removed from the Statute by the fact that the friend tendered full payment for the painting. Thus, the oral nature of the agreement is no hindrance to its validity. Moreover, a painting, by its nature, is unique, rendering the legal remedy (damages) inadequate. However, feasibility of specific performance against the gallery owner is lacking here. The salesperson sold the painting to a customer who paid value for it and was unaware that the gallery owner had already agreed to sell it to her friend. The salesperson also was unaware of the gallery owner's agreement with her friend. With the subject matter of the contract having been transferred in good faith to a third party, there is no feasible means to enforce against the gallery owner her agreement to sell the painting to her friend. Thus, the right to specific performance is cut off. Regarding (A), it is true that there was a bargained-for exchange of promises between the gallery owner and her friend. Nevertheless, specific performance is unavailable because enforcement is not feasible. (B) is incorrect because the friend's assistance to the gallery owner in moving her furniture does not form part of the basis of the consideration. The assistance given by the friend occurred before the gallery owner's promise to sell the painting, and thus was not given in exchange for the promise when made. Also, even if the assistance given did form part of the quantum of adequate consideration, specific performance would still be denied because enforcement is not feasible. (D) is incorrect because a gift is a voluntary transfer of property from one person to another without compensation or consideration. The gallery owner clearly stated that she wanted her friend to pay her $2,700 for the painting. Thus, the donative intent necessary for a gift was absent.
The owner of a furnished cottage leased it to another for one year. While this lease was in effect, the cottage owner found herself in immediate need of cash, and decided to burn down the cottage to collect the insurance on it. She waited until one evening when the tenant was away. The cottage owner then used her own key to gain access to it. To make it appear that the fire was caused accidentally by the tenant, she soaked one end of the mattress on the bed in the bedroom with gasoline and then left a lighted cigarette burning at the other end of the mattress. She planned that the cigarette would ignite the mattress and that when the fire smoldered to the area soaked in gasoline, the entire bed would burst into flames, and the resulting fire would destroy the house. However, the tenant returned home earlier than expected and discovered the fire just as the mattress burst into flames. He immediately put it out with a fire extinguisher. A police investigation revealed the cottage owner's activities. The cottage owner is guilty of: A Burglary as to the house and arson as to the mattress. B Neither burglary nor arson because she owned the structure and its contents. C Burglary and attempted arson. D Attempted arson but not burglary because she entered with her own key.
C. The cottage owner is guilty of burglary because the right of occupancy belonged to the tenant. However, the fact that there was no burning of the structure means that the cottage owner is guilty of attempted arson, rather than arson. Burglary at common law is a breaking and entering of the dwelling of another at nighttime, with the intent of committing a felony therein. A breaking requires some use of force to gain entry, but minimal force is sufficient. In determining whether the dwelling is that of another, occupancy rather than ownership is material. Thus, an owner can commit burglary of her own structure if it is rented and used as a dwelling by someone else. Here, although the cottage owner owned the cottage, the tenant had the right to occupy it pursuant to a lease. Thus, for purposes of the crime of burglary, the cottage owner is deemed to have entered the dwelling of another. Although the cottage owner used her own key to gain access to the cottage, this was still an unconsented use of force to effectuate entry, thereby constituting a breaking. This breaking and entering of the tenant's dwelling occurred in the evening. At the time of the entry, the cottage owner intended to commit the felony of arson. Consequently, all the elements of burglary are in place, making her guilty of this crime. Arson consists of the malicious burning of the dwelling of another. There is a requirement of some damage to the fiber of the wood or other combustible material. As with burglary, ownership of the structure is not material for determining whether the dwelling is that of another; rather, the right to occupancy is material.
A company operated a small amusement park on property it owned near a residential neighborhood. On a day when the park was closed, a 10-year-old girl snuck into the park with some friends by climbing over a chain link fence. While climbing on one of the carnival rides, the girl slipped and cut her leg on an exposed gear assembly, sustaining serious injuries. Through her guardian ad litem, the girl brought suit against the company to recover damages for her injuries. At trial, she presented evidence of the accident and her injuries. In defense, the company established that the girl read and understood the "No Trespassing" signs that were attached to the fence. The company also established that it had not had any previous reports of children sneaking into the park when it was closed. Before submission of the case to the jury, the company moved for summary judgment. Is the court likely to grant the company's motion? A Yes, because the girl was a trespasser who the company had no reason to anticipate would be on the property. B Yes, because the girl knew she was trespassing and was old enough to recognize the danger. C No, because the jury could find that the company should have foreseen that children would sneak into the park. D No, because the appeal of the carnival rides attracted the girl into the park.
C. The court is not likely to grant the company's motion because the jury must determine whether the attractive nuisance doctrine applies. Under this doctrine, a landowner has a duty to exercise ordinary care to avoid reasonably foreseeable risk of harm to children, including trespassing children, caused by artificial conditions on his property. Here, while the company has presented some evidence against application of the attractive nuisance doctrine, it is ultimately the trier of fact's role to determine whether the doctrine applies and whether the company exercised ordinary care. Hence, the court will likely deny the company's motion and allow the jury to make that determination. (A) is incorrect because the jury could find that even though the company was not aware of children trespassing, it should have anticipated that they might try to sneak onto the property because it was an amusement park operation. (B) is incorrect because even though the girl knew she was trespassing, the jury could find that she did not appreciate the risk of playing on the rides. (D) is incorrect because the fact that the girl was attracted onto the land by the artificial condition is just one factor for determining whether the attractive nuisance doctrine applies. Foreseeability of harm to a child is the true basis of liability.
Commercial fishing has long been one of the major industries of a coastal state. To protect the fishing industry and to promote the general welfare of the state's citizens, the legislature of the state enacted statutes requiring licenses for commercial fishing. An applicant for the license must pay a $300 fee and establish that he has been engaged in commercial fishing in the waters of the state for 10 years. A commercial fisherman residing in a neighboring state frequently takes his fishing boat up the coast. His favorite spot is approximately two miles off the coast of the legislating state. If the commercial fisherman challenges the constitutionality of the legislating state's statutes, should the court find the statutes constitutional? A Yes, because Congress has not enacted legislation regarding the subject matter of the statutes. B Yes, because economic and social regulations are presumed valid. C No, because less restrictive means are available. D No, because Congress has exclusive power to regulate foreign commerce, which includes commercial ocean fishing.
C. The court should not find the statutes constitutional, because less restrictive means are available. The statutes violate the Privileges and Immunities Clause of Article IV, which prohibits discrimination against nonresidents with respect to essential activities (e.g., pursuing a livelihood) unless (i) the discrimination is closely related to a substantial state purpose, and (ii) less restrictive means are not available. Here, other controls could be placed on fishing without discriminating against out-of-state fishermen. (A) is wrong because, even though Congress has not acted in this area, the statutes would still be unconstitutional in light of the negative implications of the "dormant" Commerce Clause. Congressional silence is, therefore, irrelevant. (B) states a due process test which, even if applicable, would not preclude a finding of unconstitutionality on other grounds. (D) is wrong because the activity here does not involve foreign commerce—this is a dispute between one state and a citizen of another state.
A State A citizen and a State B citizen were in an automobile accident in State B. The State B citizen filed a negligence action for $500,000 against the State A citizen in a federal district court located in State B. The State A defendant would prefer to litigate the case in a State B state court. The State A defendant thus filed a notice of removal, seeking to transfer the case to a State B state court. Should the federal court grant the motion? A Yes, because federal diversity jurisdiction is not needed to protect the State A citizen from the potential bias of State B courts if the State A citizen requests that the matter be litigated in the State B state courts. B Yes, because tort actions arising from accidents in State B should be litigated in State B state courts. C No, because removal to state court is not available for cases that are properly filed in federal court and that are within the federal court's subject matter jurisdiction. D No, because removal to state court is not available when one of the parties is a citizen of the state in which the action is pending.
C. The court should not grant the motion. Under 28 U.S.C. section 1441, a defendant may remove an action that could originally have been brought in the federal courts, based on either a federal question being presented or on diversity of citizenship. However, there is no similar provision that allows a case initially filed in federal court to be moved to state court. (An action that was wrongfully removed from state court to federal court may be remanded back to state court, but that procedure is not applicable here because the case was initially filed in federal court.) (A) is incorrect. As stated, although one goal behind diversity jurisdiction is to lessen an in-state prejudice against out-of-state defendants, the lack of that potential prejudice does not provide a basis for moving a case from federal court to state court. (B) is an incorrect statement of the law. A case based on diversity jurisdiction may include a tort action that arose in the jurisdiction. (D) is an incorrect statement of the law. An in-state defendant is prevented from removing a case to federal court based on diversity. The fact that the plaintiff may be a citizen of the state in which the state case was filed does not prevent removal.
A woman was arrested, given Miranda warnings, and questioned about an armed robbery. After she asked to speak with an attorney, the police stopped questioning her about the robbery. Several hours later, the police gave the woman a fresh set of Miranda warnings and began to question her about a different robbery. She did not repeat her request for an attorney and instead made several incriminating statements about the robbery. At the woman's trial for the robbery for which she made incriminating statements, the prosecution seeks to have her statements introduced into evidence. If the woman's attorney objects on appropriate grounds, how should the court rule? A Overrule the objection, because the police did not badger the woman into confessing. B Overrule the objection, because the woman did not renew her request for an attorney after receiving fresh Miranda warnings. C Sustain the objection, because the police did not honor the woman's request. D Sustain the objection, because a confession obtained in violation of a defendant's Miranda rights but otherwise voluntary may be used against the defendant.
C. The court should sustain the objection because the police did not honor the woman's request for an attorney. At any time prior to or during a custodial interrogation, the accused may invoke a Miranda (Fifth Amendment) right to counsel. If the accused invokes this right, all questioning must cease until the accused is provided with an attorney or initiates further questioning himself. Thus, the police questioning of the woman about the robbery was improper, and she can have her statements excluded. (A) is incorrect. After receiving Miranda warnings, if an accused invokes the right to remain silent, the police cannot badger the accused. However, courts have ruled that if the police scrupulously honor the request, they can rewarn the accused and later resume questioning, at least about a different crime. Here, however, the accused did not simply invoke the right to remain silent, but rather requested an attorney. After such a request, as indicated above, all questioning must cease. (B) is incorrect because the accused does not need to reassert the right to an attorney; all questioning must stop until the accused is provided an attorney or resumes the questioning herself. (D) is incorrect. It is stating the rule for impeachment - a confession obtained in violation of a defendant's Miranda rights but otherwise voluntary may be used against the defendant for purposes of impeachment, but there is no such rule for use of the confession for other purposes.
A state located in the southern half of the United States experienced a strong influx of retirees, due in part to its mild winters and in part to the generous health benefits that the state historically provided to its elderly residents who fell below the federal poverty line. The state's Office of Budget Management determined that the influx of retirees would bankrupt the state's health care benefit fund within five years. To preserve the fund and ensure the health of its citizens, the state revised its health care statute to make persons ineligible for coverage until they have lived in the state for at least one year. If a retiree who was denied benefits because she just moved to the state challenges the constitutionality of the statute in federal court, is she likely to prevail? A No, because the state has a compelling interest in maintaining the fiscal integrity of its health care fund. B No, because the states do not have a constitutional duty to provide health care benefits to retirees even if they fall below the federal poverty line. C Yes, because the requirement improperly burdens the right of interstate travel in violation of the Equal Protection Clause of the Fourteenth Amendment. D Yes, because the requirement deprives some retirees of certain privileges and immunities in violation of the Privileges and Immunities Clause of Article IV.
C. The court will likely find that the one-year residency requirement is unconstitutional because it burdens the right to travel. An individual has a fundamental right to travel from state to state, and a state law that is designed to deter persons from moving into the state is likely to violate the Equal Protection Clause (as well as the Fourteenth Amendment Privileges or Immunities Clause). When a state uses a durational residency requirement (a waiting period) for dispensing benefits, that requirement normally should be subject to the strict scrutiny test, and usually will be found not to have satisfied the test. One such requirement that has been invalidated on this basis is a one-year waiting period for state-subsidized medical care, such as the one here. [See Memorial Hospital v. Maricopa County (1974)] (A) is incorrect. The Supreme Court has specifically held that a state's interest in fiscal integrity is not sufficient to justify a one-year waiting period for welfare or health benefits. (B) contains a true statement—the states have no constitutional duty to provide health care benefits for those below the poverty line. However, once a state chooses to provide such benefits, it may not do so in a manner that violates the Constitution, and, as explained above, the restriction here violates the right to travel. (D) is incorrect because the privileges and immunities protection of Article IV prohibits discrimination by a state against nonresidents when fundamental national rights are involved. Here, the restriction differentiates between residents. While that could violate the Fourteenth Amendment Privileges or Immunities Clause, Article IV is not implicated.
The defendant was being sued for striking and seriously injuring a child with his car one evening while the child was playing in the street near the curb. At trial, the attorney for the child's parents seeks to have the defendant's wife testify that he had told her what had happened as soon as he had gotten home, and that he had said, "Between you and me, just before all this happened, I took a quick peek at the back seat to make sure I brought my briefcase home with me. If I had kept my eyes on the road, I never would've hit the kid." The wife was recently divorced from the defendant and eager to testify against him. The attorney for the child's parents also presented evidence that, unknown to either the defendant or his wife, their neighbor overheard this conversation through her open window. Assuming a proper objection by the defense attorney, will the wife be permitted to so testify? A Yes, because she and the defendant were divorced during the time between the making of the statement and the trial. B Yes, because the fact that the neighbor heard the statement removes the privileged status of the statement. C No, because the defendant's statement was a confidential marital communication. D No, because the privilege to foreclose such testimony belongs to the party-spouse.
C. The defendant's statement to his wife was made in reliance upon the intimacy of what was at that time their marital relationship. Thus, he has a privilege to prevent her from disclosing the statement. Either spouse (whether or not a party) has a privilege to refuse to disclose, and to prevent another from disclosing, a confidential communication made between the spouses while they were married. Divorce does not terminate this privilege retroactively. At the time that the defendant made the subject statement to his wife, they were married. Given that the statement essentially constituted an admission of liability by the defendant, that he prefaced it with "between you and me," and that he made the statement in the privacy of their home, it seems likely that the statement was made in confidentiality and in reliance upon the intimacy of the marital relationship. Thus, both the defendant and his wife may refuse to disclose, and may prevent the other from disclosing, the statement. Consequently, the defendant can prevent the wife from testifying to the statement. (A) is incorrect because the communication was made during the marriage, and the privilege is not abrogated by a later divorce. (B) is incorrect because the fact that the neighbor heard the statement was unknown to the defendant and his wife. If the communication is made in the known presence of a stranger, it is not privileged. However, if the statement was not made within the known hearing of a third party and it is overheard, absent a showing of negligence on the part of the speaker, it remains privileged. Nothing in these facts indicates negligence. Thus, the defendant can prevent his wife from testifying to the statement. (D) is incorrect because the privilege for confidential marital communications belongs to both spouses, rather than to just one. The trial here is a civil case, so the spousal immunity is inapplicable; this question involves the privilege for confidential marital communications. Furthermore, even if spousal immunity did apply, the federal privilege belongs to the witness-spouse, not the party-spouse.
A driver was operating her car on a city street when she was stopped by a police officer for speeding. As the police officer reached the driver's car, he saw her put something into her purse. The officer told the driver, "Ma'am, you were speeding; that's why I stopped you. I'd like your driver's license, and, by the way, what did you just put into your purse?" The driver responded, "It's just a marijuana cigarette, but don't worry, I've only had two and my driving judgment hasn't been impaired." The officer took her purse, removed the "joint," and charged the driver with possession of marijuana as well as speeding. At the driver's trial for marijuana possession, the prosecution seeks to introduce the marijuana cigarette into evidence. The driver's attorney moves to suppress the evidence. The defense motion should be: A Granted, because the cigarette is fruit of the poisonous tree. B Granted, because the police officer did not have a valid search warrant. C Denied, because the police officer's asking about the contents of the driver's purse did not constitute custodial interrogation. D Denied, provided the police officer had a reasonable suspicion of criminal activity.
C. The defense motion should be denied because the driver was not in custody when she made the statement. Persons temporarily detained for routine traffic stops are not in custody for Miranda purposes. Therefore, the driver was not entitled to Miranda warnings, and her statement about the marijuana was not tainted. Her statement thus properly provided the probable cause for the search of her purse. (A) is therefore wrong. (B) is wrong because this case falls within the automobile exception to the warrant requirement. Under that exception, if the police have probable cause to believe that a vehicle contains contraband or fruits, instrumentalities, or evidence of a crime, they may search the vehicle, including the driver's belongings, without a warrant. Here, the driver's response established probable cause to search her purse. (D) states the test for a stop, not a search. An automobile search requires probable cause.
A department store buyer and a manufacturer of food processors entered into a written contract whereby the manufacturer would sell to the buyer 50 of its top-of-the-line models for $100 each. When the delivery arrived on May 15, several days early, the buyer noticed that the food processors were a different model that did not have all of the features as the top-of-the-line model that was ordered. The buyer contacted the manufacturer and told him that he was rejecting the food processors that were delivered to him and expected the manufacturer to send 50 top-of-the-line models immediately. The manufacturer replied that because of a backlog of orders that had not yet been filled, the top-of-the-line models could not be delivered until August 15. Because the department store had contracted with a restaurant to deliver three top-of-the-line models by May 31, the buyer delivered three of the nonconforming food processors along with a promise to replace them with three top-of-the-line models in mid-August. The buyer returned the remaining food processors to the manufacturer. How much could the department store recover from the manufacturer for the three food processors that it delivered to the restaurant? A Nothing, because they were resold to another. B Nothing, because it accepted them knowing they were defective. C The difference between the market price of the top-of-the-line models and the existing food processors' actual value. D The difference between the existing food processors' actual value and the cost of the food processors that the department store must provide to the restaurant in mid-August.
C. The department store was entitled to recover contract damages from the manufacturer for the three food processors that it accepted. If the buyer accepts goods that breach one of the seller's warranties, the basic measure of damages is the difference between the value of the goods as delivered and the value they would have had if they had been according to the contract, which is best stated by choice (C). The department store's acceptance of the three food processors did not waive its right to collect damages for the defect in quality. Thus, (A) and (B) are wrong. Having accepted the nonconforming food processors, the department store's damages would be the difference between the value of the food processors as received and what they would have been worth if they had been as warranted, plus foreseeable incidental and consequential damages. (D) is wrong because the measure of damages is based on market value rather than cost. Also, the agreement with the restaurant was to accommodate the department store only and was not foreseeable by the manufacturer.
On January 15, a patient sued his doctor and his surgeon in State A state court for medical malpractice. All acts of malpractice took place in State A. The patient and the surgeon are citizens of State A; the doctor is a citizen of State B. Fifteen months later, it was learned after extensive discovery that the surgeon was only peripherally involved in the patient's treatment and was in no way negligent, so the patient dismissed the cause of action against the surgeon. Two weeks later, the doctor seeks to remove the case against him to federal court in State A, alleging diversity jurisdiction. May the doctor successfully remove the case to a federal district court? A Yes, because removal was sought within 30 days of the date that the doctor first learned that the case had become removable. B Yes, because removal was sought within one year of the case becoming removable. C No, because a case may not be removed to federal court more than one year after the action was commenced. D No, because a defendant may not seek removal if the cause of action accrued in the forum state.
C. The doctor may not remove the case. If a diversity action is not initially removable but later becomes removable (as by dismissal of a nondiverse defendant), it may not be removed more than one year after it was commenced in state court. (A) is incorrect. In all cases, the defendant has 30 days after a case becomes removable to file a notice of removal; however, for diversity cases only, removal must also occur within one year from the date the case is commenced. (B) is incorrect. The one-year limit begins when the action is commenced, not when the case first became removable. (D) is incorrect. There is no such rule.
A state statute provides a remedy for victims of employment discrimination. The statute requires complainants to bring charges before the state's fair employment commission within 180 days of the alleged unlawful employment practices. The commission then has 120 days to convene a fact-finding conference to obtain evidence, ascertain the parties' positions, and explore settlement possibilities. An employee was discharged from his job purportedly because of a physical handicap unrelated to his ability to perform his job. The employee filed a timely complaint, alleging unlawful termination of employment, as required by the statute. However, through inadvertence, the commission scheduled the fact-finding conference five days after the 120-day statutory period expired. At the conference, the employer moved that the charge be dismissed for lack of a timely conference. The commission denied the motion. The employer petitioned the state supreme court. The court held for the employer, stating that the failure to comply with the 120-day requirement deprived the commission of jurisdiction to consider the employee's charge. On appeal to the United States Supreme Court, the employee argues that his right to due process will be violated if the commission's error is allowed to extinguish his cause of action. Which of the following best describes the viability of the employee's due process claim? A The claim fails, because the employee had no protected property interest in his job. B The claim fails, because the state legislature, having conferred on claimants a remedy for claims of unfair employment practices, has the prerogative to establish limiting procedures for such claims. C The claim succeeds, because the employee had a protected property interest in the remedy. D The claim succeeds, because of the fundamental unfairness of leaving the employee without a remedy.
C. The employee has more than an abstract interest in redressing his grievance. His right to redress, guaranteed by the state through its statutory enactment, is itself a property right. Although the legislature may elect not to confer a property interest, it may not constitutionally authorize the deprivation of such an interest, once conferred, without appropriate procedural safeguards. [Logan v. Zimmerman Brush (1982)] Thus, the state statute cannot be applied so as to deprive the employee of his property interest in using the statutory procedure for possible redress of unfair employment practices without at least affording him an opportunity for an appropriate hearing. It follows that (A) and (B) are incorrect, both because they state the wrong result and because they misstate the applicable rules. As indicated, the employee has a property interest and thus (A) is wrong. (B), in turn, describes a valid general rule but does not describe this case. The state can enact specific procedures, expect the employee to follow them, and bar the claim if he fails to do so. That is, however, not what happened here. Rather, the state itself failed to act in a timely manner, and the statutory time limit operated indiscriminately to extinguish the employee's claim. Finally, (D) is incorrect because it is too general. What happened to the employee was "unfair" because it deprived him of a property right, not because the state is required to provide any remedy, or a specific remedy.
Which of the following would not make title to land unmarketable? A Evidence that a prior grantor lacked capacity to convey the property B A significant variation in the description of property from one deed to the next C The existence of a mortgage on which the statute of limitations has run D The defective execution of a prior deed in the chain of title
C. The existence of a mortgage on which the statute of limitations has run would not make title to land unmarketable. Every land sale contract contains an implied covenant that the seller will provide marketable title at closing. While it need not be perfect title, it must not present the buyer with an unreasonable risk of litigation. Generally, this means an unencumbered fee simple with good record title. Title may be unmarketable because of a defect in the chain of title. Examples include a significant variation in the description of the land from one deed to the next, the defective execution of a prior deed in the chain of title that thus fails to meet the requirements for recordation, and evidence that a prior grantor lacked capacity to convey the property. Many courts hold that an ancient lien or mortgage on the record will not render title unmarketable if the seller has proof of its satisfaction or the statute of limitations on the claim would have run under any possible circumstance, including tolling for disabilities.
A farmer who supplies several local bakeries with grains wanted to sell his rye before the growing season was over. The farmer sent the following e-mail to a local baker: "Will sell my unprocessed rye, 20 bushels maximum, best price $100 per bushel, firm for 48 hours. /s/ Farmer." Unsure how the baker would respond, and anxious to find a buyer for the rye, the farmer made the same offer to the baker's chief competitor by e-mail later that same day. The baker was delighted to receive the offer, but needed a day or so to figure out how much rye she needed. When she accepted the farmer's offer the next day, e-mailing to him an order for 20 bushels, she was aware of the farmer's offer to her competitor, and that her competitor had also e-mailed an order to the farmer for 20 bushels. Unbeknownst to the baker, the farmer has only 30 bushels of rye left in his fields. Assuming the farmer is a merchant with respect to rye, which of the following states the probable legal consequences of the correspondence between the parties? A The farmer has a contract with the baker and her competitor for 15 bushels each. B The farmer has a contract with the baker's competitor for 20 bushels and a contract with the baker for the remaining 10 bushels. C The farmer has a contract with the baker for 20 bushels and a contract with her competitor for 20 bushels. D The farmer has a contract with neither the baker nor her competitor.
C. The farmer has two contracts, one with the baker and one with the competitor, for 20 bushels each. Because his e-mail provided a firm price for 48 hours and the farmer is a merchant, the offer was an irrevocable firm merchant's offer during the 48 hours. Under the UCC, which governs here because goods are involved, a written offer signed by a merchant giving assurances that it will stay open will be irrevocable for the time stated. The farmer qualifies as a merchant of rye (one who deals in goods of that kind sold) and his offer was written and signed and contained words of firmness ("firm for 48 hours"), so it was irrevocable for 48 hours. The baker accepted the offer within the stated time. Thus, a contract was formed between the baker and the farmer. A contract was also formed between the baker's competitor and the farmer because the competitor accepted the farmer's offer. Therefore, the farmer is obligated to both the baker and her competitor for 20 bushels. If the farmer does not have the appropriate quantity in his field, he will have to procure it from somewhere else or be in breach. (A) and (B) are incorrect because they do not reflect the terms of the contracts agreed to by the parties. If a seller is unable to fully perform because of an unforeseen circumstance (i.e., impracticability), he must allocate deliveries between customers. First, this is not an unforeseen circumstance. Second, allocating between customers does not change those contracts. It is still a breach, and the customers may cancel the contract. (D) is incorrect because, as explained above, the farmer has a contract with both the baker and her competitor.
A rancher entered into a contract to sell her land to a developer for $60,000. The contract provided that the rancher agreed to convey a good and marketable title to the developer 60 days from the date of the contract. At the time set for closing, the rancher tendered a deed in the form agreed to in the contract. The developer's examination of the record prior to the date of closing disclosed, however, that the owner of record was not the rancher, but a farmer. Further investigation by the developer revealed that, notwithstanding the state of the record, the rancher had been in what the developer concedes is adverse possession for 15 years. The period of time to acquire title by adverse possession in the jurisdiction is 10 years. The developer refuses to pay the purchase price or to take possession because of the "inability" of the rancher to transfer a marketable title. In an appropriate action by the rancher against the developer for specific performance, will the rancher prevail? A Yes, because she has obtained a "good and marketable title" by adverse possession. B Yes, because the rancher's action for specific performance is an action in rem even though the farmer is not a party. C No, because the developer cannot be required to buy a lawsuit even if the probability is great that the developer would prevail against the farmer. D No, because the rancher's failure to disclose her lack of record title constitutes fraud.
C. The seller of land is obligated to deliver a title that is free from reasonable doubt either in fact or law. This does not require a perfect title, but rather one that is free from questions that might present an unreasonable risk of litigation. Title is marketable if a reasonably prudent buyer would accept it in the exercise of ordinary prudence. An inability to establish a record chain of title will generally render the title unmarketable. If the seller attempts to rely on adverse possession to show that defects have been cleared, courts traditionally do not favor such an argument, because proof of adverse possession normally rests on oral evidence, which might not be available to the buyer at a later time. Here, although the rancher may have acquired title by adverse possession, the developer should not be faced with the prospect of having to prove this in court in the future. Thus, (A) is incorrect. (If the rancher had written proof or a quiet title judgment, title would be marketable.) (D) is incorrect because it does not appear that the rancher's conduct amounted to fraud. (B) is nonsensical.
A hotelier opening a new inn in the Pacific Northwest sent letters to all known hotel and motel suppliers on June 1, alerting them to his need for such items as ice buckets, televisions, linen, and mattresses. The hotelier received a letter dated June 8 from a hotel supply company, stating that the company had 250 ice buckets left in stock and will sell them to the hotelier for $1 each. The company added that it must receive the hotelier's answer by November 1 and will hold the ice buckets for the hotelier until then. On July 1, the company sold 200 of the ice buckets to a competing hotel chain, which had recently opened a hotel on the East Coast. On July 2, the company sent the hotelier a fax stating it had only 50 ice buckets left for sale. The hotelier received the fax that day, but put it aside and never read it. On July 10, the hotelier notified the company that he was accepting the company's offer to sell 250 ice buckets. The company, upon receiving the hotelier's acceptance, shipped the remaining ice buckets. The hotelier sues the company for failing to deliver all 250 ice buckets. Will the hotelier prevail? A No, because the hotelier is not a hotel supply merchant. B No, because the company's offer was to remain open for more than three months. C Yes, because the company promised in a signed writing to hold the offer open. D Yes, because the hotelier never read the company's July 2 fax.
C. The hotelier will prevail. Ice buckets are movable goods; therefore, Article 2 of the UCC applies. The June 8 letter from the supply company is a firm offer under UCC section 2-205. No consideration is required, because the company is a "merchant" (i.e., one who ordinarily deals in goods of the kind sold) of ice buckets. Where a time period for the offer is stated, the period of irrevocability is that period, except that the period cannot exceed three months. Here, the three-month period would end on September 8. The company's fax stating that it had only 50 ice buckets left to sell constitutes an invalid attempt at revocation, because it is within the three-month period of irrevocability. (A) is incorrect because section 2-205 does not require that the offeree of a firm offer be a merchant; it requires that the offeror be a merchant, and the company is (see above). (B) is incorrect because a firm offer that states a period longer than three months is still firm for the first three months. (D) is incorrect because the hotelier's knowledge, or lack thereof, of the "revocation" of the company's offer is irrelevant because it was invalid; the fact that the company made a firm offer prevents it from revoking the offer within the stated time, not to exceed three months.
A husband was on his way to meet his wife for lunch at the restaurant in the lobby of a bank building where she worked. He had just entered the building, which was owned and operated by the bank, when he heard screams and the sound of breaking glass from the restaurant area. He immediately saw that a large piece of artwork made of stained glass had fallen onto the seating area of the restaurant. In the seating area he saw several injured persons, including his wife, lying in the wreckage of the artwork. He fainted and hit his head on the marble floor, fracturing his skull. The artwork had collapsed because the pedestal that the bank had provided for the artwork was not properly constructed. If the husband sues the bank for his injury, is he likely to prevail? A No, because he was not personally in the zone of danger of physical injury. B No, because he did not actually see the artwork collapse onto the diners. C Yes, because his wife was one of the persons he saw lying in the wreckage. D Yes, because the bank had provided the pedestal for the artwork.
C. The husband will recover for his injuries because his wife was among those injured by the collapse of the artwork. The duty to avoid negligent infliction of emotional distress may be breached when the defendant creates a foreseeable risk of physical injury to the plaintiff. In most jurisdictions, a bystander who sees the defendant negligently injuring another can recover for his own distress if (i) the plaintiff and the person injured by the defendant's negligence are closely related, (ii) the plaintiff was present at the scene of the injury, and (iii) the plaintiff personally observed or perceived the event. Observation is typically by sight, but may also be by hearing or other senses under certain circumstances. Here, the husband heard the screams and the sound of breaking glass when the artwork collapsed as he entered the lobby. Even though he evidently did not see the artwork collapse on the diners, he heard it crash where his wife was sitting and saw the immediate aftermath. Because his wife was one of the persons injured by the collapse of the artwork, he can recover damages for the injuries caused by his distress.
A witness is called in a contract action between a plaintiff and a defendant. The witness takes his oath and testifies. During cross-examination, the defendant's attorney asked the witness this question: "Isn't it true that even though you took an oath to tell the truth so help you God, you are an atheist and don't even believe in God?" Upon the proper objection, will the judge require that the witness answer this question? A Yes, because the question is relevant to the witness's character for truthfulness. B Yes, because instead of taking the oath, the witness could have requested to testify by affirmation without any reference to God. C No, because evidence of the beliefs or opinions of a witness on matters of religion is not admissible to impair credibility. D No, because an attack on the competency of a witness must be made at the time the witness is sworn.
C. The judge should not require that the witness answer the question because evidence of the religious beliefs of a witness is not admissible to challenge credibility. Lack of religious belief is no longer a basis for excluding a witness. Not only are a person's religious convictions irrelevant in determining the competence of a witness, Federal Rule 610 provides that a witness's religious beliefs or opinions are not admissible to show that the witness's credibility is thereby impaired or enhanced. Thus, (C) is correct and (A) is incorrect. (B) is incorrect. While it is true that the witness could have requested a different type of oath, Rule 610 prohibits this type of question because it would have shown his lack of religious beliefs. (D) is incorrect because, as discussed above, lack of religious belief is no longer a basis for disqualification; thus, this would not constitute an attack on the witness's competency.
A beneficiary has filed a petition in the probate court to contest the validity of a testator's will. The beneficiary contends that when the testator executed the will eight years before, he had a severe mental illness and was incapable of forming a valid testamentary intent. In support of this contention, the beneficiary seeks to offer an affidavit prepared by the testator's former attorney, which states that she was asked to prepare a will for the testator just four months before this will was made. The attorney had refused to do so because it was her opinion that the testator seemed incoherent and paranoid. How should the judge rule on the admissibility of this affidavit? A Admissible. B Inadmissible, as being violative of the attorney-client privilege. C Inadmissible, because it is hearsay not within any exception. D Inadmissible, because it is improper opinion evidence.
C. The judge should rule this affidavit to be inadmissible hearsay. This affidavit is clearly hearsay, and there is nothing in the facts that shows that it is admissible under any of the exceptions to this rule. Hence, (A) is wrong. (B) is wrong because the observations of the attorney would not be deemed a "communication received from the client." Also, while the attorney-client privilege generally survives the client's death, it does not apply to communications relevant to an issue between parties who are claiming through the same deceased client, such as in the probate proceedings here. (D) is incorrect because a lay person could probably testify to her opinion in this situation since it is rationally based on her own perception, it is helpful to a determination of a fact in issue, and it is not based on scientific, technical, or other specialized knowledge.
A landlord leased a building to a baker for 10 years, commencing January 1, at a monthly rental of $1,700. The lease stated in part, "The tenant may not sublet or assign this lease without first receiving written permission from the landlord to do so. Any attempt to sublet or assign the lease without first receiving written permission shall constitute a breach entitling the landlord to terminate this lease." Five years later, an investor approached the baker and offered to purchase the bakery if the baker would agree to sublet the premises to him. The baker agreed and executed a sublease on July 1 of that year. The investor took possession the same day. On July 3, the baker approached the landlord and asked for written permission to sublet the premises to the investor. The landlord said he had no real objection to the sublease and would execute the document requested by the baker, but only if the investor would sign a five-year extension of the existing lease. The investor refused to extend the lease, but remained in possession of the building. At no time did the landlord accept rent from the investor. After notice was given to all parties and the applicable grace period in the lease had elapsed, the landlord brought an appropriate action against the baker and the investor to evict them from the premises and to declare the lease terminated because it had been breached. How should the court rule in this action? A Against the landlord, because his withholding consent is an invalid restraint on alienation. B Against the landlord, because his conditional consent operated as a waiver of the term of the lease requiring the landlord to give written permission for subletting. C For the landlord, because the baker has breached the lease. D For the landlord, because his oral consent to sublet is not enforceable under the Statute of Frauds.
C. The landlord should prevail because the baker has breached the lease. Generally, if a tenant transfers (assigns or sublets) in violation of a prohibition in the lease against transfers, the transfer is not void. However, the landlord usually may terminate the lease under either the lease terms or a statute. Here, because the baker has breached the provision of the lease prohibiting assignment or sublease, and the lease contains a forfeiture clause, the landlord was within his rights to terminate the lease. (A) is incorrect because clauses restricting assignment or sublease are not considered to be restraints on alienation. (B) is incorrect because a conditional consent is not a waiver where the condition is not agreed to. (D) is incorrect because an oral consent, if made, is sufficient to waive the provision. Here, the landlord's consent was conditioned on the investor signing an extension of the lease, which he did not do.
The governor of an arid western state owned a vacation home and permitted his son to have a party there. At the end of the night the son failed to properly extinguish a bonfire that he and his friends had built, and within a few hours, wind-blown cinders had spread the fire to the trees east of the lodge. At the same time several miles away, a worker at a lumber mill was making emergency repairs to a pipe running between two mill buildings. He did not notice some of the sparks from his welding torch land in a pile of dried lumber and catch fire, and he failed to check the area after he was finished. By the time the fire was noticed by another employee, it was out of control. The wind blew both fires toward a landowner's hunting lodge. They merged a mile away and shortly thereafter totally consumed the lodge. For political reasons, the landowner did not bring a lawsuit against the governor or his son. He did, however, file a lawsuit against the lumber mill, alleging that its employee's negligence caused the destruction of his lodge. Evidence at trial established that either fire alone would have destroyed the lodge as well. Can the landowner recover from the lumber mill? A No, because the landowner's lodge would have been destroyed regardless of the conduct of the lumber mill's employee. B No, because the damage is indivisible and cannot be apportioned unless the landowner adds the other tortfeasor to the lawsuit. C Yes, because the negligence of the lumber mill's employee was a cause of the landowner's injury. D Yes, but the landowner can recover only 50% of his damages from the lumber mill.
C. The landowner can recover the full amount of his damages from the lumber mill because the negligence of its employee caused the destruction of the lodge. Before a defendant will be liable for a breach of duty to the plaintiff, it must be shown that the breach was the actual and proximate cause of the injury. The general test for determining whether an act or omission is the actual cause of the injury is the "but for" test, i.e., whether the injury would not have occurred but for the act or omission. Under certain circumstances, however, the "but for" test is inadequate to determine actual cause. Where several causes combine to bring about an injury—and any one alone would have been sufficient to cause the injury—the actual cause requirement is satisfied if defendant's conduct was a substantial factor in causing the injury. Under this analysis, the fire started by the lumber mill employee was an actual cause of the destruction of the landowner's lodge because it was a substantial factor in causing the harm. It was also a proximate cause of the harm because no intervening forces broke the causal connection between the act and the harm. Because its employee was acting within the scope of his employment when he caused the fire to start, the lumber mill is vicariously liable for the injury that resulted. (A) is incorrect because the "but for" test is not applicable to these facts. Under that test, neither fire would be the actual cause of the harm because, looking at either fire alone, the harm would have occurred even without it. However, under the substantial factor test, both fires are actual causes of the injury. (B) is incorrect even though it is true that the damage is indivisible. The landowner can still recover from the lumber mill even if he does not sue the other tortfeasor. (D) is incorrect because joint and several liability rules allow the landowner to recover his full damages from the lumber mill. Where two or more tortious acts combine to proximately cause an indivisible injury to plaintiff, each tortfeasor will be jointly and severally liable for that injury, even though each defendant acted entirely independently. The effect of joint and several liability is that the plaintiff may recover the entire amount of the damages from any tortfeasor, who then may have a right of contribution from the other tortfeasor. Hence, even though the negligence of another tortfeasor was also an actual cause of the destruction of the landowner's lodge, the landowner is entitled to recover all of his damages from the lumber mill.
A developer prepared and recorded a subdivision plan, calling for 100 home sites on half-acre lots. There were five different approved plans from which a purchaser could choose the design of the home to be built on his lot. Each deed, which referred to the recorded plan, stated that "no residence shall be erected on any lot that has not been approved by the homeowners' association." A lawyer purchased a lot and built a home based on one of the approved designs. However, many of the lots were purchased by investors who wanted to hold the lots for investment purposes. Two years after the lots went on the market, one such investor sold her lot to an architect by a deed that did not contain any reference to the recorded plan nor the obligation regarding approval by the homeowners' association. In fact, because very few residences had been built in the subdivision since the lots were first available for purchase, no homeowners' association meetings had been held in two years. The architect began building a very modernistic house on her one-half acre. When the lawyer noticed the house being built, he brought an action to enjoin the construction. For which party will the court rule? A The architect, because her deed contained no restrictive covenants. B The architect, because any restrictive covenant in her deed can only be enforced by the opposite party to the covenant or that person's successor in title. C The lawyer, because the recorded subdivision plan, taken with the fact that all lots were similarly restricted and the architect had notice of this, gave him the right to enforce the covenant on her property. D The lawyer, because his deed contained the restrictive covenant.
C. The lawyer will likely prevail. When a subdivision is created with similar covenants in all deeds, there is a mutual right of endorsement (each lot owner can enforce against every other lot owner) if two things are satisfied: (i) a common scheme for development existed at the time that sales of parcels in the subdivision began; and (ii) there was notice of the existence of the covenant to the party sued. Here, there was a common scheme evidenced by the recorded plan, and the fact that the covenant was in the architect's chain of title gave her constructive notice of the restriction. Therefore, not only does the covenant apply to the architect's land, but the lawyer (or any other lot owner) can enforce it as a reciprocal negative servitude. (A) is incorrect. While it is true that the architect's deed had no restrictions, those restrictions are binding if they are in her chain of title so as to give her notice of them. The restriction was in the deed from the developer to the investor, so the fact that it was omitted in the deed from the investor to the architect is of no significance. (B) is incorrect. While a covenant is normally only enforceable by the party receiving the promise (here, the developer), this is a situation of mutual rights of enforcement within a geographically defined area, a special situation that gives every lot owner in the area the right of enforcement, even though they did not directly receive the benefit of the promise. (D) is incorrect. The fact that gives the lawyer the right of enforcement is not just the fact that his deed contains the covenant, but that the same covenant was in all of the deeds from the developer, including the one to the architect's predecessor in title.
Acting with probable cause, the police arrested a man in connection with the armed robbery of a liquor store. After being given Miranda warnings, the man confessed to the robbery but denied his involvement with several other recent armed robberies of businesses in the area. He was formally charged with the one robbery and put into a cell with a paid informant working undercover for the police. The informant had been instructed to find out what he could about the other robberies but not to ask any questions. The informant began talking about a convenience store robbery in which a bystander was shot and seriously injured by the robber, and he deliberately misstated how it happened. The man, unaware that his cell mate was an informant, interrupted to correct him, bragging that he knew what really happened because he was there, and proceeded to make incriminating statements about the robbery. The man was subsequently charged with armed robbery and attempted murder in the convenience store robbery. At a motion-to-suppress hearing on that charge, if the man's attorney moves to exclude the statements made to the informant, should the motion be granted? A Yes, because the informant deliberately elicited incriminating statements in violation of the man's Sixth Amendment right to counsel. B Yes, because the informant's conduct constituted custodial interrogation in violation of the man's Fifth Amendment privilege against self-incrimination. C No, because the man had not yet been charged with the robbery of the convenience store when he made the statements to the informant. D No, because the informant's conduct did not constitute interrogation.
C. The man's motion should be denied because neither his Fifth nor Sixth Amendment rights were violated by the informant's conduct. The Sixth Amendment right to counsel applies to all critical stages of a criminal prosecution after formal proceedings have begun, but does not apply in precharge custodial interrogations. Because this right is "offense specific," the fact that the right to counsel has attached for one charge does not bar questioning without counsel for an unrelated charge. Because the man has not been charged with the convenience store robbery, his Sixth Amendment right to counsel has not been violated. The Fifth Amendment privilege against self-incrimination requires Miranda warnings and a valid waiver before any statement made by the accused during custodial interrogation can be admitted. However, this requirement does not apply where interrogation is by an informant who the defendant does not know is working for the police, because the coercive atmosphere of police-dominated interrogation is not present. [Illinois v. Perkins (1990)] Because the man was not aware of the informant's status, the informant's conduct did not constitute a police interrogation. (A) is wrong despite the fact that the informant's conduct may have been deliberately designed to elicit incriminating remarks. As discussed above, the man's right to counsel did not attach for purposes of the convenience store robbery. (B) is incorrect because, as discussed above, the Miranda warnings need not be given before questioning by a cellmate working covertly for the police. (D) is incorrect because interrogation refers not only to express questioning, but also to any words or actions on the part of the police that the police should know are reasonably likely to elicit an incriminating response from the suspect. Here, the informant, working for the police, made statements about the convenience store robbery that were intended to, and reasonably likely to, prompt a response from his cellmate. Hence, it is not the absence of "interrogation" that avoids the Miranda problem, but the fact that the man did not know that his cellmate was working for the police.
A state law prohibits physicians from practicing medicine within the state without a state license. Among other things, the grant of a state license requires a physician to have been a resident of the state for at least one year. A physician moved to the state from a nearby state and immediately applied for a license to practice medicine. Although otherwise qualified, the physician's request for a license was denied based on the residency requirement. The physician brought suit, alleging that the residency requirement violated the United States Constitution. Will the physician likely succeed? A Yes, because the requirement violates the Privileges and Immunities Clause of Article IV. B Yes, because the requirement violates the Privileges or Immunities Clause of the Fourteenth Amendment. C Yes, because the requirement violates the Due Process Clause of the Fourteenth Amendment. D No, because the state has a compelling interest in furthering the welfare of its residents.
C. The physician will succeed. The Privileges or Immunities Clause of the Fourteenth Amendment prohibits states from denying their citizens the privileges and immunities of national citizenship. This includes the right to travel, and the Court has held that the right to travel includes the right of newly arrived citizens to enjoy the same privileges and immunities as are enjoyed by other citizens of the state. A state law that distinguishes between new residents solely on the length of their residency will serve no legitimate state interest. Thus, a law limiting medical licenses to persons who have resided in the state for a year runs afoul of the clause. (A) is incorrect. The Privileges and Immunities Clause of Article IV prohibits discrimination by a state against nonresidents. Here, the physician is a resident of the state discriminating against him. Hence, the Article IV privileges and immunities protection does not apply. (C) is not as good an answer as (B) because the law here involves discrimination based on duration of residency—making the Privileges or Immunities Clause more direct. (D) is incorrect because duration of residency does not impact the welfare of residents.
Traditionally, which of the following is a defense to solicitation? A The solicitation could not have been successful due to actual circumstances. B There was no overt act committed in furtherance of the solicitation. C The solicitor is exempt from liability for the completed crime. D The solicitor withdrew the solicitation.
C. The solicitor would have a defense if she could not be guilty of the intended crime because of a legislative intent to exempt her. It is not a defense that the solicitation could not have been successful due to actual circumstances surrounding the crime. (This is factual impossibility.) The culpability of the solicitor is measured by the circumstances as she believed them to be. It is generally not a defense that the solicitation has been withdrawn once it has been made. However, note that the M.P.C. recognizes renunciation as a defense if the defendant prevents the commission of the crime, such as by persuading the person solicited not to commit the crime. For solicitation, no overt act in furtherance of the solicitation is required. (An overt act is usually required for conspiracy.)
A state statute provided that only residents of the state can be granted a license to practice medicine within the state. The statute was passed after a series of well-publicized mistakes by a nonresident physician led to a public consensus that nonresidents were less likely to be familiar with the medical standards followed in the state, making them more likely to commit malpractice. A respected surgeon who lived and was licensed in a neighboring state was offered and accepted the position of chief surgeon at a hospital in the state with the residency statute. Because he lived only 20 minutes away from the hospital, he did not wish to move. He filed an action in federal court challenging the residency requirement, alleging that the statute discriminated against nonresidents in violation of the Privileges and Immunities Clause of Article IV. Is the federal court likely to find that the statute is constitutional? A Yes, because the state has a substantial justification for the discriminatory treatment. B Yes, because a license to practice medicine is not a "privilege" under the Privileges and Immunities Clause. C No, because the statute is not necessary to achieve an important government purpose. D No, because the statute is not necessary to achieve a compelling state interest.
C. The statute is likely unconstitutional because the statute is not necessary to achieve an important government purpose (put another way, the state does not have a substantial justification for the statute). The Privileges and Immunities Clause of Article IV protects against discrimination by a state in favor of its own citizens when it affects a fundamental right, such as the pursuit of a livelihood. Any statute that results in such discrimination violates the Clause unless the state shows that it has a substantial justification for the discriminatory treatment. In effect, it must show that nonresidents either cause or are part of the problem it is attempting to solve, and that there are no less restrictive means to solve the problem. Here, the state statute clearly discriminates against nonresidents in favor of residents. The reason offered by the state to justify the discrimination, i.e., that nonresident physicians are less likely to be familiar with the medical standards imposed by the state, does not meet the test of necessary to achieve an important government purpose. The state can find less restrictive means to ensure that all physicians are familiar with its medical standards. Accordingly, (A) is incorrect. (B) is incorrect, because a "privilege" under the Privileges and Immunities Clause includes those professions that are important to the national economy, which the practice of medicine clearly is. (D) is incorrect because it states the wrong standard. The state does not have to show a compelling interest for the discrimination. It must show an "important" one, a lower standard. QUESTION ID: ML065 QUESTION ID: ML065 Skip to Assessment Navigation
After a state supreme court overturned the conviction in a murder case for failure to give proper Miranda warnings, a reporter asked the murder victim's father to comment on the case as he exited the supreme court building. The father made the following statement: "Each one of the so-called supreme court justices is worse than a murderer, because they make it possible for more sons and daughters to be murdered. I'd like to see every one of them strung up, like they should have done to the creep who was set free, and if someone will give me a rope I'll go in there and do it myself." A state statute proscribes, with criminal penalties, "the making of any threat to the life or safety of a public official for any act the official performed as part of the official's duties in office." Which of the following is correct regarding the statute? A The victim's father could constitutionally be punished under the statute, but only if the state supreme court justices heard the threats he made. B The victim's father could constitutionally be punished under the statute. C The victim's father could not be constitutionally punished under these circumstances, but the statute is constitutional on its face. D The statute is unconstitutional on its face.
C. The statute is not unconstitutional. True threats are not protected by the First Amendment. Moreover, content-based restrictions on speech are permitted in cases where the speech creates a clear and present danger of imminent lawless action. A state can forbid advocating the use of force or of law violation if such advocacy (i) is directed to producing or inciting imminent lawless action, and (ii) is likely to produce or incite such action. Thus, a statute proscribing threats to the life or safety of a public official, such as the statute here, is valid. However, it cannot constitutionally be applied to the victim's father. It is doubtful that the father's words will be interpreted as a true threat of immediate harm. In context, the speech seems to be more a political commentary, which would be protected by the First Amendment. The father appeared to be merely venting his outrage. There was no indication that the father's words were inciting imminent lawless action or were likely to produce such action. It does not appear that the father was actually threatening the justices with harm or inciting anyone to storm into the court building. Thus, his speech was protected and (B) is incorrect. (A) is incorrect because the father cannot constitutionally be punished for the reasons stated above. Moreover, whether the justices actually heard the threats would be irrelevant if the threats were otherwise punishable. (D) is incorrect because the statute is valid on its face, as discussed above.
A student and a few of his friends were making their way to spring break. Along the way, the old van that they were driving broke down. Not wanting to miss any part of spring break festivities, the student asked the mechanic on duty at the repair shop for a rush job. The mechanic provided the student with a repair estimate, and the student, on the basis of the estimate, authorized the repair and promised to pay when he came back to pick up the van. When the mechanic called the student to tell him that the van was repaired, the student, rather than paying for the repair, told one of his friends that the mechanic had agreed to finance the repair charges and that the only thing left to do was pick up the van in the garage's parking lot. The student handed the friend a key to the van and told him to go pick the van up so that they could continue their trip to spring break. The friend did so. The mechanic makes a criminal complaint against the student for larceny of the van. If the case is prosecuted, will the student likely be found guilty? A No, because it was the student's van to begin with. B No, because the friend took the van. C Yes, because the friend took the van from the mechanic without the mechanic's knowledge or permission. D Yes, because the student promised to pay the mechanic for his work when he came to get the van.
C. The student will most likely be found guilty. Larceny is the taking and carrying away of the personal property of "another" with the intent to permanently deprive the other person of the property. It is possible to commit larceny of your own property if another person, such as a bailee, has a superior right to possession of the property at that time. Because the mechanic had a right to possession of the van until he was paid, the student committed larceny when he had his friend take the van without the mechanic's consent. (B) is wrong because a person can be guilty even though he did not personally engage in the behavior if he acts through an innocent agent. (D) is also incorrect. The student is guilty, but not for the reason stated in (D). He would be guilty even if he had not made the promise to pay for the van; he incurred an obligation to pay by having the repairs don
A defendant is on trial for robbing a liquor store. The store clerk testified that the defendant came into the store at about 11 p.m., pointed a black gun with a silver grip at him, and demanded that he give him all the money in the cash register. The clerk testified that the store was well lit and that the defendant was not wearing a mask. The defendant's attorney called the clerk's employer to testify that when the clerk gave her a report of the robbery, he told her that the defendant pointed a silver gun with a black grip at him. How should the trial judge rule on the admissibility of this testimony? A Admissible, because it tends to show that the clerk is an unreliable witness. B Admissible, because it has bearing on the clerk's truthfulness and veracity. C Inadmissible, because it is extrinsic evidence of a prior inconsistent statement on a collateral matter. D Inadmissible, because it is hearsay not subject to any exceptions.
C. The testimony should be found inadmissible. Extrinsic evidence of a prior inconsistent statement may not be used to impeach a witness upon a collateral matter. The clerk testified that he could recognize the defendant's face, and so the color of the gun is not material to any issue in the case under the facts given. (A) and (B) are incorrect because, even if the evidence does have some bearing on the clerk's credibility as a witness, it will be excluded because it will possibly confuse the issues or because it is a waste of time. (D) is not correct because the defendant is not seeking evidence to prove the truth of any material issue, but to impeach the clerk. The hearsay rule does not apply
A state provided for a public school system based primarily on property tax revenues from the various districts. School districts that had a property tax base below a certain threshold received supplemental funds from the state that were derived from state lottery revenues. The school districts receiving the supplemental funds served a predominantly Hispanic population as compared to the school districts funded only from property tax revenues. To help balance its budget this year, the state legislature passed a statute terminating the supplemental funds program and earmarking the lottery revenues for deficit reduction. A group of parents of Hispanic schoolchildren in one of the school districts formerly receiving supplemental funds filed suit in federal court, alleging that the state's action in terminating the funding violates the Equal Protection Clause of the Fourteenth Amendment. Which of the following best describes the appropriate standard by which the court should review the constitutionality of the state action? A Because the state statute results in discrimination against a suspect class, the state will have to demonstrate that the statute is necessary to vindicate a compelling state interest. B Because the right to education burdened by the statute is not a fundamental right, the parents will have to demonstrate that the statute is not substantially related to an important state interest. C Because no suspect class or fundamental right is improperly burdened in this case, the parents will have to demonstrate that the statute is not rationally related to any legitimate state interest. D Because the state statute is not discriminatory in intent, the state will have to demonstrate only that the statute is rationally related to a legitimate state interest.
C. To prevail, the parents will have to show that the statute does not meet the rational basis test. Under that test, a law is presumed to be valid and will be upheld unless the challenger can make the difficult showing that it is not rationally related to a legitimate state interest. Here, the statute terminating the funds did not target a suspect class and did not burden a fundamental right, so the rational basis test applies. (A) is incorrect because it is not enough to show that legislation has a discriminatory effect on a suspect class; there must be an intent to discriminate. To establish a racial, national origin, or ethnicity classification, the party challenging the law must show that (i) the racial classification appears in the law itself (facial discrimination), (ii) the law was applied in a purposefully discriminatory manner, or (iii) the law was enacted or maintained for a discriminatory purpose. None of these situations appears to be indicated under these facts. (B) is incorrect because it states the wrong standard. As that choice indicates, the Supreme Court has not held education to be a fundamental right under the Due Process Clause, nor has it found classifications based on wealth to require strict scrutiny. Hence, the test that is applied is the rational basis standard; the standard in (B) is an intermediate scrutiny test applied to gender and legitimacy classifications. (D) is incorrect because it imposes the burden of proof on the wrong party. For a statute that does not discriminate against a suspect class, the plaintiff bears the burden of proving that the statute is not rationally related to a legitimate state interest.
Which of the following is not required for the burden of an equitable servitude to run to successors in interest? A The covenant touches and concerns the land. B The successor in interest has notice of the covenant if she has given value. C There is vertical privity between the covenantor and his successor in interest. D The covenanting parties intended that successors in interest be bound by the covenant.
C. Vertical privity between the covenantor and his successor in interest is not required for the burden of an equitable servitude to run to successors in interest. An equitable servitude is a covenant (i.e., a promise to do or not to do something on the land) that, regardless of whether it runs with the land at law, can be enforced in equity against assignees of the burdened land who have notice of the covenant. The burden of an equitable servitude will run to successors in interest if: 1. The covenanting parties intended that successors in interest be bound by the covenant; 2. The successor in interest has notice of the covenant (if she has given value); and 3. The covenant touches and concerns the land (i.e., it benefits the covenantor and his successor in their use and enjoyment of the burdened land). Horizontal privity between the original covenanting parties and vertical privity between the covenantor and his successor in interest are not required.
Which of the following is available as a defense to specific intent crimes but not general intent crimes? A Mistake of fact B Insanity C Voluntary intoxication D Involuntary intoxication
C. Voluntary intoxication is available as a defense to a specific intent crime only. The defense may be used to show that a defendant was unable to form the requisite intent due to intoxication. Involuntary intoxication and insanity are available as defenses to both specific intent and general intent crimes. Mistake of fact is available as a defense to both specific intent and general intent crimes. However, note that a mistake of fact must be reasonable to be a defense to a general intent crime. An unreasonable mistake of fact is a defense to a specific intent crime only.
Which of the following generally occurs when a mortgagee transfers a promissory note without a written assignment of the mortgage? A The mortgagee retains the rights to the mortgage. B The mortgage is separated from the obligation on the note. C The mortgage follows the note. D The mortgage is extinguished.
C. When a mortgagee transfers a promissory note without a written assignment of the mortgage, generally the mortgage follows the note. A mortgage is a security interest in real estate that secures an obligation, usually a promise to repay a loan, which is represented by a promissory note. The debtor (i.e., the mortgagor) gives the mortgage and the note to the lender (i.e., the mortgagee). The mortgagee who transfers her interest usually does so by indorsing the note and executing a separate assignment of the mortgage. While it is possible to transfer the note without the mortgage, the mortgage automatically will follow the properly transferred note. No separate written assignment of the mortgage is necessary. The mortgagee does NOT retain the rights to the mortgage when she transfers the note without a written assignment of the mortgage unless she expressly reserves the rights, which there would rarely be any reason for her to do. Generally, the mortgage follows the note; the mortgage is NOT separated from the obligation on the note, and the mortgage is NOT extinguished.
When may the spousal immunity doctrine be invoked? A In criminal or civil cases, as long as the matter concerns a confidential communication between the spouses B In criminal or civil cases, for matters that took place before or during the marriage C In criminal cases only, for matters that took place before or during the marriage D In criminal cases only, for matters that took place during the marriage only
C. When the privilege of spousal immunity is invoked, a married person whose spouse is the defendant in a criminal case may not be called as a witness by the prosecution, and a married person may not be compelled to testify against his spouse in any criminal proceeding. Spousal immunity may be invoked in criminal cases only. The privilege lasts only during the marriage and terminates upon divorce or annulment. However, if a marriage exists, the privilege can be asserted even as to matters that took place before the marriage. Spousal immunity may not be invoked in civil cases. Remember that there are two separate privileges related to marriage: spousal immunity and the marital communications privilege. This question tests your knowledge of spousal immunity
May a buyer obtain specific performance of an oral land sale contract? A Yes, provided the buyer has paid most of the purchase price B No, because an oral contract does not satisfy the Statute of Frauds C Yes, if the buyer has taken possession of and made substantial improvements to the land D No, because the buyer's remedy is damages
C. Yes, a buyer may obtain specific performance of an oral land sale contract if the buyer has taken possession of and made substantial improvements to the land. While land sale contracts must be memorialized in writing and signed by the party to be charged to be enforceable under the Statute of Frauds, courts in most states will enforce an oral contract in equity under the doctrine of part performance if the buyer has performed at least two of the following acts: 1. Taken possession of the land; 2. Made substantial improvements to the land; and/or 3. Paid all or part of the purchase price. Some courts will accept as part performance additional acts showing the buyer's detrimental reliance. A buyer might not obtain specific performance of an oral land sale contract even if the buyer has paid most of the purchase price. As explained above, most jurisdictions require at least two acts of part performance. For an oral land sale contract, the buyer's remedy is NOT damages. Only specific performance is available in equity under the doctrine of part performance. Although an oral contract does not satisfy the Statute of Frauds, a court may award specific performance if the buyer shows sufficient acts of part performance. Two theories support this remedy: (i) the buyer's acts unequivocally evidence an oral contract; and (ii) the buyer's detrimental reliance estops the seller from asserting the Statute of Frauds as a defense.
A property owner owned a tract of land that he leased to a baker for 27 years. The baker built a large bakery on the property. The baker then sold the bakery building to a buyer, assigning the lease with the property owner's approval. The buyer has failed to make a rent payment for several months and has also failed to build the cafe that the baker had agreed to build in the original lease. The landlord of the property has a cause of action against: A The buyer for the rent only, because the rent covenant runs with the land. B The buyer for the rent and the cafe, but only if the buyer expressly assumed performance of all covenants. C The buyer and the baker for both the rent and the cafe. D The buyer for the rent only, and the baker for the cafe only.
C. An assignment does not release the tenant from his contractual obligations to the landlord; thus, the baker is still liable for all of the lease provisions. Thus, (D) is incorrect. An assignee is in privity of estate with the landlord and is liable on all covenants in the lease that run with the land. His assumption of these duties is implied; it need not be expressed in the assignment. Covenants to pay money run with the land, as do covenants to perform physical acts on the property. Therefore, the buyer is liable for both the rent and the cafe even if it did not expressly assume performance of the covenants. Thus, (A) and (B) are incorrect.
During the defendant's trial for armed robbery, evidence was introduced establishing that a rifle was found in the trunk of the defendant's car when he was arrested. On direct examination, the defendant testified that when he was arrested and the rifle was found, he stated, "I keep that with me for protection." Should the court allow the testimony? A Yes, because it is a statement of a party. B Yes, because it is an excited utterance. C No, because it is hearsay not within an exception. D No, because it is a self-serving statement.
C. The court should not allow the testimony. The defendant's testimony is being offered to prove the truth of the matter asserted (i.e., that he keeps the rifle for protection) and is hearsay. Hearsay is a statement, other than one made by the declarant while testifying at the trial or hearing, offered in evidence to prove the truth of the matter asserted. A hearsay problem can arise even if the out-of-court declarant and the in-court witness are the same person, as is the case here. Because the statement is not within any exception to the hearsay rule, it must be excluded. (A) is incorrect because a party's statement is considered nonhearsay when offered by a party-opponent. This hearsay exclusion does not apply here because the defendant's testimony was elicited on direct examination—i.e., the defendant was introducing his own statement into evidence. (B) is incorrect because an excited utterance applies only if made in response to a startling event. Although the conditions of the search might have been stressful to the defendant, it is not the type of situation that gives rise to an excited utterance. (D) is incorrect. The fact that a statement is self-serving is not grounds for its exclusion. All of a criminal defendant's evidence can be considered self-serving in the sense that it furthers his claim of innocence.
The police received information linking a man to drug trafficking and went to the man's residence, where he lived with his mother. The police found the mother at home, and she told them that her son was not expected back until later. The police informed the mother that they suspected the man of selling drugs and asked if they could search his room. She replied, "I'm finished with that no-good bum; not only is he into drugs, but he has been stealing my money to pay for them, and all the time I'm making his bed and fixing his food. You can search his room. He likes to keep his private stuff under his pillow. I hope he goes to jail." The police searched the man's room and discovered a quantity of marijuana under the pillow of his bed. If before trial the man's attorney moves to suppress the marijuana on grounds that the search was invalid, should the court grant the motion? A Yes, because the man had a legitimate expectation of privacy in the area searched, and the police did not have a warrant. B Yes, because the man's mother's consent was given at a time when police knew her interests were in conflict with the man's. C No, because the man's mother had the authority to consent to the search of his room. D No, because with the mother's statement the police had probable cause to search the room.
C. The man's motion to suppress should be denied because his mother had authority to consent to the search of his room. A search of a residence can be based on the voluntary consent of the occupant. Where a parent has general access to a room occupied by a son or daughter, the parent can give a valid consent to a general search of the room even if the son or daughter is an adult. The facts in the question indicate that the man's mother had general access to his room ("and all the time I'm making his bed"). Therefore, her consent is valid and eliminates the need for probable cause and a warrant. (A) is wrong. The man had a legitimate expectation of privacy in the area searched, but the consent of his mother eliminated the need for a warrant. (B) is wrong. At one time, some courts required an "amicable relationship" between the parties before the police could rely on a third party's consent. The "amicable relationship requirement" is no longer recognized by the courts. (D) is not a good answer. It is true that with the mother's statement the police had probable cause to search the man's room. However, probable cause alone would not validate the search. The police would need probable cause plus a warrant or a valid consent. In this question the search would have to be based on consen
A state's constitution authorizes a state reapportionment board to redraw state legislative districts every 12 years. During the most recent reapportionment process, consultants had provided the board with two alternative plans for reapportionment. One plan provided for districts with less than a 3% difference in proportional representation between districts. The other plan was drawn up to conform state legislative districts as nearly as possible to county borders, resulting in differences in proportional representation between districts of up to 12%. The current apportionment of legislative districts results in differences of up to 15% between districts. The board ultimately selected the reapportionment plan based on county borders, and this plan was approved by the state legislature. A Caucasian resident and registered voter of the state brought a constitutional challenge to the reapportionment in federal court. His claim is based on the fact that, as a result of the plan that the board selected, the percentage of the African-American voting population in the district in which he lives increased from 45% to 55%. Had the other plan been selected, the percentage would have been unchanged in his district. In the absence of a federal statute applicable to the state, is the resident likely to prevail? A Yes, because an alternative plan with more equal apportionment is available. B Yes, because any legislative apportionment discriminating in favor of or against racial minority groups is subject to strict scrutiny, and there is no evidence of past discrimination or any other compelling state interest to justify adopting the plan. C No, because preserving political subdivisions is a legitimate state interest that justifies the plan's variance in representation. D No, because the reapportionment plan results in less of an overall variance between districts than the current legislative apportionment.
C. The resident will not prevail because the reapportionment plan does not violate the Equal Protection Clause of the Fourteenth Amendment. That provision has been interpreted to prohibit state dilution of the right to vote, so that whenever a governmental body establishes voting districts for the election of representatives, the number of persons in each district may not vary significantly. However, for the purpose of electing representatives to a state or local governmental body, the variance in the number of persons included in each district can be greater than that permitted for congressional districts. If the deviation from mathematical equality between districts is reasonable and tailored to promote a legitimate state interest, the law establishing the districts will likely be upheld. The Court has held that maintaining the integrity of local political subdivision lines when establishing legislative districts is a legitimate state interest, as long as the final apportionment is substantially based on population. [See Mahan v. Howell (1973)—16% variance upheld] Here, the reapportionment attempted to conform legislative districts as nearly as possible to county borders and had a maximum variance of 12%. Thus, it will probably withstand the resident's challenge. (A) is incorrect because the fact that an alternative plan has a lesser variance between the districts does not make the selected plan invalid. Because it satisfies the less stringent requirements for state and local governmental bodies discussed above, the plan does not violate the Equal Protection Clause. (B) is incorrect because race can be considered in drawing up new voting districts, even though it cannot be the predominant factor. If a plaintiff can show that a redistricting plan was drawn up predominantly on the basis of racial considerations (as opposed to the more traditional factors, such as compactness, contiguity, and community interest), the plan will violate the Equal Protection Clause unless the government can show that the plan is narrowly tailored to serve a compelling government interest (such as eliminating past discrimination). However, if a legislative redistricting map can be explained in terms other than race, the Court will not find that the law constitutes racial discrimination on its face. In such a case, the person attacking legislative districts as being based on racial classifications would have to show that district lines were drawn predominantly for a racially discriminatory purpose. Here, as discussed above, the state's interest in preserving political subdivisions (counties) is a legitimate government interest, and the resident will be unable to prove that this was not the predominant factor in the reapportionment. (D) is incorrect because the fact that the reapportionment plan reduces the existing population variance among districts does not make it constitutionally valid. The plan must satisfy the equal protection requirements established by the Court in apportionment cases.
A tenant vacated an apartment because he could no longer afford the rent. To ensure that the delinquent tenant made up for past arrearages, the landlord would not let him remove his personal property from the apartment. The tenant found a temporary place to stay with a friend, who wanted to help the tenant get his property back. The tenant remembered that the apartment would be vacant the upcoming weekend and that the landlord would be out of town, so he suggested that they break into the apartment and take the property then. They drove the tenant's pickup to the apartment, and the friend entered through an unlocked window. The friend then opened the door for the tenant, and the pair collected the personal property. While the tenant was getting ready to drive away, the friend returned to the apartment and carried out some of the fixtures to the apartment. At this point, police officers who had been alerted by neighbors arrived and arrested the pair. What is the tenant's best defense to a charge of burglary? A There was no "entry," because as an occupant of the apartment, he consented to the entry. B There was no breaking, because the window was unlocked. C There was no intent to commit a felony. D He only took his own property.
C. Absence of intent to commit a felony is the best defense. If the tenant intended merely to retrieve his property, he would have had no intent to commit a felony when he entered the apartment and thus could not be convicted of burglary. Common law burglary consists of: (i) a breaking; (ii) and entry; (iii) of the dwelling; (iv) of another; (v) at nighttime; (vi) with the intent of committing a felony therein. The tenant entered the apartment intending to retrieve his own property. Thus, the facts indicate that the only felony the tenant could have intended to commit at the time of entry would be larceny. Larceny consists of: (i) a taking; (ii) and carrying away; (iii) of tangible personal property; (iv) of another; (v) by trespass; (vi) with intent to permanently (or for an unreasonable time) deprive the person of his interest in the property. Larceny element (iv) would be missing here if the tenant intended to retrieve only his own property. Without the intent to commit a felony, no burglary exists. (A) is wrong because the tenant was no longer an occupant of the apartment and so could not consent to the entry. (B) is wrong because opening the closed but unlocked window was a breaking. (D) is not his best defense. The tenant could argue that the landlord did not have superior rights to the tenant's property (despite a possible claim of arrearages) and that therefore he did not take the property of another, and also that he did not take part in his friend's theft. However, he still would be guilty if he entered the apartment with the intent to commit a felony inside; the stronger defense of choice (C) specifically negates that intent.
The defendant was on trial for murdering his mother, who was found dead in her bathtub. At trial, the prosecutor called the nurse of the defendant's aunt to testify to what the aunt told the nurse just before the aunt died of cancer. The nurse is prepared to testify that, shortly before she died, the aunt stated, "I know I don't have much longer to live, so I must tell someone what my nephew said to me yesterday. He told me that he was very angry with his mother and that he wanted to kill her and make it look like an accident!" Should this testimony be admitted? A Yes, because it is a statement by an opposing party. B Yes, because it falls within the hearsay exception for dying declarations. C Yes, because it is a statement by an opposing party and falls within the hearsay exception for dying declarations. D No, because it is inadmissible hearsay.
D Dying declaration must include circumstances of death The testimony is inadmissible. Hearsay is an out-of-court statement offered in evidence to prove the truth of the matter asserted. An out-of-court statement that incorporates other hearsay is "hearsay within hearsay." Hearsay within hearsay is admissible only if both the outer hearsay statement and the inner hearsay statement fall within an exception to the hearsay rule. Here, the aunt's statement incorporating the defendant's statement constitutes hearsay within hearsay. Therefore, both statements must fall within an exception to the hearsay rule to be admissible. The defendant's statement to his aunt ordinarily would be hearsay because it is being offered to prove the truth of the matter asserted—that he intended to kill his mother. However, statements by a party-opponent (commonly called admissions) are admissible nonhearsay under the Federal Rules. Thus, the defendant's statement is admissible as a statement by a party-opponent. However, the aunt's statement made to the nurse regarding the defendant's admission must also fall within an exception to the hearsay rule. Because no exception applies to that statement, the entire testimony is inadmissible. (A) is incorrect because, while it correctly states that the defendant's statement is a statement by an opposing party, it incorrectly deems the statement admissible because it fails to address the hearsay within hearsay issue discussed above. (B) is incorrect. A declaration made by the now unavailable declarant, while believing that her death was imminent, is admissible if it concerns the cause or circumstances of what she believes to be her impending death. Here, the statement was made while the aunt believed that her death was imminent; however, the statement did not concern the cause or circumstances of her death and therefore does not qualify as a dying declaration. (C) is incorrect for the same reason. While the defendant's statement is an admission, the aunt's statement is not a dying declaration, as discussed above.
A company manufactured and sold a product called "True Glue." An inventor brought an action in federal court against the company, alleging that the product infringed a patent owned by him. The company denied the allegations of the inventor's complaint. In addition, it asserted a counterclaim against the inventor for breach of contract, based on a transaction between them unrelated to the inventor's claim. After the completion of discovery, the company moved for summary judgment dismissing the inventor's claim. The court granted the motion, thus leaving only the company's counterclaim for breach of contract to be adjudicated. Which of the following statements most accurately describes the inventor's right to appeal the court's grant of the motion for summary judgment? A The inventor may immediately appeal the judgment because summary judgment is a final judgment. B The inventor may immediately appeal the judgment because his claim does not arise out of the same transaction or occurrence as the company's claim. C The inventor may not appeal the judgment until the company's claim against him has been adjudicated. D The inventor may not immediately appeal the judgment unless the court provides that it is a final judgment and expressly determines that there is no just reason for delay.
D The inventor may not immediately appeal the judgment unless the court provides that it is a final judgment and expressly determines that there is no just reason for delay. When a court grants summary judgment on some but not all of the claims in an action, the court's order is not final and thus not appealable unless the court expressly determines that there is no just reason to delay entry of judgment. Unless the trial judge makes that express determination, its order determining the merits of fewer than all of the claims is not a final judgment and is not appealable.
As a general rule, a notice of appeal must be filed with the district court within ____ days from the entry of judgment. A 60 B 30 C 90 D 28
D- 28 days
A landowner and her neighbor owned adjacent parcels of land. The landowner hired a contractor to install an in-ground swimming pool on her land. The day after the contractor had excavated for the pool, the neighbor's storage shed, located on his property a few feet from the edge of the excavation, collapsed when the ground shifted. A riding tractor and patio furniture contained within the shed were damaged. The neighbor sued the landowner for damages. At trial, the neighbor established that the landowner's project caused the subsidence and the damage to his property. What else must the neighbor establish to prevail? (A) No additional facts. (B) That his land would have been damaged without the storage shed. (C) That the contractor was negligent. (D) That his land would have been damaged without the storage shed or that the contractor was negligent.
D) For the neighbor to prevail, he must show either that his land would have been damaged without the storage shed or that the contractor was negligent. A landowner has a right to have his land supported in its natural state by adjoining land. If, however, the land has buildings on it, an excavating adjacent landowner is strictly liable for damage to the buildings caused by the excavation only if the excavation would have caused the land to subside even in its natural state (i.e., without buildings). Even if the land would not have subsided in its natural state, the excavating landowner is liable for the damages if she was negligent. Here, the neighbor has established that the landowner's excavation caused the damage to the neighbor's storage shed. To prevail, the neighbor will need to show either that the subsidence would have occurred even if the land had been unimproved or that the contractor hired to excavate for the pool was negligent. (A) is therefore incorrect. (B) is incorrect because the neighbor need not show that the land would have subsided without the weight of the storage shed if he can show that the contractor was negligent. Conversely, (C) is incorrect because the neighbor need not show that the contractor was negligent if he can show that the land would have subsided without the storage shed
Twenty years ago, a nephew received a deed to a farm as a gift from his aunt. The aunt's deed was properly executed and delivered, and the nephew recorded. The nephew had no interest in farming, so he left the property alone for 10 years. At that point, he transferred all his rights in the farm by quitclaim deed to a neighbor who owned the adjacent land. The neighbor had a full-time job and did not farm the land but lived in the farmhouse for five years. He then took a job in a distant city for five years. When he returned, he learned that the aunt who made the initial transfer to her nephew had not in fact been the fee simple owner of the farm. The true owner of the property had just died and her daughter and sole heir demanded possession from the neighbor. The statutory period for adverse possession in the jurisdiction is seven years. In a quiet title action, how should the court rule? (A) The neighbor takes clear title to the farmhouse and the entire farm by virtue of his adverse possession for over 10 years. (B) The neighbor takes clear title to the farmhouse but not to the rest of the farm that was not actually possessed by him, which goes to the daughter. (C) The nephew takes clear title because he had record title to the farm for over 10 years, more than the seven-year statutory period for adverse possession. (D) The daughter is the fee simple owner of the farm.
D) The daughter owns the farm. Because the aunt was not the legal owner at the time she transferred to her nephew, the nephew and any claimant through the nephew must rely on adverse possession to defeat the daughter's claim. The nephew did not get title by adverse possession, despite the fact that he recorded his deed, because he never possessed the property, and adverse possession requires actual possession. The neighbor did have possession of the house, although not the rest of the farm, but his possession was interrupted by his five-year job in a distant city. He did not continuously possess any part of the farm for seven years. Because all adverse possession claims fail, the daughter must own the property in fee simple. (A) and (B) are incorrect. As indicated above, the neighbor did not continuously adversely possess any part of the farm for the necessary seven years. Thus, he has no claim to any of the farm, even the farmhouse he occupied. (C) is incorrect. The nephew never took possession of the farm for that 10-year period. The mere fact that he claimed it and had recorded a deed to it during that time will not serve as a basis for adverse possession.
Question 24 A 12-year-old child was injured in an automobile accident. The child's father brought the child to see an attorney to bring suit against the defendant. During the paid consultation with the attorney, the seriousness of the child's injuries was discussed with candor. After the discussion, the attorney told the father and child that they would be better off with a lawyer who specialized in personal injury work. Eventually, another attorney was hired to bring the child's lawsuit against the defendant. Defense counsel has reason to believe that the child's injuries are not serious at all. She therefore subpoenas the first attorney for an oral deposition. During the course of the deposition she asks the attorney about his discussion with the child regarding the child's injuries. May the attorney invoke the attorney-client privilege? (A) No, because the child never hired the attorney as her counsel. (B) No, because the privilege is held by the client rather than the attorney. (C) Yes, because the child paid the attorney for consultation. (D) Yes, because the presence of a third party did not negate the privilege.
D, (D) The attorney may invoke the attorney-client privilege because the presence of a minor client's parent does not waive the privilege. A client has a privilege to refuse to disclose, and to prevent others from disclosing, confidential communications between herself and her lawyer. The attorney-client privilege requires that, at the time of the communication, the client be seeking the professional services of the attorney. Disclosures made before the lawyer has decided to accept or decline the case are covered if the other requirements of the privilege are met. A communication is confidential if it is not intended to be disclosed to third persons; thus, communications made in the known presence and hearing of a stranger are not privileged. However, statements made in front of third persons whose presence is reasonably necessary to the consultation (e.g., this client's parent) are still considered confidential. Here, the child was consulting with the attorney for the purpose of seeking the attorney's professional services. During this consultation, the child made disclosures concerning her injuries that were not intended to be disclosed to third persons. Thus, the child's communication would be deemed confidential. This confidentiality would not be lost by virtue of the fact that the communication was made in the presence of the father, whose presence was reasonably necessary given the child's age. Because the elements of the attorneyclient privilege are thus satisfied, the attorney may invoke the privilege on behalf of the child to refuse to disclose his discussion with the child concerning the child's injuries. (A) is incorrect because the attorney-client privilege does not depend on an actual hiring of the attorney. The requisite relationship exists simply by virtue of the fact that, at the time of the communication, the child was seeking the attorney's professional services. (B) is incorrect because the person who was the attorney at the time of the communication can claim the privilege on behalf of the client. The attorney's authority to do this is presumed in the absence of any evidence to the contrary. (C) is incorrect because application of the privilege does not hinge on payment for services. The confidential communications of a client receiving a professional consultation free of charge are protected to the same extent as those made to a lawyer charging for his time.
Under a state aid-to-education statute, a private school receives: (i) free textbooks from the state, (ii) an exemption from state taxes, and (iii) 20% of its operating budget in the form of state grants. The remaining 80% of the school's budget is covered by tuition fees and by donations from alumni and others. The school is licensed by the state, but the state has no requirement for certification and licensure of teachers in private schools. A teacher was hired and given the standard three-year contract given to teachers in their first stint at the school. In the fall term of his second year, the teacher gave a lecture to his students criticizing the school's use of school uniforms and encouraging the students to organize a protest against the uniform policy. After the speech, the teacher was called to the administrative office by the headmaster and fired on the spot, despite the teacher's protests that he had almost two years left on his contract. The teacher requested a hearing and was told to leave the premises of the school immediately. If the teacher files suit in federal district court alleging that his constitutional rights have been violated, is the teacher likely to succeed? A Yes, because the school's action violated the teacher's freedom of speech rights. B Yes, because the teacher has been denied due process of law. C No, because the teacher was not in his position long enough to acquire property rights in his job. D No, because assistance and involvement by the state did not result in the private school's action being conduct by the state.
D. The teacher's constitutional arguments will fail because his firing by the school did not constitute state action. The Fourteenth Amendment Due Process Clause, which makes many of the provisions of the Bill of Rights applicable to the states, does not apply to purely private conduct that interferes with these rights. Thus, unless the private individual (i) was performing exclusively public functions, or (ii) took actions with significant state involvement, the individual's action is not unconstitutional. In this case, the school is a private institution performing a function—education—that has never been considered to be an exclusively public function. [See Pierce v. Society of Sisters (1925)] Furthermore, its licensing by the state and receipt of state funds do not constitute significant state involvement with regard to its personnel matters; thus, the teacher cannot establish that the school exercised state action. [See Rendell-Baker v. Kohn (1982)] (A) is incorrect because constitutional protection for freedom of speech does not extend to actions taken by private individuals. Furthermore, even a public school probably could have fired the teacher for his speech (if its termination procedures were otherwise proper). Because public schools generally are not public forums, reasonable restrictions based on legitimate pedagogical concerns rather than the content of the speech are permissible. [Bethel School District No. 403 v. Fraser (1986] The school reasonably could argue that urging the students to organize a protest against a school policy would disrupt discipline and interfere with its educational process. (B) is incorrect for the same reason as (A): The constitutional right to due process of law does not apply to private conduct. Had the teacher been fired by a public school, he would be able to claim a property right in his employment for the balance of his contract. The school probably would have had to provide him with a pretermination opportunity to respond to the charges against him. [Cleveland Board of Education v. Loudermill (1985)] (C) is incorrect; the teacher did have property rights in his job because he had a three-year contract with the school. Had the teacher instead been an employee-at-will, without a contract, he would have had no property interest in continued employment even if his employer had been a public school. [Bishop v. Wood (1976)]
A defendant is being prosecuted in federal court for illegally transporting persons across state lines for immoral purposes. The prosecutor alleges that her route was from New York to Tampa. The court takes judicial notice of the fact that it is impossible to get from New York to Tampa without crossing a state line. What is the effect of the court's action? (A) The fact judicially noticed is conclusively established. (B) The burden of persuasion is shifted to the defendant. (C) The burden of proof is shifted to the defendant. (D) The prosecutor's burden of producing evidence on this point is satisfied.
D. (D) The court's action satisfies the prosecutor's burden of producing evidence on this point. This question involves the effect of taking judicial notice of a fact. Judicial notice allows a party to "prove" a fact by the court's recognizing that the fact is a matter of common knowledge within the jurisdiction or is able to be quickly determined by resorting to sources whose accuracy cannot reasonably be questioned. [Fed. R. Evid. 20l(b)] Since judicial notice functions are a substitute for more formal evidence (i.e., testimonial evidence), it has the same effect as more formal evidence. If the prosecutor had presented competent testimonial evidence to establish that it is impossible to get from New York to Tampa without crossing a state line, the prosecutor's burden of producing evidence on that point would have been satisfied. Thus, the prosecutor's burden of production is likewise satisfied if the point is established through judicial notice. (A) is incorrect because this is a criminal case. (A) would be correct if this were a civil case because, in civil cases, a fact judicially noticed is conclusively established, and thus binding on the jury. However, in criminal cases, a judicially noticed fact is not binding on the jury; the jury is permitted to find facts that have been judicially noticed, but never is required to do so. [Fed. R. Evid. 20l(g)] If a jury were required to find specific facts in a criminal case, even facts that had been judicially noticed, the defendant's right to trial by jury would be undermined. (B) and (C) are also incorrect. Since this is a criminal case, the burden of establishing the defendant's guilt always stays with the prosecution. If the interstate nature of the New York-Tampa route were established through more formal evidence, the burden of establishing the defendant's guilt would stay with the prosecution. The prosecution would not be relieved of this burden if judicial notice, rather than more formal evidence, were used to establish the interstate nature of the route. (These choices were questionable from the outset because they draw a distinction between "burden of persuasion" and "burden of proof," two concepts that are often treated as interchangeable.)
Under the Due Process Clause, the following government acts are considered deprivations of liberty except: A Denial of the right to engage in gainful employment B Loss of a freedom provided by the Constitution C Defamation without a tangible loss D Denial of the right to vote
D. A government act that causes defamation without a tangible loss is not considered a deprivation of liberty under the Due Process Clauses. The term "liberty" includes more than just freedom from bodily restraints. A deprivation of liberty occurs if a person (i) loses significant freedom of action; or (ii) is denied a freedom provided by the Constitution or a statute. Damage to one's reputation generally does not involve a loss of significant freedom of action or of a freedom provided by law. Therefore, it generally does not constitute a deprivation of liberty. (However, it can, if the damage is so severe that employment or associational opportunities are lost.) Government acts that cause denial of the right to engage in gainful employment, loss of a freedom provided by the Constitution, or denial of the right to vote are within the definition of deprivation of liberty under the Due Process Clause.
What separates voluntary manslaughter from murder is: A The killing is "caused" by the commission of a felony B The killing is "caused" by the commission of a crime not amounting to a felony C The lack of express malice D Adequate provocation
D. A killing committed under adequate provocation is voluntary manslaughter. A killing committed during the commission of an unlawful act not amounting to a felony is involuntary manslaughter. Committing a felony that results in death is murder. The intent to commit a felony satisfies the malice requirement to classify a killing as murder. A killing committed without express malice may be murder if malice is implied. "Malice aforethought" for common law murder can be satisfied by (i) the intent to kill; (ii) the intent to inflict great bodily injury; (iii) a reckless indifference to an unjustifiably high risk to human life; or (iv) the intent to commit a felony. The intent to kill is deemed to be "express malice," whereas, in the latter three, malice is implied. Although "implied," the latter three states of mind also satisfy the malice requirement for common law murder.
Which of the following activities involving public schools is valid under the Establishment Clause? A A school policy whereby students themselves decide whether to hold a student invocation ceremony prior to athletic events. B Posting of the Ten Commandments in a public school pursuant to legislature's declaration that the posting is for a secular purpose. C A voluntary moment of silent prayer or meditation at the beginning of the schoolday. D A program whereby, once weekly, the school day ends one hour early so that interested students may participate in voluntary religious classes in a separate location from the school.
D. A program whereby, once weekly, the schoolday ends one hour early so that interested students may participate in voluntary religious classes in a separate location from the school is valid. (A similar program held in the school itself has been struck down, because turning the classrooms over to religious instructors was found to promote religion.) A voluntary moment of silent prayer or meditation at the beginning of the schoolday is invalid as promoting religion. It is irrelevant that the exercise is voluntary and that it is silent. Posting of the Ten Commandments in a public school pursuant to legislature's declaration that the posting is for a secular purpose is invalid. The Supreme Court ruled that this program clearly served a religious purpose in spite of the legislature's statement. A school policy whereby students themselves decide whether to hold a student invocation ceremony prior to athletic events in invalid. The policy promotes religion and allowing the students to vote does not negate the policy's effect.
A recording act that provides: "Any conveyance of an interest in land, other than a lease for less than one year, shall not be valid against any subsequent purchaser for value, without notice thereof, unless the conveyance is recorded," is a: A race statute B race-notice statute C statute of frauds D notice statute
D. A recording act that provides: "Any conveyance of an interest in land, other than a lease for less than one year, shall not be valid against any subsequent purchaser for value, without notice thereof, unless the conveyance is recorded," is a notice statute. Under a notice statute, a later purchaser of land will prevail over an earlier grantee if she takes without actual or constructive (e.g., record) notice of the earlier grant. The above language is not a race-notice statute. An example of a race-notice statute is: "Any conveyance of an interest in land, other than a lease for less than one year, shall not be valid against any subsequent purchaser for value, without notice thereof, whose conveyance is first recorded." Under a race-notice statute, a later purchaser will prevail over an earlier grantee only if she takes without actual or constructive (e.g., record) notice of the earlier grant and records before he does. The above language is not a pure race statute. An example of a pure race statute is: "Any conveyance of an interest in land, other than a lease for less than one year, shall not be valid against any subsequent purchaser whose conveyance is first recorded." Under a race statute, notice is irrelevant. The first party to record, regardless of the date of her conveyance, wins. The Statute of Frauds is not a recording act. Every conveyance of an interest in land with a duration long enough to bring into play a particular state's Statute of Frauds (typically one year) must be evidenced by a writing, signed by the party to be charged
A man owned a tract of land in fee simple. Fifteen years ago, he built a barn on five acres that he believed were part of his property. One year later, the man discovered that the five acres on which he had built his barn were not part of his property. The five acres actually belonged to the woman who owned the adjoining property. The year following the discovery that the five acres belonged to the woman next door, the woman died, leaving all of her property to her one-year-old daughter. The man has brought a quiet title action against the now 14-year-old daughter. The statutory period for adverse possession in this jurisdiction is 10 years. The man has not paid any additional property taxes to account for the five acres for any of the past fifteen years. Who will prevail? A The daughter, because the man did not pay the property taxes on the five acres. B The daughter, because her status as a minor tolls the statute until she reaches her majority. C The man, because he honestly believed that the five acres were part of his land. D The man, because he was in continuous possession of the five acres for the statutory period.
D. Because the man was in continuous possession for the statutory period and has met all of the other requirements of adverse possession, he would be declared the owner of the five acres. To establish title by adverse possession, the possessor must show (i) an actual entry giving exclusive possession that is (ii) open and notorious, (iii) adverse (hostile), and (iv) continuous throughout the statutory period. Here, the man possessed the property by building a barn on it, something that is clearly visible to the public, so the possession was actual and open and notorious. The man did not share the barn with anyone, so the possession was exclusive. The man did not have the true owner's permission to build the barn, so the possession was hostile. And the possession was continuous for more than 10 years. (A) is incorrect because only a minority of states require the adverse possessor to pay property taxes. (B) is incorrect because the disability of the woman's successor in interest will not keep the statute from running. For a disability, such as status as a minor, to stop the clock, the disability must have been in existence on the day the adverse possession began. Here, the daughter was not yet alive when the adverse possession began. Thus, her status as a minor will not stop the running of the statute. (C) is incorrect because the man's state of mind is irrelevant under the majority view. Even if he had possessed the land knowing he was trespassing, he could still claim it by adverse possession.
A local news station broadcast a live interview with a bystander about his views concerning the state of local education. The bystander responded by saying that the principal of his daughter's high school had been embezzling school funds for years. The principal saw the telecast and also recorded it. He sued the owner of the station for defamation. At trial, the principal sought to testify to the defamatory statement made in the interview. Will the principal's testimony likely be held to be admissible? A No, because the testimony would be hearsay not within any exception. B No, because a recording of the interview exists. C Yes, because the statement is being offered to show its effect on the principal. D Yes, because the principal personally saw the interview on television.
D. Because the principal had firsthand knowledge that the statement was made, his testimony will be admissible unless there is a specific rule excluding the evidence. Witnesses are generally presumed competent to testify until the contrary is demonstrated. While a witness may not testify to a matter unless evidence is introduced to support a finding that the witness has personal knowledge of the matter, this evidence may consist of the witness's own testimony. (A) is incorrect. Hearsay is a statement, other than one made by the declarant while testifying at the trial or hearing, offered in evidence to prove the truth of the matter asserted. In a defamation action, evidence of the statement alleged to be defamatory is not hearsay because the evidence is by definition not offered to prove the truth of the matter asserted. It is offered only to show that the actionable statement was made. (B) is incorrect. Because the principal had firsthand knowledge of the event he can testify about the event, even though there might exist a recording that would be better proof of the event. The "best evidence rule" does not apply because the recording is not an essential repository of the facts recorded. (C) is incorrect. Although the statement is not hearsay, it is not being offered to show its effect on the hearer (e.g., knowledge, motive), but rather to show that the statement was made, as explained above.
Which of the following statements is correct regarding covenants against assignment or sublease? A If a landlord consents to one transfer that violates a covenant against assignment or sublease, he waives his right to avoid future transfers. B If a tenant transfers her interest in violation of a covenant against assignment or sublease, the transfer is void. C A covenant against assignment or sublease is an unreasonable restraint on alienation. D A covenant against assignment prevents the tenant from subleasing her interest.
D. If a landlord consents to one transfer that violates a covenant against assignment or sublease, he waives his right to avoid future transfers. This is the Rule in Dumpor's Case. The landlord may reserve the right to avoid future transfers, but such reservation must take place at the time of granting consent. A covenant against assignment or sublease is NOT an unreasonable restraint on alienation. All jurisdictions permit and enforce such covenants. A covenant against assignment does NOT prevent the tenant from subleasing her interest. Covenants against assignment or sublease are strictly construed against the landlord. Thus, a covenant prohibiting assignment does not prohibit subleasing and vice versa. If a tenant transfers her interest in violation of a covenant against assignment or sublease, the transfer is NOT void. However, the landlord usually may terminate the lease under the lease terms or a statute or sue for damages.
In general, a party who fails to tender performance on the closing date: A Is excused from performance B Has no liability for even incidental damages C Is in total breach and loses her right to enforce the contract D Has a reasonable time after the closing date to tender performance
D. In general, a party who fails to tender performance on the closing date has a reasonable time after the closing date to tender performance and avoid breach. Generally, the time of performance stated in a land sale contract is not absolutely binding. A party, even though late in tendering her own performance, can still enforce the contract if she tenders within a reasonable time after the stated date. Courts presume that time is not of the essence. However, this presumption may be overcome if: (i) The contract states that time is of the essence; (ii) The circumstances indicate that the parties intended that time is of the essence; or (iii) One party notifies the other within a reasonable time before the closing date that time is of the essence. If time is of the essence, a party who fails to tender performance on the closing date is in total breach and loses her right to enforce the contract . However, even if time is not of the essence, a party who is late in tendering performance is NOT excused from performance absent repudiation or impossibility, and will be liable for incidental damages (e.g., additional mortgage interest or taxes).
Which of the following statements is correct regarding the ability of police officers to stop automobiles for investigatory purposes? A The police may stop an automobile for investigatory purposes without cause because automobiles are not areas protected by the Fourth Amendment. B The police may not stop an automobile for investigatory purposes unless they have probable cause to believe that the driver has broken a law. C The police may not stop an automobile for investigatory purposes unless they have reasonable suspicion that the driver has broken a law. D The police may stop automobiles for investigatory purposes even without reasonable suspicion if they make the stops on a neutral, articulable basis to investigate a problem closely related to the mobility of automobiles.
D. It is true that the police may stop automobiles on a neutral, articulable basis to investigate a problem closely related to the mobility of automobiles. Generally, to make an investigatory stop of an automobile, the police must have at least reasonable suspicion that the driver or an occupant violated or was about to violate some law (i.e., the normal Terry standard). However, the Supreme Court has recognized that the mobility of automobiles can cause special problems, and allow the police to set up roadblocks to stop automobiles even without individualized suspicion to investigate these problems. A good example would be a roadblock that stops every car to determine whether the drivers are intoxicated. The police do not need to have probable cause to believe that the driver has broken the law to stop an automobile for investigatory purposes, because automobiles can be stopped on reasonable suspicion or even without any individualized suspicion, as discussed above. The police do not need to have even reasonable suspicion that the driver has broken the law to stop an automobile for investigatory purposes. First, the driver is not the only possible object of reasonable suspicion. And second, as discussed above, automobiles can be stopped even without any individualized suspicion, to investigate a problem closely related to automobiles and their mobility, as long as the stops are made on a neutral and articulable basis. The choice indicating that automobiles are not areas protected by the Fourth Amendment is incorrect. The Fourth Amendment protects people rather than places. And the courts have held that people do have a privacy interest (albeit a diminished one) when in their automobiles and are protected by the Fourth Amendment.
Which of the following will not establish the general intent requirement of a crime? A The doing of the act B The intent to cause a harmful result to a different person or object C Proof that the defendant is acting in a proscribed way and that any attendant circumstances required by the crime are present D Motive
D. Motive will not establish the general intent requirement of a crime. Motive is merely a reason or explanation for committing the offense, and is generally immaterial to substantive criminal law. A jury can infer the required general intent merely from the doing of the act. It is not necessary that evidence specifically proving the general intent be offered by the prosecution. The intent to cause a harmful result to a different person or object is sufficient to establish intent to cause a similar harmful result to the person or object actually harmed. This is known as transferred intent. General intent requires that the defendant be aware that he is acting in a proscribed way and that any attendant circumstances required by the crime are present. Thus, proof that this is true will establish general intent.
The states are sovereignties apart from the federal government. When the federal government acts within a state, the state may: A Directly tax federal property and instrumentalities within the state B Charge a sales tax on items purchased by contractors acting as purchasing agents for the federal government C Require federal employees who drive as part of their jobs to have a valid driver's license, even when performing job duties D Impose an income tax on persons who receive a salary from the federal government
D. States MAY impose an income tax on persons who receive a salary from the federal government, as long as the tax is nondiscriminatory and does not burden the federal government. States may NOT directly tax federal property and instrumentalities within the state. States may NOT charge a sales tax on items purchased by contractors acting as purchasing agents for the federal government. This would be a prohibited direct tax. However, states can charge a sales tax to a contractor who is working on a federal project on a cost-plus profit basis, even though the tax ultimately will be borne by the federal government, because, in such a case, the tax is indirect. States may NOT require federal employees to have a valid driver's license to drive within the state while performing job duties. The states may not regulate the federal government without the federal government's consent. Thus, instrumentalities and agents of the federal government are immune from state regulations relating to performance of their federal functions.
A homeowner and a contractor duly executed a contract providing that the contractor was to construct a residence on a specified lot. No date was included in the contract for completion of the home. After the contractor completed 5% of the residence, a tornado demolished the construction but left the lot undamaged. Which of the following states the probable legal consequences of the tornado damage? A The contract is void because the subject matter of the contract was destroyed through no fault of the parties. B The contractor's duty of performance is discharged because of impossibility. C The contractor remains obligated to construct the residence, but he is entitled to a quantum meruit recovery for the work done prior to the tornado. D The contractor remains obligated to perform under the original contract without any compensation for the work done prior to the tornado.
D. The contractor remains bound under the original contract, and he is not entitled to compensation for the work that was destroyed. The general rule is that a contractor is responsible for destruction of the premises under construction prior to completion. Once the residence is completed, risk of loss shifts to the owner. (A) is wrong because the subject matter was not destroyed. Note that even if the subject matter were destroyed, it would not void the contract; it would merely discharge the contractor's duties under the contract. (B) is wrong because performance is not impossible; the contractor can rebuild the residence.
Acting on a hunch, a police officer went to a young woman's apartment, broke in, and searched it. The officer found exactly what she was looking for under the woman's bed: a sack filled with jewels. The attached note read, "Sweetheart, here are the goods from the estate heist. Your loving boyfriend." It was well known in the community that the woman's boyfriend was a jewel thief. The officer also knew that the estate of a local socialite had been burglarized three days ago. Just as the officer finished reading the note, the woman returned. The officer immediately placed the woman under arrest as an accessory to the estate burglary. Based on the evidence obtained from the woman's apartment, a search warrant was issued for her boyfriend's apartment. The search yielded burglar tools and more jewels from the estate. The boyfriend was immediately arrested and charged with the estate burglary. At the boyfriend's trial for the estate burglary, his attorney files a motion to suppress the evidence consisting of the bag of jewels and note, the tools, and the jewels from the boyfriend's apartment. How should the court rule on the motion? A Grant the motion as to the bag of jewels and note, but deny it as to the evidence found in the boyfriend's apartment. B Grant the motion, because all of this evidence is fruit of the poisonous tree. C Deny the motion, because the police would have caught the boyfriend with the goods eventually. D Deny the motion, because the police had a warrant to search the boyfriend's apartment.
D. The court should deny the motion to suppress because the police had a warrant to search the boyfriend's home. The boyfriend's expectation of privacy extended only to his own home, which was searched under a warrant. He does not have standing to assert a Fourth Amendment claim regarding the search of his girlfriend's apartment because her apartment was not his home, and he did not own it or have a right to possession of it. Thus, (A) is incorrect. Because the boyfriend cannot object to the search that provided the probable cause for the search of his apartment, (B) is also incorrect. (C) is not a valid justification because there is nothing to indicate that the seizure would fall under the "inevitable discovery" exception to the exclusionary rule.
A landowner died, validly devising her land "to my son for life, then to my brother and sister in fee simple." Without obtaining the brother and sister's consent, the son borrowed $20,000 from a bank, secured by a mortgage on the land, to make improvements to the land. Five years later, the son died. The brother and sister took possession of the land, but failed to make any mortgage payments. If the bank sues to recover the delinquent payments, the court should render judgment for: A The bank, because a life tenant is obligated to make repairs. B The bank, because the remaindermen are obligated to pay the principal of a debt. C The brother and sister, because the son committed ameliorative waste. D The brother and sister, because the mortgage does not encumber the fee simple.
D. The court should render judgment for the brother and sister because the mortgage does not encumber the fee simple. A life tenant is entitled to all the ordinary uses and profits of the land, which includes encumbering his own interest, but he cannot lawfully do any act that would injure the interests of the remaindermen. Permissive waste occurs when the life tenant allows the land to fall into disrepair or fails to take reasonable measures to protect the land. Although a life tenant is obligated to pay interest on any encumbrances of the fee simple estate to the extent of the income or profits from the land (or in their absence to the extent of the reasonable rental value of the land), he is liable for both principal and interest payments if the encumbrance is on the life estate alone. Here, because the son did not obtain the consent or joinder of the remainderman when he mortgaged the land, the mortgage attaches only to the life estate. Thus, the remaindermen (the brother and sister) are not liable for the mortgage payments. (A) is incorrect because the son made improvements rather than repairs to the land. Although a life tenant is obligated to preserve the land and structures in a reasonable state of repair, to the extent of the income or profits from the land (or in their absence to the extent of the reasonable rental value of the land), he is under no obligation to make permanent improvements on the land. (B) is incorrect because remaindermen are obligated to pay the principal of a debt only if the debt encumbers the entire fee simple estate. Here, only the life estate is encumbered. (C) is incorrect because ameliorative waste, which occurs when the use of the property is substantially changed but the change increases the value of the property, may have been an appropriate cause of action for the brother and sister to bring against the son for the improvements, but it does not affect the bank's rights.
The defendant's neighbor owned an authentic major league baseball signed by Babe Ruth. The defendant asked if he could show it to some friends who were visiting. The neighbor agreed as long as he kept it in the display case, which the defendant promised to do. In fact, the defendant intended to use the ball in a pickup game. During the game, the ball was hit over the fence and into a yard with a guard dog, which had chewed up several other balls that had previously landed in the yard. The dog did the same to that ball. When the neighbor learned what happened to the ball, he pressed charges against the defendant. If the defendant is convicted, he will most likely be found guilty of what crime? A Common law larceny. B Embezzlement. C False pretenses. D Larceny by trick.
D. The defendant is guilty of larceny by trick because he obtained possession of the baseball by means of a misrepresentation. Larceny is the taking and carrying away of tangible personal property of another by trespass, with intent to permanently (or for an unreasonable time) deprive the person of her interest in the property. The taking must be without the consent of the person in possession of the property. If such consent is induced by a misrepresentation of a past or existing fact, the consent is not valid. The resulting larceny is called larceny by trick. Here, the defendant obtained possession of the baseball with the owner's consent. However, this consent was obtained by means of the defendant's misrepresentation about friends visiting. This was a false statement of an existing fact, made with the intent that his neighbor rely on the statement, and the misrepresentation induced his neighbor's consent. At the time of this taking, the defendant intended to deal with the baseball in a manner that involved a substantial risk of damage or loss. This suffices as intent to permanently deprive. Therefore, all the elements are in place for larceny by trick.
A craftsperson entered into a written agreement with an electrician to install electrical wiring in her standalone garage so that she could convert the garage into a workshop. The contract contained a clause requiring all electrical work to be completed within two days and provided that the craftsperson would pay the electrician $700 for his work. After the first day, approximately half of the job was completed. That evening, a piece of a defunct satellite reentered the Earth's atmosphere and a large chunk of it crashed directly into the craftsperson's garage, catching the garage on fire and destroying it. Which of the following best describes the obligations of the electrician and the craftsperson after the crash? A Neither the electrician nor the craftsperson is discharged from their obligations under the contract. B Neither the electrician nor the craftsperson has any further obligations. C The craftsperson is obliged to pay the electrician the full contract price of $700. D The electrician is discharged from his obligation but is entitled to recover from the craftsperson the fair value of the work he performed prior to the destruction of the garage.
D. The destruction of the garage discharges the electrician's duties due to impossibility, but the electrician has a right to recover for the reasonable value of the work he performed. Contractual duties are discharged where it has become impossible to perform them. The occurrence of an unanticipated or extraordinary event may make contractual duties impossible to perform. If the nonoccurrence of the event was a basic assumption of the parties in making the contract, and neither party has assumed the risk of the event's occurrence, duties under the contract may be discharged. If there is impossibility, each party is excused from duties that are yet to be performed. If either party has partially performed prior to the existence of facts resulting in impossibility, that party has a right to recover in quasi-contract for the reasonable value of his performance. While that value is usually based on the benefit received by the defendant (unjust enrichment), it also may be measured by the detriment suffered by the plaintiff (the reasonable value of the work performed). Here, the garage that the electrician was wiring burned down after a chunk of a satellite crashed into it. That event was of such an unexpected nature that its nonoccurrence was a basic assumption of the parties, and neither party was likely to have assumed the risk of its occurrence. Thus, it has become objectively impossible for the electrician (or anyone else) to complete the job. This impossibility will discharge both the craftsperson and the electrician from performing any contractual duties still to be fulfilled. Therefore, the electrician need not finish the wiring work, and the craftsperson is not obligated to pay the entire amount of $700. However, the electrician can recover under quasi-contract. (A) is incorrect because both parties are discharged. (B) is incorrect because it fails to account for the fact that the craftsperson will have to pay the electrician for the value of the work already performed. (C) is incorrect because, as discussed above, the craftsperson is obligated to pay for the value of the electrician's services to date, not the full contract price.
A felon intending to rob a market waited outside until there were no customers. When he saw that the market was empty, he went inside and walked up to the counter with his hand in his jacket pocket to simulate a gun. Before the clerk could turn around to see what the felon wanted, another customer entered the market, startling the felon, who turned and ran out the door. Should the felon be found guilty on a charge of attempted robbery? A No, because he used no actual force on the clerk nor threatened any. B No, because he withdrew successfully from the robbery attempt. C No, because he never entered the zone of perpetration. D Yes, regardless of whether he totally abandoned his plan when the customer entered the market.
D. The felon should be found guilty of attempted robbery. With the specific intent to commit a robbery, the felon went beyond mere preparation for the offense. Once a person has gone beyond preparation, abandonment is not a defense to attempt. A criminal attempt is an act that, although done with the intention of committing a crime, falls short of completing the crime. The defendant must have the intent to perform an act and obtain a result that, if achieved, would constitute a crime. Also, the defendant must have committed an act beyond mere preparation for the offense. If a defendant has, with the required intent, gone beyond preparation, the general rule is that abandonment is not a defense. Even in those jurisdictions in which abandonment is a defense, such abandonment must be: (i) fully voluntary and not made because of the difficulty of completing the crime or because of an increased risk of apprehension; and (ii) a complete abandonment of the plan made under circumstances manifesting a renunciation of criminal purpose, not just a decision to postpone committing it or to find another victim. Here, the felon intended to take money from the clerk at the market by means of the threat of having a gun (i.e., by simulating a gun). Thus, the felon intended to commit a robbery.
A motorist drove home from work late one night, and fell asleep behind the wheel of his car. His car drifted across the middle of the road and struck another car. The other driver was killed instantly in the collision. Angered by the noise of the collision, a homeowner fired a gun out the window of his house at the car. The bullet struck and killed a bystander. Both the motorist and the homeowner were arrested and charged with common law murder. Which of the defendants likely would be found guilty? A Both the motorist and the homeowner. B The motorist. C Neither the motorist nor the homeowner. D The homeowner.
D. The homeowner likely would be found guilty. At common law, murder was the unlawful killing of a human being with malice aforethought. Malice aforethought could be established with any one of the following states of mind: intent to kill; intent to cause serious bodily harm; the depraved heart killing (a reckless indifference to an unjustifiably high risk to human life); or the commission of a felony. The homeowner would be guilty of murder. Firing a gun out of his window at a car would demonstrate a reckless indifference to a high risk to human life. Thus, (B) and (C) are incorrect. It is unlikely that the motorist would be guilty of murder. While his action might be classified as negligent or even reckless, it would not represent a depraved heart (reckless indifference to life) state of mind. Thus, (A) and (B) are incorrec
A landowner and her neighbor owned adjoining parcels of land. The landowner's property was situated to the west of the neighbor's property. A highway ran along the east of the neighbor's property. Twelve years ago, the landowner asked the neighbor if it would be all right for the landowner to use an eight-foot strip along the northern part of the neighbor's land to access the highway. The only other way for the landowner to get to the highway was to use a one-lane unpaved road that meandered through the woods for two miles. The neighbor agreed, and the landowner used the strip of land regularly to access the highway. The statutory period for adverse possession in this jurisdiction is 10 years. What is the landowner's interest in the neighbor's eight-foot strip of land? A An easement appurtenant. B An easement by necessity. C An easement by prescription. D Not an easement.
D. The landowner's interest in the neighbor's eight-foot strip of land is not an easement. In effect, the landowner only has a "license" (i.e., a revocable privilege) to use the land. The answer is best reached by the process of elimination. Because an easement is an interest in land, the Statute of Frauds applies. Here, the agreement between the landowner and the neighbor was not in writing; thus, the Statute of Frauds requirements for the creation of an express easement were not met. Therefore, (A) is incorrect. (B) is incorrect because an easement by necessity is created when the owner of land sells a part of it and deprives the part sold of access to the public road. Here, the facts do not indicate that the landowner's and the neighbor's parcels were once part of a common tract, and the landowner has an alternate, albeit inconvenient, way to access the highway—the one-lane road. Thus, the landowner does not have an easement by necessity. (C) is incorrect because the landowner's use of the land was permissive. To acquire a prescriptive easement, the use must be open and notorious, adverse, and continuous and uninterrupted for the statutory period. Although the landowner used the strip for the requisite 10-year period, she does not meet the adverse requirement necessary to obtain a prescriptive easement
Intending to encourage long-time resident aliens to become American citizens, a state passed a law denying numerous state and municipal jobs to persons who had been resident aliens for longer than 10 years. Those already in the state had to apply for American citizenship within a year after the law took effect. Persons who had acquired resident alien status prior to achieving the age of majority had until age 30 to acquire such status or be automatically disqualified from obtaining such a job. A 40-year-old man who has been a resident alien in the state for 15 years applied for a job as a police emergency response telecommunications expert. He had not filed for citizenship within the one-year grace period. May the state constitutionally rely on the statute to refuse to hire the man? A Yes, because a police department performs an integral governmental function and the state law does not discriminatorily classify resident aliens by race or ethnicity. B Yes, because aliens are not entitled to the privileges and immunities of state citizenship. C No, because the law does not apply equally to all aliens. D No, because the reasons for application of the law to the man do not appear compelling.
D. The law probably is unconstitutional as applied to the man in question. An equal protection issue is involved. Under the Equal Protection Clause, state classifications based on alienage are subject to strict scrutiny and so must serve a compelling interest to be constitutional. No compelling purpose seems to be present here. Thus, (D) is correct. (A) is incorrect because, although there is an exception from the strict scrutiny standard where a state or local government discriminates against aliens when hiring persons for jobs involving "self-government" processes, the job here (emergency communications for a police department) is a technical position and probably would not be found to be related to the self-government process; in any event, the statute applies to all positions and not just to jobs involving only the self-government process. (B) is incorrect because, although it is true that aliens are not entitled to the privileges and immunities of state citizenship, the law here is still unconstitutional under the Equal Protection Clause, as discussed above, which applies to aliens. (C) is incorrect because the discrimination would be unconstitutional even if it did apply to all aliens equally, as discussed above.
When the government takes private property for public use, which of the following statements is correct regarding the remedy? A Just compensation is measured by the fair market value of the property as of the date of the court ruling that a taking has occurred. B The government may not avoid payment of just compensation by rescinding the regulation. C One who purchases property after a regulation that amounts to a taking was enacted does not have standing to make a claim for just compensation. D The measure of damages is the loss the owner incurs, rather than the gain the taker receives.
D. The measure of damages required for just compensation is the loss the owner incurs, rather than the gain the taker receives. This is the proper measure of "reasonable value" that the courts look to in determining just compensation. Just compensation is measured by the fair market value of the property as of the date of the court ruling that a taking has occurred is incorrect. The time to be considered for just compensation valuation purposes is the time that the taking itself has occurred. The statement that the government may not avoid payment of just compensation by rescinding the regulation is incorrect. Once the court decides that a taking has occurred, the government may be required to pay just compensation to the property owner, or may be required to terminate the regulation and pay the property owner so-called temporary taking damages—compensation for any damage resulting during the time the regulation was effective. One who purchases property after a regulation that amounts to a taking was enacted MAY have standing to make a claim for just compensation. If the regulation remains in effect after the purchase of the property, the new owner may make a taking claim.
An intentional deprivation of life, liberty, or property requires fair procedures and an unbiased decision maker. Whether an individual is entitled to notice, a hearing, or some other fairness consideration depends on the circumstances of the particular deprivation. Which of the following is not a factor a court will consider in determining the type of procedural due process that is required? A The importance of the individual's interest that is involved. B The government's interest in fiscal and administrative efficiency. C The value of specific procedural safeguards of the individual's interest. D The opportunity for the individual to be represented by counsel.
D. The opportunity for the individual to be represented by counsel is not something the courts consider in determining what type of procedure is required. There is no across-the-board right to counsel under the Constitution in any event. The level of due process that is required depends on the circumstances surrounding the deprivation of the interest. The courts consider the factors raised in the remainder of the choices: (i) The importance of the individual's interest that is involved, (ii) The value of specific procedural safeguards of the individual's interest, and (iii) The government's interest in fiscal and administrative efficiency. Normally, the person whose interest is being deprived should also receive notice of the government's action and have an opportunity to respond before termination of the interest. However, the court may allow a post-termination hearing in situations where a pre-termination hearing is highly impracticable
The owner of a stationary bicycle wrote a letter to her friend offering to sell her stationary bicycle to him for $150. The friend received the letter on January 18. On January 19, he mailed a letter back saying that he was not interested in purchasing the bike because he had just purchased a gym membership. However, the friend changed his mind the next day and mailed a letter to the owner accepting her offer to sell the bicycle and enclosing a certified check for $150. The owner received the friend's rejection letter on January 21 but put it aside without reading it. The next day, she received the friend's acceptance letter, which she opened and read immediately. Do the parties have a contract? A Yes, because under the mailbox rule an acceptance is effective on dispatch, while a rejection is effective on receipt. B Yes, because the friend paid for the bicycle when he accepted the offer to buy it. C No, because the acceptance was dispatched after the rejection. D No, because the mailbox rule does not apply—whichever is received first controls.
D. The parties do not have a contract, because the mailbox rule does not apply when the offeree sends a rejection, followed by an acceptance. In such a case, whichever is received first controls. Under the mailbox rule, acceptance by mail or similar means creates a contract at the moment of posting, with a couple of exceptions not relevant here. Rejection, on the other hand, is effective when received. So, if the mailbox rule had applied, there would have been a contract, because the friend's acceptance was mailed before his rejection letter was received. But because the mailbox rule does not apply here, and the matter is decided based on which letter was received first, there is no contract, because the friend's rejection letter was received by the bicycle owner a day before his acceptance letter was received by her. (A) is incorrect because, as discussed above, the mailbox rule does not apply when a rejection is sent before an acceptance; rather, whichever is received first controls. The fact that the bicycle owner did not read the rejection does not matter; it still was received by her before the acceptance. [See Restatement (Second) Contracts �68] (B) is incorrect because whether the friend paid for the bicycle is irrelevant. He sent the certified check (and his acceptance) after he sent his rejection, and the rejection was received first. (C) is incorrect because when a rejection by mail is followed by an acceptance by mail, the rule is that whichever is received first controls, not whichever is dispatched first. Thus, although it is true that there is no contract between the parties, it is because the friend's rejection letter was received by the bicycle owner first, rather than because it was mailed first.
The privilege against self-incrimination is waived: A by a witness once she takes the stand, but a defendant must affirmatively waive the privilege B by a defendant or witness once she takes the stand C by a witness once she discloses incriminating information, but she cannot be compelled to reveal anything further D by a defendant when she takes the stand, but only to the extent that she is subject to cross-examination
D. The privilege against self-incrimination is waived by a defendant when she takes the witness stand, but only to the extent that she is subject to cross-examination. By taking the stand, the defendant waives the privilege to the extent necessary to subject her to any cross-examination proper under the rules of evidence. The choices indicating that a witness waives the privilege once she takes the stand are incorrect. A defendant's taking the stand constitutes a waiver because a defendant may refuse to take the stand completely. A witness, on the other hand, cannot refuse to take the stand, but instead must take the stand and assert the privilege, if applicable, to the specific questions posed. A witness will be held to have waived the privilege only to the extent that she discloses incriminating information. If a witness discloses incriminating information, she CAN be compelled to reveal further information as long as the disclosure does not increase her risk of conviction or create a risk of conviction for a different offense.
Justice Jackson's opinion in Youngstown Sheet & Tube v. Sawyer presents a framework regarding the President's power over internal affairs. The following are part of that framework EXCEPT: A Where the President acts with the express or implied authority of Congress, his authority is at its maximum and his actions likely are valid B Where the President acts where Congress is silent, his action will be upheld as long as the act does not take over the powers of another branch of the government or prevent another branch from carrying out its tasks C Where the President acts against the express will of Congress, he has little authority and his action likely is invalid, provided Congress has acted constitutionally D Where the President acts to preserve funds set aside by Congress to be spent, his action likely will be held valid because of his powers under the Take Care Clause
D. The statement, where the President acts to preserve funds set aside by Congress to be spent, his action likely will be held valid because of his powers under the Take Care Clause is neither true nor part of the Youngstown framework. The Take Care Clause is in Article II, Section 3. It provides that the President shall take care that the laws be faithfully executed. Under the third prong of the Youngstown framework and the Take Care Clause, if Congress has provided that the President is to spend certain funds and the President refuses, the President's actions will likely be held invalid. The other choices set out the Youngstown framework: (i) Where the President acts with the express or implied authority of Congress, his authority is at its maximum and his actions likely are valid; (ii) Where the President acts where Congress is silent, his action will be upheld as long as the act does not take over the powers of another branch of the government or prevent another branch from carrying out its tasks; and (iii) Where the President acts against the express will of Congress, he has little authority and his action likely is invalid, at least if Congress has acted constitutionally.
Congress enacted a statute requiring state-supported institutions of higher education that provide federal student loan funds to their students to fund women's sports according to a complex formula intended to fairly support woman's athletics and remedy past funding discrimination. Under the formula, a particular state military school will be required to allocate 25% of its athletic budget to its female athletics programs even though only 10% of the school population is female. A male student whose athletic program will be discontinued because of the budget allocation filed suit in federal court challenging the federal statute on various constitutional grounds. Is the court likely to find that the statute is constitutional? A No, because the government will be unable to prove that the discriminatory funding requirements required by the statute are necessary to achieve a compelling government interest. B No, because the federal government does not have the power to dictate the budget allocations of state-supported educational institutions. C Yes, because remedying past discrimination is a legitimate government interest, and the student will be unable to prove that the statute's funding requirements are not rationally related to that interest. D Yes, because the government will be able to prove that the statute's funding requirements are substantially related to an important government interest.
D. The statute will likely be held constitutional because the attempt to compensate for past discrimination against women is substantially related to an important government objective. When examining federal government action involving classifications of persons, the Supreme Court, using the Due Process Clause of the Fifth Amendment, applies the same standards that it applies to state actions under the Fourteenth Amendment Equal Protection Clause. When analyzing government action based on gender classifications, the Court will apply an intermediate standard and strike the action unless the government proves, by an exceedingly persuasive justification, that the action is substantially related to an important government interest. Applying this standard, the Court has generally upheld classifications benefiting women that are designed to remedy past discrimination against women, because remedying past gender discrimination is an important government interest. Here, the federal statute establishes a formula designed not only to ensure current "gender equity" in funding of intercollegiate athletic programs but also to correct specific past inequities, and the school's required funding allocation in favor of women is designed to correct inequitable allocations by the school in prior years. Hence, even though the statute's allocation requirement may discriminate against the student and other males at the school, the government can satisfy its burden of showing a substantial relationship to an important government interest. (A) is incorrect because classifications based on gender are subject to an intermediate standard rather than a strict scrutiny standard; in other words, the government need not show that the classification is necessary to achieve a compelling interest, only that it is substantially related to an important interest. Furthermore, if the classification were one subject to strict scrutiny, remedying past discrimination based on the classification would probably be considered a compelling government interest. (B) is incorrect because Congress may "regulate" states through the spending power by imposing conditions on the grant of money to state governments. Even if Congress lacked the power to directly regulate the activity that is the subject of the spending program, attaching conditions on the spending does not violate the states' Tenth Amendment rights as long as the conditions are (i) clearly stated, (ii) related to the purpose of the program, and (iii) not unduly coercive. (C) is incorrect because it imposes the burden of proof on the wrong party and relies on the wrong standard. Because the statute results in gender discrimination, the government has the burden of proof, and that burden is to prove that the statute is substantially related to an important government interest.
On December 6, the owner of an electronics store sent a written request to a computer manufacturer asking for the price of a certain laptop computer. The manufacturer sent a written reply with a catalog listing the prices and descriptions of all of his available computers. The letter stated that the terms of sale were cash within 30 days of delivery. On December 14, by return letter, the store owner ordered the computer, enclosing a check for $4,000, the listed price. Immediately on receipt of the order and check, the manufacturer informed the store owner that there had been a pricing mistake in the catalog, which should have quoted the price as $4,300 for that computer. The store owner refused to pay the additional $300, arguing that his order of December 14 in which the $4,000 check was enclosed was a proper acceptance of the manufacturer's offer. In a suit for damages, will the manufacturer prevail? A Yes, because his first communication stated terms calling for cash within 30 days of delivery. B Yes, because of the mistake as to price. C Yes, because his first communication did not constitute an offer. D No, because the store owner's December 14 letter was a proper acceptance of the manufacturer's offer.
D. The store owner's December 14 letter was an acceptance. Whether the letter was an acceptance depends on whether the manufacturer's letter was an offer, because an acceptance is a manifestation of assent to an offer. For a communication to be an offer, it must create a reasonable expectation in the offeree that the offeror is willing to enter into a contract on the basis of the offered terms. There must be a promise, undertaking, or commitment to enter into a contract with certain and definite terms. Courts usually hold that if a statement is made broadly, such as in an advertisement or catalog, it will not constitute an offer because it is not reasonable to expect that the sender intended to make offers to all who received the advertisement; rather, the courts usually find such advertisements to be invitations seeking offers. However, price quotations may be considered as offers if given in response to a specific inquiry. The courts will look to the surrounding circumstances, and here a court would probably determine that the catalog that the manufacturer sent was an offer because it was sent in response to the store owner's specific inquiries about prices on a specific computer and it included delivery terms and conditions of sale. (A) is incorrect because although the letter called for payment in cash, tender by check is sufficient unless the seller demands legal tender and gives the buyer time to obtain cash. Moreover, because the contract called for payment within 30 days of delivery, even if the check was not sufficient, the store owner still had time under the contract to obtain cash. (B) is incorrect because the mistake was unilateral. Generally, a unilateral mistake will not be grounds to rescind a contract unless the nonmistaken party knew or should have known of the mistake. Here, nothing in the facts indicates that the store owner knew of the mistake, and the mistake was not so large that it could be said that he should have known of it. (C) is incorrect because, as explained above, the manufacturer's catalog was sent in response to the store owner's request for information and his terms for sale constituted an offer.
man and a woman purchased a parcel of land, taking title as joint tenants. Two years later, they married and had a son. Several years after that, the man and woman divorced. After the divorce, the woman and her son continued to occupy the land, although title remained in the names of both the man and the woman. The man moved out of the state and conveyed all of his title and interest in the land by deed to the son. Shortly thereafter, the man was killed in an automobile collision. The man died intestate. Who has title to the land? A The woman. B The woman owns one-half and the man's heirs own one-half. C The woman and her son as joint tenants. D The woman and her son as tenants in common.
D. The woman and her son have title to the land as tenants in common. The man and the woman took title to the land as joint tenants. An inter vivos conveyance by one joint tenant of his undivided interest severs the joint tenancy, so that the transferee takes the interest as a tenant in common and not as a joint tenant. Here, there was an inter vivos conveyance by the man to the son of all of the man's interest in the property held in joint tenancy with the woman. This conveyance destroyed the joint tenancy, so that the son takes his interest in the property as a tenant in common with the woman, rather than as a joint tenant. (A) is incorrect because the severance of the joint tenancy destroyed the right of survivorship. A joint tenancy carries with it a right of survivorship, whereby the death of one joint tenant frees the property from his concurrent interest, so that the surviving joint tenant retains an undivided right in the property that is no longer subject to the interest of the decedent. Had the man died without having conveyed his interest in the land, the woman would have held an undivided interest in the property, free of the man's interest. However, because the joint tenancy had been terminated prior to the man's death, there is no right of survivorship. Note also that the estate held by the man and the woman was not a tenancy by the entirety, which is a marital estate similar to a joint tenancy between a husband and wife. This estate arises presumptively (in some states) in any conveyance made to a husband and wife, and carries a right of survivorship. Here, the man and the woman took title to the land prior to their marriage, and their subsequent marriage does not affect the nature of their title. This is important because in a tenancy by the entirety, one spouse cannot convey any interest. (B) is incorrect because the man conveyed his interest in the land to the son. Thus, there is no interest or right in the land to which the heirs of the man can succeed under the intestacy laws. In addition, even if there had been no conveyance, the man's heirs would not have succeeded to his interest in the land. Rather, the woman would have taken an undivided interest in the property by means of the right of survivorship. (C) is incorrect because one joint tenant cannot convey his right of survivorship. When a joint tenant conveys his interest, it automatically becomes a tenancy in common interest. This is because the unity of time (one of the four unities required for creation of a joint tenancy) is lacking. To be joint tenants, the interests of the co-tenants must vest at the same time. Here, the woman and the man's interest vested at the same time, but the son's vested much later. Thus, the woman and her son cannot be joint tenants.
To acquire a prescriptive easement on property, the claimant's use must be: A Under color of title B In good faith and with the owner's permission C Exclusive, open and notorious, and adverse for the statutory period D Open and notorious, adverse, and continuous for the statutory period
D. o acquire a prescriptive easement on property, the claimant's use must be open and notorious, adverse, and continuous for the statutory period. Acquiring an easement by prescription is analogous to acquiring title to property by adverse possession, except that the use need NOT be exclusive (i.e., the user may share the use with the owner or other easement claimants). There is no requirement that the use be in good faith, and adverse use means the user does NOT have the owner's permission. Moreover, the claimant's use need not be under color of title. Color of title is a document that purports to give title, but for reasons not apparent from its face, does not. Under certain circumstances, color of title may be needed to establish adverse possession, but it is not required to acquire a prescriptive easement.
A man had rented a woman's home from her for seven years. When the time came to sign a new lease, the woman decided that because the man had always been a quiet tenant, she would continue to charge him only $350 per month rent instead of the $500 to $550 she could probably get otherwise. The new lease was for a period of five years, and by its terms, the man was specifically prohibited from assigning the lease without the woman's specific written consent. About a year later, the man got married and moved into his new wife's home. Instead of giving up his lease, the man sublet the property to a friend for $500 a month. The man did not get the woman's permission to sublease the property. If the woman brings an action to either eject the friend from the premises or to recover damages from the man for subletting the premises without her consent, what is the most likely result? A The woman will be able to recover damages and to eject the new tenant. B The woman will be able to eject the new tenant only, because she has suffered no money damages. C The woman will not be able to eject the new tenant because, although the man did not have the right to sublet, he had the power, but she will be entitled to recover the full rent paid by the new tenant because it would be unfair to let the man profit from his wrongful act. D The woman will have no cause of action for either ejectment or damages.
D. The woman will most likely have no cause of action for either ejectment or damages. There are two ways for a tenant to transfer the right to possession under a lease: assignment (transferring the entire period of time remaining under the lease) and sublease (transferring only a portion of the time remaining under the lease). Restraints on alienation are traditionally strictly construed. Thus, a covenant prohibiting assignment does not prohibit subleasing and vice versa. Hence, this prohibition against assignment would not be read to include a prohibition against subleasing. Therefore, the woman would have no cause of action against the man, and (A) and (B) are incorrect. (C) is incorrect. If a tenant transfers (assigns or sublets) in violation of a prohibition in the lease against transfers, the transfer is not void, but the landlord usually may terminate the lease under either the lease terms or a statute. Here, however, there is no cause of action because subleasing was not prohibited.
Under which theory can the mortgagee take possession of the mortgaged property upon the mortgagor's default? A The title theory only B Either the lien theory or the intermediate theory C The lien theory only D Either the title theory or the intermediate theory
D. Under either the title theory or the intermediate theory, the mortgagee may take possession of the mortgaged property upon the mortgagor's default. Under the title theory, followed in a minority of states, legal title is in the mortgagee until the mortgage has been satisfied or foreclosed. Thus, the mortgagee is entitled to possession upon demand at any time, which means the mortgagee can take possession as soon as the mortgagor defaults. The same is true in the few states that follow the intermediate theory, under which legal title transfers from the mortgagor to the mortgagee on default. Under the lien theory, followed in a majority of the states, the mortgagee is deemed to hold a security interest in the land and the mortgagor is considered the owner until foreclosure. Thus, the mortgagee may not take possession of the land before foreclosure.
A landowner advertised in the newspaper that he wished to sell 40 acres of land at $10,000 per acre. A rancher who was looking to expand his holdings was interested, so he came out to inspect the property. After the inspection, the rancher agreed to purchase the land for $400,000. A contract for the sale of the 40 acres was prepared and signed by the landowner and the rancher. The contract failed to state the purchase price. Later, the rancher had a change of heart and refused to complete the purchase. In the landowner's lawsuit for breach of contract, for which party would the court likely hold? A The landowner, because the parol evidence rule will not bar testimony that the rancher agreed to pay $400,000. B The landowner, because the Statute of Frauds can be satisfied by combining the original advertisement and the written contract. C The rancher, because the parol evidence rule will bar all evidence that he agreed to pay $400,000 for the land. D The rancher, because the Statute of Frauds would require the contract to contain the price in order to be enforced.
D. Under the Statute of Frauds, contracts for the sale of land must be in writing. The writing must contain all essential terms, and the price is considered an essential term. (A) is wrong because although the parol evidence rule might not bar the testimony, the Statute of Frauds will prevent recovery. (B) is wrong; the advertisement was not signed by the rancher, the party charged with breaking the contract. Thus, it is not a memorandum. Furthermore, the ad could not be considered part of the contract because there is nothing in the question indicating that it was attached to or referred to in the contract, or that it was assented to by the parties as part of the contract. In fact, an ad is a mere offer to deal; the actual price term may be very different by the time parties to a contract reach an agreement. (C) is wrong; the parol evidence rule would not bar the testimony, and in any event, that is not the reason the rancher will win.
Which of the following is NOT required for the burden of an equitable servitude to run to successors in interest? A The covenant touches and concerns the land. B The successor in interest has notice of the covenant if she has given value. C The covenanting parties intended that successors in interest be bound by the covenant. D There is vertical privity between the covenantor and his successor in interest.
D. Vertical privity between the covenantor and his successor in interest is not required for the burden of an equitable servitude to run to successors in interest. An equitable servitude is a covenant (i.e., a promise to do or not to do something on the land) that, regardless of whether it runs with the land at law, can be enforced in equity against assignees of the burdened land who have notice of the covenant. The burden of an equitable servitude will run to successors in interest if: 1. The covenanting parties intended that successors in interest be bound by the covenant; 2. The successor in interest has notice of the covenant (if she has given value); and 3. The covenant touches and concerns the land (i.e., it benefits the covenantor and his successor in their use and enjoyment of the burdened land). Horizontal privity between the original covenanting parties and vertical privity between the covenantor and his successor in interest are not required.
When a government classification intentionally discriminates on the basis of gender or legitimacy, the classification will be found unconstitutional unless the government can prove that the classification is at the least: A Necessary to achieve a compelling or overriding government purpose B Substantially related to a compelling or overriding government purpose C Necessary to achieve an important government purpose D Substantially related to an important government purpose
D. When a classification based on gender or legitimacy is challenged, an intermediate scrutiny standard is applied, under which the classification will be found invalid unless the government can show it is substantially related to an important government purpose. This is an intermediate level of scrutiny. The Supreme Court applies intermediate scrutiny to equal protection challenges involving intentional discrimination on the basis of gender or legitimacy. While the classification would be upheld if the government proved it was necessary to achieve a compelling or overriding government purpose, this standard is more demanding than is required, so it is not true that the classification will be found invalid unless the government can prove that it is necessary to achieve a compelling or overriding government purpose. The choice indicating that the classification must be substantially related to a compelling or overriding government purpose is incorrect because it pairs the "substantially related" language of the intermediate scrutiny test with the "compelling interest" needed under strict scrutiny. As indicated above, the intermediate scrutiny test applies. Similarly, the choice indicating that the classification must be necessary to achieve an important government purpose mixes strict scrutiny's "necessary" language with intermediate scrutiny's "important government purpose" standard and so is incorrect.
Which of the following is true when a seller of land dies before the contract closes? A The contract is voidable by the seller's estate. B The successors to the seller's personal property must give up equitable title at closing. C The contract is voided by the seller's death. D The successors to the seller's real property must give up legal title at closing.
D. When a seller of land dies before the contract closes, the successors to the seller's real property must give up legal title at closing. Under the doctrine of equitable conversion, the buyer of land is considered to own (i.e., hold equitable title to) the real property once the contract is signed. The seller is entitled to the proceeds of sale. Equity regards the seller as holding bare legal title in trust for the buyer as security for the debt owed. When a party to the contract dies before closing, her interest passes accordingly. The deceased seller's personal property takers are thus entitled to the sale proceeds on closing but must surrender legal title at that time. The successors to the seller's personal property do NOT give up equitable title at closing when a seller of land dies before the contract closes. As stated above, under the doctrine of equitable conversion, the buyer obtains equitable title to the land upon the signing of the contract. If the seller dies before closing, the bare legal title she held passes to the takers of her real property, who must surrender it to the buyer at closing (when both legal and equitable titles merge in the buyer). The takers of the seller's personal property succeed only to the proceeds of the sale, not the title to the real property. When a seller of land dies before the contract closes, the contract is NOT voidable by the seller's estate. It can be enforced against the takers of her real property when closing occurs. Furthermore, the contract is NOT voided by the seller's death. As is explained above, the doctrine of equitable conversion affects the passage of title when a party to a land sale contract dies before the closing. On closing, the seller's estate must surrender legal title to the buyer, and the estate is entitled to the proceeds of the sale.
As between two mortgages, what is the effect on the junior mortgage when the mortgagor accepts an advance of funds from the senior mortgagee? A The junior mortgage is given priority over the entire senior mortgage if the advance was optional B The junior mortgage is given priority over the advance if the advance was optional C The junior mortgage is given priority over the advance if the senior mortgagee was contractually obligated to make it D The junior mortgage is given priority over the entire senior mortgage if the senior mortgagee was contractually obligated to make it
D. When the mortgagor accepts an advance of funds from the senior mortgagee, the junior mortgage is given priority over the advance if the advance was optional. Priority among mortgages on the same real estate is normally determined by chronology: The earliest (i.e., senior) mortgage is first in priority, the next (i.e., junior) mortgage is second, and so on. Generally, if the mortgage obligates the mortgagee to make further advances of funds after the mortgage is executed, such advances will have the same priority as the original mortgage. However, if a junior mortgage is placed on the property and the senior mortgagee later makes an "optional" advance (i.e., one it was not contractually bound to make) while having notice of the junior mortgage, the advance will lose priority to the junior mortgage. Numerous states have reversed this rule by statute, but it remains the majority view. Thus, the junior mortgage is NOT given priority over the advance if the senior mortgagee was contractually obligated to make it. Furthermore, an advance would not jeopardize the priority of the entire senior mortgage itself; thus, the junior mortgage is NOT given priority over the entire senior mortgage, regardless of whether the advance was optional or the senior mortgagee was contractually obligated to make it.
Three thieves agreed to rob a bank. The first was to steal a car to be used for the getaway, the second agreed to procure weapons, and the third would check the bank for cameras. The car thief stole a car and parked it in a lot behind his girlfriend's apartment building. While visiting her the night before the robbery, the car thief suffered a series of convulsive seizures. He was rushed to the hospital where he was placed in the intensive care unit and heavily sedated. Meanwhile the two other thieves, unaware of their accomplice's illness, met and decided to rob the bank on their own, despite the absence of a getaway car and driver. They robbed the bank, but were quickly apprehended as they tried to escape and implicated the car thief under police questioning. The car thief can be charged with: A Theft of the car only. B Conspiracy to commit robbery and theft of the car only. C Robbery and theft of the car only. D Theft of the car, conspiracy to commit robbery, and robbery.
D. car that was to be used for the getaway. Conspiracy requires: (i) an agreement between two or more persons; (ii) the intent to enter into the agreement; and (iii) the intent to achieve the objective of the agreement. Most states also require an overt act in furtherance of the conspiracy. The car thief conspired to commit robbery, because he entered into an agreement with the two other thieves to rob the bank, intending both to enter into such an agreement and to achieve the objective thereof. The car thief's theft of the car constituted an overt act in furtherance of the conspiracy. Thus, the car thief can be charged with conspiracy. Furthermore, each member of a conspiracy is liable for the crimes of all other conspirators if: (i) such crimes were committed in furtherance of the objectives of the conspiracy; and (ii) such crimes were a natural and probable consequence of the conspiracy. A conspirator may limit his liability for subsequent acts of the other members of the conspiracy if he withdraws from the conspiracy by performing an affirmative act that notifies all members of the conspiracy in time for them to have the opportunity to abandon their plans. The car thief's absence from the robbery scene with the getaway car was due to his sudden illness, rather than any voluntary decision on his part to withdraw from the conspiracy. Thus, the car thief failed to make a legally effective withdrawal from the conspiracy. Certainly, the robbery of the bank, which was the sole object of the conspiracy, was a crime committed in furtherance of the conspiracy's objectives and was a natural and probable consequence of the conspiracy. Therefore, the car thief is liable for this robbery committed by his co-conspirators. Because the car thief can properly be charged with all three crimes, (A), (B), and (C) are incorrect.
A seller conveyed her residential city property to a buyer by a general warranty deed. On taking possession of the property, the buyer discovered that the garage of his neighbor encroached six inches onto his property. If the buyer wishes to compel the seller to assist him in a suit against the neighbor, which of the following covenants may he rely on to do so? A Seisin and encumbrances. B Warranty and further assurances. C Seisin and warranty. D Encumbrances and further assurances.
D. The buyer would rely on the covenants of warranty and further assurances to compel the seller to assist him in a suit against his encroaching neighbor. Under the covenant of warranty, the grantor agrees to defend, on behalf of the grantee, any lawful or reasonable claims of title by a third party, and to compensate the grantee for any loss sustained by the claim of superior title. The covenant for further assurances is a covenant to perform whatever acts are reasonably necessary to perfect the title conveyed if it turns out to be imperfect. These covenants are "continuous" (run with the land) and require the grantor to assist the grantee in establishing title. The covenants of seisin and encumbrances do not require such assistance. A covenant of seisin is a covenant that the grantor has the estate or interest that she purports to convey. Both title and possession at the time of the grant are necessary to satisfy this covenant. The covenant against encumbrances is a covenant assuring that there are neither visible encumbrances (easements, profits, etc.) nor invisible encumbrances (mortgages, etc.) against the title or interest conveyed. While the seller may have violated these two covenants because of the garage encroachment, they do not provide the basis to compel her to assist the buyer in a title suit. Instead, the buyer merely has a cause of action against the seller for their breach. Therefore, (A), (C), and (D) are wrong.
While cross-examining a defendant on trial for robbery and assault with a deadly weapon, the prosecutor asks him whether he was convicted of fraud within the previous year. Is this question proper? A No, because fraud is not probative of a tendency to commit violence. B No, unless the proper foundation was laid. C Yes, because fraud is a form of stealing, and so it will tend to show that the defendant could commit robbery. D Yes, because it tends to show that the defendant would lie.
D. While cross-examining a defendant on trial for robbery and assault with a deadly weapon, the prosecutor asks him whether he was convicted of fraud within the previous year. Is this question proper? response - correct A No, because fraud is not probative of a tendency to commit violence. B No, unless the proper foundation was laid. C Yes, because fraud is a form of stealing, and so it will tend to show that the defendant could commit robbery. D Yes, because it tends to show that the defendant would lie.
A landowner leased 150 acres of farmland to a produce company for 15 years. The produce company used the land for crops along with several other contiguous acres that it owned or leased. About four years into the lease, the state condemned a portion of the leased property because it intended to build a highway. As a result, too little property remained for the produce company to profitably farm, although there still existed the farmhouse on the property, which was being used by one of its foremen. The produce company gave the landowner 30 days' written notice that it considered the lease to have been terminated because of the condemnation. In a suit for breach of contract, is the landowner likely to win? A No, because the condemnation made it economically undesirable for the produce company to continue to lease the property. B No, because when there is a condemnation, the tenant's obligation to pay rent is extinguished. C Yes, because the produce company can still use the farmhouse, and the rental value would be adjusted accordingly. D Yes, because the condemnation did not affect the produce company's obligation to pay the full rental price, although it is entitled to share in the condemnation award.
D. The landowner probably will win in a breach of contract suit. In partial condemnation cases, the landlord-tenant relationship continues, as does the tenant's obligation to pay the entire rent for the remaining period of the lease. The tenant is, however, entitled to share in the condemnation award to the extent that the condemnation affected the tenant's rights under the lease. Therefore, (B) and (C) are incorrect. (A) is not correct because the law of landlord and tenant traditionally refuses to recognize frustration of purpose as grounds for termination of a lease.
Removal Jurisdiction
DEFENDANT can remove state action to federal courts. All defendants must join in or consent to the removal. Venue=whatever court embraces original state court. Defendant may remove separate and independent FQ claim, but federal court must sever and remand state court claims that don't invoke diversity or FQ. Limitations: when jurisdiction of federal court based solely on diversity and one of defendants is citizen of state in which state action brought, action not removable. Cannot be removed on basis of diversity more than one year after it was commenced in state court. Must be removed no later than 30 days after defendant discovered that case can become removable. Does not apply if plaintiff acted in bad faith (i.e., fraudently joining nondiverse party in order to prevent defendant from removing action.)
Effect of Failure to Answer-Default and Default Judgment
Default is simply notation in case file by clerk that defendant has not answered within time permitted by rules. Once default entered, party may not proceed with action until default haws been set aside by court. Default judgment: clerk may sign and enter judgment for amount and costs against defendant if: plaintiffs claim against defaulted D is for sum certain, default entered because defendant failed to appear, defaulted D is not infant or incompetent person, damage amount requested not greater than amount requested in complaint. Notice Required- if defendant has appeared, even if he has not answered, must be notified at least 7 days before hearing on application for default judgment.
Rule 11 sanctions
May consist of monetary penalties or nonmonetary directive and may be imposed on law firms, attorneys, or PARTIES.
Proximate Cause (Indirect Cause)
Facts indicate force came into motion after time of defendant's negligence and combined with negligent act to cause injury to plaintiff; intervening forces present. Whether it cuts off defendant's liability determined by foreseeability; 1) Foreseeable Results Caused by Foreseeable Intervening Forces: Defendant liable. Examples: subsequent medical malpractice, negligence of rescuers, efforts to protect person or property, "reaction" forces (stampeding pedestrians), subsequent disease, subsequent accident. Independent intervening forces: where defendant's negligence increased risk that these forces would harm plaintiff. Negligent acts of third persons, criminal acts and intentional torts of third persons, acts of god
Interpleader (General and Diversity)
Federal Impleader Statute: only that among the parties there be two or more adverse claimants of diverse citizenship; any two is fine. Under Federal Rules: complete Diversity Generally: joinder by defendant in originally action (usually called third party plaintiff) of another person not originally party to action; asserts that th ethird party defendant is or may be liable to defendant for all or part of plaintiff's claim against D). May assert claim against plaintiff if claim arises from same T/O and plaintiff may assert claim against him UNLESS supplemental jurisdiction would not fulfill complete diveristy.
Erie Doctrine
Federal court in the exercise of its DIVERSITY JURISDICTION is required to apply the substantive law of the state in which it is sitting, including state's conflict of law rules; however, still applies federal procedural law (rule of civil procedure) 1) is there a federal statute on point? 2) substantive or procedural; (statute of limitations, choice of law, elements of claim or defense all substantive and court must apply state law) Test: ocutcome determinative, issue is outcome determinative if it substantially affects outcome of case, and forum shopping deterence.
A private citizen may make felony arrest without warrant if:
Felony IN FACT had been committed and reasonable grounds for believing person arrested did it.
When is an action commenced?
Filing a complaint; unless state rule that an action is commenced for purposes of statute of limitations only upon service of process in diversity cases.
Pleadings: Complaint
Filing commences an action. Requirements: 1) statement of grounds of SMJ; 2) Short and plain statement of the claim, showing entitle to relief; 3) demand for relief sought. Standard of Pleading: must plead facts supporting a plausible claim (three things that must plead with particularity or specificity - fraud, mistake, special damages)
Intervention of Right
Given when: 1) intervenor claims interest relating to property transaction that is subject of the action; 2) disposition of action may adversely affect the interest 3) current parties do not adequately represet interventors rights.
Transfer of Venue
If venue proper --> where also has jurisdiction + look at convenience and interests of justice (transferor state) If venue improper --> where has jurisdiction + interests of justice (apply law of transferee state) Public Factors = law applies, forum interest, local controversy/local court, Private Factors = where W, where evidence, where COA arose
Duty to Invitees
Invitee is a person who enters onto premises in response to express or implied visitation. Two types: those who enter as members of public/land held open to public AND those who enter for purpose connected with business. Lose status of invitee if you exceed scope of invitation. Duty: to use ordinary or reasonable care in keeping property reasonably safe for benefit of invitee; includes duty to warn or make safe non-obvious dangerous conditions, but ALSO to make reasonable inspects. Warnings may suffice.
Defendant's Answer
Must contain specific detail or admission of each averment of complaint; failure to deny constitutes admission. Answer must state any affirmative defenses. If no Rule 12 Motion made, defendant must present answer within 21 days; if waived service, 60 days; If Rule 12 motion made, responsive pleading to be served within 14 days of court's denial or postponement of motion. If same T/O, compulsory counterclaim.
Duty of possessor to those off premises
Natural conditions: no duty. Artificial conditions: no duty EXCEPT for unreasonably dangerous conditions (huge construction site with hole) or passerbys
Question 6 An uncle who owned a large tract of wooded land in fee simple had always opened his land to hunters from the local hunting club, and had often expressed the wish that they be permitted to continue to use it after he died. On his 75th birthday, he conveyed the land to his niece and nephew, who were members of the hunting club. The deed was a general warranty deed conveying the property in fee simple absolute. A few days later, the niece had a serious dispute with the nephew and the other members of the hunting club, and she quit the club. The uncle then executed a written agreement with the nephew stating that the conveyance of the land to the niece and nephew was in trust for the benefit of the local hunting club for a period of 10 years, with the niece and nephew as trustees, and then to the niece and nephew in fee simple. Several months later, the uncle died. When the next hunting season drew near, the nephew told the niece that members of the hunting club were once again planning to hunt on the property. The niece threatened to have anyone hunting on the property other than the nephew arrested for trespassing. The nephew brought an action for appropriate legal or equitable relief to establish his rights and the rights of the hunting club. What, if any, relief should the court provide? (A) Deny the nephew any relief, because the niece has done nothing that would constitute an ouster of the nephew. (B) Partition the land into two separate tracts so that the nephew may permit the hunting club to use his half. (C) Order the niece to permit the hunting club to hunt on the land, because the uncle created an inter vivos trust with the requisite formalities for the benefit of the hunting club. (D) Order the niece to permit the hunting club to hunt on the land, because the nephew is equally entitled to possession of all of it and can allow members of the hunting club to hunt on the property as his guests.
Once you transfer land, its gone. (B) The court should partition the land into two separate tracts. A court will presume that the devise to the niece and nephew gave them a tenancy in common. Hence, each of them has the right to possess all portions of the property; neither of them has the right to exclusive possession of any part. However, any tenant in common has a right to judicial partition of the property, either in kind or by sale and division of the proceeds. When co-tenants are squabbling and cannot come to any agreement, the remedy of partition terminates the co-tenancy and divides the common property. Since the niece and nephew cannot agree on the use of the land by members of the hunting club, the court will probably partition the property. (A) is wrong because, even though there is no indication that the niece has ousted the nephew, i.e., wrongfully excluded him from possession of the property, the nephew is entitled to the remedy of partition because the parties cannot agree on the use of the property. (C) is wrong because the uncle did not own the land at the time he attempted to convey the property in trust. While the instrument the uncle executed was in writing and properly identified the trustees and an appropriate beneficiary, the uncle had already conveyed away in fee simple his interest in the property; thus, he did not have the power to create a trust. (D) is wrong despite the fact that the nephew is entitled to possession and use of all of the land to the same extent as the niece. Since the co-tenants cannot agree on this particular use of the land by members of the hunting club, a court will grant whichever tenant is before it the remedy of partition.
Assault
Prima Facie Case 1) Act by defendant creating reasonable apprehension in plaintiff of immediate harmful or offensive contact to the plaintiff's person; 2) intent on the part of the defendant to bring about in the plaintiff's apprehension of immediate harmful or offensive contact with the plaintiff's person 3) Causation "Apprehension"; reasonable, but not the same as fear, simply "expectation." Knowledge of act required. Defendant must have APPARENT ability to carry out the threat. An overt act is required and words may negate an assault. Apprehension must be of immediate harmful or offensive contact. Transferred Intent applies. No requirement of damages.
Battery
Prima Facie Case: 1) Act by defendant which brings about a harmful or offensive contact to the plaintiff's person; 2) intent on the part of the defendant to bring about harmful and offensive contact; 3)causation Contact harmful/offensive if would be to reasonable person. "Contact" means anything connected to the plaintiff's person. Actual damages not required, nominal damages okay. Transferred Intent applies.
Defamation
Prima Facie Case: i) defamatory language; ii) of or concerning the plaintiff; iii) publication to a third person iv) damage to plaintiff's reputation Public figure, plaintiff must prove: falisity and fault.
Misrepresentation
Prima facie cause (intentional misrepresentation, fraud, or deceit) 1) misrepresentation 2) Scienter (made it knowingly or recklessly) 3)intent to induce plaintiff's reliance on misrepresentation 4) causation 5) justifiable reliance 6) damages
A plaintiff, a citizen of State A, sued a defendant, a citizen of State B, in state court in State B for breach of a contract to build a house for $200,000. The defendant counterclaimed for $300,000, alleging that the plaintiff breached an earlier contract by failing to pay for a house that the defendant had built. The plaintiff files a notice of removal to federal court in State B. Can the case properly be removed to the federal court in State B? A Yes, because all plaintiffs are of diverse citizenships from all defendants. B Yes, unless the defendant objects to removal. C No, unless the defendant joins in the removal. D No, because only defendants may remove.
Removal is not proper. A plaintiff may not remove on the basis of a counterclaim against him that could have been brought in federal court. Additionally, only defendants may remove a case to federal court. For these reasons, (A) and (B) are incorrect. (C) is incorrect because removal by a plaintiff is improper even if the defendant joins in removal. An argument could be made that, because the defendant has her own right to seek removal, the court should treat her "joining" the plaintiff's removal request as her own request for removal. Here, however, the defendant has no independent right of removal because she is a citizen of the forum state.
Defendant Pre-Answer Motions
Rule 12(b): may file for lack of SMJ, lack of PJ, improper venue, insufficient process, insufficient service of process, failure to state claim upon which relief can be granted, failure to join necessary or indispensable party. Lack of SMJ can be raised at any time, even on appeal, lack of PJ, improper venue/process/service must be raised AT TIME FILES MOTION OR ANSWER, whichever is first, or waived. failure to state claim or failure to join party any time before trial or at trial. Motion for More Definitive Statement: before responding to pleading, opposing party has 14 days unless court fixes different time, if not obeyed court may strike. Party may move to have stricken within 21 days after service of pleading any insufficient defense, or any redundant, immaterial, impertinent or scandalous matter; may be made upon court's initiative at any time.
Preliminary Injunctions (Civ. Procedure)
Sought by party prior to trial on merits of complaint. Requirements: plaintiff will suffer irreparable harm if injunction not granted, harm to plaintiff if injunction not granted outweighs harm to the defendant if injunction is granted, plaintiff shows that he is likely to be successful on merits, and public interest favors granting injunction.
A backgammon player was upset after losing a match against the club champion. Rushing out of the club, he inadvertently grabbed the champion's board, which looked very much like his own but which was much more expensive. The player left the backgammon board in the trunk of his car, as was his usual practice. During the night, the car was stolen and along with it, the champion's expensive backgammon board. In an action by the champion against the player to recover the value of the backgammon board, is the champion likely to recover? A Yes, because when the player took the backgammon board he committed a trespass to the champion's chattel. B Yes, because when the backgammon board was stolen along with the car, the player became liable for conversion of the champion's chattel. C No, because the player believed in good faith that the board was his when he took it from the backgammon club. D No, because the backgammon board was lost through no fault of the player's.
The champion will recover from the player for conversion. Conversion consists of (i) an act by defendant interfering with plaintiff's right of possession in the chattel, (ii) intent to perform the act bringing about the interference with plaintiff's right of possession, (iii) causation, and (iv) damages—an interference that is serious enough in nature or consequence to warrant that the defendant pay the full value of the chattel. Intent to trespass is not required; intent to do the act of interference with the chattel is sufficient for liability. Therefore, the player was guilty of conversion when he intentionally (i.e., volitionally) took the champion's board, which resulted in its loss, even though the player did not intend to lose it or even realize that he had taken the property of another.
An art collector was interested in buying a painting from his neighbor. The neighbor told the collector that he could have the painting for $30,000. The collector wanted to think the purchase over. Therefore, the two agreed in writing that the neighbor would keep the offer open for 30 days in exchange for $500, which the collector paid. The terms of the written agreement provided that the offer would expire at 11:59 p.m. on September 30 if the collector failed to accept by that time. On September 20, the collector telephoned his neighbor and told him, "The more I think about it, the less I think that I want your painting." The neighbor responded, "That's your decision to make." On September 26, one of the neighbor's friends was visiting him, saw the painting, and offered his friend (the neighbor) $35,000 for it. On September 27, the neighbor mailed a $50 check to the collector with a letter stating that he was terminating his offer to the collector regarding the painting and refunding 10% of the money that the collector paid him to keep the offer open. He mailed the letter at 11:59 p.m. on September 27. The collector received the letter at 11:30 a.m. on September 29. On September 28, at 9:30 a.m., the collector mailed a letter to his neighbor stating that he had decided to purchase the painting and a certified check in the amount of $30,000 was enclosed. Two hours later, the neighbor sold the painting to his friend for $35,000. The neighbor received the collector's letter on October 1 and immediately mailed the check back to the collector. Can the collector maintain a successful legal action against his neighbor? A Yes, because the neighbor sold the painting after the collector's effective acceptance, and before the neighbor's revocation became effective. B Yes, because in his revocation the neighbor did not refund the full $500 to the collector. C No, because the neighbor effectively revoked his offer before the collector accepted. D No, because the collector's power to accept lapsed before he effectively accepted.
The collector's power to accept lapsed because the option contract specified that the offer would expire at 11:59 p.m. on September 30. Hence, the power had to be exercised prior to that time and it was not. The mailbox rule does not apply to the exercise of options. In such cases, acceptance is effective when received by the offeror, here on October 1. Thus, (D) is correct. (A) is wrong because, for the reasons discussed above, the collector did not effectively accept before his option expired. (C) is wrong for two reasons: (i) a revocation is not effective until received; and (ii) because the contract is an option, the offeror's power to terminate the offer through revocation is limited. Even if the revocation had arrived earlier, the neighbor lacked the power to revoke. (B) is irrelevant. Returning the consideration, in and of itself, would not give the offeror the power to revoke in an option situation.
A developer constructed several small stores in a commercial district. She received a bid from a contractor to install awnings on the front windows of the stores. The developer had heard that the contractor did shoddy work, but the price was right and the contractor expressly assumed all of the risk of any liability. The developer subsequently sold one of the stores to a barber. A few months later, an awning collapsed without warning, injuring a customer who was about to enter the barbershop. An investigation by the building inspector revealed that the awning collapsed because the brackets used by the contractor were cheaper and weaker than the required brackets, although they looked the same. The developer and the contractor are now both bankrupt. If the customer sues the barber for his injuries, is the customer likely to prevail? A Yes, because the barber had a nondelegable duty to keep the premises safe for customers and those passing by. B Yes, because the developer did not exercise reasonable care in hiring the contractor to install the awnings. C No, because the contractor assumed all of the risks from his work. D No, because the barber had no opportunity to oversee the contractor's actions.
The customer is not likely to prevail because the barber had no opportunity to oversee the contractor's work. A property owner owes a duty to those adjacent to the premises to take due precautions to protect them from dangerous conditions, and a business owes its customers a duty to make reasonable inspections to discover and make safe any dangerous conditions. Further, that duty cannot be delegated to an independent contractor; the owner remains vicariously liable. However, nothing in the facts establishes that the barber breached his duty to the customer. There was nothing wrong with the brackets evident from a reasonable inspection, and the awning collapsed without warning. Further, the barber was not involved in hiring or supervising the contractor and would not be responsible for the contractor's negligence. Hence, because no facts point to negligence by the barber, the customer is not likely to prevail.
Transferred Intent
The intent to commit a tort against one person is transferred to the other tort or to the injured person. - Can only be used on Assault; Battery; False Imprisonment; Trespass to land; and Trespass to chattels.
A small business owner decided to retire, so she offered her long-time employee a chance to buy the business for $1 million. She promised in writing to keep the offer open to him for 90 days and to give him enough time to secure financing once he accepted the offer. Over the next few days, the employee cashed out all his retirement accounts and took a second mortgage on his home to raise the funds to purchase the business. When he approached the business owner to discuss the details of the sale, she said that she changed her mind and was revoking her offer because she did not want to retire after all. Was the owner's revocation of her offer proper? response - incorrect A Yes, because it was an offer that could be revoked at will. B No, because the owner created an option contract by promising to keep the offer open for 90 days. C No, because the employee detrimentally relied on the offer. D No, because the offer constitutes a merchant's firm offer.
The owner's revocation of her offer was proper because the offer could be revoked at will. Generally, offers can be revoked at will by the offeror, even if she has promised not to revoke for a certain period of time. There are limitations on the offeror's power to revoke, but none of those exceptions apply in this case. (B) is incorrect because an option contract requires that the offeree give consideration for the promise by the offeror to keep the offer open, and no consideration is indicated by the facts. (C) is also incorrect. Detrimental reliance can limit an offeror's power to revoke where the offeror could reasonably expect that the offeree would rely to his detriment on the offer, and the offeree does so rely. However, this usually is limited to those situations in which the offeror would reasonably contemplate reliance by the offeree in using the offer before it is accepted; e.g., when a general contractor uses a subcontractor's bid in making its own offer. Here, the offer itself included a promise by the owner to give the employee time to secure financing after the offer was accepted. Therefore, the owner had no reason to anticipate that the employee would take immediate steps to raise the purchase money before he even accepted the offer. (D) is incorrect because these facts are not an example of a merchant's firm offer. A merchant's firm offer does not apply to any offer by a merchant; it applies only to an offer under the UCC for the sale of goods where a signed writing gives assurances that the offer will be held open.
Question 10 A hunter residing in State A visited a website created and operated by a hunting equipment company. The company is a State B corporation, and its headquarters and all its facilities are in State B. It sells its products in several stores in State B and through its website. Its marketing efforts and most of its sales are in State A and State B. Using the company's website, the hunter ordered a hunting stand. The hunter paid for the stand and shipment to State A by providing his credit card information on the company's website. After the stand arrived at the hunter's home in State A, the hunter used the stand while on a hunting trip in State C—a very popular hunting destination. The stand collapsed, causing severe injuries to the hunter. The hunter filed a products liability action against the company in a State C court. The company filed a motion to dismiss the action on the ground that the State C court lacks personal jurisdiction over it. Although the company's website is accessible in State C, the company does not otherwise advertise there, and it does no business there. There is no evidence that the company has sold any products in State C. Does the company have sufficient contacts with State C such that a State C court may exercise personal jurisdiction over it? (A) Yes, because the company's website is continuously accessible in State C so that people there may purchase products from the company at any time. (B) Yes, because the company could foresee that its customers would carry the company's hunting products on hunting trips to other states—particularly states where hunting is popular. (C) No, because personal jurisdiction cannot be based on Internet contacts. (D) No, because the company does not have sufficient purposeful contacts with State C.
Watch for contacts being the result of the defendant's UNILATERAL ACTIVITY. (D) The company does not have sufficient contacts for personal jurisdiction. In addition to the state having a statute authorizing the court to exercise personal jurisdiction over a defendant, the exercise of personal jurisdiction must be constitutional, meaning that the defendant must have purposeful contacts with the forum state such that it would be fair and reasonable to exercise personal jurisdiction over him. Personal jurisdiction may be "specific," meaning that the court has personal jurisdiction over the defendant only for the plaintiff's cause of action, or it may be "general," meaning that the court has personal jurisdiction over the defendant for all causes of action. The close connection between the plaintiff's cause of action and the contact from which the case springs is usually sufficient for the court to exercise "specific" jurisdiction over the defendant. As to "general" jurisdiction, recent Supreme Court decisions require the defendant to be "at home" in the jurisdiction, a very difficult standard to meet. Here, the company had no purposeful contact with State C. It was the hunter's unilateral act that brought the stand to State C from State A. Such an act does not give the forum court personal jurisdiction over a company that places a movable object into the stream of commerce [World-Wide Volkswagen Corp. v. Woodsen]. (A) is incorrect because the mere accessibility of a company's website in a state, without more, is usually insufficient contact to support personal jurisdiction, and the claim does not arise from that contact. (B) is incorrect because the stand's transport to State C is the result of the plaintiff's unilateral activity and thus not purposeful on the part of the company. (C) is incorrect because Internet contacts can be sufficient to create personal jurisdiction in appropriate cases (e.g., when they are purposeful and the claim arises from the contacts).
Joinder: Should absentee be joined?
When complete relief cannot be accorded amount other parties without absentee made party; when absentee has such interest in subject matter that a decision in his absence will (common on bar) i) as a practical matter, impair or impede ability to protect the interest, or leave any of the other parties subject to substantial risk of incurring multiple or inconsistent obligations.
Lien theory v. title theory on mortgages
Whether a mortgage creates a severance or not depends on whether the state follows the lien theory or the title theory of mortgages. Lien theory means no severance; title theory means severance. Because this is a lien theory state (majority rule on the MBE), there was no severance; thus, the joint tenancy remained intact. On the first partner's death, the joint tenancy ended and the first partner's interest instantly passed to the second partner. The first partner's estate got nothing; hence, the daughter could get nothing. (B) would be the correct answer if the execution of the mortgage had created a severance of the joint tenancy. The severance would have changed the joint tenancy to a tenancy in common. The second partner would keep her one-half, free of the mortgage she did not sign, and the daughter would inherit the first partner's one-half, subject to the mortgage the first partner signed. Because we are told this is a lien theory state, there was no severance when the mortgage was executed so the joint tenancy remained intact until the first partner's death. (C) is incorrect. Not only is the joint tenancy unsevered so that it remains intact to give title to the entire property to the second partner, but also under no circumstances could the second partner be held liable for a mortgage she did not enter into. (D) is incorrect. When the first partner died, the property passed free and clear to the surviving joint tenant. The mortgage signed by the first partner did not sever the joint tenancy because this is a lien theory state. The mortgage can be held only against the property the first partner has; when the first partner died, the right of survivorship operated to end the first partner's interest and automatically vested it in the survivor.
The Privileges or Immunities Clause prohibits states from infringing upon their citizens': A Right to travel B Right to earn a living C Right to an attorney D Freedom of speech
a. The Privileges or Immunities Clause prohibits states from denying their citizens the rights of national citizenship, which includes the right to travel. Other rights protected include the right to petition Congress for redress of grievances, the right to vote for federal officers, and the right to enter public lands. Freedom of speech is not a right of national citizenship under the Privileges or Immunities Clause of the Fourteenth Amendment. In The Slaughterhouse Cases, the Supreme Court held that rights of national citizenship do not include all of the rights in the Bill of Rights, although some rights under the Bill of Rights may be held applicable to the states as incidents of due process. The right to earn a living is not a right of national citizenship under the Privileges or Immunities Clause of the Fourteenth Amendment, although the right is protected under the Privileges and Immunities Clause of Article IV, which limits states from discriminating against nonresidents. The right to an attorney is not a right of national citizenship under the Privileges or Immunities Clause of the Fourteenth Amendment. Although the right is included in the Bill of Rights under the Sixth Amendment, as discussed above, the Supreme Court held in The Slaughterhouse Cases that rights of national citizenship do not include the Bill of Rights.
In contrast to product liability case based on negligence, a products liability case based on strict liability does not require of the suppliers:
that the suppliers had opportunity to inspect. For negligence, they do require that suppliers have the opportunity to inspect. In SL, liable either way. Plaintiff may recover personal injury damages and property damages.